Attachment Paper-3 Lyst8178

You might also like

Download as pdf or txt
Download as pdf or txt
You are on page 1of 439

CA - FOUNDATION

BUSINESS MATHEMATICS
[ PAPER-3 ]
PART-A
&
LOGICAL REASONING &
STATISTICS
[ PAPER-3 ]
PART-B & C
CONTENTS
PART - A : BUSINESS MATHEMATICS

1. RATIO AND PROPORTION, INDICES, LOGARITHMS ........................... .................................................... 1-43


UNIT 1 : RATIO & PROPORTION ............................. ................................................................................. 1
CLASS WORK ........................... ................................................................................................... 5
HOME WORK-1 ....................... ................................................................................................... 8
HOME WORK-2 ....................... ................................................................................................... 14
UNIT 2 : INDICES .................................... .................................................................................................. 16
CLASS WORK ........................... ................................................................................................... 18
HOME WORK-1 ....................... ................................................................................................... 22
HOME WORK-2 ....................... ................................................................................................... 29
UNIT 3 : LOGARITHMS ............................ .................................................................................................. 31
CLASS WORK ........................... ................................................................................................... 35
HOME WORK-1 ....................... ................................................................................................... 39
HOME WORK-2 ....................... ................................................................................................... 43
2. EQUATIONS AND MATRICES ........................... ......................................................................................... 44-72
UNIT I: EQUATIONS ................................. ................................................................................................. 44
CLASS WORK ............................. ................................................................................................. 51
HOME WORK-1 ........................ .................................................................................................. 60
HOME WORK-2 ........................ .................................................................................................. 62
UNIT II: MATRICES ................................. ................................................................................................... 46
CLASS WORK ............................. ................................................................................................. 63
HOME WORK-1 ........................ .................................................................................................. 68
HOME WORK-2 ........................ .................................................................................................. 71
3. LINEAR INEQUALITIES ............................... ................................................................................................ 73-103
CLASS WORK .................................... ........................................................................................................ 74
HOME WORK-1 ............................ ............................................................................................................ 85
HOME WORK-2 ............................ ............................................................................................................ 101
4. TIME VALUE OF MONEY (MATHS OF FINANCE) ............................ .......................................................... 104-122
CLASS WORK ........................................ .......................................................................................................... 114
HOME WORK-1 ............................ ............................................................................................................ 118
HOME WORK-2 ............................ ............................................................................................................ 121
5. BASIC CONCEPTS OF PERMUTATIONS AND COMBINATIONS ............................... ................................ 123-139
CLASS WORK .............................. ............................................................................................................... 126
HOME WORK-1 ............................ ............................................................................................................ 137
HOME WORK-2 ............................ ............................................................................................................ 139
6. SEQUENCE AND SERIES
(ARITHMETIC AND GEOMETRIC PROGRESSIONS) .......................... ....................................................... 140-164
CLASS WORK ................................ ............................................................................................................. 152
HOME WORK-1 ............................ ............................................................................................................ 159
HOME WORK-2 ............................ ............................................................................................................ 163
7. SETS, FUNCTIONS AND RELATIONS ............................ .............................................................................. 165-182
CLASS WORK ............................... .............................................................................................................. 171
HOME WORK-1 ............................ ............................................................................................................ 180
HOME WORK-2 ............................ ............................................................................................................ 182
8. BASIC CONCEPTS OF DIFFERENTIAL AND INTEGRAL CALCULUS ........................... ............................... 183-212
CLASS WORK ............................... .............................................................................................................. 191
HOME WORK-1 ............................ ............................................................................................................ 195
HOME WORK-2 ............................ ............................................................................................................ 199
PART - B : LOGICAL REASONING

9. NUMBER SERIES, CODING AND DECODING AND ODD MAN OUT ........................ ................................ 213-219
CLASS WORK ................................. ........................................................................................................... 215
HOME WORK-1 ............................ ............................................................................................................ 218
HOME WORK-2 ............................ ............................................................................................................ 219
10. DIRECTION TESTS ............................. ........................................................................................................ 220-229
CLASS WORK ............................ ................................................................................................................. 221
HOME WORK-1 ............................ ............................................................................................................ 226
HOME WORK-2 ............................ ............................................................................................................ 229
11. SEATING ARRANGMENTS ............................. ............................................................................................. 230-239
CLASS WORK ............................. ................................................................................................................ 233
HOME WORK-1 ............................ ............................................................................................................ 236
HOME WORK-2 ............................ ............................................................................................................ 238
12. BLOOD REALATION .............................. ..................................................................................................... 240-247
CLASS WORK ............................. ................................................................................................................ 242
HOME WORK-1 ............................ ............................................................................................................ 246
HOME WORK-2 ............................ ............................................................................................................ 247
13. SYLLOGISM .............................. .................................................................................................................. 248-266
CLASS WORK ............................. ................................................................................................................ 260
HOME WORK-1 ............................ ............................................................................................................ 262
HOME WORK-2 ............................ ............................................................................................................ 265
PART - C : STATISTICS

14. STATISTICAL DESCRIPTION OF DATA ......................... ................................................................................ 267-283


CLASS WORK ................................. ........................................................................................................... 271
HOME WORK-1 ............................ ............................................................................................................ 279
HOME WORK-2 ............................ ............................................................................................................ 282
15. MEASURES OF CENTRAL TENDENCY AND DISPERSION ...................................................... .................... 284-322
CLASS WORK ............................ ................................................................................................................. 290
HOME WORK-1 ............................ ............................................................................................................ 310
HOME WORK-2 ............................ ............................................................................................................ 321
16. CORRELATION AND REGRESSION ........................... .................................................................................. 323-354
CLASS WORK ............................. ................................................................................................................ 332
HOME WORK-1 ............................ ............................................................................................................ 344
HOME WORK-2 ............................ ............................................................................................................ 353
17. PROBABILITY ............................. ................................................................................................................. 355-378
CLASS WORK ................................ ............................................................................................................. 358
HOME WORK-1 ............................ ............................................................................................................ 370
HOME WORK-2 ............................ ............................................................................................................ 378
18. PROBABILITY DISTRIBUTION .............................. .................................................................................... 379-402
CLASS WORK ............................. ................................................................................................................ 386
HOME WORK-1 ............................ ............................................................................................................ 395
HOME WORK-2 ............................ ............................................................................................................ 402
19. INDEX NUMBER AND TIME SERIES ....................... .................................................................................... 403-434
UNIT 1 : INDEX NUMBER ............................ ............................................................................................. 403
CLASS WORK ............................. ................................................................................................................ 409
HOME WORK-1 ............................ ............................................................................................................ 422
HOME WORK-2 ............................ ............................................................................................................ 427
UNIT 2 : TIME SERIES ................................ ................................................................................................ 428
CLASS WORK ............................. ................................................................................................................ 430
HOME WORK-1 ............................ ............................................................................................................ 433
HOME WORK-2 ............................ ............................................................................................................ 434
SECTIONS-WISE WEIGHTAGE

Foundation Paper 3: Business Mathematics,


Logical Reasoning and Statistics (100 Marks)
Sections Weightage Content Area

Part A : Business Mathematics (40 Marks)

I 20%-30% 1. Ratio and Proportion, Indices and Logarithms


(i) Ratio and proportion and Business Applications to Ratio
and Proportion problems, Laws of Indices, Exponents and
Logarithms and Anti Logarithms.
2. Equations and Matrices
(i) Equations: Linear Simultaneous linear equations up to three
variables, Quadratic and Cubic equations in one variable.
(ii) Matrices: Algebra of Matrices, Inverse of a Matrix and
determinants, solving system of (Cramer’s rule) involving not
more than three variables equations using matrix method
3. Linear Inequalities: Linear Inequalities in one variable and the
solution space and optimal Solution
II 30%-40% 4. Time Value of Money
(i) Simple Interest
(ii) Compound interest and annuity applications,
(iii) Perpetuity
(iv) Depreciation
(v) Effective Rate of Interest
(vi) Present Value
(vii) Calculations of Returns:
(a) Nominal Rate of Return
(b) Effective Rate of Return
(c) Compound Annual growth rate (CAGR)
III 30%-50% 5. Permutations and Combinations
Introduction, the factorial, permutations, results, circular
permutations, permutations with restrictions, Combinations with
standard results.
6. Sequence and Series
Introduction Sequences, Series, Arithmetic and Geometric
progression, Relationship between AM and GM and Sum of n terms
of special series
7. Sets, Relations and Functions
8. Basic applications of Differential and Integral calculus (Excluding
the trigonometric applications) Applications of Marginal Cost and
Marginal Revenue etc.,

Part B: Logical Reasoning (20 Marks)

I 60%-70% 9. Number series coding and Decoding and odd man out.
10. Direction Tests
11. Seating Arrangements
II 30%-40% 12. Blood Relations13. Syllogism

Part C: Statistics (40 Marks)

I 45%-50% 14. Statistical Representation of Data, Diagrammatic representation


of data, Frequency distribution, Graphical representation of
Frequency Distribution –Histogram, Frequency Polygon, Ogive, Pie-
chart
15. Measures of Central Tendency and Dispersion: Mean Median, Mode,
Mean Deviation, Quartiles and Quartile Deviation, Standard
Deviation, Co-efficient of Variation, Coefficient of Quartile
Deviation.
II 25%-30% 16. Probability: Independent and dependent events; mutually exclusive
events.25% Total and Compound Probability; Bayes’ theorem; and
Mathematical Expectation.
17. Theoretical Distribution: Binomial Distribution, Poisson distribution
– basic application and Normal Distribution – basic applications.
III 10%-15% 18. Correlation and Regression: Scatter diagram, Karl Pearson’s
Coefficient of Correlation Rank Correlation, Regression lines,
Regression equations, Regression coefficients.
IV 10%-15% 19. Index Numbers and Time series
(i) Uses of Index Numbers, Problems involved in construction of
Index Numbers, Methods of construction of Index
Numbers.
(ii) Time Series Analysis – basic application including Moving
Average Method
CHAPTER-1
RATIO & PROPORTION, INDICES, LOGARITHM

UNIT I : RATIO & PROPORTION


Features :
1. Comparison : A ratio is a comparison of the sizes of two or more quantities of the same kind by
division.
2. Same Unit : The two quantities should be of the same unit.
3. Lowest Terms : Ratio is expressed in lowest terms.
4. Fraction : If ‘a’ and ‘b’ are the quantities of the same kind, then the fraction a/b is called the ratio
of ‘a’ to ‘b’.
5. Antecedent : The first term ‘a’ is called Antecedent.
6. Consequent : The second term ‘b’ is called Consequent.
7. Multiplication / Division : Ratios remain unaffected if both the Antecedent and Consequent are
multiplied or divided by the same term.
8. Order : The order of the terms in a ratio is important. Example : 7:8 is not same as 8:7.
Original Quantity
9. If a Quantity increases or decreases in the ratio a : b then, New Quantity = b x .
a
The fraction by which the Original Quantity is multiplied to get a New Quantity is called the Factor-
Multiplying Ratio.
 TYPES OF RATIO
1. Compounded ratio [Multiplication of given Ratio]

a b c d
, , ,
b c d e

a b c d
Compounded ratio × × × = a:e
b c d e
2. Duplicate Ratio [Square of given Ratio]:
Duplicate ratio of a:b = a2:b2
3. Triplicate Ratio [Cube of given Ratio]:
Triplicate Ratio of a:b = a3: b3
4. Sub-Duplicate ratio (Square root of given ratio)

RATIO AND PROPORTION, INDICES, LOGARITHMS 1


Sub-Duplicate ratio of a:b = a: b
5. Sub-Triplicate Ratio (Cube root of given ratio)

Sub-Triplicate Ratio of a:b = 3 a : 3 b


6. Commensurable Ratio (Ratio of rational numbers)

4 1.5
Example : = 4:5, = 15:25 = 3:5
5 2.5
7. Un- Commensurable ratio (Ratio or irrational numbers) :

3
Example :
5

UNIT II : PROPORTION
 PROPORTION

a c
=
b d
 a, d are known as extreme values (terms) b, c are know as Mean terms

a c
 = Or a:b = c:d
b d

a c
:: Or a:b : : c:d
b d
 Continued proportion :

a b c
= =
b c d
 a, b, c, d are in continued proportion
 a, b,c are in Proportion

a b
= 
b c
A. Features of Proportion :
1. Equality of two ratios is called a proportion. If the term a, b, c, d are in proportion, then a:b = c:d, or
ad = bc.
2. a, b, c, d are the terms of the proportion,
(a) a and d are the Extremes
(b) b and c are the Means

2 BUSINESS MATHEMATICS, LOGICAL REASONING & STATISTICS (Paper 3) [CA Foundation]


3. Cross Product Rule : Product of Extremes = Product of Means.
a b c d
4. Continuous Proportion : Quantities are said to be in continuous proportion if = = = = ....
b c d e
5. Same Kind : In proportions, all the terms need not be of the same kind. If a and b are of the same
kind and c and d are of the same kind then the proportion is properly expressed.
B. Terminologies :
a b
1. If a, b, c are in continued proportion, then = ,
b c
(a) First Proportional - a is the first proportional.
(b) Second Proportional - b is the second proportional.
(c) Third Proportional - c is the third proportional.
(d) Mean Proportional - Refers to the Geometric Mean of n terms.
a b
Mean Proportional of three terms : If a, b, c are in proportion then, = . Therefore, b2 = ac.
b c

ab  b = ac . Hence b is itself the Mean Proportional between a and c.


(e) Fourth Proportional - If a, b, c, d are in proportion, then d is the fourth proportional.

 Principles of proportion

a c b d
1. Invertendo =  =
b d a c

2 4 3 6
=  =
3 6 2 4

a c a b
2. Alternendo =  =
b d c d

2 4 2 3
=  =
3 6 4 6

a c a+b c+d
3. Componendo =  
b d b d

3 6 3+2 6+4
=  =
2 4 2 4

a c a-b c-d
4. Dividendo =  =
b d b d

3 6 3-2 6-4
=  =
2 4 2 4

RATIO AND PROPORTION, INDICES, LOGARITHMS 3


5. Componendo-dividendo
a c a+b c+d
=  =
b d a-b c-d

3 6 3+2 6 + 4
=  =
2 4 3- 2 6 - 4

a c a+ c
6. Addendo = 
b d b+d

3 6 3+6
= 
2 4 2+ 4

a c a-c
7. Substractendo = 
b d b-d

3 6 3-6 -3 3
=  = =
2 4 2-4 -2 2
Some Important formulas :
2
 a - b = a2 - 2ab + b 2

2
  a +b = a2 + 2ab +b2

3
  a-b = a3 -b3 - 3ab  a -b

3
  a +b = a3 + b3 + 3ab  a +b 

 a2 - b2 =  a - b  a + b 

 a3 + b 3 =  a + b  a2 - ab + b 2
 
 
a3 -b3 =  a -b a2 + ab + b2 
2
  a+ b + c = a2 +b2 + c2 + 2ab + abc + 2ca
 a3 + b3 + c3 = (a + b + c)3- 3 [(a + b + c)(ab + bc + ca) - abc]

4 BUSINESS MATHEMATICS, LOGICAL REASONING & STATISTICS (Paper 3) [CA Foundation]


10. The ratio compounded of 4 : 9, the duplicate
ratio of 3 : 4, the triplicate ratio of 2 : 3 and 9 : 7
is
CLASS WORK
(a) 2:7 (b) 7 : 2
(c) 2 : 21 (d) none of these
1 The inverse ratio of 11 : 15 is
11. The ratio compounded of duplicate ratio of 4 :
(a) 15 : 11 (b) 5, triplicate ratio of 1 : 3, sub duplicate ratio of
11 : 15
81 : 256 and sub-triplicate ratio of 125 : 512 is
(c) 121 : 225 (d) none of these
(a) 4 : 512 (b) 3 : 32
2 The ratio of two quantities is 3 : 4. If the
(c) 1 : 12 (d) none of these
antecedent is 15, the consequent is
12. If a : b = 3 : 4, the value of (2a+3b) : (3a+4b) is
(a) 16 (b) 60
(a) 54 : 25 (b) 8 : 25
(c) 22 (d) 20
(c) 17 : 24 (d) 18 : 25
3. The ratio of the quantities is 5 : 7. If the
consequent of its inverse ratio is 5, the 13. Two numbers are in the ratio 2 : 3. If 4 be
antecedent is subtracted from each, they are in the ratio 3 :
5. The numbers are
(a) 5 (b) 5 (a) (16, 24) (b) (4, 6)
(c) 7 (d) none of these (c) (2, 3) (d) none of these
4. The ratio compounded of 2 : 3, 9 : 4, 5 : 6 and 8 14. The angles of a triangle are in ratio 2 : 7 : 11.
: 10 is The angles are
(a) 1 : 1 (b) 1 : 5 (a) (20O , 70O , 90O) (b) (30O , 70O , 80O)
(c) 3 : 8 (d) none of these (c) (18O , 63O , 99O) (d) none of these
5. The duplicate ratio of 3 : 4 is 15. Division of ` 324 between X and Y is in the
ratio 11 : 7. X & Y would get Rupees
(a) 3:2 (b) 4 : 3
(a) (204, 120) (b) (200, 124)
(c) 9 : 16 (d) none of these (c) (180, 144) (d) none of these
6. The sub-duplicate ratio of 25 : 36 is 16. Anand earns `80 in 7 hours and Promode `90
(a) 6 : 5 (b) 36 : 25 in 12 hours. The ratio of their earnings is
(c) 50 : 72 (d) 5 : 6 (a) 32 : 21 (b) 23 : 12
7. The triplicate ratio of 2 : 3 is (c) 8 : 9 (d) none of these
(a) 8 : 27 (b) 6 : 9 17. The ratio of two numbers is 7 : 10 and their
(c) 3 : 2 (d) none of these difference is 105. The numbers are
8. The sub-triplicate ratio of 8 : 27 is (a) (200, 305) (b) (185, 290)
(a) 27 : 8 (b) 24 : 81 (c) (245, 350) (d) none of these
18. P, Q and R are three cities. The ratio of average
(c) 2:3 (d) none of these
temperature between P and Q is 11 : 12 and
9. The ratio compounded of 4 : 9 and the that between P and R is 9 : 8. The ratio between
duplicate ratio of 3 : 4 is the average temperature of Q and R is
(a) 1:4 (b) 1 : 3 (a) 22 : 27 (b) 27 : 22
(c) 3:1 (d) none of these (c) 32 : 33 (d) none of these

RATIO AND PROPORTION, INDICES, LOGARITHMS 5


19. If x : y = 3 : 4, the value of x2y + xy2: x3+ y3 is 29. The number which has the same ratio to 26
(a) 13 : 12 (b) 12 : 13 that 6 has to 13 is
(c) 21 : 31 (d) none of these (a) 11 (b) 10
20. If p : q is the sub-duplicate ratio of p–x2: q–x2 (c) 21 (d) none of these
then x2 is 30. The fourth proportional to 2a, a2, c is
(a) ac/2 (b) ac
p q
(a) (b) (c) 2/ac (d) none of these
p+q p+q
31. If four numbers 1/2, 1/3, 1/5, 1/x are
pq proportional then x is
(c) (d) none of these (a) 6/5 (b) 5/6
p+q
(c) 15/2 (d) none of these
21. If 2s : 3t is the duplicate ratio of 2s – p : 3t – p
32. The mean proportional between 12x 2 and
then
27y2is
(a) p2 = 6st (b) p = 6st
(a) 18xy (b) 81xy
(c) 2p = 3st (d) none of these
(c) 8xy (d) none of these
22. If p : q = 2 : 3 and x : y = 4 : 5, then the value of (Hint: Let z be the mean proportional and z =
5px+ 3qy : 10px+ 4qy is
(a) 71 : 82 (b) 27 : 28 12x x27y 
2 2

(c) 17 : 28 (d) none of these


33. If A = B/2 = C/5, then A : B : C is
23. The number which when subtracted from each
(a) 3 : 5 : 2 (b) 2 : 5 : 3
of the terms of the ratio 19 : 31 reducing it to 1
(c) 1 : 2 : 5 (d) none of these
: 4 is
34. If a/3 = b/4 = c/7, then a + b + c/c is
(a) 15 (b) 5
(a) 1 (b) 3
(c) 1 (d) none of these
(c) 2 (d) none of these
24. Daily earnings of two persons are in the ratio 35. If p/q = r/s = 2.5/1.5, the value of ps : qr is
4:5 and their daily expenses are in the ratio 7 :
(a) 3/5 (b) 1:1
9. If each saves `50 per day, their daily earnings
(c) 5/3 (d) none of these
in `are
36. If x : y = z : w = 2.5 : 1.5, the value of (x + z)/(y +
(a) (40, 50) (b) (50, 40)
w) is
(c) (400, 500) (d) none of these
(a) 1 (b) 3/5
25. The ratio between the speeds of two trains is
(c) 5/3 (d) none of these
7 : 8. If the second train runs 400 kms. in 5
hours, the speed of the first train is 37. If (5x – 3y)/(5y – 3x) = 3/4, the value of x : y is
(a) 10 Km/hr (b) 50 Km/hr (a) 2 : 9 (b) 7 : 2
(c) 70 Km/hr (d) none of these (c) 7:9 (d) none of these
26. The fourth proportional to 4, 6, 8 is 38. If A : B = 3 : 2 and B : C = 3 : 5, then A : B : C is
(a) 12 (b) 32 (a) 9 : 6 : 10 (b) 6 : 9 : 10
(c) 48 (d) none of these (c) 10 : 9 : 6 (d) none of these
27. The third proportional to 12, 18 is 39. If x/2 = y/3 = z/7, then the value of (2x – 5y + 4z)/
(a) 24 (b) 27 2y is
(c) 36 (d) none of these (a) 6/23 (b) 23/6
28. The mean proportional between 25, 81 is
(c) 3/2 (d) 17/6
(a) 40 (b) 50
(c) 45 (d) none of these

6 BUSINESS MATHEMATICS, LOGICAL REASONING & STATISTICS (Paper 3) [CA Foundation]


40. If x : y = 2 : 3, y : z = 4 : 3 then x : y : z is 50. The mean proportional between 1.4 gms and
(a) 2:3:4 (b) 4 : 3 : 2 5.6 gms is
(c) 3:2:4 (d) none of these (a) 28 gms (b) 2.8 gms
41. Division of `750 into 3 parts in the ratio 4 : 5 : 6 (c) 3.2 gms (d) none of these
is
a b c a+b+ c
(a) (200, 250, 300) (b) (250, 250, 250) 51. If = = then is
4 5 9 c
(c) (350, 250, 150) (d) 8 : 12 : 9
(a) 4 (b) 2
42. The sum of the ages of 3 persons is 150 years.
(c) 7 (d) none of these.
10 years ago their ages were in the ratio 7 : 8 :
9. Their present ages are 52. Two numbers are in the ratio 3 : 4; if 6 be added
to each terms of the ratio, then the new ratio
(a) (45, 50, 55) (b) (40, 60, 50)
will be 4 : 5, then the numbers are
(c) (35, 45, 70) (d) none of these
(a) 14, 20 (b) 17, 19
43. The numbers 14, 16, 35, 42 are not in
(c) 18 and 24 (d) none of these
proportion. The fourth term for which they will
be in proportion is a b
53. If = then
(a) 45 (b) 40 4 5
(c) 32 (d) none of these
a+4 b-5 a+ 4 b+ 5
44. If x/y = z/w, implies y/x = w/z, then the process (a) = (b) = =
is called a-4 b+5 a-4 b-5
(a) Dividendo (b) Componendo a-4 b+5
(c) = (d) none of these
(c) Alternendo (d) none of these a+4 b-5
45. If p/q = r/s = p – r/q – s, the process is called
(a) Subtrahendo (b) Addendo a b
54. If a : b = 4 : 1 th e n + is 
(c) Invertendo (d) none of these b a
46. If a/b = c/d, implies (a + b)/(a – b) = (c + d)/(c – (a) 5/2 (b) 4
d), the process is called (c) 5 (d) none of these
(a) Componendo (b) Dividendo
x y z
(c) Componendo and Dividendo 55. If = =
b + c - a c + a -b a + b - c
(d) none of these
(b – c)x + (c – a)y + (a – b)z is
47. If u/v = w/p, then (u – v)/(u + v) = (w – p)/(w +
p). The process is called (a) 1 (b) 0
(a) Invertendo (b) Alternendo (c) 5 (d) none of these
(c) Addendo (d) none of these 
48. 12, 16, *, 20 are in proportion. Then * is
(a) 25 (b) 14
(c) 15 (d) none of these
49. 4, *, 9, 13½ are in proportion. Then * is
(a) 6 (b) 8
(c) 9 (d) none of these

RATIO AND PROPORTION, INDICES, LOGARITHMS 7


(a) 9, 12, 15 (b) 12, 16, 20
(c) 15, 20, 25 (d) 3, 4, 5
HOME WORK-1 10. The ratio of two terms is 1 : 25. If 36 added to
second term then the ratio becomes 1 : 28.
The first terms is
1. If a : b = 3 : 8 then (7a + 2b) : (7a – 2b) is (a) 12 (b) 10
(a) 5 : 37 (b) 37 : 5 (c) 8 (d) 9
(c) 21 : 16 (d) 16 : 21 11. Three friends divide ` 391 in three
2. If (3a + 2b) : (3a – 2b) = 9 : 5 then a : b is 1 2 3
(a) 7 : 2 (b) 3 : 2 proportional parts : : . How much is the
2 3 4
(c) 7 : 3 (d) 2 : 3 last share ?
3. If a : b : c = 2 : 1 : 3 and 2a – 3b + c = 8 then a + b (a) 135 (b) 153
+ c is (c) 100 (d) none
(a) 12 (b) 6
12. What least number must be added to each one
(c) 18 (d) 24 of 6, 14, 18, 38 to make them in proportion ?
4. If (a2 + b2) : (a2 – b2) = 41 : 9 then (a3 + b3) : (a3 –
(a) 2 (b) 3
b3) is
(c) 4 (d) none
(a) 5 : 4 (b) 125 : 64
(c) 189 : 61 (d) 41 : 9 13. If A’s money is to B’s money as 4 : 5 and B’s
money is to C’s money as 2 : 3 and C has 1500 `
5. If ( 13  x + 13  x ) : ( 13  x – 13  x ) = 5 then A has.
then x is (a) 1000 (b) 800
(a) 4 (b) 5 (c) 1200 (d) 2000
(c) 6 (d) 3 14. A bag contains Rs. 600 in the form of one –
1 a rupee, 50 paise and 25 paise coins in the ratio
6. If ab = 1 then is 3 : 4 : 12. The number of 25 paise coins is
1 b
a 1 1 b (a) 900 (b) 800
(a) (b) (c) 1200 (d) 1000
b 1 a 1
15. A mixture contains milk and water in the ratio
a 1 1 a
(c) (d) 5 : 1. On adding 5 litres of water, the ratio of
1 b 1 b
milk and water becomes 5 : 2. The quantity of
7. A car covers a distance of 150 km in 2 hrs and 40 milk in the original mixture is
minutes, while a motorcycle covers the same
(a) 25 litres (b) 20 litres
distance in 2 hours, what is the ratio of their
speeds ? (c) 30 litres (d) 15 litres
(a) 3 : 4 (b) 4 : 3 16. Out of the ratios 7 : 20, 13 : 25, 17 : 30 and 11 : 15
(c) 4 : 5 (d) none the largest one is ?
8. The marks obtained by a student in 3 subject (a) 11 : 15 (b) 7 : 20
1 1 3 (c) 17 : 30 (d) 13 : 25
a, b and c are in the ratio of : : . If his
2 3 5 17. The incomes of a and b are in the ratio 3 : 2 and
total score is 430 then his marks in c is their expenditures in the ratio 5 : 3. If each
(a) 150 (b) 100 saves ` 1500 then B’s income is
(c) 180 (d) none (a) 3000 (b) 9000
9. How many 50, 25 and 10 paise coins of which (c) 6000 (d) none
the numbers are in the ratio 3 : 4 : 5 are together
worth Rs. 15 ?
8 BUSINESS MATHEMATICS, LOGICAL REASONING & STATISTICS (Paper 3) [CA Foundation]
18. The ratio compounded of duplicate ratio of 4 : (a) (q + p/2p (b) (q - p)2p
5, triplicate ratio of 1 : 3, sub-duplicate rate 2 2
(c) (q - p )/2p (d) none
of 81: 256 and sub-triplicate ratio of 125 : 512 is
27. 94 is divided into two parts in such a way that
(a) 4:512 (b) 3:32 the fifth part of the first and the eighth pan of
(c) 1:12 (d) none the second are in the ratio 3 : 4. The first part is
19. P, Q and R are three cities. The ratio of average (a) 27 (b) 30
temperature between P and Q is 11: 12 and
that between P and R is 9: 8. The ratio between (c) 36 (d) 48
the average temperature of Q and R is: 28. The salaries of A, B, C are in the ratio 2 : 3 : 5. If
(a) 22 : 27 (b) 27: 22 the increments of 15%, 10% and 20% are
(c) 32 : 33 (d) none allowed respectively in their salaries, then
what will be the new ratio of their salaries?
20. The ratio of the prices of two types of cars was
16: 23, Two years later when the price of the (a) 3 : 3 : 10
first has increased by 10% and that of the (b) 10 : 11: 20
second by Rs. 477, the ratio of the prices (c) 23 : 33 : 60
becomes 11 : 20. Find the original prices of the
(d) Cannot be determined.
two types of cars
(a) (848, 1219) (b) (748, 1319) 29. Two numbers are respectively 20% and 50%
(c) (948, 1119) (d) none more than a third number. The ratio of the two
numbers is:
21. If 2s : 3t is the duplicate ratio of 2s - p : 3t - p,
then (a) 2 : 5 (b) 3 : 5
(a) p2 = 6st (b) p = 6st (c) 4 : 5 (d) 6 : 7
(c) 2p = 3st (d) none 30. ` 4850 have been divided among A, B, C such
22. If A : B := 8:15, B : C = 5 : 8 and C : D = 4 : 5, then that if their shares be diminished by ` 15, ` 10
A : D is equal to : and ` 25 respectively, the remainders are in
the ratio 3 : 4 : 5. Then B’s share is :
(a) 2 : 7 (b) 4 : 15
(a) ` 1595 (b) ` 1610
(c) 8 :15 (d) 15 : 4
(c) ` 1626.66 (d) ` 1600
23. If 2A = 3B and 4B = 5C, then A : C is :
31. A sum of money is to be distributed among A.
(a) 4 : 3 (b) 15 : 8
B, C, D in the proportion of 5 : 2 : 4: 3. If C gets `
(c) 8 :15 (d) 3 : 4 1000 more than D, what is B’s share?
a b c abc (a) ` 500 (b) ` 1500
24. If   , then is equal to :
3 4 7 c (c) ` 2000 (d) none
(a) 7 (b) 2 32. ` 1360 have been divided among A. B, C such
that A gets (2/3) of what B gets and B gets (1/4)
1 1 of what C gets. Then B’s share is :
(c) (d)
3 5 (a) ` 120 (b) ` 160
25. The ratio between two numbers is 3: 4. If their (c) ` 240 (d) ` 320.
l.c.m is 180 the numbers are: 33. ` 1980 are divided among A, B, C so that half of
(a) 36, 48 (b) 45, 60 A’s part, one-third of B’s part and one- sixth of
(c) 15, 20 (d) none C’s part are equal. Then, B’s part is :
26. What quantity must be added to the terms of (a) ` 660 (b) ` 360
the ratio p + q : p - q to make it equal to (p + q)2 (c) ` 1080 (d) ` 540.
: (p - q)2 ?

RATIO AND PROPORTION, INDICES, LOGARITHMS 9


34. The fourth proportional to the numbers 60, 48, 42. Given that 24 carat gold is pure gold; 18 carat
30 is : gold is (3/4) gold and 20 carat gold is (5/6) gold,
(a) 36 (b) 24 the ratio of the pure gold in 18 carat gold to
the pure gold in 20 carat gold is :
(c) 48 (d) none
(a) 5 : 8 (b) 9 : 10
35. The ratio of the number of boys and girls in a
college is 7 : 8. If the percentage increase in (c) 15 : 24 (d) 8 : 5
the number of boys and girls be 20% and 10% 43. An amount of ` 2430 is divided among A, B and
respectively, what will be the new ratio? C such that if their shares be reduced by ` 5,
(a) 8:9 (b) 17:18 `10 and.` 15 respectively, the remainder shall
be in the ratio of 3 : 4 : 5. Then, B’s share was
(c) 21: 22
(a) ` 605 (b) ` 790
(d) Cannot be deterthined.
(c) ` 800 (d) ` 810
36. The mean proportional between 234 and 104
is : 44. A and B are two alloys of gold and copper
prepared by mixing metals in the ratio 7 : 2
(a) 12 (b) 39
and 7 : 11 respectively. If equal quantities of
(c) 156 (d) none the alloys are melted to form a third alloy C,
37. Ratio of earnings of A and B is 4 : 7. If the the ratio of gold and copper in C will be :
earnings of A increase by 50% and those of B (a) 5 : 7 (b) 5 : 9
decrease by 25 %, the new ratio of their (c) 7 : 5 (d) 9 : 5
earnings becomes 8 : 7. What is A’s earnings ?
45. A piece of string is 40 centimeters long. It is
(a) ` 21,000 (b) ` 26,000 cut into three pieces+ The longest piece is 3
(d) ` 28,000 (d) Data inadequate times as long as the middle-sized and the
38. The ratio of the number of boys and girls in a shortest pieces is 23 centimeters shorter than
school is 3 : 2. If 20% of the boys and 25% of the the longest piece. The length of the shortest
girls are scholarship holders, what percentge piece (in cm) is :
of the students does not get the schoLarship? (a) 27 cm (b) 5 cm
(a) 56% (b) 70% (c) 4 cm (d) 9 cm
(c) 78% (d) 80% 46. The least whole number which when
39. 60 kg of alloy A is mixed with 100 kg of alloy B. subtracted from both the terms of the ratio 6 :
If alloy A has lead and tin in the ratio 3 : 2 and 7 gives a ratio less than 16 : 21 is
alloy B has tin and copper in the ratio 1: 4, then (a) 3 (b) 2
the amount of tin in the new alloy is : (c) 4 (d) 6
(a) 36 kg (b) 44 kg 47. If one star equals four circles and three circles
(c) 53 kg (d) 80 kg. equal four diamonds, then the ratio of star :
40. The ratio of third proportional to 12 and 30 and diamonds is :
the mean proportional between 9 and 25 is (a) 3/16 (b) 1/3
(a) 2 :1 (b) 5 :1 (c) 3/4 (d) 16/3
(c) 7 :15 (d) 9 :14 48. If two numbers are in the ratio 6 :13 and their
41. The prices of a scooter and a T.V. are in the least common multiple is 312, the sum of the
ratio 7 : 5. If the scooter costs ` 8000 more numbers is
than a T.V. set, then the price of a T.V. set is : (a) 75 (b) 57
(a) ` 20,000 (b) ` 24,000 (c) 76 (d) 67
(c) ` 28,000 (d) ` 32,000.

10 BUSINESS MATHEMATICS, LOGICAL REASONING & STATISTICS (Paper 3) [CA Foundation]


49. The third proportional to (a2 - b2) and (a + b)2 57. If q : p is the sub duplicate ratio of q–x2 : p–x2
is: then x2 is
ab ab p+q pq
(a) (b) (a) (b)
ab ab pq p-q

(a  b)2 (a  b)3 pq
(c) (d) (c) (d) None
ab a b p+q
50. In an express train the passenger travelling in 58. If a : b = c : d = 3 : 5, then what are the values of
A.C. class, First class and Second class are in ad : bc is
the ratio 1 : 2 : 7. and the rate for each class is in (a) 3:5 (b) 5:3
the ratio 5 : 4 : 2. If the total income from this (c) 1:1 (d) None
train is ` 54000. then the income from the A.C. 59. The number ________ has same ratio to 7/33
class is : that 8/21 does to 4/9
(a) ` 10,000 (b) ` 15,000 (a) 2/11 (b) 3/11
(c) ` 7,000 (d) ` 5,000 (c) 2/21 (d) None
51. A fraction bears the same ratio to 1/27 as 3/7
does to 5/9. The fraction is : 60. If 5 = then value of x is
(a) 7/45 (b) 1/35
(c) 45/7 (d) 5/21 (a) 10 (b) 20
52. If (a + b) : (b + c) : (c + a) = 6 : 7 : 8 and a + b + c = (c) 5 (d)
14, then the value of c is : 2
(a) 8 (b) 7 61. The highest score in an innings was of the
9
(c) 6 (d) 12.
2
53. If the ratio of boys to girls in a class is B, the total score and the next highest was of the
ratio of girls to boys is G, then B + G is : 9
(a) equal to 1 (b) greater than 1 remainder. These score differed by 8 runs.
What was the total score in the innings?
(c) less than 1 (d) none
(a) 162 (b) 152
xa xb 2ab (c)142 (d) 1132
54. If the value of : + , when x =
x a x b ab 62. The mean proportional between 9 and 25 is –
; a  b , is : (a) 16 (b) 10
(a) 3 (b) 4 (c) 15 (d) None
(c) 1 (d) 2 63. One-third of a number is greater than one-
fourth of its successor by 1. Find the number.
55. The ratio between the speeds of two trains is
7 : 8. If the second train runs 400 Kms. in 5 (a) 51 (b) 21
hours, the speed of the first train is (c) 15 (d) None
(a) 10 Km/hr. (b) 50 Km/hr. 64. 1230 baskets of mangoes were loaded in three
trucks. When unloaded, it was found that 5, 10
(c) 71 Km/hr. (d) None and 15 baskets were rotten in the trucks
56. The ratio of two quantities is 5:9. If the respectively, but the remaining baskets were
antecedent is 25, the consequent is in the ratio of 3:4:5. How many baskets were
(a) 9 (b) 45 loaded initially in 3rd truck?
(c) 40 (d) None (a) 575 (b) 515
(c) 565 (d) None
RATIO AND PROPORTION, INDICES, LOGARITHMS 11
65. Two vessels contain equal quantity of mixtures 75. An employer reduces the number of
of milk and water in the ratio 5:2 and 6:1 employees in the ratio of 9:8 and increases
respectively. Both the mixtures are now mixed their wages in the ratio of 14:15. In what ratio
thoroughly. Find the ratio of milk to water in is the wages bill decreased?
the new mixture so obtained. (a) 20:22 (b) 20:33
(a) 3:11 (b) 11:3 (c) 21:20 (d) None
(c) 12:13 (d) None 76. A, B and C have to distribute Rs. 1,000 between
66. The vessels contain water and milk in the ratio them, A and C together have Rs. 400 and B and
1:2 and 2:5 are mixed in the ratio 1:4. The C Rs. 700. How much does C have?
resulting mixture will have water and milk in (a) Rs. 100 (b) Rs. 200
the ratio.
(c) Rs. 150 (d) None
(a) 31:74 (b) 31:75
77. The mean annual salary of all employees in a
(c) 30:77 (d) None
company is Rs. 25,000. The mean salary of male
67. The sub duplicate ratio of 16:49 is and female employees is Rs. 27,000 and Rs.
(a) 4:7 (b) 256:2401 17,000 respectively. Find the percentage of
(c) 4:9 (d) None males and females employed by the company.
68. Duplicate ratio of 4:5 is (a) 60% and 40% (b) 75% and 25%
(a) 16:25 (b) 2: (c) 70% and 30% (d) 80% and 20%
5
78. The fourth proportional to (a2– ab + b2), (a3 +
(c) 64:125 (d) None
b3) and (a-b) is equal to _________
69. Triplicate ratio of 3:5 is
(a) a2 + b2 (b) a2 – b2
(a) 27:125 (b) 9:25
(c) 1 (d) None
1 1
(c) 33 : 53 (d) 125:27
70. The sub–triplicate ratio 8:125 is
(a) 2:6 (b) 2:5
(c) 5:2 (d) None
71. The 4th proportion of 6, 8 and 15 is –
(a) 40 (b) 30
(c) 20 (d) None
72. Two numbers are in the Ratio 5:6. If 5 is
subtracted from each number, the ratio
becomes 4:5. Then the number are:
(a) 20, 30 (b) 20, 25
(c) 25, 30 (d) None
73. What must be subtracted from each term of
the ratio 27:43 to make it equal to 7:15?
(a) 13 (b) 15
(c) 17 (d) None
74. There are n ARITHMETIC MEANs between 3
and 31 such that the ratio of 3rd mean to
(n–1)th mean is 1:3, then the value of n is
(a) 12 (b) 15
(c) 13 (d) None

12 BUSINESS MATHEMATICS, LOGICAL REASONING & STATISTICS (Paper 3) [CA Foundation]


ANSWER KEYS

1. (b) 2. (c) 3. (a) 4. (c) 5. (b) 6. (c) 7. (a)


8. (c) 9. (c) 10. (a) 11. (b) 12. (a) 13. (b) 14. (c)
15. (a) 16. (a) 17. (c) 18. (d) 19. (b) 20. (a) 21. (a)
22. (b) 23. (b) 24. (b) 25. (b) 26. (c) 27. (b) 28. (c)
29. (c) 30 (b) 31. (c) 32 (c) 33. (d) 34 (b) 35. (c)
36 (c) 37. (d) 38. (c) 39. (b) 40. (b) 41. (a) 42 (b)
43. (d) 44 (c) 45. (c) 46. (a) 47. (d) 48. (c) 49. (d)
50. (a) 51. (b) 52. (c) 53. (b) 54. (d) 55. (d) 56. (b)
57 (c) 58. (c) 59. (a) 60. (b) 61 (a) 62. (c) 63. (c)
64. (b) 65. (b) 66. (a) 67 (a) 68. (a) 69. (a) 70. (b)
71. (c) 72. (c) 73. (a) 74. (c) 75. (c) 76. (a) 77. (d)
78. (b)

RATIO AND PROPORTION, INDICES, LOGARITHMS 13


9. The ages of A and B are in the ratio 3:1. Fifteen
years hence, the ratio will be 2:1. Their present
ages are
HOME WORK-2 (a) 30 years, 10 years
(b) 45 years, 15 years
(c) 21 years, 7 years
1. Ratio exists only between quantities of ____
kind. (d) 60 years, 20 years
(a) Same (b) bigger 10. The first, second and third month salaries of a
person are in the ratio 2:4:5. The difference
(c) Smaller (d) None
between the product of the salaries of first 2
2. 42.5 : 23 is same as months & last 2 months is 4,80,00,000. Find
(a) 4 : 1 (b) 2:1 the salary of the second month
(c) 16 : 1 (d) 80 : 1 (a) ` 4,000 (b) `6,000
3. Ratio between 25 minutes and 45 seconds (c) `12,000 (d) `8,000
(a) 100 : 3 (b) 5:9 11. If 50 is the third proportional to 8 and X then
(c) 4 : 5 (d) 3 : 10 the value of X is
4. The ratio compounded of the Duplicate Ratio (a) 20 (b) 2
of (c) 10 (d) 1
5 : 6 and the Triplicate Ratio of 3 : 5 is
12. If b be the mean proportional between a & c
(a) 4 : 75 (b) 2 : 15
then
(c) 9 : 50 (d) 3 : 10
(a) b x b = ac (b) b = (a + c) / 2
5. The ratio of numbers is 1:2:3 and sum of their
(c) b = a + c (d) b = (a - c) / 2
squares is 504, then the numbers are -
(a) 6, 12, 18 (b) 3, 6, 9 a b
13. If a:b = 4:1, then  is
(c) 4, 8, 12 (d) 5, 10, 15 b a
6. A person on a tour has `9600 for his expense. (a) 1 (b) 5/4
But the tour was extended for another 16 days, (c) 4/5 (d) None
so he has to cut down his daily expenses by
`20. The original duration of the tour had x 3 2x + y
14. If = , then the value of is
been? 2y 2 x -2y
(a) 48 days (b) 64 days (a) 5 (b) 7
(c) 80 days (d) 96 days (c) 2 (d) 7.1
7. A bag contans 23 number of coins in the form
of 1 rupee, 2 rupee and 5 rupee conis. The total x + 5 + x -16 7
15. If  then x equals
sum of the coins is `43. The ratio between 1 x + 5  x -16 3
rupee and 2 rupees coins is 3:2. Then the (a) 10 (b) 20
number of 1 rupee coins.
(c) 30 (d) 40
(a) 12 (b) 8
(c) 10 (d) 16 a3 + 3a 91
16. If 2  then ‘a’ equals
8. A recipe for 4 servings requires salt and pepper 3a +1 37
to be added in the ratio of 2:3. If the recipe is (a) 8 (b) 7
adjusted from 4 to 8 servings, what is the ratio (c) 6 (d) None
of the salt and pepper that must now be
added?
(a) 4 : 3 (b) 2:6
(c) 2 : 3 (d) 3:2
14 BUSINESS MATHEMATICS, LOGICAL REASONING & STATISTICS (Paper 3) [CA Foundation]
17. A Dealer mixes Tea costing `6.92 per kg with 24. What least number must be added to each one
Tea costing `7.77 per kg and sells the mixture 6, 14, 18, 38 to make them in proportion ?
at `8.80 per kg and earns a profit 17.5% on his (a) 1 (b) 2
Sale Price. In what proportion does he mix (c) 3 (d) 4
them ?
25. For p, q, r, s > 0 the value of each ratio is
(a) 1 : 2 (b) 3:2
(c) 3 : 4 (d) 5:3 P q r s
= = =
18. 20 litres of a mixture contains milk and water q +r r + s s + p p + q
in the ratio 5:3. If 4 litres of this mixture be (a) 1/2 (b) 1/3
replaced by 4 litres of milk, the ratio of milk to (c) 1/4 (d) 1
water in the new mixture would be 26. Let x, y and z are three positive numbers and P
(a) 2 : 1 (b) 7:3
x+y+z
(c) 8 : 3 (d) 4:3 = if (p -x): (p-y): (p-z) = 3:5:7 then the
2
19. Three containers have their volumes in the ratio of x:y:z is
ratio 3:4:5. They are full of mixtures of milk
(a) 4:5:6 (b) 6:5:4
and water. The mixtures contain milk and
water in the ratio of (4:1), (3:1) and (5:2) (c) 3:5:7 (d) 7:5:3
respectively. The contents of all these three 27. A dealer mixes tea costing `6.92 per kg. with
containers are poured into a fourth container. tea costing `7.77 per kg and sells the mixture
The ratio of milk and water in the fourth 1
container is at `8.80 per kg and earns a profit of 17 % on
2
(a) 4 : 1 (b) 15 : 48 his sale price. In what proportion does he mix
(c) 157 : 53 (d) 5:2 them?
20. What must be added to each of the numbers (a) 3 : 2 (b) 2:3
10, 18, 22, 38 to make them proportional: (c) 1 : 3 (d) None
(a) 5 (b) 2
(c) 3 (d) 9
21. If one type of rice of cost `13.84 is mixed with ANSWER KEYS
another type of rice of cost `15.54, the mixture
is sold at `17.6 with a profit of 14.6% on selling
price then in which proportion the two types 1 A 10 D 20 B
of rice mixed ?
2 A 11 A 21 A
(a) 3 : 7 (b) 5:7
3 A 12 A 22 C
(c) 7 : 9 (d) None
4 C 13 D 23 B
22. If 4, x and 9 are in proportional then x is equal 5 A 14 B 24 B
to
6 B 15 B 25 A
(a) 36 (b) 6.5 7 A 16 B 26 B
(c) 6 (d) 24 8 C 17 B 27 A
23. The ratio of the third proportional to 4 and 6 9 B 18 B
and the mean proportional between 9 and 25 19 C
is -
(a) 5 : 3 (b) 3:5
(c) 8 : 5 (d) 5:8
---0---0---

RATIO AND PROPORTION, INDICES, LOGARITHMS 15


UNIT II : INDICES

INTRODUCTION
We know that when two or more numbers are multiplied together the result is called the continued product.
Each number separately is called a factor of the product. But when a product consists of the same factor
repeated several times, it is called a power of the factor.
For example :
(i) In the product a x b x c; a, b, c are called the factors of the product.
(ii) The product a x a x a x a = a5, a5 is called the fifth power of a.

EXPONENT OR INDEX OF THE POWER


If a number b is multiplied by itself n times, n being a positive integer, i.e., then the continued product b x b
x b x b ... n times ...(i) is called the nth power of b and is written as bn, b is known as base and n is called the
Exponent or Index power of b.
Similarly, 5 x 5 x 5 x 5 = 54 .. (ii)
5
3x3x3x3x3=3 ...(iii)
2 x 2 x 2 x 2 x 2 x 2 = 26 ...(iv)
In (i) b is multiplied n times; in (ii) 5 is mutliplied 4 times; in (iii) 3 is multiplied 5 times; in (iv) 2 is multiplied
6 times. In all such cases a factor which multiplies is called the base and the number of times it is multiplied
is called the power or index. For Example in (i) the base is b and power is n ; in (ii) the base is 5 and the power
is 4; in (iii) the base is 3 and power is 5 : in (iv) the base is 2 and power is 6.
LAWS OF INDICES
The seven laws of indices are :
(i) am x a n = a m + n (ii) am  an = a m-n
(iii) (am)n = amn (iv) (a . b)m = am . bm

m
a am 1
(v)    ,b  0 (vi) a-n = , a 0
b bm an

p
(viii) ap/q =  a
q q
 ap . (viii) If am = an then m = n

(ix) If am = bm then a = b.

16 BUSINESS MATHEMATICS, LOGICAL REASONING & STATISTICS (Paper 3) [CA Foundation]


MEANING OF ZERO INDEX
Since the index law is assumed to be true for all rational values of m and n, i;e.,
am x an = am+n Also a0 x an = a0+n = an.

an
Dividing both sides by an, if a  0, we get : a0 = = 1.
an
 a0 = 1.
Hence any non-zero number raised to the power zero is equal to unity.
MEANING OF FRACTIONAL INDEX
p
ap/q = q
ap =  a  ; for all p, q  N, q  0.
q

MEANING OF NEGATIVE INDEX

1
a -r = , a  0 , r is any positive integer or fraction.
ar

RATIO AND PROPORTION, INDICES, LOGARITHMS 17


10. The value of ya–b × yb–c × yc–a × y–a–b is
(a) ya+b (b) y
CLASS WORK (c) 1 (d) 1/ya+b
11. The True option is
1. 4x–1/4 is expressed as
(a) x2/3 = 3
x2 (b) x2/3 = x3
(a) –4x1/4 (b) x–1
(c) 4/x1/4 (d) none of these (c) x2/3 > 3
x2 (d) x2/3 < 3
x2
2. The value of 81/3 is
12. The simplified value of 16x–3y2 × 8–1x3y–2 is
(a) 32 (b) 4 (a) 2xy (b) xy/2
(c) 2 (d) none of these (c) 2 (d) none of these
3. 1/5
The value of 4/(32) is 13. The value of (8/27) × (32/243)–1/5 is
–1/3

(a) 8 (b) 2 (a) 9/4 (b) 4/9


(c) 4 (d) none of these (c) 2/3 (d) none of these
1/3
4. The value of (8/27) is 6
 (x + y)2/3 (x - y)3/2 
(a) 2/3 (b) 3/2 
14. The value of 3  is
(c) 2/9 (d) none of these  x + y (x - y) 
–1/8
5. The value of 2(256) is (a) (x + y)2 (b) (x – y)
(a) 1 (b) 2 (c) x + y (d) none of these
(c) 1/2 (d) none of these 15. [{(2) . (4) . (8) . (16)7/8 . (32)9/10}4]3/25 is
1/2 3/4 5/6

6. 2½ . 4¾ is equal to (a) A fraction (b) an integer


(a) a fraction (c) 1 (d) none of these
(b) a positive integer
(c) a negative integer 1
1 n+1
(d) none of these  n n- n 
 81x 4 
1
4
16.  x   is equal to
7.  -8  has simplified value equal to  
 y 
(a) xn (b) xn+1
(a) xy 2 (b) x 2 y
(c) x n–1 (d) none of these
(c) 9xy2 (d) none of these
3 3 3
17. Using (a–b) = a –b –3ab(a–b) tick the correct
0
 2p2q3  of these when x = p1/3 – p–1/3
8. The value of   where p, q, x, y  0 is
 3xy  (a) x3+3x = p + 1/p (b) x3 + 3x = p – 1/p
equal to (c) x3 + 3x = p + 1 (d) none of these
(a) 0 (b) 2/3 18. On simplification, 1/(1+a m–n+am–p) + 1/(1+an–
m
(c) 1 (d) none of these +an–p) + 1/(1+ap–m+ap–n) is equal to

9. {(3 ) × (4 ) × (5 ) } / {(32)3 × (43)2 × (52)3} is


3 2 2 3 3 2 (a) 0 (b) a
(a) 3/4 (b) 4/5 (c) 1 (d) 1/a
(c) 4/7 (d) 1

18 BUSINESS MATHEMATICS, LOGICAL REASONING & STATISTICS (Paper 3) [CA Foundation]


7
- 5 2 1
26. If ax 3 + bx 3 + c = 0 then the value of
 6-172  2  6-273  2
19. The value of  2 -4  × 3 -5  is 3 2 3 3
6 7  6 7  a x + b x + c is given by
(a) 0 (b) 252 (a) 3abcx (b) –3abcx
(c) 250 (d) 248 (c) 3abc (d) –3abc

2-9 2 5 27. If ap = b q = Cr and b2 = ac the value of


x 7
x 5 x 7 z 6
20. The value of 1 × 2 × 2 -3 is q (p+ r)/pr is given by
z 2 z 3 z 3 x 5 (a) 1 (b) –1
(a) 1 (b) –1 (c) 2 (d) None
(c) 0 (d) None
c 1 1 2
28. If 2a = 3b =  12  then the - - reduces to
a2 b2 c2 c b a
21. Show that
 a+b  x ×
b+c  x
×  c+a
x
(a) 1 (b) 0
b2 c2 a2
x x x
(c) 2 (d) None
reduces to
c
(a) 1 (b) 0 29. If 3a = 5b =  75  ,then the value of ab-c
(c) –1 (d) None (2a+b) reduces to
(a) 1 (b) 0
 xb   x c   x a 
(c) 3 (d) 5
22. Show that  xc  ×  xa  ×  b  reduces to
    x  c
30. If 2a = 3b =  12  , then the value of ab-c (a+2b)
(a) 1 (b) 3 reduces to
(c) 0 (d) 2 (a) 0 (b) 1
23. Show (c) 2 (d) 3
1 1 1 31. If 2a = 4b = 8c andabc=288 then the value
 xb  bc  xc  ca  xa  ab
that  c  × a  ×  reduces to
b 1 1 1
x  x  x  + + is given by
2a 4b 8c
(a) –1 (b) 0
(c) 1 (d) None 1 1
(a) (b) 
8 8
1 1 1
24. + c -a + would
b -c a -b 11 11
x + x +1 x + x +1 x + x +1 (c) (d) 
reduce to one if a+b+c is given by 96 96
(a) 1 (b) 0 32. If ap = bq = cr = ds and ab = cd then the value
(c) –1 (d) None
1 1 1 1
1 -1 of + - - reduces to
25. If x = 4 3 + 4 3 prove that 4x -12x is given
3 p q r s
by 1
(a) a (b) 1 b
(a) 12 (b) 13
(c) 15 (d) 17 (c) 1 (d) 0

RATIO AND PROPORTION, INDICES, LOGARITHMS 19


y x
33.  
If m = bx ,n = b y and m n = b2 the value of
40. If a =
3+ 2
,b =
3 2
then the value of
xy is given by 3- 2 3 2
(a) –1 (b) 0 1 1
+ is
(c) 1 (d) None a2 b2
1 -1 (a) 10 (b) 100
34. If a = 3 + 2 2 then the value of a 2 + a 2 is
(c) 98 (d) 99
(a) 2 (b)  2
41. The square root of 11 - 120 is given by
(c) 2 2 (d) -2 2
(a) 6+ 5 (b) 6 5
1 -1
35. If a = 3 + 2 2 then the value of a 2 - a 2 is (c) (d) 2 3 + 3 2
2 3 -3 2

(a) 2 2 (b) 2  243 


4 / 5

42. The value of   is


 32 
(c) 2 2 (d) -2 2
(a) 4 9 (b) 16 81
36. The square root of 3 + 5 is
(c) 2 3 (d) 3 2
(a) 5 +
2
1
2 (b) -  5 +
2
1
2  x

(c) Both the above (d) None


43.  
If 4
x
= 256 then x is

(a) 4 (b) 2
(c) 8 (d) 2
37. If x = 2 - 2 - 2 ....... the value of x is given
2n  2n  1
by 44. The value of n  1 is
2  2n
(a) -2 (b) 1
(a) 2 (b) 8
(c) 2 (d) 0
(c) 3 2 (d) 2 3
3+ 2 3 2
38. If a = ,b = then the value of x m  3n . x 4m  9n
3- 2 3 2 45. Simplification of is
x 6m  6n
a + b is
(a) xm (b) x–m
(a) 10 (b) 100
(c) xn (d) x–n
(c) 98 (d) 99
46. If 2x – 2x – 1 = 4 then the value of xx is
3+ 2 3 2 (a) 2 (b) 1
39. If a = ,b = then the value of (c) 64 (d) 27
3- 2 3 2
2 2
a + b is 47. If 4x = 5y = 20z then z equal to
(a) 10 (b) 100 xy
(a) xy (b)
(c) 98 (d) 99 xy

1 xy
(c) xy (d) x  y

20 BUSINESS MATHEMATICS, LOGICAL REASONING & STATISTICS (Paper 3) [CA Foundation]


48. If a . 3b = x1/ 6 then x = 6n  2  30  6n  1
59. Value of is
(a) a2b3 (b) a1/2 b 1/3 6n  10
(c) a3b2 (d) ab
31
a (b - c ) c (a -b ) (a) 3 (b)
X .X 10
49. Simplification of b (a -c )
is
X 36
(c) (d) none
(a) x (b) 1 10
(c) xa + b + c (d) 0 1 1 1
50. x x
If 9 – 10.3 + 9 = 0 then x is 60. If 2x = 3y = 6–z then + + is
x y z
(a) 0 or 2 (b) 0 or 3 (a) 0 (b) 1
(c) – 1 or 3 (d) 2 or 3 (c) 36 (d) -1
4 4
51. If 2 x  3 x =8  27 then x is
(a) 81 (b) 8 a 1 a6

52.
(c) 16
n+1 n–3
(d) 27
If 2 + 3.2 = 76 then n is
61. If    
x
y
=
y
x
then a is

(a) 3 (b) 2 (a) 7 (b) 7 2


(c) 5 (d) 4
53. –1 –1
Find x if x = 3 – 4 .–1 (c) 5 2 (d) 5

(a) 112 (b) 1 4 1


5
1
62. If 3x. 27 =3 5 then x is
(c) 12 (d) none
(a) 2 5 (b) – 1 5

54. Solve 22x×3y = 48 and 3x×5y = 45 (c) 1 5 (d) – 2 5


(a) x = 2, y = 0 (b) x = 2, y = 1
(c) x = 0, y = 1 (d) x = 0, y = 2 4n  25  (8)3
63. If = 8 then n is
X 2  (16 )4
55. If x x =  X then x is
(a) 2 (b) 1
(a) 1 (b) 4
(c) 0 (d) 3
(c) 1 or 4 (d) any value
64. If ( 2 )5× (4)2 = 2x ×2 2 then x is
56. 3
If 5  x = 3 then x is
(a) 5 (b) 2
(a) 64 (b) 8 (c) 3 (d) 4
(c) 4 (d) 1 65. If ax–3 × ay + 2 = a2 × ax and ay = a4 – x then
x+3 3x – 4
57. If 6 = (36) then x is (a) x = 3, y = 1 (b) x = 1, y = 3
(a) 5 (b) 11 5 (c) x = 1, y = 1 (d) x = 3, y = 3
66. If x = ya, y = zb and z = xc then abc is
(c) 511 (d) 4 3
(a) 2 (b) 1
58. If 42x – 22x = 12 then x is
(c) 3 (d) 4
(a) 4 (b) 1
(c) 2 (d) 0

RATIO AND PROPORTION, INDICES, LOGARITHMS 21


67. Which is smaller 4 6 or 3 4 ? HOME WORK-1
(a) 4 6 (b) 3 4 1. Simplification of 9x+3 = 27x-1 gives :
(c) Can’t Say (a) 8 (b) 7
x2 (c) 9 (d) None
2. [(18)  (27) ] x 6 = 2x then the value of x is
3.5 3.5 3.5
68. If 4 5  4 = 256 then x is
(a) 3.5 (b) 4.5
(a) 5 (b) – 5 (c) 6 (d) 7
(c) (a) or (b) (d) any value
x 1 x 3
a b
   
3. If,  b  a then the value of x is
---0---0---0----
(a) 1/2 (b) 1
(c) 2 (d) 7/2

4. The value of

1 1 1
2
 3
 1
   is;
 216  3  256  4  32  5

(a) 102 (b) 105


(c) 107 (d) 109

5. If (25)7.5  (5)2.5  (125)1.5 = 5x, then x =


(a) 8.5 (b) 13
(c) 16 (d) None
3
9 n x 3 5 x  27 
6. If 4 =27, then the value of n
3 x  81
is :
(a) 0 (b) 2
(c) 3 (d) 4.
1 4
 
7. 64 2
  32 5  x, then the value of x

is

1 3
(a) (b)
8 8
1 3
(c) (d)
16 16

22 BUSINESS MATHEMATICS, LOGICAL REASONING & STATISTICS (Paper 3) [CA Foundation]


8. The value of is 13. If m and n are whole numbers such that mn =
121, then the value of (m - 1)n+1 is : (m, n >1)
0.13
0.07
 243 x  243 
0.25 0.075 0.2 (a) 1 (b) 10
7 x  49  x  343  (c) 121 (d) 1000.
0 1
 0.6    0.1
1 3 1
3 7  3  3  1
(a) (b) 14. The quantity is
7 3  3      
2  4  3
3 2 equal to
(c) 1 (d) 2
7 7
n 4 24 1
2  2  2n 3 (a) (b) 
9. The quantity n  3 
 2 is equal 5 2
2 x 2
2 3
to; (c) (d)
3 2
9 n
(a) 2n+1 (b)  2 
8 
ab bc ca
 n1 1  xa   xb   xc 
(c)  2   (d) 1 15. If  b  c  a = x. then
 8
x  x  x 
the value of x is :

10. If ax-2 (a2x+2 + a1 - x) = a -3 (a9 + a2), then the value


(a) 0 (b) xabc
of x is;
(c) xa+b+c (d) 1
(a) 0 (b) a fraction
(c) a positive integer
16. The number of prime factors in
(d) a negative integer.
3 2 1

 243 
n
5 x3
2n 1  216 5 x  2500  x  300
5 5 is:

11. If n
= x, then the value of x
9 x 3 n 1
(a) 6 (b) 7
is: (c) 8 (d) 9
(a) 1 (b) 3
1 1 1
(c) 9 (d) 3n 17. If = yq =
x p
z and xyz = 1 then p + q + r is
r

12. Given that 100.48 = x, 100.70 = y and xz = y2. then


the value of z is close to: (a) 1 (b) 0
(a) 1.45 (b) 1.88 (c) 1/2 (d) none
(c) 2.9 (d) 3.7

RATIO AND PROPORTION, INDICES, LOGARITHMS 23


18. The number of prime factors in
1
28 16 (a) 0 (b)
612 x  35  x 15  2
12 11 is
14  x  21 (c) 1 (d) am n
(a) 56 (b) 66
a b
(c) 112 (d) None  a ab b
b a

19. x+3 3x-4
If 5 = (25) then the value of x is:  x x
 b 
24.  a  , then the value of z
a b b a
(a)
5
(b)
11  x x 
11 5
is:
11 13
(c) (d) .
3 5 (a) 0 (b) xabc
(c) xa+b+c (d) 1.
bca cab abc 
 xb   xc   xa 
20. If  c   a  b = x,
x  x  x  1

1
 ba   c a   a b 
 x
c b
then the value of x is 1 x x 1 x
25. 1
abc
 b  c 
(a) x (b) 1 1 x  x  a c 
ab+bc+ca
(c) x (d) xa+b+c
= z then the value of z is;

(a) 0 (b) 1
21. The value ofx for which 2 x + 4 - 2 x + 2 = 3 is:
(c) xa-b-c (d) None
2 -1 -1 -1/2
26. [1 - {1 - (1 - x ) } ] simplified the term
(a) 0 (b) -2
(a) 0 (b) x
(c) 2 (d) -1. -1
(c) x (d) 1
1 1 1
 1
 ac  1
 ba  1
 c b  1 1 1  24
a b bc c a 27. If 2x = 4y = 8Z and     7 ,
22. x  xx  xx  ,  2x 4y 6z 
     
then the value of z is:
find the value of

(a) 1 (b) 0 7 7
(a) (b)
(c) -1 (d) none 16 32

1 1 (c)
7
(d)
7
23. if
 = x, then the 48 64
1  a  1  a 
nm mn

value of x is:

24 BUSINESS MATHEMATICS, LOGICAL REASONING & STATISTICS (Paper 3) [CA Foundation]


28. If 5 5 x 53 x 52  5a2 then the value of xa b
2 1 1
c
xa
1 2 1
b c
xa
1 1 2
b c
simplifies
a is
to
(a) x (b) x2
(a) 4.5 (b) 5
(c) x3 (d) x4.
(c) 6 (d) 8.
36. Assuming that x, y, z are positive real numbers
29. If a . a = a . a and a . ay = a4 then
x-3 y+2 2 x x
and the exponents are all rational numbers,

(a) x = y = 0 (b) x = y = 1
then
(c) x > y = 0 (d) x < y
a 2  ab  b 2 b 2  bc  c 2
 xa   xb 
30. If ax = by = cz and b2 = ac, then y equals:  b  c
x  x 
c 2  ac  a2
xy zx  xc 
(a) (b) 2 x  z  a
xz   x 

zx 2xz equals:
(c) 2 z  x (d)
   x  z
(a) -1 (b) 1
x (c) 0 (d) 2
31. The value of x for which if xx x

= x x 
find x 37. The value of x 1 y . y 1z . z 1x is:

(a) 4/9 (b) 9/4 (a) 1 (b) -1


(c) - 9/4 (d) none (c) 0 (d) 2.

32. If 2n-1 +2n+1 = 320, then n is equal to; 38. Show that
(a) 6 (b) 8 x x 1
(c) 5 (d) 7. 16  32   23x 2 . 4x 1 5 5 
x 1 x

15  2  16  52 x
33. Simplified value of
is given by:
2/3
125  3
x 25 x 5 x 5 3 1/ 2

(a) 1 (b) -1
(a) 3 (b) 5
(c) 4 (d) 0.
(c) 1 (d) none

2 1 1 2 1 1 2 1 1
34. If m = xa, n = xb, p = xc then m b-c n c-a p a-b equals;
(a) 0 (b) 1
39. xa b c
xb c a
xc a b
- x 3 would

(c) -1 (d) 2. reduce to zero if a + b + c is given by

35. If a + b + c = 0 Then the expression (a) 1 (b) -1


(c) 0 (d) None.

RATIO AND PROPORTION, INDICES, LOGARITHMS 25


40. The expression 45. If a = xym-1, b = xyn-1 c = xyp-1 then the value of a
n-p
x b p-m x c m-n is :
a2  ab  b2 b2  bc  c 2
 xa   xb  (a) 1 (b) -1
 b  x  c  (c) 0 (d) None.
x  x  46. If a = 31/4 + 3-1/4 and b = 31/4 - 3-1/4 then the value
c 2  ac  a2
 x  c of (a2 + b2) 2 is :
x  a  (a) 67 (b) 65
x 
(c) 64/3 (d) None
reduces to :
47. If a = 3 2 1  3
2  1 , then the value
(a) 1 (b) 
 2 a 2  b2  c 2 
x of a3 + 3a - 2 is
(a) 3 (b) 0
(c) 
2 a 3  b3  c 3  (d) 
2 a3  b3  c3 
x x (c) 2 . (d) 1
41. On simplification 48. On simplification
b + c

1 1 ab a+b  b 2 + c2  ca c+ a
 x   x
  x 
  2  x x
 
a b a c b c b a a + b2  bc
 c 2 + a2
1 z z 1 z z  x  x  x 
 
1 reduces to:
 reduces to
c a c b
1 z z
3 3
(a) x 2a (b) x 2a
1 1
(a) 2 abc 

 (b) 
 abc 
2 a3  b3  c3 
   
z z
(c)  
(c) 1 (d) 0. x
42. if x = 5 + 5 prove that 5x3 - 15x is given by.
1/3 -1/3
2 a3  b3  c3 
 
(a) 25 (b) 26 (d)  
x
(c) 27 (d) 30.
49. If ax2/3 + bx1/3 + c = 0, then the value of a3 x 2 +
a/b b3x + c3 is given by:
a
43. If ab = ba + then the value of    aa/b 1
b (a) 3abcx (b) -3abcx
(c) 3abc (d) -3abc.
(a) - 1 (b) 0 50. If abc = 1 then find the value of
(c) 1 (d) None. 1 1 1
1 + 1 +
xy y z 1 a  b 1 b  c 1  b  a 1
 mx   my 
44. On simplification  y  x z
m 
m    (a) 0 (b) 1
 3(mxmz)x-z reduces to (c) -1 (d) None.

9 y.32 (3 y )1  27 y 1


(a) 3 (b) -3 51. If 3x 3
 then x-y is given
3 .2 27
1 1 by;
(c)  (d)
3 3

26 BUSINESS MATHEMATICS, LOGICAL REASONING & STATISTICS (Paper 3) [CA Foundation]


(a) 0 (b) -1
a a-1
(c) 3 (d) 1. (a) (b)
a-1 2
52. On simplification
2 2
x+3 2x-y x+y+3 y+1 (c) (d)
2 3 5 6 a-1 1-a
x+1 y+3 x reduces to:
6  10  15
59. If 1176 = 2p.3q.7r Find the value of p, q, r.
(a) 1, 2, 3 (b) 1, 3, 2
(a) -1 (b) 0
(c) 3, 1, 2 (d) 1, 3, 5
(c) 1 (d) 10. x y z
-1/2 –1
60. If 3 = 2, 5 =3 and 2 =5 find the value of multiply
53. Value of 2x .3x if x = 4 is
of x.y.z
(a) 3/4 (b) 4
(a) 0 (b) 1
(c) 3.5 (d) 4.5
(c) 2 (d) None
-1/ 7
61. The value of 2×(128) is
54. Find the value of p from =
(a) 1 (b) 2
2p (c) 14 (d) None
(a) 16 (b) 8
216 x 310 x 54
(c) –8 (d) 4 62. The value of is equal to
212 x 36 x 53
55. If P+ = 6/25 then value of P is equal to
o
(a) 2160 (b) 6480
(a) 1/5 (b) 2/5 (c) 648 (d) 3240
(c) 1/25 (d) 2/25 -1 4 1/ 4
63. The value of 5 (5 ) is ______
0 1 4
 1   (a) 1 (b) 5
  (64)  (32)
2 5
56. The value of  (c) 0 (d) None
 64 
64. The value of 3(256)–1/8 is _______
1 3
(a) 17 (b) 17 2 3
8 8 (a) (b)
3 2
7
(c) 11 (d) None (c) 3 (d) None
8 65. Which is True ?
1 1 (a) 20 >(1/2)0 (b) 20 <(1/2)0
57. If a2 + b2 = 45 & ab = 18, then + is
a b (c) 20 =(1/2)0 (d) None
1 2
(a) (b)
3 3

1
(c) (d) None
2

58. The value of

RATIO AND PROPORTION, INDICES, LOGARITHMS 27


ANSWER KEYS

1. (c) 2. (d) 3. (c) 4. (a) 5. (b) 6. (c) 7. (c)


8. (a) 9. (d) 10. (c) 11. (c) 12. (c) 13. (d) 14. (a)
15. (d) 16. (b) 17. (b) 18. (b) 19. (b) 20. (b) 21. (b)
22. (a) 23. (c) 24. (d) 25. (b) 26. (b) 27. (c) 28. (a)
29. (d) 30 (d) 31. (b) 32 (d) 33. (d) 34 (b) 35. (c)
36 (b) 37. (a) 38. (a) 39. (c) 40. (c) 41. (c) 42 (b)
43. (b) 44 (d) 45. (a) 46. (c) 47. (b) 48. (a) 49. (a)
50. (b) 51. (d) 52. (b) 53. (a) 54. (c) 55. (c) 56. (a)
57 (c) 58. (d) 59. (c) 60. (b) 61 (a) 62. (b) 63. (a)
64. (b) 65. (c)

28 BUSINESS MATHEMATICS, LOGICAL REASONING & STATISTICS (Paper 3) [CA Foundation]


3 12
8. The value of x × 3 x 6 is
(a) x7 (b) x6
HOME WORK-2 (c) 1 (d) None of these
9. If 49 × 49 × 49 × 49 = 7n ,then n equals

1. If the index of any power function is zero, then (a) 4 (b) 7


the value of that function is (c) 8 (d) 16
(a) 0 (b) 1
10. If 16 × 8 n+2= 2m ,then m is equal to
(c) -1 (d) 
(a) n +8 (b) 2n+10
 150 146  (c) 3n +2 (d) 3n +10
2. The value of  10  ÷  10 
 is

1
(a) 1,000 (b) 10,000  m 1-m1  m-1
(c) 1,00,000 (d) 106 11. Simplification of   x   gives
 
1
3. The value of 5 4 × 125 0.25 is (a) x (b) 1
(c) 0 (d) None of these
(a) 5 (b) 5 3 3 3ab a+b
xa .xb .x
25

35
12. Solve for x if = 25  
(c) (d) 25
and a + b = 5.
4. 3 3x-4 2
x +5 (a) 2 (b) 3
 3 52 8 Find the value of x.
(c) 1 (d) 0
(a) 1 (b) 3
13. If abc = 2 then the value of
(c) 1/3 (d) 0
1 1 1
5. If 3 x-y  = 27 and 3 x+y  = 243 ,then x is equal -1
+ + =
1 + a + 2b b -1 1 + a-1 + c
1+ +c
to - 2
(a) 0 (b) 2 (a) 1 (b) 2
(c) 4 (d) 6
1 3
6. Find the value of (27+2a)/(33a+11)for a = -4. (c) (d)
2 4
(a) 2/3 (b) 3/2
14. If a,b,c are real number, then the value of
(c) 1 (d) -2/3
-1 -1 -1
1 1 a b , b c , c a is
7. If x
2 × 85 = 2 5 ,then the value of x is (a) 1 (b) abc

1 1 1
(a) 5 (b) 5 (c) (d) None
abc
15. If xyp-1= a,zyq-1=b,and xyr-1= c the aq-r br-p cp-q is
2 2
(c) (d)  equal to
5 5 (a) 1 (b) 0
(c) p +q +r -1 (d) None of these

RATIO AND PROPORTION, INDICES, LOGARITHMS 29


b
16. Solve for ‘b’ if b b =  b
(a) 1
(b) 3
(c) 1 or 3
(d) Any positive integers
n n
17. If xm = xm and n=3, then the value of ‘m’
 
is
(a) 3 (b) 3

(c) 3 (d) Any value

ANSWER KEY

1. (b) 2. (b) 3. (b)

4. (b) 5. (c) 6. (b)

7. (d) 8. (b) 9. (c)

10. (d) 11. (a) 12. (a)

13. (a) 14. (a) 15. (a)

16. (c) 17. (c)

30 BUSINESS MATHEMATICS, LOGICAL REASONING & STATISTICS (Paper 3) [CA Foundation]


UNIT 3 : LOGARITHMS

INTRODUCTION
1. Solve the following equations :

(i) 2x = 16 (ii) 3x = 243 (iii) 5x = 125 (iv) 5x = 127


(i) 2x = 16

 2x = 24  x=4

(ii) 3x = 243

 3x = 35  x=5

(iii) 5x = 125

 5x = 125  x=3

In the first three equations the values of x can be easily obtained, but in the fourth equation 5x = 127, it
is not convenient to express 127 as 5 raised to some number. The equation therefore cannot be solved.
For solving such equations a technique of logarithms was given by John Napier.
i.e. if 5x = 127, then x = is given by

x = log5127

Definition of Logarithm
If ax = n (a > 0 and a  1, n > 0) then x = logan

Here x is said to be logarithm of number n to the base a.

e.g. 25 = 32  log232 = 5
102 = 100  log10100 = 2

RATIO AND PROPORTION, INDICES, LOGARITHMS 31


1 1
4-3 =
64  log4 =-3
64
x0 = 1  logx 1 = 0
It should be carefully noted that Log 1 to any base is zero.

i.e. Loga 1 = 0, Log5 1 =0, Log101 = 0 etc.


and Logarithm of a number to the same base is 1

i.e. Loga a = 1, Logxx = 1, Log55 = 1, Log1010 = 1, Logee = 1 etc.

2. System of Logarithms :
Depending on the base, following two different system of logarithms are used
(i) Napierian Logarithms - the base is ‘e’ (  2.7183)
(ii) Common Logarithms - the base is 10.
The first was suggested by John Napier (1557-1610) which is used mostly for theoretical purposes and
second was given by Henry Briggs (1561-1630) which is used mostly for common calculations.
When no base is given, it is generally assumed to be 10. Logarithm of Napierian base can be converted
in logarithm of common base by using logeN = log10N x loge10.
3. Change to Base :
If the logarithm of a number to any base is given them the logarithm of the same Number to any other
base can be determined from the following relation :
loga N = logb N.logab

log a N
i.e. logbN =
log a b

If N = a then logaa = logba.logab

1
i.e. 1 = logba.logab i.e. logba =
log a b

4. Rules of Logarithms :
1. Shifting rule
logb a = x  a = bx
x log x 16 = 256 log x 256 = 8
 2 =8

log 2 8 = x 256 8
x = 16 x = 256

32 BUSINESS MATHEMATICS, LOGICAL REASONING & STATISTICS (Paper 3) [CA Foundation]


x log x 16 = 256 logx 256 = 8
2 =8
2. Brahmasthra

log a log 3
 logba =  = log 2 3
log b log 2

log b log 16
log cb =  log 2 16 =
log c log 2

3. log xn = n log x  5 log 2 = log 25 = log 32

4. loga a=1 log25 25 =1

log 25 25 = 1

5. logx1 = 0

Proof: log x x 0 = 0 log x x eg. log 50001 = 0 × 1


=0x1
=0

log 2 1 = 0 , log 51 = 0 , log x 1 = 0

 log10 10 = 1 , log10 1000 = 3 , log10 1,00,000 = 5

log10 100 = 2 , log10 1000 = 4

6. log  MN = log M+ log N

log  MN  log  M+ N

log m.log n  log m + log n

log m.log n  log  m + n 

log  2 × 3× 5  = log 2 + log 3 + log 5

a b c d  a b c d
log + log + log + log = log  × × × 
b c d a b c d a
= log 1 = 0

7.  N = log M - log N
log M

 2  = log 5 - log 2
log 5

RATIO AND PROPORTION, INDICES, LOGARITHMS 33


8. alog a x = x
log b log a
9. a x =b x
_________________________________________________________________________________
 Characteristic = no of digits-1
 Characteristic +1 = no of digit
 Characteristic shows the position of 1st Significant digit
eg. 312 eg. log 2000
= 12 log 3 = 3.3028 Mantissa (always positive)
= 12 (0.4771) where, 3 = Characteristic (May be + , -)
0.3028 = Mantissa (always positive)
= 5.7252
No. of digit= 5+ 1 = 6
5. Characteristic and Mantissa :
the logarithm of a number consists of two parts-intergral and fractional.
i.e. log N = Integer + Fraction

The integral part is known as characteristic and the fraction part is known as Mantissa. Characteristic
may be positive, negative or zero but Mantissa is always positive and less than 1. If number N is greater
than 1 then characteristic equal to number of digits in integral part minus 1. If number N is less than 1
then characteristic is negative then numerically one more than the no. of zeros to the right of the
decimal point and a horizontal bar (-) is put over the value of characteristic. The following table will
illustrate :

Number Characteristic Number Characteristic

3874.5 3 0.38745 -1 or 1

387.45 2 0.03874 -2 or 2

38.74 1 0.00387 - 3 or 3

3.8745 0 0.000387 -4 or 4

The mantissa of all numbers consisting of the same digits in the same order are the same, whatever
may be position of decimal point. e.g. Mantissa of 4875, 487.5, 48.75, 4.875, 0.4875, 0.004875 will be
same and its value can be obtained from the log table.

34 BUSINESS MATHEMATICS, LOGICAL REASONING & STATISTICS (Paper 3) [CA Foundation]


10. log 0.0625 to the base 2 is equal to
(a) 4 (b) 5
(c) 1 (d) none of these
CLASS WORK 11. Given log 2 = 0.3010 and log 3 = 0.4771 the value
of log 6 is
1. log 6 + log 5 is expressed as (a) 0.9030 (b) 0.9542
(a) log 11 (b) log 30 (c) 0.7781 (d) none of these
(c) log 5/6 (d) none of these 12. The value of log2 log2 log2 16
2. log28 is equal to (a) 0 (b) 2
(a) 2 (b) 8 (c) 1 (d) none of these
(c) 3 (d) none of these 1
13. The value of log to the base 9 is
3. log 32/4 is equal to 3
(a) log 32/log 4 (b) log 32 – log 4 (a) –½ (b) ½
3
(c) 2 (d) none of these (c) 1 (d) none of these
4. log (1 × 2 × 3) is equal to 14. If log x + log y = log (x+y), y can be expressed as
(a) log 1 + log 2 + log 3 (a) x–1 (b) x
(b) log 3 (c) x/x–1 (d) none of these
(c) log 2 15. The value of log2 [log2 {log3 (log3273)}] is equal
(d) none of these to
5. The value of log 0.0001 to the base 0.1 is (a) 1 (b) 2
(a) –4 (b) 4 (c) 0 (d) none of these
(c) 1/4 (d) none of these
6. If 2 log x = 4 log 3, the x is equal to 16. If log2x + log4x + log16x = 21/4, these x is equal
(a) 3 (b) 9 to
(c) 2 (d) none of these (a) 8 (b) 4
(c) 16 (d) none of these
7. log 2
64 is equal to
17. Given that log102 = x and log103 = y, the value of
(a) 12 (b) 6 log1060 is expressed as
(c) 1 (d) none of these (a) x–y+1 (b) x + y + 1
(c) x–y–1 (d) none of these
8. log 2 3 1728 is equal to
18. Given that log102 = x, log103 = y, then log101.2 is
expressed in terms of x and y as
(a) 2 3 (b) 2 (a) x + 2y – 1 (b) x + y – 1
(c) 2x + y – 1 (d) none of these
(c) 6 (d) none of these 19. Given that log x = m + n and log y = m – n, the
9. log (1/81) to the base 9 is equal to value of log 10x/y 2 is expressed in terms of
mand n as
(a) 2 (b) ½
(a) 1 – m + 3n (b) m – 1 + 3n
(c) –2 (d) none of these
(c) m + 3n + 1 (d) none of these

RATIO AND PROPORTION, INDICES, LOGARITHMS 35


20. The simplified value of 2 log 105 + log 108 – ½
1 1 1
log104 is + +
30. logab  abc  logbc  abc  logca  abc 
(a) 1/2 (b) 4
(c) 2 (d) none of these is equal to.
21. log [1 – {1 – (1 – x2)–1}–1]–1/2 can be written as (a) 0 (b) 1
(c) 2 (d) -1
(a) log x2 (b) log x
(c) log 1/x (d) none of these 1 1 1
31. + +
1 + log a  bc  1 + logb  ca 1 + logc  ab 
22. The simplified value of log 4 3 -1 -4/3 is
729 9 .27
is equal to.
(a) log 3 (b) log 2 (a) 0 (b) 1
(c) log ½ (d) none of these (c) 3 (d) –1
23. The value of (logba × logcb × logac)3 is equal to
1 1 1
(a) 3 (b) 0 + +
(c) 1 (d) none of these
32. log a x  log b x  log c x 
b c a
24. The logarithm of 64 to the base 2 2 is is equal to.
(a) 0 (b) 1
(a) 2 (b) 2 (c) 3 (d) –1
(c) ½ (d) none of these 33. log ba . log cb . log ac is equal to
25. The value of log825 given log 2 = 0.3010 is (a) 0 (b) 1
(a) 1 (b) 2 (c) –1 (d) None
(c) 1.5482 (d) none of these
26. log (1 + 2 + 3) is exactly equal to  1  2
34. logb  a  .log c(b ).log a  c 3  is rqual to.
2 3
(a) log 1 + log 2 + log 3    
   
(b) log (1×2×3)
(a) 0 (b) 1
(c) Both the above
(d) None (c) –1 (d) None
c a
27. The logarithm of 21952 to the base of 2 7 and 35. The value of is alogbc . blog a . Clog b is.
19683 to the base of 3 3 are
(a) 0 (b) 1
(a) Equal
(c) –1 (d) None
(b) Not equal
(c) Have a difference of 2269 36. The value of
(d) None logc a log a
logb c .
8 16
bc   ca .  ab  b is
28. The value of is 4 log - 3 log - log5 is
25 125 (a) 0 (b) 1
(a) 0 (b) 1 (c) -1 (b) None
(c) 2 (d) –1
logb-logc logc-loga loga-logb
29. a ×b ×c has a value of
(a) 1 (b) 0
(c) –1 (d) None

36 BUSINESS MATHEMATICS, LOGICAL REASONING & STATISTICS (Paper 3) [CA Foundation]


n
bn acn 1 1
37. The value of log n + log n + log n is 45. The value of log (ab) + log (ab) is
b c a a b
(a) 0 (b) 1 (a) 0 (b) 1
(c) –1 (d) None (c) –1 (d) None

a2 b2 c2 1 1 1 1
38. The value of log + log + log is 46. If   
bc ca ab log a t logb t log c t log z t then the
(a) 0 (b) 1 value if z is given by
(c) –1 (d) None (a) abc (b) a+b+c
(c) a(b+c) (d) (a+b)c
39. log (a9 ) + loga=10 if the value of a is given by
47. If l=1+loga bc, m=1+logbca,n=1 + logcab then the
(a) 0 (b) 10
1 1 1
(c) –1 (d) None value of + + - 1 is
l m n
loga logb logc (a) 0 (b) 1
40. = = the value of abc is
y -z z-x x -y (c) –1 (d) 3
(a) 0 (b) 1 48. 2 3 4
If a=b =c =d then the value of loga (abcd) is
(c) –1 (d) None
1 1 1 1 1 1
(a) 1+ + + (b) 1 + + +
loga logb logc 2 3 4 2! 3! 4!
41. If = = the value of
y -z z-x x-y (c) 1+2+3+4 (d) None
y+z . bz+x . x+y is given by 49. The sum of the series logab+ log a2 b2 +
a c
(a) 0 (b) 1
log a3 b3+........ log an bn is given by
(c) -1 (d) None
(a) log ab n (b) log anb
1 1
42. If log a = log b= logc the value of a4 b3 c-2 is (c) nlog ab (d) (a) & (c) both
2 5
(a) 0 (b) 1 en - e-n
50. If x= n -n then the value of n is
(c) -1 (d) None e +e

1 1 1 1 1+ x 1+x
43. If loga = log b= logc the value of a4 -bc is (a) loge (b) loge
2 3 5 2 1-x 1- x
(a) 0 (b) 1 1- x 1 1-x
(c) -1 (d) None (c) loge (d) loge
1+x 2 1+ x
1 1 1 51. The value of the following expression
44. If log 2a= log 2b=- log 2c the value of
4 6 24 logab,logbc.logcd.logd t is gioven by
3 2
a b c is a
(a) 0 (b) 1 (a) t (b) abcdt
(c) -1 (d) None (c) (a + b +c + d + t) (d) None

RATIO AND PROPORTION, INDICES, LOGARITHMS 37


52. For any three consecutive integers x y z the
625
equation log (1+xz)-2logy =0 is (c) (d) None
16
(a) True (b) False
(c) Sometimes true 1 1
60. If (4.8)x =(0.48)y = 1,000 then the value of - is
(d) cannot be determined in the cases of x y
variables with cyclic order
(a) 3 (b) -3
a+b 1
53. If log = loga + logb  then the value of 1 1
3 2 (c) (d) 
3 3
a b
+ is
b a x
61. If x2a-3y2a=x6-ay5a then the value of alog   is
(a) 2 (b) 5 y
(c) 7 (d) 3 (a) 3 log x (b) log x
54. If a 2+b 2 =7ab then the value of is (c) 6 log x (d) 5 log x
a + b loga logb
log - -
3 2 2
(a) 0 (b) 1 ---0---0---
(c) –1 (d) 7
3 3
55. If a +b =0 then the value of log(a+b)-
1
loga + logb + log3  is equal to
2
(a) 0 (b) 1
(c) -1 (d) 3
56. If x = logabc ; y = logbca ; z = logcab then the
value of xyz-x-y-z is
(a) 0 (b) 1
(c) -1 (d) 2
57. On solving the equation logt + log (t-3) = 1,
we get the value of t as
(a) 5 (b) 2
(c) 3 (d) 0
58. On solving the equation log3[log2(log3t)]=1 we
get the value of t as
(a) 8 (b) 18
(c) 81 (d) 6561

59. On sloving the equation log 1 2 [logt (log432)]=2


we get the value of t as
5 25
(a) (b)
2 4

38 BUSINESS MATHEMATICS, LOGICAL REASONING & STATISTICS (Paper 3) [CA Foundation]


21
9. If log2x + log4x + log16x = then x is
4
HOME WORK-1 (a) 4 (b) 8
(c) 16 (d) 32
10. If log m + log n = log (m + n) then m is
n n2
(a) (b)
1. The value of log 5
10,000 is n 1 n 1

(a) log4 (b) 1 n 1


(c) (d)
(c) log 4 – 1 (d) none n 1 n 1
11. If log16x + (log16x)2 + (log16x)3 + ...............  =
2. The base in logarithm of loga32 = 10 3 then a is
1 then x is
equal to 3
(a) 2 (b) 4
(a) 4 (b) 8
2 (c) 12 (d) 1 4
(c) 2 (d) 3
12
12. If log2 [log3(log2x)] = 1 then x is
3. The number of digits in 3 is (given log3 =
(a) 256 (b) 512
0.4771)
(c) 81 (d) 128
(a) 5 (b) 6 13. If log4x + log8x + log2x = 11 then x is
(c) 12 (d) 11 (a) 8 (b) 16
(c) 64 (d) 128
log10 10000
4. The value of 10 is 14. Addition of 4 .74628 and 2.42367 is
(a) 100 (b) 10
(a) 0.16995 (b) 1 .16995
(c) 1 (d) 10000 (c) 1.16995 (d) none
1 15. Subtraction of 3 .62493 from 1 .24567 is
5. If log100000x = – then the value of x is
5 (a) 0.62074 (b) 1.62074
1 1 (c) 1 .62074 (d) none
(a) (b)
10000 1000 16. (a) Multiplication of 2 .7780 by 5 is
1 1 (a) 7 .8900 (b) 10 .8900
(c) (d)
100 10 (c) 9 .8900 (d) 8 .8900
6. If log10 [log10(log10x)] = 0 then the value of x is 17. Division of 13 .15836 by 5 is
(a) 1 (b) 10 (a) 2 .43167 (b) 1 .43167
(c) 1010 (d) 100
(c) 3 .43167 (d) 4 .43167
7. If log10 xy + log10 yz = log10 xyz then y equal to
(a) xz (b) x2 m n
18. If log   + log   = log (m + n) then
n m
(c) z2 (d) 1
(a) m + n = 0 (b) m + n = 1
1 1
8. + logxyz yz + is equal to m
log xy ( xyz) log zx ( xyz) (c) =1 (d) m – n = 1
n
(a) 1 (b) 2 19. The characteristic in log (6.7472 x 10–5) is
(c) 3 (d) 4 (a) – 4 (b) – 6
(c) – 5 (d) 5

EQUATIONS AND MATRICES 39


20. If 10x = 1.73 and log10 173 = 2.2380 then x is 29. If x = loga bc, y = logb ca, z = logc ab, then
(a) 0.2380 (b) 2.2380 (a) xyz = x + y + z + 2
(c) 1.2380 (d) 1 .2380 (b) x y z = x + y + z + 1
(c) x + y + z = 1
21. Value of 16 log 4 5 is
(d) xyz = 1.
(a) 5 (b) 25
log x log y log z
(c) 125 (d) 5 64 . 30. If = = , then the value of
bc c a a b
22. If x = log2aa, y = log3a2a and z = log4a3a, then the xyz is :
value of yz (2 - x) is :
(a) 1 (b) 0
(a) 1 (b) -1
(c) 2 (d) none
(c) 2 (d) - 2 x y z w
31. If a = b = c = d , then loga (bcd) =
23. If log 2 = 0.3010, then the position of first
significant digit in the value of 2-37 is :
(a) 11th (b) 12th 1 1 1 1 
(c) 14th (d) 13th. (a)    
xy z w
log x log y log z
24.   , then 1 1 1 
l + m - 2n m  n  2l n  l  2m (b) x    
the value of x2y2z2 is : y z w
(a) 2 (b) -1
(c) 4 (d) 1 yzw
25. 2 log a + 2 log a2 + 2 log a3 + ......... + 2 log an = (c)
x
(a) n log 2 (b) n (n + 1) log a
(d) none
(c) 2 (n + 1) log a (d) none
32. The difference between the logarithms of sum
of the squares of two positive numbers A and
26. The value of log3 2 log4 3 log5 4 .... log15 14 log16 B and the sum of logarithms of the individual
15 is : numbers is a constant C. If A = B, then C is
1 1 (a) 2 (b) 1.3031
(a) (b)
3 2 (c) log 2 (d) none
33. If
1 1
(c) (d)
5 4
log x log y log z
27. If x2 + y2 = 11xy, then 2 log (x - y) = 2 2
 2 2
 2 2,
a  ab  b b  bc  c c  ca  a
(a) log 3 + log x + log y
then xa - b. yb - c. zc - a =
(b) 3 log 3 + log x + log y
(a) 1 (b) —1
(c) 2 log 3 + log x + log y
(c) 0 (d) 2.
(d) none
34. If x = 1og 35, y = log 1725, which one of the
28. If 0 < a  x, the minimum value of
following is correct ?
loga x + logx a is :
(a) x < y (b) x = y
(a) 1 (b) 2
(c) x>y (d) none
(c) 3 (d) 5

40 BUSINESS MATHEMATICS, LOGICAL REASONING & STATISTICS (Paper 3) [CA Foundation]


35. If log 1 (x-1) > 2, then x lies in the interval (a) 0 (b) 1
2

(a) (3/2,  ) (b) (-  , 3/2) (c) 2 (d) 3


44. If 3 + log5 x = 2 log25 y, then x =
(c) (3/2, 1) (d) none
(a) y/125 (b) y/25
36. The value of log2 [log2 {log3 (log3 (273)}] is : 2
(c) y /625 (d) 3 - y2/25
(a) 1 (b) 0
(c) 2 (d) 3 45. log7 log7 7( 7 7 ) equals :
xy 1 (a) 3 log2 7 (b) 1 - 3 log3 7
37. If log = (log x + log y) then, :
7 2 (c) 1 - 3 log7 2 (d) none

x y x y loga logb logc


(a)  = 48 (b)  = 49 46. If   , then aa. bb cc is equal
y x y x b c c a ab
to :
x y (a) 2 (b) 3
(c)  = 47 (d) none
y x (c) 1 (d) none
38. If log 3 = 0.4771, then the position of first  3x  3y 
significant digit in 3-20 is : 47. If log (x + y) = log   , then log x - log y
 2 
(a) 9th (b) 10th
=
(c) 11th (d) 8th.
(a) log 2 (b) log 3
39. If log3 x logy3 log2 y = 5, then x equals :
(c) log 5 (d) log 6.
(a) y 5 (b) 243
2 3 4
48. If a = b = c = d then the value of loga (abcd)
(c) 32 (d) none
40. If log303 = a, log305 = b, then log308 = 1 1 1 1 1 1
(a) 1+ + + (b) 1 + + +
(a) 3(1- a - b) (b) a - b + 1 2 3 4 2! 3! 4!
(c) 1 - a - b (d) 1(a - b+ 1). (c) 1 + 2 +3 + 4 (d) none
1 1 2
41. If log x  log x  log x then a, b, c are in 49. If loge2 logb 625 = log10 16. loge10, then b =
a c b

(a) G.P. (b) A.P. (a) 4 (b) 5


(c) H.P. (d) None (c) 1 (d) e.
50. If log (2a - 3b) = log a - log b, then a =
 1  1
42. The value of log5  1   + log5  1  
 5  6 3b 2 3b
(a) (b)
2b  1 2b  1
 1  1 
+ log5  1   +.... + log5  1   is :
 7  624  b2 3b 2
(c) (d)
(a) 5 (b) 4 2b  1 2b  1
(c) 3 (d) 2 51. On solving the equation: log1/2 [logt (log4 32)] =
43. If x = 1983 !, then the value of the expression : 2 we get the value of ‘t’ as
(a) 5/2 (b) 25/4
1 1 1 1
+   .......  (c) 625/16 (d) none
log 2x log3 x log4 x log1983 x
is equal to

EQUATIONS AND MATRICES 41


 x 60. The value of 0.356 is
52. If x2a - 3 y2a = x6 - a y5a, then the value of a log  
 y 356 353
(a) (b)
is : 999 999
(a) 3 log x (b) log x 353
(c) 6 log x (d) 5 log x. (c) (d) None
990
53. If log abc = x, log bca = y, log cab = z , then the
61. The log of 64 to base 2 2 is
1 1 1
value of   is : (a) 2 (b) 2
x 1 y 1 z 1
(a) 0 (b) 1 (c) 1 (d) 4
2
(c) -1 (d) none 62. If log 2 = 0.3010 then value of log825 is
54. If a = log2412, b = log3624, and c = 1og4836, then (a) 1 (b) 2
1 + abc is equal to : (c) 1.5482 (d) None
(a) 2bc (b) 2ab
(c) 2ac (d) none 63. log 0.0625 to the base 2 is equal to
55. If a = 1 + logxyz, b = 1 + logyzx, c = 1 + logzxy, then (a) 4 (b) 5
ab + bc + ca = — (c) 1 (d) -4
(a) 0 (b) 2abc
2 2 2
(c) a + b + c (d) abc. 
56. If log 2 = a, log 3 = b, log 7 = c and 6x =7 x + 4, then
x= ANSWER KEYS
4ab 4c
(a) (b) 1. (d) 2. (b) 3. (b) 4. (a)
bac abc
5. (d) 6. (c) 7. (d) 8. (b)
4b 4a
(c) (d) 9. (b) 10. (c) 11. (a) 12. (b)
cab abc
13. (c) 14. (b) 15. (b) 16. (a)
57. If = ; then value of x is 17. (b) 18. (b) 19. (c) 20. (a)
21. (b) 22. (c) 23. (b) 24. (d)
(a) 8 (b) 32
25. (b) 26. (d) 27. (c) 28. (b)
(c) 16 (d) 64
58. If ax = by = cz and x, y, z are in G.P. then log a, log 29. (a) 30 (a) 31. (b) 32 (c)
b and log c are in 33. (a) 34 (c) 35. (d) 36 (b)
(a) A.P. (b) G.P.
37. (c) 38. (b) 39. (c) 40. (a)
(c) A.P. and G.P. both (d) None
41. (a) 42 (c) 43. (b) 44 (a)
59. The value of log(1+ 2+ 3+.......+ n) is equal to
(a) log 1+ log 2 + …. + log n 45. (c) 46. (c) 47. (c) 48. (a)
(b) log n + long (n+1) – log 2 49. (b) 50. (a) 51. (c) 52. (a)
(c) 0 53. (b) 54. (a) 55. (d) 56. (b)
(d) 1 57 (b) 58. (b) 59. (b) 60. (c)
61 (d) 62. (c) 63. (d)

42 BUSINESS MATHEMATICS, LOGICAL REASONING & STATISTICS (Paper 3) [CA Foundation]


9. log (a-9)+ log a =1 , the value of ‘a’ is -
(a) 0 (b) 10
HOME WORK - 2 (c) -1 (d) None of these
10. Find the value of lm + mn +nl - lmn,
if l =1 +logabc, m=1 + logbca, n=1 + logcab
1. log  3 - 2  is equal to
(a) 0 (b) 1
(a) 4 (b) 3
(c) -1 (d) 3
(c) 0 (d)  11. The equivalent form of the equation log (x-2)
2
2. Evaluate log (logx )-log(logx) + log (x + 3) =0 is
(a) 2 (b) log 2 (a) x2+ x -5 = 0 (b) x2 - x -5 = 0
(c) log x (d) log x (c) x2 + x -6 = 0 (d) None of these
12. Value of log3 2 log4 3.........log1514 log1615 is
loga logba   (a) 1/3 (b) 1/2
3. The value of is (c) 1/5 (d) 1/4
loga  log ab 
3 2
13. If logx+2(x -3x -6x + 8)=3,then x =
(a) -1 (b) 1 (a) 2 (b) -2
(c) log ab (d) log a(ab) (c) 1/2 (d) None of these
75 5 32 x y
4. log - 2log + log reduces to- x+y 1
16 9 243 14. If log = logx + logy  then y
+
x
=
5 2
(a) 2 log 2 (b) 5 log 2
(a) 20 (b) 23
(c) log 2 (d) 4 log 2
(c) 22 (d) 21
 16   25   81 
5. 7log   + 5log   + 3log   is equal to  1
 15   24   80  15. log log aba +  equals
 logbab 
(a) 0 (b) 1
(c) log 2 (d) log 3 (a) log ab (b) 1
(c) 0 (d) None of these
6.
3 2
 
log a2 × b3 Simplifies to
ANSWER KEY
(a) 3/2 log a + 2/3 log b
(b) 6 (log a + log b)
1. (c) 2. (b) 3. (a) 4. (c)
(c) 2/3 log a + 3/2 log b
5. (c) 6. (c) 7. (c) 8. (c)
(d) None of these
9. (b) 10. (a) 11. (c) 12. (d)
7. The simplified value of log2 log2 log216 is
13. (b) 14. (b) 15. (c)
(a) 0 (b) 2
(c) 1 (d) None of these
8. Find the value of [logyx logzy logxz]3
(a) 0 (b) -1
(c) 1 (d) 3

EQUATIONS AND MATRICES 43


CHAPTER-2
EQUATIONS AND MATRICES

UNIT I : EQUATIONS

INTRODUCTION
Equation is defined to be a mathematical statement of equality. If the equality is true for certain value
of the variable involved, the equation is often called a conditional equation and equality sign ‘=’ is
used; while if the equality is true for all values of the variable involved, the equation is called an
identity.
x +2 x +3
For Example: + = 3 holds true only for x =1.
3 2
x + 2 x + 3 5x + 13
So it is a conditional. On the other hand, + =
3 2 6
is an identity since it holds for all values of the variable x.
Determination of value of the variable which satisfies an equation is called solution of the equation
or root of the equation. An equation in which highest power of the variable is 1 is called a Linear (or
a simple) equation. This is also called the equation of degree 1. Two or more linear equations involving
two or more variables are called Simultaneous Linear Equations. An equation of degree 2 (highest
Power of the variable is 2) is called Quadratic equation and the equation of degree 3 is called Cubic
Equation.
For Example: 8x+17(x–3) = 4 (4x–9) + 12 is a Linear equation.
3x2 + 5x +6 = 0 is a Quadratic equation.
4x3 + 3x2 + x–7 = 1 is a Cubic equation.
x + 2y = 1, 2x + 3y = 2 are jointly called Simultaneous equations.
For quadratic equaction ax2 + bx + c = 0, a  0

1. factors Formulas

-b ± Δ
x= where Δ = determinant
2a
2
Δ = b -4ac

44 BUSINESS MATHEMATICS, LOGICAL REASONING & STATISTICS (Paper 3) [CA Foundation]


2. Δ  Nature of roots
2
Δ =b -4ac
Δ 0 Δ>0 Δ=0 Δ<0
Real Roots Real and unequal Real and equal Imaginary and
unequal
3. EquationFormat:

x 2 -  α + β  x + αβ = 0

α + β = Sum of Roots = -b
a

αβ = Product of Roots = c
a
4. One root exceeds the other
difference ,between the roots

b2 - 4ac
α -β =
a
5. One root is reciprocal of other
C=a
6. roots are same but opposite in sign
α = -β
α +β = 0
7. Roots are in the ratio:
2
b2 p + q 
=
ac pq

2 2 2
8. α + β =  α + β  - 2αβ

3 3 3
9. α + β =  α + β  - 3αβ  α + β 

3 3 3
10. α - β =  α - β  + 3αβ  α + β 

EQUATIONS AND MATRICES 45


UNIT 2 : MATRICES

INTRODUCTION
Matrices applications are used in Business, Finance and Economics. Matrices applications are helpful to
solve the linear equations with the help of this cost estimation, sales projection, etc., can be predicted. In
this chapter, we shall find it interesting fundamental applications of matrices.
Matrix :
Ram, Sita and Laxman are three friends. Ram has 5 books, 3 pencils and 2 pens. Sita has 10 books, 8 pencils
and 5 pens. Laxman has 15 books, 10pencils and 2 pens. The above information about three friends can be
represented in the following form:
Books Pencils Pen
Ram 5 3 2
Sita 10 8 5
Laxman 15 10 2
We can express the above information in the following form:

An arrangement or display of information in the above form is called a matrix.


Matrix (Definition)
A rectangular array of numbers (real/complex) denoted by:

46 BUSINESS MATHEMATICS, LOGICAL REASONING & STATISTICS (Paper 3) [CA Foundation]


A is rectangular matrix with m rows and n columns. The numbers aij, i = 1,2 ……..m; j = 1,2,…..n of this array are
called its elements aij, is associated. We shall denote a matrix either using by using brackets
[ ]; or ( ).
Notes:
1. It is to be noted that a matrix is just an arrangement of elements without any value in rows and
columns.
2. The plural matrix is matrices.

Order of a Matrix: A matrix A with m rows and n columns is called a matrix of order (m, n) or m × n (read as m
by n).

1 2 2
 
 4 6 5
Consider the matrix A = 
 7 9 8 
 

It is a matrix of order 3 × 3. Here the 9 occurs in the third row and second column. The elements 5 occurs in the
second row and third column. Thus in notations we may write: a32 = 9 and a23 = 5.

EQUAL MATRICES:
Two matrices are said to be equal if they satisfy the following conditions:
(1) The number of rows in both matrices should be equal and the number of columns should also be
equal i.e.the order of both the matrices must be the same.
(2) The corresponding elements in both the matrices are equal if they are equal if they are equal in all
respects.
Transpose of a matrix:
If we interchange rows and columns of a marix A.the new matrix so obtained is known as the transpose of
matrix A and it is denoted by AT or A’

1 2 5 0
A=3 5 2 2
e.g. If  
4 1 0 0

1 3 4
2 5 1
AT =  
5 2 0
0 1 0

EQUATIONS AND MATRICES 47


The order of A is 3 x4 and that of AT is 4 x3
i.e... If A is of order (m x n ) then AT is of order (n x m)
2. Special types of matrices:
(i) Row matrix: A matrix in which there is only row and any number of columns is said to be a r o w
matrix.
A=[ a11,a12,a13........a10] is a row matrix.
It is clear that order of this row matrix is l x n
e.g. [ 1 2 3 ] is a row matrix of order 1 x3
(ii) Column matrix: A matrix in which there is only one column and any number of rows is said to be a
column matrix

 a11 
a 
 21 
a 
A =  31 
e.g.  ....  is a column matrix of order m x 1.
.... 
 
am1 

0 
 5
 
Similarly  2  is a column matrix of oder 4 x 1
 1 

(iii) Zero matrix or Null matrix: If all the elements of matrix are zero,it is said to be a null matrix or a zero
matrix.

0 0 0 0
e.g.  is a null matrix of order 2 x 4
0 0 0 0
(iv) Square matrix:A matrix in which number of rows and number of column are equal is said to be a square
matrix

 a11 a12 a13 


 
e.g.  A = a21 a22 a 23  is a square matrix of order 3 x 3
 a31 a 32 a33 
 

Similarly  is a square matrix of order 2 x 2

48 BUSINESS MATHEMATICS, LOGICAL REASONING & STATISTICS (Paper 3) [CA Foundation]


(v) Symmetric matrix: If the transpose of a square matrix gives the same matrix. it is known as a symmetric
matrix i.e. for a symmetric matrix.
aij= element of ith row and jth column
= element of jth row and ith column]
=aji

1 2 3
A= 2 4 5
e.g.   is a symmetric matrix of order 3 x 3
3 5 6

a21 = a12 = 2
a13 = a31 = 3
Here
a23 = a32 = 5

 AT=A
[Note: Every symmetric matrix must be a square matrix.]
(vi) Skew symmetric matrix : If in a square matrix aij= -aij i.g. the elements of ith row and jth column and jth
row and jth column are equal in magnitude but opposite in sign,the matrix is known as a skew
symmetrixc matrix.
Obviously each diagonal element of a skew symmetric matrx must be zero.


i.e. a11 = a22 = a33 = ....... = 0 
0 -2 -3
A= 2 0 1
eg.   is a skew symmetric matrix of order 3 x3.
3 -1 0
its diagonal elements are all zero.In a skew symmetric matrix AT = -A
(vii) Unit matrix or Identity matrix:A square matrix in which all diagonal elements are unity and all other
elements are unity all other elements are zero is known as a unit matrix or an identity matrix. and it is
denoted by I

1 0 0
I=0 1 0
eg.   is a unit matrix of order 3 x 3
0 0 1

1 0
Similarly I=   is a unit matrix of order 2 x 2
0 1
[ Note: Unit matrix holds special importance in the study of matrices .]

EQUATIONS AND MATRICES 49


(viii) Diagonal matrix:If all elements except diagonal elements of a square matrix are zero the matrix is said
to be a diagonal matrix.

2 0 0
A= 0 -3 0
eg.   is a diagonal matrix.
0 0 1
(ix) Scalar matrix:A diagonal matrix in which all the elements on principal diagonal are equal is called a
scalar matrix e.g.

5 0 0
A= 0 5 0
  is a 3 x 3 scalar matrix.
0 0 5
(x) Triangular matrix: A square matrix is said to be a triangular matrix if each element above or below the
principal diagonal is zero.If each element above the principal diagonal is zero, it is called Lower triangular
matrix and if each element below the principal diagonal is zero it is called Upper triangular matrix.

3 0 0
A= 4 5 0
eg.   is a lower triangular matrix
3 2 1

3 2 1
A= 0 5 4
and B=   is a upper triangular matrix.
0 0 3

50 BUSINESS MATHEMATICS, LOGICAL REASONING & STATISTICS (Paper 3) [CA Foundation]


2 xy
7. The solution of : (2x + 3y) = 3 + ;
CLASS WORK 13 4
4 y  5x 1
= 2x + 7 is :
UNIT I : EQUATION 3 6
(a) (-7/2, 9/2) (b) (7/2, 9/2)
(c) (7/2, -9/2) (d) none
1. Which of the following equation is not a linear
equation ? 8. The solution of the simultaneous equations :
(a) 2x + 3 = 7 4 3
+ 5y = 7; + 4y = 5, x  0 is :
4 x x
(b) ( ) x + (7/2) = 6 + (1/2) x
3 1 1
(a) ( , 1) (b) ( , -1)
3 3
1
(c) x + =3 (d) none 1
2x (c) (- , 1) (d) none
3
2. Which of the following equation is not a linear
equation ? 9. The solution of the set of equations :

(a) x + 3x = 0 (b) x + 5 2 15 7
x =3  = -1;  = 10,
xy xy xy xy
(c) 7 x+4 7 = 3
7 (d) none
where x + y  0 and x - y  0 is :
3. The solution of the equation : 2x + 2 = 3x - 4 (a) (2, 3) (b) (3, 2)
(c) (-2, 3) (d) none
- 3 2 is :
10. The solution of the system of equations :
(a) 4 (1 - 2) (b) 4 (1 + 2) x y x y
 = a + b; 2  2 = 2 is :
(c) 3 (1 - 2) (d) 3 (1 + 2) a b a b
4. The root of the equation (a) (a2, b2) (b) (-a2, b2)
2 2
(c) (a , -b ) (d) none
4 5 2 1  10 11. The solution of the following system of
: x    x    is :
5 6 3 4 9
xy yz xz
(a) x = 25/12 (b) x = 12/25 equations = 70; = 140;
xy y+z x+z
(c) x = 23/12 (d) 17/12.
= 84 is :
y  11 y  1 y  7 (a) 210, 105, 320 (b) 95, 210, 420
5. Solve for y :   and the
6 9 4 (c) 105, 210, 420 (d) none
value of y is : 12. A man leaves half his property to his wife, one
(a) -1 (b) 7 third to his son and the remaining to his
daughter. If the daughter’s share is ` 15,000,
(c) 1 (d) - 1/7. how much money did his wife get ?
6. The solution of the equation is : (a) ` 45,000 (b) ` 30,000
x y x y (c) ` 9,000 (d) None
+ -1= + = 22 is :
4 5 5 4
(a) (60, 40) (b) (40, 60)
(c) (40, -60) (d) none

EQUATIONS AND MATRICES 51


13. By selling a car for ` 72,000, a person made a 20. The age of the father is 3 years more than three-
profit of 20%. what was the cost price of the times the age of the son. Three years hence
car ? the age of father will be 10 years more than
(a) ` 50,000 (b) ` 60,000 twice the age of the son. Their present ages in
(c) ` 56,000 (d) None years are:
14. On the occassion of Diwali, Kahdi Bhandar (a) (33, 10) (b) (31, 12)
allows a discount of 20% on all textiles and (c) (30, 13) (d) none
25% on ready-made garments. Hari paid ` 180 21. If twice the son’s age in years is added t the
for a gown. what was the marked price of the father’s age, the sum is 70. But if twice the
gown ? father’s age is added to the son’s age, the um
(a) ` 240 (b) ` 260 is 95. The ages of father and son in years are :
(c) ` 275 (d) None (a) (40, 15) (b) (15, 40)
15. A and B can do a piece of work in 8 days, which (c) (25, 30) (d) none
A alone can do in 12 days. In how many days 22. The sum of the ages (in years) of a son and his
can B alone do the same work ? father is 35 and the product of their ages is
(a) 20 days (b) 24 days 150. Their ages are :
(c) 18 days (d) 16 days. (a) 7 years, 42 years (b) 5 years, 30 years
16. A streamer goes downstream from one part to (c) 6 years, 36 years (d) none
another in 4 hours. It covers the same distance 23. The age of a person is twice the sum of the
upstream in 5 hours. If the speed of the stream ages of his two sons and five years ago his
be 2 km per hour, find the distance between age was thrice the sum of their ages. Find
the two parts. his present age.
(a) 60 km. (b) 80 km
(a) 60 years (b) 52 years
(c) 70 km (d) 55 km
17. A man rowing at the rate of 5 km an hour in (c) 51 years (d) 50 years
still water takes thrice as much time in going 24. y is older than x by 7 years 15 years back x’s
40 km up the river as in going 40 km down. age was 3/4 of y’s age. Their present ages
Find the rate at which the river flows. are:
(a) 9 km/hr (b) 10 km/hr
(a) (x=36, y=43) (b) (x=50, y=43)
(c) 12 km/hr (d) none
18. 5 years ago, the age of a man was 7 times the (c) (x=43, y=50) (d) (x=40, y=47)
age of his son. the age of the man will be 3 25. Two man start from points A and B, 42 km apart.
times the age of his son in five years from now. One walks at 4 km per hour and meets the
How old is he now ? other, going in the opposite direction after 6
(a) 40 years (b) 30 years hours. Find the rate at which the second man
(c) 35 years (d) non is walking.
(a) 3 km/hr (b) 5 km/hr
19. Ten years ago the age of a father was four
(c) 7 km/hr (d) none
times of his son. Ten years hence the age of
the father will be twice that of his son. The 26. In an election for a corporation seat, there
were two candidates. A total of 9791 votes
present ages of the father and the son are.
were polled, 116 votes were declared invalid.
(a) (50, 20) (b) (60, 20) The successful candidate got 5 votes for every
(c) (55, 25) (d) none of these 4 votes his opponent had. By what margin did
the successful candidate win ?
(a) 1175 votes (b) 1050 votes
(c) 1075 votes (d) 1250 votes.

52 BUSINESS MATHEMATICS, LOGICAL REASONING & STATISTICS (Paper 3) [CA Foundation]


27. A man invested ` 35,000, a part of it at an annual (a) 39 (b) 92
interest rate of 12% and the rest at 14%. If he (c) 93 (d) 94
received a total interst of ` 4460, how much
did he invest at the rate of 14% ? 35. A number between 10 and 100 is five times
the sum of its digits. If 9 be added to it the
(a) ` 13,000 (b) ` 22,000
digits are reversed find the number.
(c) ` 17,000 (d) ` 19,000
(a) 54 (b) 53
28. If a scooterist drives at the rate of 24 km per
hour, he reaches his destination 5 minutes (c) 45 (d) 55
late. If he drives at the rate of 30 km per hour, 36. A number consisting of two digits is four times
he reaches his destination 4 minutes soon. the sum of its digits and if 27 be added to it the
How far is his destination ? digits are reversed. The number is :
(a) 18 km (b) 16 km (a) 63 (b) 35
(c) 20 km (d) 15 km (c) 36 (d) 60
29. A train travel a distance of 300 km at a constant
37. The sum of the digits in a three digit number is
speed. If the speed of the train is increased by
12. If the digits are reversed the number is
5 km an hour, the journey would have taken 2
increased by 495 but reversing only of the ten’s
hours less. The original speed of the train is:
and unit digits increases the number by 36.
(a) 25 km/hr (b) 28 km/hr
The number is
(c) 27 km/hr (d) none
(a) 327 (b) 372
30. A number consists of two digits,the digit in
the ten’s place is twice the digit in the unit’s (c) 237 (d) 273
place. If 18 be subtracted from the number the 38. Two numbers are such that twice the greater
digits are reversed. The number is : number exceeds twice the smaller one by 18
(a) 63 (b) 42 and 1/3rd of the smaller and 1/5th of the
(c) 84 (d) none greater number are together 21. The numbers
31. The sum of a two digit number and the number are:
obtained by reversing the order of digits is 121, (a) (36, 45) (b) (45, 36)
and the two digits differ by 3. The number is :
(c) (50, 41) (d) (55, 46)
(a) 47 (b) 57
39. Of two numbers, 1/5th of the greater is equal
(c) 63 (d) none
to 1/3rd of the smaller and their sum is 16.
32. A certain number of two digits is 4 times the
sum of the digits. If 18 be added to the number The numbers are:
the digits are reversed. The number is (a) (6, 10) (b) (9, 7)
(a) 42 (b) 24 (c) (12, 4) (d) (11, 5)
(c) 36 (d) none 40. The fourth part of a number exceeds the sixth
33. The sum of the digits of a two digit number is part by 4. The number is :
10. If 18 be subtracted from it the digits in the (a) 84 (b) 44
resulting number will be equal. The number is (c) 48 (d) none
(a) 37 (b) 73 1
(c) 75 (d) none of these 41. If a number of which the half is greater than
5
34. A number consists of two digits. The digits in of the number by 15 then the number is
the ten’s place is 3 times the digit in the unit’s (a) 50 (b) 40
place. If 54 is subtracted from the number the
(c) 80 (d) none of these.
digits are reversed. The number is

EQUATIONS AND MATRICES 53


42. A pharmacist needs to strengthen a 15% 49. A man starts his job with a certain monthly
alcohol solution to one of 32% alcohol salary and earns a fixed increment every year.
solution. How much pure alcohol should be If his salary was ` 1500 after 4 years of service
added to 400 ml of 15% alcohol solution ? and ` 1800 after 10 years of service, what was
(a) 70 ml (b) 90 ml his starting salary and what is the annual
(c) 100 ml (d) 80 ml increment in rupees ?
43. There are benches in a classroom. If 4 students (a) ` 1300, `50 (b) ` 1100, ` 50
sit on each bench, three benches are left (c) `1500, ` 30 (d) none
vacant and if 3 students sit on each bench, 3 50. The taxi charges in a city comprise of a fixed
students are left standing. What is the total charge together with the charge for the
number of students in the class ? distance covered. For a journey of 10 km. the
(a) 46 (b) 56 charges paid is ` 75 and for a journey of 15 km.,
(c) 48 (d) 66 the charges paid is ` 110. What will a person
have to pay for travelling of distance of 25 km?
44. The denominator of a fraction exceeds the
numerator by 5 and if 3 be added to each term, (a) ` 160 (b) ` 200
the fraction becomes 3/4. Find the fraction. (c) ` 180 (d) none
(a) 12/17 (b) 13/18 51. The largest angle of triangle is twice the sum
(c) 16/21 (d) none of the other two. The smallest is one-fourth of
the largest. The smallest angle in degrees is
45. Find a fraction which becomes (1/2) when 1 is
subtracted from the numerator and 2 is added (a) 30° (b) 60°
to the denominator; and becomes (1/3) when (c) 450 (d) none
7 is subtracted from the numerator and 2 from 52. The sum of two numbers is 52 and their
the denominator. difference is 2. The numbers are
(a) 15/26 (b) 13/15 (a) 17 and 15 (b) 12 and 10
(c) 2 1/23 (d) none (c) 27 and 25 (d) none of these
46. The denominator of a fraction exceeds the
53. Divide 56 into two parts such that three times
numerator by 2.If 5 be added to the numerator
the first part exceeds one third of the second
the fraction increases by unity. The fraction is.
by 48. The parts are.
5 1 (a) (20, 36) (b) (25, 31) ]
(a) (b)
7 3 (c) (24, 32) (d) none of these
54. The product of two numbers is 3200 and the
7 3
(c) (d) quotient when the larger number is divided
9 5 by the smaller is 2.The numbers a re
47. Find the fraction which is equal to 1/2 when (a) (16, 200) (b) (160, 20)
both its numerator and denominator are
(c) (60, 30) (d) (80, 40)
increased by 2. It is equal to 3/4 when both are
increased by 12. 55. One student is asked to divide a half of a
number by 6 and other half by 4 and then to
(a) 3/8 (b) 5/8
add the two quantities. Instead of doing so
(c) 2/8 (d) 2/3 the student divides the given number by 5. If
48. A lady has only 25 paise and 50 paise coins in the answer is 4 short of the correct answer then
her purse. If in all she has 40 coins valued the number was
` 12.75. How many of each type does she have? (a) 320 (b) 400
(a) (18, 23) (b) (30, 8) (c) 480 (d) none of these.
(c) (29, 11) (d) none

54 BUSINESS MATHEMATICS, LOGICAL REASONING & STATISTICS (Paper 3) [CA Foundation]


56. The demand and supply equations for a certain (a) no solution
commodity are 4q + 7p = 17 and (b) unique solution
(c) infinite number of solutions
q 7
p= + . respectively where p is the market (d) none
3 4
64. The system of equations : 6x + 5y = 11; 9x + (15/
price and q is the quantity then the
2) y = 21 have :
equilibrium price and quantity are:
(a) unique solution
3 1 (b) infinite number of solutions
(a) 2, (b) 3,
4 2 (c) no solution
(d) none
3
(c) 5, (d) None of these. 65. The system of equations : 4x + 7y = 10; 10x +
5 (35/2) y = 25 have :
57. The diagonal of a rectangle is 5 cm and one of (a) Infinite number of solution
at sides is 4 cm. Its area is (b) unique solution
(a) 20 sq.cm. (b) 12 sq.cm. (c) no solution
(c) 10 sq.cm. (d) none of these (d) none
58. Three persons Mr. Roy, Mr. Paul and Mr. Singh 66. The value of k for which the system of
together have `51. Mr. Paul has `4 less than equations : kx + 2y = 5; 3x + y = 1 have unique
Mr. Roy and Mr. Singh has got `5 less than Mr. solution is :
Roy. They have the money as. (a) k6 (b) k = 4
(a) (`20, `16, `15) (b) (`15, `20, `16) (c) k = 3 (d) k = 1.
(c) (`25, `11, `15) (d) none of these 67. The system of linear equations : ax + by + c = 0
59. Monthly incomes of two persons are in the and lx + my + n = 0 will admit a unique solution
ratio 4 : 5 and their monthly expenses are in if
the ratio 7 : 9. If each saves `50 per month find (a) am = bl (b) am  bl
their monthly incomes. (c) ab = ml (d) none
(a) (500, 400) (b) (400, 500) 68. Determine the value of  for which the
system of linear equations
(c) (300, 600) (d) (350, 550)
 x + 3y =  - 3; 12x +  y =  has an infinite
60. The wages of 8 men and 6 boys amount to `33. number of solutions :
If 4 men earn `4.50 more than 5 boys
(a) 6 (b) 7
determine the wages of each man and boy.
(c) -6 (d) 8
(a) (`1.50, `3) (b) (`3, `1.50)
69. Determine the value of a and b for which the
(c) (`2.50, `2) (d) (`2, `2.50) following system of linear equations has
61. The solution of the set of equations : x + y = 7; infinitely many solutions : 3x - (a + 1) y = 2b - 1;
5x + 12y = 7 is : 5x + (1 - 2a) y = 3b.
(a) (11, - 4) (b) (4, -11) (a) a = 8, b = 5 (b) a = -8, b = 5
(c) (-11, - 4) (d) none (c) a = 8, b = -5 (d) a = -8, b = -5
62. The value of x and y satisfying the equations 70. The roots the equation : 6x2 + x - 2 = 0 are :
8x + 5y = 9; 3x + 2y = 4 are given by : (a) (1/2, - 2/3) (b) (-1/2, 2/3)
(a) (2, 5) (b) (-2, 5) (c) (-1/2, -2/3) (d) (1/2, 2/3)
(c) (2, -5) (d) none
63. The system of equations : 2x + 3y = 7; 6x + 5y =
11 have :
EQUATIONS AND MATRICES 55
71. Solving the equation : 3x2 - 14x + 8 = 0, we get (a) – 3 (b) – 1
roots as : (c) 1 (d) – 2
(a) 4 (b) 2 80. The equation x2 –(p+4)x+ 2p + 5 = 0 has equal
(c) (4, 2/3) (d) none roots the values of p will be.
72. Solving the equation : x2 - 24x + 135 = 0, we (a) ± 1 (b) 2
find value(s) of x as :
(c) ±2 (d) –2
(a) 9, 6 (b) 9, 15 (c) 15,
6 (d) none 81. The roots of the equation x2 + (2p–1)x + p2 = 0
2x+ 3 2 x are real if.
73. If 2 – 3 . 2 + 1 = 0 then values of xare
(a) p >1 (b) p <4
(a) 0, 1 (b) 1, 2
(c) p  1/44 (d) p  1/44
(c) 0, 3 (d) 0, – 3
82. If x = - 4 is a root of the equation : x2 + px - 4 =
74. The solution of the equation 3x2 –17x+ 24 = 0
0 and the equation x2 + px + q = 0 has equal
are
roots, then the values of p and q are :
 2 (a) (3, 9/4) (b) (- 9/4, 3)
(a) (2, 3) (b)  2, 3  (c) (- 3, 9/4) (d) none
 3
83. 2
Solving the equation : x - (a + b) x + ab = 0, we
 2  2 find the value(s) of x as :
(c)  3, 2  (d)  3,  (a) (a, b) (b) a
 3  3
(c) b (d) none
75. If L + M + N = 0 and L, M, N are rationals the
roots of the equation 84. If one root of an equation is 2 + 3 then the
(M+N–L)x2 +(N+L–M)x+(L+M–N) = 0 are equation is :
(a) real and irrational (a) x2 - 4x + 1 = 0 (b) x2 + 4x + 1 = 0
(b) real and rational (c) x2 - 4x + 11 = 0 (d) none
(c) imaginary and equal 85. If p  q and p = 5p – 3 and q2 = 5q – 3 the
2

(d) real and equal p q


equation having roots as and is
76. The value of p for which the equation : px2 + q p
4x + 1 = 0 has real roots is :
(a) x2 – 19x+ 3 = 0 (b) 3x2 – 19x– 3 = 0
(a) p  4 (b) p  4
(c) 3x2 – 19x+ 3 = 0 (d) 3x2 + 19x+ 3 = 0
(c) p = 2 (d) p  2 86. If α,β be the roots of the equation : 2x2 -4x - 3
77. The values of k for which the equation : (k - 12)
x2 + 2 (k - 12) x + 2 = 0 has real and equal roots = 0, then the value of α2 + β2 is :
are: (a) 5 (b) 7
(a) (12, 14) (b) (-12, 14) (c) 3 (d) - 4
(c) (12, -14) (d) (-12, -14) 87. If p and q are the roots of x2 + x + 1 = 0, then the
78. Find the positive value of k for which the values of p3 + q3 becomes
equations : x2 + kx + 64 = 0 and x2 - 8x + k = 0 will (a) 2 (b) - 2
have real roots. (c) 4 (d) -4
(a) 12 (b) 16
(c) 18 (d) 22
79. If the roots of the equation 2x2 + 8x– m3 = 0 are
equal then value of m is

56 BUSINESS MATHEMATICS, LOGICAL REASONING & STATISTICS (Paper 3) [CA Foundation]


88. If  and  are the roots of x 2 = x + 1, then
1
2
2 2 1
2
value of α - β is : 96. The values of 1 are :
β α 2
2  ...
(a) 2 5 (b) 5
(a) 1± 2 (b) 2  5
(c) 3 3 (d) -2 5
89. If the sum of the roots of the quadratic (c) 2  3 (d) none
equation : ax2 + bx + c = 0 is equal to the sum of
the squares 97. The value of 6  6  6.....  is :

b2 bc
of their reciprocal, then  is equal to : (a) -3 (b) 2
ac a 2
(c) 3 (d) 4
(a) 2 (b) -2
(c) 1 (d) -1 1
4
90. If the difference between the roots of x2 + ax - 1
b = 0 is equal to the difference between the 4
98. The values of 1
roots of x2 - px + q = 0, then the value of a2 - p2 4
4  .....
in terms of b and q is :
(a) - 4 (b + q) (b) 4(b + q) (a) 1 (b) 2+ 5
2
(c) 4(b - q) (d) none
91. If one root of the equation : x 2 - 8x + m = 0 (c) 2± 5 (d) none of these
exceeds the other by 4, then the value of m is
99. If x = m is one of the solutions of the equation
:
: 2x2 + 5x - m = 0, the possible values of m are:
(a) m = 10 (b) m = 11
(a) (0, 2) (b) (0, -2)
(c) m = 9 (d) m = 12
92. If the roots of x2 - px + q = 0 differ by unity, then (c) (0. 1) (d) (1. -1)
1 + 4q = ..... 100. The solutions of the equation : x2 - 2x + 2 =
(a) p (b) p2 l3x - 2l are :
(c) pq (d) p3 (a) 4, 1 (b) -4, -1
93. 2
If the roots of ax + bx + c = 0 are in the ratio p/ (c) -4, 1 (d) 4, 2
q, then the value of b2/(ca) is : 101. A solution of the quadratic equation
(a) (p + q)2/(pq) (b) (p + q)/(pq) (a+b–2c) x2+ (2a–b–c)x+ (c+a–2b) = 0 is
(c) (p - q)2/(pq) (d) (p - q)/(pq) (a) x= 1 (b) x= –1
94. The condition that one of the roots of ax2 + bx (c) x= 2 (d) x= – 2
+ c = 0 is thrice the other is :
102. The values of x in the equation
(a) 3b2 = l6ca (b) b2 = 9ca
(c) 3b2= -16ca (d) b2 = -9ca. 7(x+2p)2+ 5p2= 35xp + 117p2 are
95. The roots of the equation : lx2 + mx + n = 0 are (a) (4p, –3p) (b) (4p, 3p)
in the ratio 3 : 4, then 12m2 = knl, where k = (c) (–4p, 3p) (d) (–4p, –3p)
(a) 32 (b) 49 103. The solutions of the equation
(c) 43 (d) 59 6x 6(x + 1)
+ = 13
x +1 x
(a) (2, 3) (b) (3, –2)
(c) (–2, –3) (d) (2, –3)
EQUATIONS AND MATRICES 57
104. The satisfying values of xfor the equation 111. Solving 9x = 3y and 5x + y + 1 = 25xy, we get the
following roots :
1 1 1 1
= + + are (a) 1, 2 (b) 0, 1
x +p+q x p q (c) 0, 3 (d) 1, 3
(a) (p, q) (b) ( –p, –q) 112. The sum of two numbers is 8 and the sum of
(c) (p, –p) (d) ( –p, q) their squares is 34. Taking one number as x
105. The values of xfor the equation x2 + 9x+ 18 = 6 form an equation in x and hence find the
– 4xare numbers. The numbers are
(a) (1, 12) (b) (–1, –12) (a) (7, 10) (b) (4, 4)
(c) (1, –12) (d) (–1, 12) (c) (3, 5) (d) (2, 6)
106. The values of x satisfying the equation 113. The difference of two positive integers is 3
and the sum of their squares is 89. Taking the
(2x 2 + 5x - 2) - (2x 2 + 5x - 9) = 1
smaller integer as x form a quadratic equation
(a) (2, –9/2) (b) (4, –9) and solve it to find the integers. The integers
(c) (2, 9/2) (d) (–2, 9/2) are.
107. The equation (a) (7, 4) (b) (5, 8)
3(3x2 + 15) 2x2 + 96 (c) (3, 6) (d) (2, 5)
+ 2x2 + 9 = + 6 has got 114. Five times of a positive whole number is 3 less
6 7
than twice the square of the number. The
the solution as
number is
(a) (1, 1) (b) (1/2, –1)
(a) 3 (b) 4
(c) (1, –1) (d) (2, –1)
(c) –3 (d) 2
 l-m  2  l+m  115. Divide 50 into two parts such that the sum of
108. The equation  x -  x+m=0 their reciprocals is 1/12. The numbers are
 2   2 
has got two values of x to satisfy the equation (a) (24, 26) (b) (28, 22)
given as (c) (27, 23) (d) (20, 30)
116. There are two consecutive numbers such that
 2m   m  the difference of their reciprocals is 1/240.
(a)  l,  (b)  l, 
 l-m   l-m  The numbers are
(a) (15, 16) (b) (17, 18)
 2l   l  (c) (13, 14) (d) (12, 13)
(c)  l,  (d)  l, 
 l-m   l-m  117. The sum of two numbers is 45 and the mean
x x  1 34 proportional between them is 18. The
109. The roots of theequation : + = , numbers are
x 1 x 15
(a) (15, 30) (b) (32, 13)
x  0, x  -1 are :
(a) (3/2, -5/2) (b) (3/2, 5/2) (c) (36, 9) (d) (25, 20)
(c) (-3/2, 5/2) (d) none 118. The sum of two irrational numbers multiplied
1 by the larger one is 70 and their difference is
1 1 1 multiplied by the smaller one is 12; the two
110. Solving + 2 - 13 = 0 and + -5 = 0, we
x2 y x y numbers are
get the roots asunder : (a) 3 2 ,2 3 (b) 5 2 ,3 5
1 1 1 1
(a) , (b) , (c) 2 2 ,5 2 (d) none of these
8 5 2 3
119. A distributor of apple Juice has 5000 bottle in
1 1 1 1 the store that it wishes to distribute in a
(c) , (d) , month. From experience it is known that
13 5 4 5
58 BUSINESS MATHEMATICS, LOGICAL REASONING & STATISTICS (Paper 3) [CA Foundation]
demand D (in number of bottles) is given by 128. The roots of the cubic equation : x3 + 7x2 - 21x -
D = –2000p2 + 2000p + 17000. The price per bottle 27 = 0 are :
that will result zero inventory is (a) (-3, -9, -1) (b) (3, -9, -1)
(a) `3 (b) `5 (c) (3, 9, 1) (d) (-3, 9, 1).
(c) `2 (d) none of these. 129. The solution of the cubic equation x 3 –6x 2
120. The area of a rectangular field is 2000 sq.m and +11x–6 = 0 is given by the triplet :
its perimeter is 180m. Form a quadratic (a) (–1, 1 –2) (b) (1, 2, 3)
equation by taking the length of the field as (c) (–2, 2, 3) (d) (0, 4, –5)
xand solve it to find the length and breadth of 130. The cubic equation x + 2x2 – x– 2 = 0 has 3 roots
3

the field. The length and breadth are namely.


(a) (205m, 80m) (b) (50m, 40m) (a) (1, –1, 2) (b) (–1, 1, –2)
(c) (60m, 50m) (d) none (c) (–1, 2, –2) (d) (1, 2, 2)
121. Two squares have sides p cm and (p + 5) cms. 131. x, x– 4, x+ 5 are the factors of the left–hand
The sum of their squares is 625 sq. cm. The side of the equation.
sides of the squares are (a) x3 + 2x2 – x– 2 = 0
(a) (10 cm, 30 cm) (b) (12 cm, 25 cm) (b) x3 + x2 – 20x= 0
(c) 15 cm, 20 cm) (d) none of these (c) x3– 3x2– 4x+ 12 = 0
122. The hypotenuse of a right–angled triangle is (d) x3– 6x2+ 11x– 6 = 0
20cm. The difference between its other two 132. The equation 3x3 + 5x2 = 3x+ 5 has got 3 roots
sides be 4cm. The sides are and hence the factors of the left–hand sideof
(a) (11cm, 15cm) (b) (12cm, 16cm) the equation 3x3+ 5x2– 3x– 5 = 0 are
(c) (20cm, 24cm) (d) none of these (a) x– 1, x– 2, x– 5/3
123. The sides of an equilateral triangle are (b) x– 1, x+1, 3x+ 5
shortened by 12 units 13 units and 14 units (c) x+ 1, x– 1, 3x- 5
respectively and a right angle triangle is (d) x– 1, x+ 1, x– 2
formed. The side of the equilateral triangle is
133. The roots of x3 + x2 – x– 1 = 0 are
(a) 17 units (b) 16 units
(a) (– 1, – 1, 1) (b) (1, 1, – 1)
(c) 15 units (d) 18 units
(c) (– 1, – 1, – 1) (d) (1, 1, 1)
124. The product of two consecutive integers is 72. 3 2
134. If 4x +8x –x–2=0 then value of (2x+3) is given
The integers are :
by
(a) (8, 9) (b) (- 8, - 9)
(a) 4, –1, 2 (b) –4, 2, 1
(c) both (A) and (B) (d) none
125. The real roots of the equation : 2 x4 - 3x3 — x2 - (c) 2, –4, –1 (d) none of these.
3x + 2 = 0, x  0 are :
(a) (2, 1/2) (b) (3, 1/3)
(c) (2, -1/2 (d) (3, -1/3)
126. The satisfying values of the equation x 3 +x2 -
20x = 0 are :
(a) (1,4, - 5) (b) (2, 4, - 5)
(c) (0, - 4, 5) (d) (0, 4, - 5)
127. The rational root of the equation : 2x3 - x2 - 4x +
2 = 0 is :
(a) 1/2 (b) -1/2
(c) 2 (d) -2

EQUATIONS AND MATRICES 59


8. If 3 times of Ramesh’s age 6 year ago be
subtracted from twice his present age the
HOME WORK-1 result would be equal to his present age. Find
Ramesh’s age
(a) 15 (b) 9
(c) 16 (d) 8
1. If _______, the roots are real and equal. 9. The sum and product of the equation 3x2–2x–
(a) b2 – 4ac= 0 (b) b2 – 4ac> 0 6=0 are
(c) b2 – 4ac<0 (d) b2 – 4ac < 0 (a) 2/3, –2 (b) –2/3, 2
2
2. If b – 4ac < 0 then the roots are _____________. (c) 2, –2/3 (d) –2, –2/3
(a) Real and equal 10. Find the value of m if one of root is –3/2 of the
(b) Imaginary and unequal equation x2+x–m=0
(c) Real and unequal
(a) 1 (b) 0
(d) Irrational and unequal
(c) 3/4 (d) –2
3. If b2– 4ac >0 then the roots are ______ and
______ . 11. The roots of the equation x2 - x +1= 0 are —
(a) Real, Equal (a) Imaginary and unequal
(b) Real, Unequal (b) Real and unequal
(c) Imaginary, Unequal (c) Real and equal
(d) Imaginary, Equal (d) Imaginary and equal
4. If b2–4ac = 0 the roots are ______ and ______.
12. A fraction is such that if the numerator is
(a) Real, Unequal
multiplied by 3 and the denominator is
(b) Real, Equal
reduced by 3, we get 18/11, but if the
(c) Irrational, Unequal
numerator is increased by 8 and the
(d) Rational, Unequal
denominator is doubled, we get 2/5. Then the
5. The denominator of a fraction exceeds the
fraction is
numerator by 7 and if the 2 is added to the
(a) 13/25 (b) 17/25
denominator
(c) 12/25 (d) None
then the fraction becomes . Find the fraction. 13. When a number is added to another number
the total becomes 150 percent of the second
number. What is the ratio between the first
and the second number.
(a) (b)
(a) 1:2 (b) 1:3
(c) 2:3 (d) None
14. The sum of three consecutive even numbers
(c) (d) is 15 less than three fourth of 60. What is
middle number?
6. If a number of which the half is greater than 1/ (a) 15 (b) 10
10th of number by 20 then the number is
(c) 12 (d) None
(a) 50 (b) 40
15. If one-fifth of one-third of one-half of number
(c) 80 (d) None is 15, then number is
2
7. For Equation ax +bx+c=0 the sum of roots are (a) 400 (b) 450
equal to _______
(c) 500 (d) None
(a) ac (b) – b/a
(c) c/a (d) None

60 BUSINESS MATHEMATICS, LOGICAL REASONING & STATISTICS (Paper 3) [CA Foundation]


16. Four-fifths of three-eighths of a number is 240. (a) 0 (b) 1
What is 25% of that number? (c) -1 (d) None
(a) 100 (b) 160
25. If  ,  are the roots of equation x2 -5x+6=0
(c) 1120 (d) 200
and  >  then the equation with roots (  +
17. A student on being asked to multiply 16 / 17 of
a certain fraction made the mistake of dividing  ) and (  -  ) is
the fraction by 16 / 17 and so got an answer
(a) x2 -6x+5=0 (b) 2x2 -6x+5=0
which exceeded the correct answer by 33 / 340
Find the correct answer. (c) 2x2 -5x+6=0 (d) x2 -5x+6=0
(a) 64 / 85 (b) 46 / 58 26. If  ,  are the roots of equation x2-5x+6=0
(c) 64 / 58 (d) None and  >  then the equation with roots (
5 4 2 4  2+  ) and (  +  2) is
18. of of a number is 8 more than of
7 15 5 9 (a) x2 -9x+99=0 (b) x2 -18 x +90=0
of the same number. What is half of that (c) x2 -18x+77=0 (d) None
number?
27. If  ,  are the roots of equation x2 -5x+6=0
(a) 630 (b) 210
(c) 105 (d) None and  >  then the equation with roots
19. Find the number which when multiplied by 36 (   +  +  ) and (   -  -  ) is
is increased by 1050.
(a) x2 -12x+11=0 (b) 2x2 -6x+12=0
(a) 40 (b) 30
(c) x2 -12x+12=0 (d) None
(c) 50 (d) None
x 17 x+y
20. = , what is equal to
x+y 23 x-y 
(a) 11/23 (b) 17/32
(c) 23/11 (d) None
ANSWER KEYS
25 x
21. If 1+ = 1 + , then x is
144 12 1 (a) 2 (b) 3 (b) 4 (b)
(a) 1 (b) 2
(c) 3 (d) None 5 (a) 6 (a) 7 (b) 8 (b)
22. The sum of two numbers is 75 and their
difference is 20. Find the difference of their
9 (a) 10 (c) 11 (a) 12 (c)
squares.
(a) 1500 (b) 1600
(c) 1550 (d) None 13 (a) 14 (b) 15 (b) 16 (d)
23. The sum of two numbers is 13 and the sum of
their squares is 85. Find the numbers. 17 (a) 18 (d) 19 (b) 20 (c)
(a) 7, 6 (b) 8, 10
(c) 5, 4 (d) None 21 (a) 22 (a) 23 (a) 24 (b)

25 (a) 26 (c) 27 (a)


24. Solving equation
we get roots as follows

EQUATIONS AND MATRICES 61


7. Let  and  be the roots of X2 + 7X + 12=0.

HOME WORK-2 2 2 



Then the value of   will be
   
1. The roots of the equation x2 - x +1= 0 are —
49 144 7 12
(a) Imaginary and unequal (a) + (b) +
144 49 12 7
(b) Real and unequal
(c) Real and equal 91
(c)  (d) None
(d) Imaginary and equal 12
2. A number consist of three digit of which the 8. If one root of 52x + 13x + p = 0 be reciprocal of
middle one is zero and the sum of other digits the other then the value of p is
is 9. The number formed by interchanging the (a) –5 b) 5
first and third digits is more than the original (c) 1/5 (d) –1/5
number by 297 find the number?
(a) 306 (b) 309
(c) 603 (d) 307
3. Find the quadratic equation Sum of whose ANSWER KEYS
roots is 3 and the Sum of the cubes of roots is 7
(a) 9x2 - 27x + 20 = 20
(b) 21x2 + 147x +20 = 0 1 (a) 2 (a) 3 (a) 4 (a)

(c) 21x2 - 147x +20 = 0


5 (a) 6 (a) 7 (c)
(d) - 21x2 - 147x +20 = 0
4. Find the quadratic equation given that 5+ 3 8 (b)
is one root
(a) x2 - 10x + 22 = 0 (b) x2 + 10x - 22 = 0
(c) x2 - 10x - 22 = 0 (d) -x2 - 10x + 22 = 0
5. If and ß are the roots of the equation
 
3x2 - 5x + 3 = 0 then the value of 
  is
(a) 7/9 (b) -7/9
(c) 8/9 (d) -8/9
6. When two roots of quadratic equation are
1
 ,  then what will be the quadratic
equation:
2
(a)  x2 - (  +1) X +  = 0
2
(b)  x2 -  x+1 = 0
(c)  x2 - (  2 +1)x + 1 = 0
(d) None of these

62 BUSINESS MATHEMATICS, LOGICAL REASONING & STATISTICS (Paper 3) [CA Foundation]


8. If Aand Bare matrices, then which from the
UNIT : II MATRICES following is true?
(a) A + B  B+A (b) (At)t  A
CLASS WORK (c) AB  BA (d) all are true
9. What is a, if
1. If a matrix has 16 elements; what are the
possible orders it can have 2 3
A=  is a singular matrix?
(a) 2 × 8 ; 8 × 1; 4 × 4; 1 × 16; 16 × 1 4 a
(b) 2 × 8 ; 8 × 2; 4 × 4; 1 × 16; 16 × 1 (a) 5 (b) 6
(c) 2 × 8 ; 8 × 2; 4 × 1; 1 × 16; 16 × 1 (c) 7 (d) 8
(d) 2 × 4 ; 8 × 2; 4 × 4; 1 × 16; 16 × 1
2. Transpose of a rectangular matrix is a  2i 3i 
10. if A =   (i2 = -1) then |A| = ?
(a) rectangular matrix  2i -i 
(b) diagonal matrix (a) 2 (b) 8
(c) square matrix (d) scaler matrix (c) 4 (d) 5
3. Transpose of a row matrix is 11. If
(a) zero matrix
 a11 a12  b11 b12 b13 
(b) diagonal matrix    
(c) column matrix a21 a22  A = b21 b22 b23 
a31 a  b31 b b33 
(d) row matrix  32   32 
4. Two matrices A and B are multiplied to get AB then order of matrix A= ?
if (a) 2 x 2 (b) 2 x 3
(a) both are rectangular (c) 3x 2 (d) 3 x 3
(b) both have same order
(c) no. of columns of A is equal to rows of B  2 -3  1 5  2 5
Using 12-16 Let A =   B=   C=  
(d) no. of rows of A is equal to no. of columns 4 5   6 -7  3 4
of B 12. Find A + B.
5. If |A| = 0, then A is
(a) zero matrix (b) singular matrix
 3 2  3 2
(a) 10 -2  (b)  
(c) non-singular matrix    -10 -2
(d) 0
 2 3  3 -1
6. If A is a symmetric matrix, then At = (c)  -10 -2 (d)  
   -10 -2
(a) A (b) |A|
(c) 0 13. Find A–B.

(d) diagonal matrix 1 2  1 -8 


7. If the order of matrix Ais m × p. And the order (a)   (b)  
 -2 -2   -2 12 
of Bis p × n. Then the order of matrix ABis?
(a) m× n (b) n× m 1 8   1 -8 
(c) n× p (d) m× p (c)  -2 -12  (d)  
   -12 -2 
EQUATIONS AND MATRICES 63
14. 3A – C (a)

 4 -14   4 -14   a2 + b2 + 2ab b2 + c2 - 2bc 


(a)  9 11  (b)  -9 -11   2 2  or
     a + c + 2ac a2 + b2 + 2ab 

 4 -14   2 -3
(c)  -9 11  (d)   (a + b)2 (b - c)2 
  4 5   
 (a + c)2 (a + b)2 
15. AB
(b)
 -16 31  16 31 
(a)  34 -15 (b)    (a - b)2 (b + c)2 
  34 -15
or  
(a + c)2 (a + b)2 
16 31  2 -3
(c) 34 5  (d)  
  4 5   a2 + b2 - 2ab b2 + c2 - 2bc 
16. BA (c)  2 2 
a + c + 2ac a2 + b2 + 2ab 
 22 22   -22 22 
(a)  -16 -53 (b)   (a -b)2 (b - c)2 
   16 -53  
(d) 2 2
 (a - c) (a + b) 
22 -11 22 -33
(c) 16 53  (d)  
  16 53   l m   -p q 
19. n o    r s 
   
 a -b   a b 
17.  b a   -b a 
    l + p m + q 
(a) n + r
 s 
a2 + b2 0 
(a)  2 2 l + p m + q 
 0 a +b 
(b) n - r
 s 
 -a2 - b2 0 
(b)  2 2 l + p m - q 
 0 a +b  (c) n + r
 s 
a2 - b2 0 
(c)  2 2
 l-p m+q 
 0 a +b  (d) 
s 
n + r
a2 -b2 0   a b  a b 
(d)  2 2 20.  -b a    b -a 
 0 a -b     

 a2 + b2 b2 + c2   -2ab -2bc  a2 + b2 0 


18.  2 2 +  (a)  
a +c
 a2 + b2   +2ac +2ab   0 -a2 - b2 

64 BUSINESS MATHEMATICS, LOGICAL REASONING & STATISTICS (Paper 3) [CA Foundation]


a2 + b2 0 
(b)  2 2  x + xy 2x + y2 3x + yz 
 0 -a + b 
(b)  
2 + 3x 4 + 3y 6 + 3z 
 -a2 -b2 0 
(c)  2 2  x + 2xy 2x + y2 12 yz 
 0 -a + b 
(c)  
 2 + 3x 4 + 3y 6 + 3z 

 -a2 - b2 0
(d)    x - xy 2x - y2 3x - yz 
 0 a2 + b2  (d)  
2 + 3x 4 + 3y 6 + 3z 
1
   1 -2 3   1 3 5 
21.  2  x (3 4 5 6)    
 5
  23. 4 5 6 0 2 4
 7 8 9  3 0 5
   
3 4 5 6
 6 8 10 12   10 -1 12   10 -1 28 
(a)      
15 20 25 30  (a)  22 22 70  (b) 
22 -2 70 
 34 37 112   34 -5 112 
   

3 5 4 6  10 1 28   10 1 28 
 6 8 10 12     
(b)   (c)  22 -2 -70  (d) 
22 -2 -70 
12 16 20 24   34 -5 112   34 -5 -112 
   

3 4 5 6  -2 3 
 6 8 10 12   -3 -1 3   
(c)   24.  1 0 2 1 0
   3 1
12 16 20 24   

14 -6  14 -6 
(a)   (b)  
3 4 5 6  4 -5   4 5
 6 8 10 12 
(d)  
24 16 16 12  14 -6   -14 -6 
(c)  -4 5  (d)  
   -4 5 
x y  1 2 3
22. 2 3  x y z  1 2 3 0
    3 1 2   
 2 3 0 1
25. if A=  ,B=
 x + 2xy 3x + y2 3xyz  2 0 4  3 0 1 2
 
(a)  
 2 + 3x 4 + 3y 6 + 3z  Find AB. Does BA exist?

EQUATIONS AND MATRICES 65


(a) AB exists but BA not Exists
 -1 12 11
(b) AB not exists BA Exists  -1 7 8 
(c) Both Ab and BA not Exists  
 0 5 5 
(d) None of these
(b) C 23 = 8, C 32= =5, C 22= 7, C 31 = 0 and AB =
0 3
1
 2   -1 12 11
0 2 2 3  -1 7 8 
26. if A=   ; B = 2 1  
3 2 1 0    0 5 5 
3 0

(a) AB (c) C23 = 8, C32= =-1, C22= 7, C31 = 0 and AB =


 BA
(b) AB= BA  -1 12 11
(c) AB exists BA not exists  -1 7 8 
 
(d) AB not exists BA exists  0 5 5 
0 i  (d) C23 = 8, C32= =-1, C22= 7, C31 = 5 and AB =
27. If A=   ; where i2 = -1
 i 0  -1 12 11
 -1 7 8 
 -1 0  3  0 -i   
(a) A2 =   A = -i 0   0 -5 -5 
 0 1  

 -1 0  3  0 -i 
(b) A2 =   A = -i 0  29. Using matrix Cramers method
 0 -1  
 x=1,  y = -1 ,  z= 1,  = 1, find x, y and z
values
1 0  3  0 -i 
2
(c) A =  A = -i 0  (a) X = 1 , y = –1 and z= -1
 0 -1    (b) X = –1 , y = 1 and z = 1
(c) X = 1 , y = –1 and z= 1
(d) X = –1 , y = –1 and z = 1
 1 0  3  0 -i 
(d) A2 =   A = -i 0  30. x + y + z = 4; 2x + 5y - 2z = 3; x + 7y - 7z = 5 then the
0 1    values of x, y, z using crammers rule
28. F ind the elements C 23 , C 32 , C 31, C 22 in the (a) X = 1 , y = –1 and z = 1
product C=AB. (b) inconsistent
(c) X = 1 , y = –1 and z = 1
2 3 4   1 3 0 (d) none of these
   
 1 2 3  -1 2 1  31. x + y = –1; y + z = 1; z + x = 0
Where A = ,B=
1 1 2   0 0 2 (a) X = –1; y = 0; z = 1
   
(b) X = 1; y = 0; z = 1
(a) C23= 8, C32= =-1, C22= 7, C31 = 5 and AB = (c) X = 1; y = 0; z = –1
(d) X = –1; y = 0; z = –1

66 BUSINESS MATHEMATICS, LOGICAL REASONING & STATISTICS (Paper 3) [CA Foundation]


6 5
32. If A =   , find (A’)’
3 9 
(a) A (b) -A
2
(c) A (d) none of these
33 Chose the correct alternative;

x y   -2 3 
if 2   9   18 I
z p  1 0
(a) X=18 ; z= 9/2 (b) x= 0 , z= -9/2
(c) X= 0 ; z= 9/2 (d) None of these

 0 3 -4 
 -3 0 -5 
34.   is a
 4 4 8 

(a) Symmetric matrix


(b) Null matrix
(c) Skew - symmetric matrix

(d) None of these

6 10 
35. if A =  
3 5 
(a) Is a singular matrix

(b) Non-singular matrix


(c) Identity matrix
(d) Symmetric matrix

LINEAR INEQUALITIES 67
(a) identity or unit matrix
(b) skew symmetric matrix
HOME WORK-1 (c) square matrix
(d) diagonal matrix
1 Mention the type of matrix (vi) Mention the type of matrix
0 1 2  0 -3 -2 
 1 -4 -5 3
(i)   0 -1 
 2 -5 0   
 2 1 0 
(a) symmetric matrix
(a) identity or unit matrix
(b) colum matrix
(b) skew symmetric matrix
(c) row matrix
(c) square matrix
(d) null matrix
(d) diagonal matrix
(ii) Mention the type of matrix
(vii) Mention the type of matrix
4 
2   3 -4 
   -3 4 
 
 6 
(a) identity or unit matrix
(a) symmetric matrix
(b) skew symmetric matrix
(b) colum matrix
(c) row matrix (c) square matrix
(d) null matrix (d) diagonal matrix
(ii) Mention the type of matrix (viii) Mention the type of matrix

0 1 0 1 1 0 0
0 -2 0
(a) symmetric matrix  
(b) colum matrix  0 0 0 
(c) row matrix
(a) identity or unit matrix
(d) null matrix
(iv) Mention the type of matrix (b) skew symmetric matrix
(c) square matrix
0 0 0 0 (d) diagonal matrix
0 0 0 0 (ix) Mention the type of matrix
 
0 0 0 0
3 0 0
 0 0 0 0  2 5 0
 
(a) symmetric matrix  1 2 6 
(b) colum matrix
(a) identity or unit matrix
(c) row matrix
(d) null matrix (b) skew symmetric matrix
(v) Mention the type of matrix (c) lower triangular matrix
(d) diagonal matrix
1 0 0
0 1 0
 
 0 0 1 

68 BUSINESS MATHEMATICS, LOGICAL REASONING & STATISTICS (Paper 3) [CA Foundation]


5. Solve the following equation using the
 1 -1 1  1 2 3  method of matrix inversion:
A =  -3 .2 -1  B = 2 4 6  2x-3y+5=0 3x+y=9
2.    
 -2 1 0  1 2 3  (a) x = 3, y = 2 (b) x = 4, y = 2
find BA (c) x=2,y=3 (d) x=2,y=4
0 0 0   -11 6 -1  6. Find adjoint of A where
0 0 0   -22 12 -2 
(a)   (b)   1 0 7
 0 0 0   -11 6 -1  A =  2 2 5
 
 6 10 14   0 3 6 
10 14 11 
(c)   (d) None
 -3 21 -14 
 7 4 18   -12 6 9 
(a)  
3. Find Adoint of matrix
 6 -3 2 

 5 -6 4   -3 -12 6 
7 4 -3  21 6 -3
  (b)  
2 1 6   -14 9 2 

 2 -2 -4   27 40 2 
 0 -1 1   -48 22 43 1 0 7
(a)   (b)    2 2 5
(c)   (d) None
 -2 1 3   -1 -17 62   0 3 6 
 27 -48 -1  7. A manufacture produces four products A,B,C
 40 22 -17  and D These products are sold in two shops.The
(c)   (d) None monthly sale in the two shops are given below:
 2 43 62 
Products
4. Find Inverse of
A B C D
 1 -2 3  Shop I 800 600 400 100
 2 1 -3 II 600 500 400 200
 
 -1 1 2  If the seling price of the products are respectively
Rs. 10,20,30 AND 40,find the total monthly
 5 -1 3  -7 11 5 revenue of each shop.using matrix multiplication.
1  1
7 5 1 -8 14 5 (a) Shop I=36000
(a) 16   (b) 5  
-1 1 2   -6 13 5 Shop II=48000
(b) Shop I= 36000
 -7 -1 3   5 7 3 Shop II= 36000
1 1 
2 1 -3 -1 5 9  (c) Shop I=48000
(c) 5  (d) 16  
 -1 1 2   3 1 5  Shop II=36000
(d) None

LINEAR INEQUALITIES 69
8. Find the determinant value of the following
matrices.

ANSWER KEYS

1(i) (a) (ii) (b)


(a) 0 (b) 1
(c) -1 (d) None (iii) (c) (iv) (d)

(v) (a) (vi) (b)

(vii) (c) (viii) (d)

 (ix) (c) 2 (b)

3 (b) 4 (d)

5 (c) 6 (a)

7 (b) 8 (a)

70 BUSINESS MATHEMATICS, LOGICAL REASONING & STATISTICS (Paper 3) [CA Foundation]


5 6
HOME WORK-2 6. If A= 10 12  then find A3
 

1445 3468 
 1 -1 1  1 2 3  (a) 2890 1734 
A =  -3 .2 -1  B = 2 4 6   
1.    
 -2 1 0  1 2 3  2890 3468 
find BA (b) 1445 1734 
 
0 0 0   -11 6 -1 
0 0 0   -22 12 -2  1734 3468 
(a)   (b)   (c) 2890 1445
 
 0 0 0   -11 6 -1 
1445 2890
1734 
 6 10 14  (d) 1734
10 14 11   2890 3468 
(c)   (d) None
 7 4 18  20 6  4 1 
7. If AB=   , A=   find B
2. Transpose of a rectangular Matrix is  7 3 7 2
(a) Rectangular Matrix
(b) Diagonal Matrix
(c) Square Matrix  33 9   9 33 
(d) Scalar Matrix (a)  -112 - 30  (b)  -112 - 30 
   
 -5 2 
3. If A =  1 -3 , then adj A is  33 9   9 33 
  (c)  -30 - 112  (d)  -10 - 112 
   
 -3 -2  5 1
(a)  -1 -5 (b) 2 3
    7 8 
8. Find value of  
6 - 3
 3 -2  3 2 
(c)  -1 5  (d) 1 5 (a) 96 (b) -96
   
(c) 69 (d) -69
5 x
T
4. If A  y 0  = and A= A , then x -1 1 

9. Find value of  2 x+1
(a) x = 0, y = 5 (b) x = y x 
(c) x+y=5 (d) None
(a) 1 (b) -1
5. Let AT be the transpose of matrix A having order
(c) x2-1 (d) x2+1
m x n, then AT A is a matrix of order
(a) nxn (b) mxm
(c) mxn (d) nxm

LINEAR INEQUALITIES 71
1 3 4 1 3 1
5 4 2  4
10. Find the value of   14. Find the co-factor of the matrix 2 4 
 5 1 0  3 3 1 

(a) -32 (b) -1


 -8 10 - 6   -8 10 6 
(c) 16 (d) -16  0 -2 6   
(a)   (b)  0 6 - 2 
1 4   3 0  8 - 2 - 2   8 - 5 2 
11. If A=   and B =   then find AB
3 5 4 1 
 -9 1 6   -8 3 6 
(a) 21 (b) -21 10 4 - 2   
(c) 12 (d) 11 (c)   (d)  4 5 - 1
 7 - 5 2   1 - 5 7 
1 2 3
 
12. If A =  4 5 6  then find adj A
 7 8 9  ANSWER KEYS
 -3 6 - 3  3 8 3
 6 - 12 3    1 (b) 2 (a) 3 (a)
(a)   (b) 6 12 3
 -3 3 - 3  3 3 3
4 (b) 5 (a) 6 (d)

7 (a) 8 (d) 9 (b)


 -3 6 - 3  -3 6 - 3
 6 - 12 6   
(c)   (d)  8 - 12 8  10 (a) 11 (b) 12 (c)
 -3 6 - 3  -3 6 - 3
13 (c) 14 (a)

 -2 4 
13. Find the inverse of the matrix  3 - 5
 

5 2   2 5 
2
(a)  3  (b)  3 2 
 1 2   2 1 

5 2  2 5 
 2   2
(c)  3 1 (d) 1 3 
 2   2

72 BUSINESS MATHEMATICS, LOGICAL REASONING & STATISTICS (Paper 3) [CA Foundation]


CHAPTER-3
LINEAR INEQUALITY

INTRODUCTION
PRACTICAL STEPS INVOLVED IN DRAWING A GRAPH OF INEQUALITY
Step 1 - Write down the inequality as an equality.
Step 2 - Form a table of values of the equation ax + by = c.
Step 3 - Plot the points of the table obtained in Step 2 in XOY plane and join them.
(a) If the inequality is < or > the line drawn should be thick.
(b) If the inequality is < or >, the line drawn should be dotted.
(c) This line will divide XOY plane in two regions.
Step 4 - Determine the region in which the given inequation is satisfied. This is done by taking
any arbitrary point (x1, y1) in one of the regions. Generally, (x1, y1) is taken as (0, 0).
Substitute its corrdinates in the given inequation.
(i) If the given inequation is satisfied by (x1, y1), then the region containing this
point is the desired region.
(ii) If the point (x1, y1) does not satisfy the given inequation, then the region not
containing this point is the desired region.
Step 5 - Shade the desired region.

Tutorial Notes
1. Thick line is a part of the solution set but dotted line is not.
2. The suitable point T (also called Testing Point) is generally taken be the origin (0, 0).
3. For more than one inequations repeat the above procedure, for each of these.

LINEAR INEQUALITIES 73
CLASS WORK

Q-1 An employer recruits experienced (x) and fresh workmen (y) for his firm under the condition that he
cannot employ more than 9 people. x and y can be related by the inequality
(a) x + y  9 (b) x + y  9 x  0, y  0
(c) x + y  9 x  0, y  0 (d) none of these
Q-2 On the average experienced person does 5 units of work while a fresh one 3 units of work daily but the
employer has to maintain an output of at least 30 units of work per day. This situation can be expressed
as
(a) 5x + 3y  30 (b) 5x + 3y >30
(c) 5x+3y  30 x  0, y  0 (d) none of these
Q-3 The rules and regulations demand that the employer should employ not more than 5 experienced
hands to 1 fresh one and this fact can be expressed as

y
(a) y  x/5
5 (b) 5y  x (c) x (d) none of these
5
Q-4 The union however forbids him to employ less than 2 experienced person to each fresh person. This
situation can be expressed as
(a) x  y/2
2 (b) y  x/2
2 (c) y  x /2
2 (d) x > 2y
Q-5 The graph to express the inequality x + y  9 is

(a) (b)

74 BUSINESS MATHEMATICS, LOGICAL REASONING & STATISTICS (Paper 3) [CA Foundation]


(c) (d) none of these

Q-6 The graph to express the inequality 5x + 3y  30 is

(a) (b)

(c) (d) none of these

LINEAR INEQUALITIES 75
1
Q-7 The graph to express the inequality y    x is indicated by
2

(a) (b)

(c) (d)

76 BUSINESS MATHEMATICS, LOGICAL REASONING & STATISTICS (Paper 3) [CA Foundation]


Q-8

L1 : 5x + 3y = 30 L2 : x+y = 9 L3 : y = x/3 L4 : y = x/2


The common region (shaded part) shown in the diagram refers to
(a) 5x + 3y  30 (b) 5x + 3y  30 (c) 5x + 3y  30 (d) 5x + 3y > 30, x + y  9
x+y  9 x+y  9 x +y <9
y  1/5 x y  x/33 y  x/3
3 y 9
y  x/2
2 y  x/2
2 y  x/2
2 y  x/2
2
x  0,y  0 x  0,y  0 x  0,y  0
(e) None of these
Q-9 A dietitian wishes to mix together two kinds of food so that the vitamin content of the mixture is at
least 9 units of vitamin A, 7 units of vitamin B, 10 units of vitamin C and 12 units of vitamin D. The
vitamin content per Kg. of each food is shown below:
A B C D
Food I : 2 1 1 2
Food II: 1 1 2 3
Assuming x units of food I is to be mixed with y units of food II the situation can be expressed as
(a) 2x + y  9 (b) 2x + y  30 (c) 2x + y  9 (d) 2x + y  9
x + y 7 x+y  7 x + y 7 x+y  7
x 2y  10 x + 2y  10 x + y  10 x + 2y  10
2x +3y  12 x + 3y  12 x +3y  12 2x + 3y  12
x > 0,y>0 x  0, y  0.

LINEAR INEQUALITIES 77
Q-10 Graphs of the inequations are drawn below :

L1 : 2x +y = 9 L2 : x + y = 7 L3 : x+2y= 10 L4 : x + 3y = 12
The common region (shaded part) indicated on the diagram is expressed by the set of inequalities
(a) 2x + y  9 (b) 2x + y  9 (c) 2x + y  9 (d) none of these
x + y 7 x + y 7 x + y 7
X + y  10 x + 2 y  10 x + 2y  10
x +3 y  12 x + 3y  12 x + 3y  12
x  0, y  0
Q-11 The common region satisfied by the inequalities
L1: 3x + y  6, L2: x + y  4, L3: x +3y  6, and L4: x + y < 6 is indicated by

78 BUSINESS MATHEMATICS, LOGICAL REASONING & STATISTICS (Paper 3) [CA Foundation]


(a) (b)

(c) (d) None of these

Q-12 The region indicated by the shading in the graph is expressed by inequalities

LINEAR INEQUALITIES 79
(a) x1 + x 2  2 (b) x1 + x2  2 (c) x1 + x2  2 (d) x1 + x2  2
2x1 + 2x2  8 x2x1 +x2  4 2x1 + 2x2  8 2x1 + 2x2>8
x1 „d 0 , x2  0
Q-13 The inequalities x1  0, x2  0, are represented by one of the graphs shown below:

(a) (b)

(c) (d)

80 BUSINESS MATHEMATICS, LOGICAL REASONING & STATISTICS (Paper 3) [CA Foundation]


Q-14

The region is expressed as


(a) x1 – x2  1 (b) x1 + x2  1 (c) x1 + x2  1 (d) none of these

Q-15 The inequality –x1 + 2x2  0 is indicated on the graph as

(a) (b)

(c) (d) none of these

LINEAR INEQUALITIES 81
Q-16

The common region indicated on the graph is expressed by the set of five inequalities
(a) L1 : x1  0 (b) L1 : x1  0 (c) L1 : x1  0 (d) None of these
L2 : x2  0 L2: x2  0 L2: x2  0
L3 : x1 + x2  1 L3 : x1+x2  1 L3 : x1+x2  1
L4 : x1 -x2  1 L4 : x1-x2  1 L4: x1-x2  1
L5 : -x1+ 2x2  0 L5 : -x1+ 2x2  0 L5 : -x1+ 2x2  0
Q-17 A firm makes two types of products : Type A and Type B. The profit on product A is Nu. 20 each and that
on product B is Nu. 30 each. Both types are processed on three machines M1, M2 and M3. The time
required in hours by each product and total time available in hours per week on each machine are as
follows:
Machine Product A Product B Available Time
M1 3 3 36
M2 5 2 50
M3 2 6 60
The constraints can be formulated taking x1 = number of units A and x2 = number of unit of B as
(a) x1 + x2  12 (b) 3x1 + 3x2  36 (c) 3x1 + 3x2  36 (d) none of these
5x1 +2x2  50 5x1 + 2x2  50 5x1 +2x2  50
2x1 +6x2  60 2x1 +6x2  60 2x1 +6x2  60
x1  0,x2  0 x1  0, x2  0
Q-18 The set of inequalities L1: x1 + x2  12, L2: 5x1 + 2x2  50, L3: x1 + 3x2  30, x1  0, and x2  0 is represented
by

82 BUSINESS MATHEMATICS, LOGICAL REASONING & STATISTICS (Paper 3) [CA Foundation]


(a)

(b)

LINEAR INEQUALITIES 83
(c)

Q-19 The common region satisfying the set of inequalities x  0, y  0, L1: x+y  5, L2: x +2y  8 and L3: 4x
+3y  12 is indicated by

(a) (b)

(c) (d) none of these

84 BUSINESS MATHEMATICS, LOGICAL REASONING & STATISTICS (Paper 3) [CA Foundation]


HOME WORK - 1

Q-1 Graph of the Inequation are drawn below:

L1 :15 x + 6y = 300, L2 : 5x + 4y= 120,L3 : x + 2y = 50.


The common region (shaded part) shown in the figure 8. 13 refers to the unequalities to :
(a) 15 x + 6y < 300 (b) 15x + 6y > 300 (c) 15 x + 6y < 300
5x + 4y < 120 5x +4y < 120 5x+4y > 120
x + 2y < 50 x + 2y < 50 x + 2y < 50
x > 0, y > 0. x > 0,y > 0 x > 0, y > 0
(d) none of these.

LINEAR INEQUALITIES 85
Q-2 Graphs of Inequations are drawn below:
The shaded part shown in the figure 8.14 in the diagram refers to the inequalities.

<

86 BUSINESS MATHEMATICS, LOGICAL REASONING & STATISTICS (Paper 3) [CA Foundation]


Q-3 Graphs of Inequations are drawn below :
L1 : 5x + 3y =30 ; L2 : x + y = 9; L3 : y = x/3, L4: y = x/2.

The common region (shaded part) shown in the diagram refers to the inequalities

(a) 5x + 3y < 30 (b) 5x + 3y > 30 (c) 5x + 3y > 30 (d) 5x + 3y > 30

x+y<9 x+y<9 x+y>9 x+y<9

y < 1/5 x y > x/3 y < x/3 y > x/3

y < x/2 y < x/2 y > x/2 y > x/2

x > 0, y > 0 x > 0, y > 0 x > 0, y > 0 x > 0, y > 0.

LINEAR INEQUALITIES 87
Q-4 Graphs of Inequations are drawn below :

L1 : x + 2y = 1000 ; L2 x + y = 800 ; L3 : x = 400 ; L4 y = 700:

The shaded part shown in the diagram refers to:

(a) x + 2y < 1000 (b) x + 2y > 1000 (c) x + 2y < 100

x + y < 800 x + y < 800 x + y > 800

x < 400 x < 400 x < 400

y < 700 y < 700 y < 700

x > 0,y > 0. x > 0,y > 0 x > 0,y > 0

(d) none of these.

88 BUSINESS MATHEMATICS, LOGICAL REASONING & STATISTICS (Paper 3) [CA Foundation]


Q-5 Graphs of the inequations are drawn below:

L1 : 2x + y = 9; L2 x + y = 7 ; L3 : x + 2y = 10 ; L4 x + 3y = 12.

The common region (shaded part) indicated on the diagram is expressed by the set of inequalities:
(a) 2x + y > 9 (b) 2x + y > 9 (c) 2x + y > 9
x+y> 7 x+y< 7 x+y>7
x + 2y > 10 x + 2y > 10 x + 2y > 10
x + 3y > 12 x + 3y > 12 x + 3y > 12
x > 0, y > 0
(d) none of these.

LINEAR INEQUALITIES 89
Q-6

90 BUSINESS MATHEMATICS, LOGICAL REASONING & STATISTICS (Paper 3) [CA Foundation]


Q-7 The common region satisfied by the inequalities L1 : 3x + y > 6, L2 : x + y > 4, L3 : x + 3y > 6, and L4 : x + y <
6 is indicated by:

(a) (b)

(c) (d) None of these.

Q-8 Graphs of the Inequations are drawn below :

LINEAR INEQUALITIES 91

L1 : 3x + 3y = 36; L2 = 5x + 2y = 50 ; L3 : 2x + 6y = 60 ; x > 0, y > 0.


The shaded region M O D E F shown in the figure 8.20 refers to the inequalities:
(a) 3x + 3y < 36 (b) 3x + 3y < 36 (c) 3x + 3y > 36
2x + 6y > 60 2x + 6y < 60 2x + 6y < 60
5x + 2y < 50 5x + 2y < 50 5x + 2y > 0
x > 0, y > 0 x > 0, y > 0 x > 0, y > 0
(d) none of the above.

Q-9 The inequality - x + 2y < 0 is indicated on the graph as :

Q-10 Graphs of the inequations are drawn below :

The common region indicated on the graph is expressed by the set of five inequalities given

92 BUSINESS MATHEMATICS, LOGICAL REASONING & STATISTICS (Paper 3) [CA Foundation]


(a) L1 : x1 > 0 (b) L1 : x1 < 0 (c) L1 : x1 > 0
L2 : x2 > 0 L2 : x2 > 0 L2 : x2 > 0
L3 : x1+ x2 < 1 L3 : x1+ x2 > 1 L3 : x1+ x2 > 1
L4 : x1- x2 > 1 L4 : x1- x2 > 1 L4 : x1- x2 < 1
L5 : - x1+ 2x2 < 0 L5 : - x1+ 2x2 < 0 L5 : - x1+ 2x2 < 0.
(d) None of these.
Q-11 A scooter company manufactures scooters of two models A and B. Model A requires 15 man-hours for
assembly, 5 man-hours for painting and finishing and 1 man-hour for checking and testing. Model B
requires 6 man-hours for assembly, 4 man-hours for painting and finishing and 2 man-hours for checking
and testing. There are 300 man-hours available in the assembly shop, 120 man-hours in painting and
finishing shop and 50 man hours are available in checking and testing division.
The constraints can be formulated by taking x = number of scooters of model A; and y = number of
scooters of model B as :
(a) 15x + 6 y < 300 (b) 15x + 6y > 300 (c) 15x + 16y < 300
x + 2y < 50 x + 2y < 50 x + 2y < 50
5x + 4y < 120 5x + 4y < 120 5x + 4y > 120
x>0,y>0 x > 0, y > 0 x > 0, y > 0.
(d) None of the these.
Q-12 A company produces two types of leather belts, say A and B. Belt A is of superior quality and belt B is
of lower quality. Each belt of type A requires twice as much time as required by a belt of type B. If all
belts were of type B, the company could produce 1000 belts per day. But the supply of leather is
sufficient only for 800 belts per day. Belt A requires fancy buckles and only 400 fancy buckles are
available per day. For belt of type B only 700 buckles are available per day.
Constraints can be formulated by assuming that the company produce x units of belt A and y units of
belt B as :

(a) 2x + y < 1000 (b) 2x + y < 1000 (c) 2x + y > 1000


x + y > 800 x + y < 800 x + y < 800
x < 400; y < 700 x < 400; y < 700 x < 400; y < 700
x>0,y>0 x > 0, y > 0 x > 0, y > 0.
(d) None of the these.
Q-13 A man makes two types of furniture : chairs and tables. Profits are ` 20 per chair and ` 30 per table. Both
the products are processed on two machines M1 and M2. The time required for each product in hours
and total time available in hours for each machine are as follows:
Machine Chair Table Available Time
M1 3 3 36
M2 5 2 50
Constraints can be formulated by taking x = the number of chairs and y = the number of tables produced as

LINEAR INEQUALITIES 93
:
(a) x + y < 12 (b) x + y < 12, (c) x + y > 12
5x+2y > 50 5x+2y < 50 5x + 2y < 50
x > 0;y > 0 x > 0;y > 0 x > 0;y > 0
(d) None of these.
Q-14 A firm makes two types of products : Type A and Type B. The profit on product A is ` 20 each and that on
product B is ` 30 each. Both types are processed on three machines M1, M2 and M3. The time required in
hours by each product and total available in hours per week on each machine are as follows :
Machine Product A Product B Available Time
M1 3 3 36
M2 5 2 50
M3 2 6 60
The constraints can be formulated by taking x1 = number of units A and x2 = number o of B as :
(a) x1 + x2 < 12 (b) 3x1 + 3x2 > 36, (c) 3x1 + 3x2 < 36,
5x1 + 2 x2 > 50 5x1 + 2 x2 < 50 5x1 + 2 x2 < 50
2x1 + 6 x2 > 60 2x1 + 6 x2 > 60 2x1 + 6 x2 < 60
x1 > 0; x2 > 0 x1 > 0; x2 > 0 x1 > 0; x2 > 0
(d) None of these
Q-15 A factory manufactures two articles X and Y. To manufacture article X, a certain machine has to be
worked for 1.5 hours and in addition, a craftsman has to work for 2 hours. To manufacture the article Y,
the machine has to be worked for 2.5 hours and in addition, the craftsman has to work for 1.5 hours. In
a week, the factory can avail of 80 hours of machine and 70 hours of the craftsman’s time. Let x units of
article X and y units of article Y be produced, then the constrains are :
(a) 1.5x + 2.5y > 80 (b) 1.5x + 2.5y < 80 (c) 1.5x + 2.5y < 8
2x + 1.5y < 70 2x + 1.5y < 70 2x + 1.5y > 70
x > 0;y > 0 x > 0;y > 0 x > 0;y > 0
(d) None of these.
Q-16 Suppose a man needs a minimum of 50 units of carbohydrate, 40 units of proteins per month for good
health. He is taking food at two places, viz., A and B, food at A contains 4 and 5 units of carbohydrates
and proteins respectively. Food at B contains, and 3 units of carbohydrates and 3 units of proteins
respectively. Express this in the form of a linear inequalities assuming the man is keeping good health.
(a) 4x1 + 3x2 > 50 (b) 4x1 + 3x2 > 50 (c) 4x1 + 3x2 < 50
5x1 + 3x2 > 40 5x1 + 3x2 < 40 5x1 + 3x2 > 40
x1 > 0; x2 > 0 x > 0; x2 > 0 x1 > 0; x2 > 0
(d) None of these.

94 BUSINESS MATHEMATICS, LOGICAL REASONING & STATISTICS (Paper 3) [CA Foundation]


Q-17 A dietitian wishes to mix together two Kinds of food so that the vitamin contents of the mixture is at
least 9 units of vitamin A, 7 units of vitamin B, 10 units of vitamin C and 12 units of vitamin D. The
vitamin content per Kg. of each food is shown below

A B C D
Food I : 2 1 1 2
Food II : 1 1 2 3
Assuming x units of food I is to be mixed with y units of food II the situation can be expressed as :
(a) 2x + y < 9 (b) 2x + y > 30 (c) 2x + y > 9 (d) 2x + y > 9
x+y < 7 x+y < 7 x+y > 7 x+y > 7
x + 2y < 10 x + 2y >10 x + y < 10 x + 2y > 10
2x + 3y < 12 2x + 3y > 12 x + 3y > 12 2x + 3y > 12
x > 0; y > 0 x > 0; y > 0 x > 0; y > 0 x > 0; y > 0
Q-18 A small manufacturing firm produces two types of gadgets A and B. which are first processed in the
foundary, and then sent to another machine for finishing. The number of man-hours of labour required
in each shop for the production of each unit of A and B, and the number of man-hours for the firm
available per week are as follows :
Foundary Machines-shop
Gadget A 10 5
Gadget B 6 4
Firms capacity per
Week in man hours 1000 600

Let the firm manufacture x units of A and y units of B. The consraints are:
(a) 10x + 6y < 1000 (b) 10x + 6y < 1000 (c) 10x + 6y > 1000
5x + 4y < 600 5x + 4y > 600 5x + 4y < 600
x > 0; y > 0 x > 0; y > 0 x > 0; y > 0.
(d) None of these.
Q-19 Vitamins A and B are found in food F1 and F2. One unit of F1 contains 20 units of vitamin A and 30 units
of vitamin B. One unit of food F2 contains 60 units of A and 40 units of B. Costs per unit of F1 and F2 are
` 3 and ` 4 respectively. The minimum daily requirement of vitamin A and B is 80 and 100 units
respectively. Problem is to determine the mixture of food F1 and F2, which meets the requirement at
minimum cost by assuming that x1 units of food F1 and X2 units of food F2 are required to fulfill the need
of vitamins. The constraints are :
(a) 20x1 + 60x2 > 80 (b) 20x1 + 60x2 < 80 (c) 20x1 + 60x2 > 80
30x1 + 40x2 > 100 30x1 + 40x2 > 100 30x1 + 40x2 < 100
x1 > 0; x2 > 0 x1 > 0; x2 > 0 x1 > 0; x2 > 0.
(d) None of these.

TIME VALUE OF MONEY 95


Q-20 On solving the inequalities 6x + y > 18, x + 4y > 12, 2x + y > 10,x > 0, y > 0, we get the following situation
(a) (0, 18), (12, 0), (84/23, 54/23), (4, 2) and (7, 6)
(b) (3, 0), (0, 3), (84/23, 54/23), (4, 2) and (7, 6)
(c) (5, 0), (0, 10), (84/23. 54/23), (4, 2) and (7, 6)
(d) (0, 18), (12, 0), (4, 2), (0, 0) and (7, 6).
Q-21 The graphs of the inequalities is given below :

L1: x+y = 30, L2: x - y = 0, L3: x = 20, L4 : y = 12, L5 : y = 3


The shaded region of figure 8.23 belongs to
(a) x +y < 30 (b) x + y > 30 (c) x +y < 30
x-y>0 x-y >0 x -y < 0
0 < x < 20 0 < x < 20 0 < x < 20
3 < y < 12 3 < y < 12 3 < y < 12
x>0;y>0 x > 0, y > 0 x > 0, y > 0
(d) None of the

96 BUSINESS MATHEMATICS, LOGICAL REASONING & STATISTICS (Paper 3) [CA Foundation]


Q-22 Graphs of four inequalities are given below :

L1: x+2y =3 L2 : x + y = 4 L3: x = 5/2 L4 : y = 3/2


The shaded region ABC belongs to the constraints :
(a) x + 2y > 3 (b) x + 2y < 3 (c) x + 2y < 3
x+y<4 x+y <4 x +y > 4
0 < x < 5/2 0 < x < 5/2 0 < x < 5/2
0 < y < 3/2 0 < y < 3/2 0 < y < 3/2
x>0;y>0 x > 0, y > 0 x > 0, y > 0
(d) None of the
Q-23 Graph of some constraints are given by the figure 8.25 :

The shaded region OAEC belongs to the constraint ]

(a) 3x + y < 9 (b) 3x + y < 9 (c) 3x + y > 9

x + 2y > 8 x + 2y < 8 x + 2y < 8

x > 0; y > 0 x > 0; y > 0 x > 0; y > 0

(d) None of these.

TIME VALUE OF MONEY 97


Q-24 Graph of some contraints are given by the figure 8.26 :

L1: x + 2y = 10; L2=x + y =6; L3: x - y = 2; L4 : x - 2y = 1


The shaded region belongs to the constraints given by :
(a) x + 2y < 10 (b) x + 2y < 10 (c) x + 2y < 10

98 BUSINESS MATHEMATICS, LOGICAL REASONING & STATISTICS (Paper 3) [CA Foundation]


x+y>6 x+y<6 x+y<6
x-y <2 x-y <2 x -y> 2
x - 2y < 1 x - 2y < 1 x - 2y < 1
x > 0; y > 0 x > 0; y > 0 x > 0; y > 0
(d) none of these.
Q-25 On solving the inequalities 2x + 5y < 20, 3x + 2y < 12, x > 0, y > 0, we get the following situation:
(a) (0, 0), (0, 4), (4,0) and (20/11, 36/11) (b) (0,0), (10,0), (0.6) and (20/11, 36/11)
(c) (0, 0), (0, 4), (5, 0) and (2, 3) (d) (0, 0), (10, 0), (0, 6) and (2, 3)
Q-26 On the average experienced person does 5 units of work while a fresh one 3 units of work daily but the
employer has to maintain an output of at least 30 units of work per day. This situation can be expressed
as
(a) 5x + 3y < 30 (b) 5x + 3y >30 (c) 5x + 3y > 30 (d) None
Q-27 An employer recruits experienced (x) and fresh workmen (y) for his firm under the condition that he
cannot employ more than 9 people. x and y can be related by the inequality
(a) x + y 9 (b) x + y < 9 (c) x + y > 9 (d) None
Q-28 Mr. A plans to invest upto Rs.30,000 in two stocks X and Y. Stock X(x) is priced at Rs.175 and Stock Y(y) at
Rs.95 per share. This can be shown by
(a) 175x+95y < 30,000 (b) 175x+95y > 30,000
(c) 175x+95y = 30,000 (d) None
Q-29 A firm plans purchase hens (x) for its canteen. There cannot be more than 20 hens. This can be shown
by
(a) x < 20 (b) x = 20 (c) x > 20 (d) None
Q-30 A dealer has only Rs.5760 to invest in fans (x) and sewing machines (y). The cost per unit of fans and
sewing machine is Rs.360 and Rs.240 respectively. This can be shown by
(a) 360x + 240y > 5760 (b) 360x + 240y < 5760
(c) 360x + 240y = 5760 (d) None
Q-31 In a class of boys (x) and girls (y) the maximum seating capacity is 360. This can be shown by
(a) x + y < 360 (b) x + y > 360
(c) x + y  360 (d) None
Q-32 An employer recruits experienced and fresh workmen for his firm under the condition that he cannot
employ more than 12 people. It can be related by the inequality
(a) x + y  12 (b) x + y < 12
(c) x + y > 12 (d) None
Q-33 The inequalities x > 0, y > 0 indicates
(a) First quadrant (b) Second quadrant
(c) Third quadrant (d) Fourth quadrant
Q-34 The inequalities x < 0, y > 0 indicates ______
(a) Third quadrant (b) First quadrant
(c) Second quadrant (d) Fourth quadrant

TIME VALUE OF MONEY 99


Q-35 The inequalities x < 0, y < 0 represents _____
(a) First quadrant (b) Second quadrant
(c) Third quadrant (d) Fourth quadrant
Q-36 The inequalities x > 0, y < 0 represents _________
(a) First quadrant (b) Second quadrant
(c) Third quadrant (d) Fourth quadrant

ANSWER KEYS

1. (a) 2. (c) 3. (b) 4. (a) 5. (c) 6. (b) 7. (a)


8. (b) 9. (a) 10. (c) 11. (a) 12. (b) 13. (b) 14. (c)
15. (b) 16. (a) 17. (d) 18. (a) 19. (a) 20. (a) 21. (a)
22. (a) 23. (b) 24. (b) 25. (a) 26. (c) 27. (b) 28. (a)
29. (a) 30 (b) 31. (a) 32. (b) 33. (a) 34 (c) 35. (c)
36. (d)

---0---0---0---

100 BUSINESS MATHEMATICS, LOGICAL REASONING & STATISTICS (Paper 3) [CA Foundation]
HOME WORK - 2

(b)
1. -6X < -18 implies-
(a) x < 3 (b) x>3
(c) X = 0 (d) x = 3
2. A fertilizer company produces two types of
fertilizers called Grade I and Grade II. Each of
these types is processed through two critical
chemical plant units.
Plant A has maximum 120 hrs available in a
week and Plant B has maximum of 180 hrs
available in a week.
Manufacturing one bag of grade I fertilizer (c)
requires 6 hours in Plant A and 4 hours in Plant
B.
Manufacturing one bag of Grade II fertilizer
requires 3 hrs in Plant A and 10 hours in Plant
B.
(a) 6x + 3y < 120, 4 x +10 y = 180
(b) 6x + 3y = 120, 4 x 10 y > 180
(c) 6x + 3y < 120 , 4x + 10 y < 180
(d) 6x + 3y < 120, 4x + 10 y < 180
3. The inequalities
X1 + 2X2 < 5, X1+ X2 > 1, X1 > 0,X2 > 0 (d)
represents the region

(a)

TIME VALUE of Money 101


4. By lines x+ y = 6, 2x-y = 2, the common region
shown is the diagram refers to

(c)

(d) None of these


6. Given conditions x + y > 5, x + y < 5, 0 < x < 4 and
0< x> 2

(a) x + y > 6 , 2x- y < 2, x > 0, y > 0


(b) x + y < 6 , 2x-y < 2, x > 0, y > 0
(c) x + y < 6, 2x-y > 2, x > 0, y > 0
(d) None of these
5. Which of the following graph represents the
then the commonregion under these
in equality x + y ≤ 6 is conditions is
(a) ECDE (b) EOABCE
(c) Line segment CD
(d) Line segment BC
7. The Solution of x in the equality
(a) 8x+6 < 12x+14 is
(a) (-2,2) (b) (0, -2)
(c) (2,  ) (d) (-2,  )

ANSWER KEY

1. (b) 2. (c) 3. (a)


4. (b) 5. (b) 6. (b)
(b)
7. (d)

102 BUSINESS MATHEMATICS, LOGICAL REASONING & STATISTICS (Paper 3) [CA Foundation]
CHAPTER-4
MATHEMATICS OF FINANCE (Time Value of Money)

INTRODUCTION
All money dealings are associated with three factors, i.e., Principal ; Rate of Interest and Time. Computations
of simple interest, compound interest, present and future values of annuities and nominal and effective
rate of interest etc. are directly based on the above three factors. These computations are known as
mathematics of finance. We give below a flow chart for Mathematics of Finance.

Mathematics of Finance
Calculation of

  
Interest Value of Annuities Rate of Interest
  
     
Simple Compound Present Future Nominal Effective
Interest Interest Value Value Rate Rate

KINDS OF INTERESTS
There are two kinds of interests
(i) Simple Interest (ii) Compound Interest
Simple Interest: When the interest is calculated only on the principal initially invested, for each year of its
use, it is called simple interest. In other words, if the interest is calculated uniformly on the original principal
throughout the loan period, it is called a simple interest.
Compound Interest: If a borrower does not pay the interest at the end of the prescribed period as in the case
of simple interest but adds it to the principal and then pays interest at the same rate on the new principal
during the next period and so on for further periods, the interest thus calculated is called compound interest.
The period at which interest is added is usually quarterly, half-yearly or yearly.

TIME VALUE of Money 103


 Simple Interest
Points to be considered in Simple Interest
1. SI is always calculated on Principal Amount
2. Every Year the Amount of SI will Remain Same
3. If during the period, Any Repayment of loan is made then first payment will be Adjusted against
Interest will be paid.
4. Interest on interest is neither allowed and interest on interest will not be paid.

Loan 1,00,000  Principal


+
+ 1st Year Interest @ 10 %
Amount to be paid at 10,000  Interest the end of 1st Year
the end of first year 1,10,000  Amount
+ 2nd Year Interest @ 10 % 10,000
Amount to be paid at
the end of 2nd Year 1,20,000
- loan Paid 50,000
70,000
+ 3rd Year Interest @ 10 % 7,000
Amount to be paid at ______
the end of 3rd Year 77,000

 Compound interest
Points to be considered in Compound Interest
1. CI is always calculated on the opening balance of each year
2. Every Year the Amount of CI will go on increasing
3. No Bifurcation of payment is required
4. Interest on interest is allowed and interest on interest will be paid.

104 BUSINESS MATHEMATICS, LOGICAL REASONING & STATISTICS (Paper 3) [CA Foundation]
Loan 1,00,000  Principal
+
+ 1st Year Interest @ 10 %
Amount to be paid at 10,000  Interest the end of 1st Year
the end of first year 1,10,000  Amount
+ 2nd Year Interest @ 10 % 11,000
Amount to be paid at ________
the end of 2nd Year 1,21,000
- loan Paid 50,000
71,000
+ 3rd Year Interest @ 10 % 7,000
Amount to be paid at ______
the end of 3rd Year 78,100
 Simple Interest
PRN
 I= ; P = Principal amount = sum = Present value
100
R = Rate of interest
N = number of Year
I = Interest
A = Amount = Accumulated amount
= Future value
= Final value
 A = P + I
 I = A - P
eg. P = 1,00,000 R = 12% N = 3%

PRN 1, 00, 000 × 12 × 3


I = = = 36,000
100 100
A = P+I = 1,00,000 + 36,000 = 1,36,000

PRN
 A = P+I = P+
100

 RN 
A = P 1+ 
 100 

TIME VALUE of Money 105


 12  3  
A = 1,00,000 1 + 
 100 

A = 1,36,000
I = A - P = 1,36,000- 1,00,000
I = 36,000

eg. N1 = 5 R1 = 5 P = 1, 00,000

N2 = 3 R2 = 15 % A =P

N3 = 2 R3 = 20% I=P

 R N +R N +R N 
 A = P 1 + 1 1 2 2 3 3 
 100 

 10  5  + 15  3  + 20  2  
A = P 1 + 
 100 
A = 2,35,000
I = A-P
I = 2,35,000-1,00,000
I = 1,35,000

P R1N1 P R2N2 P R3N3


or I = + =
100 100 100

 R N +R N +R N 
= P 1 1 2 2 3 3 
 100 

 50 + 45 + 40 
= 1, 00, 000  
 100 
I = 1,35,000
A = P + I = 1,00,000 + 1,35,000 = 2,35,000

106 BUSINESS MATHEMATICS, LOGICAL REASONING & STATISTICS (Paper 3) [CA Foundation]
COMPOUND INTEREST
n
 A = P i + r 

R
r=
100
R = 5%  r = 0.05
R = 12%  r = 0.12
R = 3.5%  r = 0.035
R = 1.25%  r = 0.125
R = 75%  r = 0.0075
eg. (1) P = 1,00,000 R = 12 % N = 3
A = 1,00,000 + 12% + 12% + 12%
= 1,40,493
I = 1,40,493-1,00,000
= 40,493
(2) P = 1,00,000 R = 9 % N = 30 A = ? , I = ?
= 1,00,000 (1.09)30
= 13,26,768
I = 13,26,768 - 1,00,000
= 12,26,768
(3) A = 5,00,000 R = 1 % N = 240
n
A = P (1 + r)
5,00,000 = P ( 1.01)240
4
10 6 
5,00,000 = P  1.01
  
 

30 8
= 
P  1.01  
20 12
= 
P  1.01  
10
8 3
= P  1.01
  

P= 45903

TIME VALUE of Money 107


eg. find log with formula
(4) P = 1,00,000 A = 3,00,000 N = ? R = 12 %
n
A = P (1 + r)
3,00,000 = 1,00,000 (1.12)n
3 = (1.12)n
log 3 = n log 1.12
(0.4771)= n (0.0492)
n = 9.7 Yrs.
n = 9 Yrs + ( 0.7 x 12 months)
= 9 Yrs + 8.4 months
= 9 Yrs + 8 months + 0.4 x 30 days
= 9 Yrs + 8 months + 12 days
 Formula to find out value of log in calculator
13 times -1 x 3558
log 2 = 0.3010
log 1.5 = 0.1761
eg. P = 1,00,000 R = ? A = 2,75,000 N=7
A = P (1 + r )7
2,75,000 = 1,00,000 [ 1 + r]7
2.75 = (1 + r )7
7 2.75 = 1 + r

1.1555 = 1 + r
r = 15.55 %
 Formula to find out nth root in calculator

12 times - 1 ÷ nth root + 1x = x = x = 12 times

eg. 7 128 = 2 6 729 = 3

eg. (1) A = 1,00,000 R = 12 % N = 2.25 P=?

225 9
N= =
100 4
n
A=P(i+r)
1,00,000 = P ( 1.12)9/5
9
1,00,000 = P 4  1.12 

1, 00, 000
=P
MRC
P= 77,492

108 BUSINESS MATHEMATICS, LOGICAL REASONING & STATISTICS (Paper 3) [CA Foundation]
eg. N1 = 5 R1 = 10 % P = 1, 00, 000

N2 = 3 R2 = 15% A=9

N3 = 2 R3 = 20% I=?
n n n
A =P 1+r1  1 ×1+r2  2 ×1+r3  3 
5 3 2
= 1, 00,000  1.10  ×  1.15  ×  1.20 
A = 1,00,000 x 1.6105 x 1.5208 x 1.44
A = 3,52,711
OR P = 1,00,000
A = 1,00,000 + 10 % + 10 % +10 % +10% +10% + 15 % +15 % + 15 % + 20 % + 20 %
A = 3,52,711
I = 3,52,711 - 1,00,000
= 2,52,711
 Compounded no. of times during the Year
Interest Received
Half Yearly K=2
(semi Annually )
quartrly K= 4
R
Bimonthly K=6
K
( every 2 months) nxk
Monthly K = 12
Fort nighty K =24
Weekly K = 52
Daily K = 365
eg. P = 1,00,000 R= 12 N=2 K=2

R 12%
= 6% =R
K 2
N = NXK= 2X2=4
A = 1,00,000 + 6 % + 6 % + 6 % + 6 %
= 1,26,248
I = 1,26,248 - 1,00,000
= 26,248

TIME VALUE of Money 109


 EFFECTIVE RATE OF INTEREST
[ equivalent rate of Inetest, actual rate of Interest]
ERI ( always take n =1)
 
SI CI

ERI = R %
 
Compounded annually Compounded otherwise
ERI = R % ERI = [ i +r )n-1] x 100

12%
eg. K = 4 R= = 3% n= 1 x 4 =4
4
A=P
100 + 3 % + 3 % + 3 % +3 % = 112.55-100
I = 12.55
1. Time value of money: Time value of money means that the value of a unity of money is different in
different time periods. The sum of money received in future is less valuable than it is today. In other
words the present worth of money received after some time will be less than a money received today.
Since a money received today has more value rational investors would prefer current receipts to
future receipts. If they postpone their receipts they will certainly charge some money i.e. interest.
2. Opportunity Cost: The lender has a choice between using his money in different investments.
If he chooses one he forgoes the return from all others. In other words lending incurs an opportunity
cost due to the possible alternative uses of the lent money.
3. Inflation:Most economies generally exhibit inflation. Inflation is a fall in the purchasing power of
money. Due to inflation a given amount of money buys fewer goods in the future than it will now. The
borrower needs to compensate the lender for this.
4. Liquidity Preference: People prefer to have their resources available in a form that can immediately be
converted into cash rather than a form that takes time or money to realize.
5. Risk Factor: There is always a risk that the borrower will go bankrupt or otherwise default on the loan.
Risk is a determinable factor in fixing rate of interest.
SINKING FUND
It is the fund credited for a specified purpose by way of sequence of periodic payments over a time period
at a specified interest rate. Interest is compounded at the end of every period. Size of the sinking fund
deposit is computed from A = P.A(n, i) where A is the amount to be saved, P the periodic payment, n the
payment period.
Leasing
Leasing is a financial arrangement under which the owner of the asset (lessor) allows the user of the asset
(lessee) to use the asset for a defined period of time(lease period) for a consideration (lease rental) payable
over a given period of time. This is a kind of taking an asset on rent. How can we decide whether a lease
agreement is favourable to lessor or lessee, it can be seen by following example.

110 BUSINESS MATHEMATICS, LOGICAL REASONING & STATISTICS (Paper 3) [CA Foundation]
Capital Expenditure (investment decision)
Capital expenditure means purchasing an asset (which results in outflows of money) today in anticipation of
benefits (cash inflow) which would flow across the life of the investment. For taking investment decision
we compare the present value of cash outflow and present value of cash inflows. If present value of cash
inflows is greater than present value of cash outflows decision should be in the favour of investment. Let us
see how do we take capital expenditure (investment) decision.
Valuation of Bond
A bond is a debt security in which the issuer owes the holder a debt and is obliged to repay the principal and
interest. Bonds are generally issued for a fixed term longer than one year.
PERPETUITY
Perpetuity is an annuity in which the periodic payments or receipts begin on a fixed date and continue
indefinitely or perpetually. Fixed coupon payments on permanently invested (irredeemable) sums of money
are prime examples of perpetuities.
The formula for evaluating perpetuity is relatively straight forward. Two points which are important to
understand in this regard are:.
(a) The value of the perpetuity is finite because receipts that are anticipated far in the future have
extremely low present value (today’s value of the future cash flows).
(b) Additionally, because the principal is never repaid, there is no present value for the principal.
Therefore, the price of perpetuity is simply the coupon amount over the appropriate discount rate or yield.
ANNUITY
Year Amt.Deposited in to bank
1 10,000
2 10,000
3 10,000
4 10,000
5 10,000

ANNUITY

Future value Annuity Present value Annunity


 If Benefit will be received in future  If Benefit is received at present

 All Investment cases are covered here  All loan cases /Hire Purchase / lease transaction
are covered here.

 i + r n - 1 (1 + r) n -1
  Annuity × n
 F.V. = Annuity ×  P.V = r x i + r 
r
 P.V. = Annuity x PVIFA @ r % for ‘n’ yrs.

BASIC CONCEPTS OF PERMUTATIONS AND COMBINATIONS 111


PVIFA = P.V Interest factor Annuity
@ r % for ‘n’ yrs.
e.g. PVIFA @ 10% for ‘5’ yrs.

1
= ............. 5 times G.T.Ans : 3.7908
1.10
 PVIFA @ 12% for 30 yrs.
1
= ........30 times G.T = 8.0552
1.12
Depreciation
* SLM WDV
1-.4.01 original cost 1,00,000 1,00,000
@ 10% depreciation 10,000 10,000
90,000 90,000

Depreciation for 2nd year @ 10% 10,000 9000

80,000 81,000
Depreciation for 3rd year @ 10% 10,000 8100
70,000 72,900
WDV Method of depreciation

Scrap value = original cost x (1-r)n


S = C x (1-r)n
Pension
* Pension

Uncommuted Commuted Pension


Pension
 Pension given in equal  Pension given in Lumpsum Amt.
instalments up to the or
death of employee full payment at once
or
Single sum at once
at the time of retirement

 It is nothing but annuity  P.V of commuted pension


 P.V of Annuity
112 BUSINESS MATHEMATICS, LOGICAL REASONING & STATISTICS (Paper 3) [CA Foundation]
Perpetuity

Meaning : Perpetuity means annuity forever


F.V = 
Perpetuity
P.V =
r

300
=
0.10
= ` 3000
Growing Perpetuity

Perpetuity  1st.Installment 
P.V. =
r-g
r = discount rate g = growth rate

Types of Annuity
Annuity x [(1 +r)n -1]
Annuity Regular If Installments are paid at the F.V =
r
(ordinary Annuity) end of each period. P.V = Annuity x PVIFA @ r% for ‘n’ yrs.

(1 + r) n -1
Annuity Immediate If Installment are F.V.= Annuity ×  (1 +r)
r
(Annuity due) paid at the Begining
of each period
Or
“ Starting from to day” P.V. = Annuity x PVIFA @ r% for ‘n’ yrs. x (1 + r )
word is used in question

Perpetuity Annuity forever F.V. = 

Perpetutiy
P.V. =
r
---0---0---

BASIC CONCEPTS OF PERMUTATIONS AND COMBINATIONS 113


10. A sum of money doubles itself in 10 years. The
number of years it would triple itself is
CLASS WORK (a) 25 years. (b) 15 years.
(c) 20 years (d) none of these
11. If P = ` 1,000, R = 5% p.a, n = 4; What is Amount
1. S.I on ` 3,500 for 3 years at 12% per annum is and C.I. is
(a) ` 1,200 (b) ` 1,260 (a) ` 1,215.50, ` 215.50 (b) ` 1,125, ` 125
(c) ` 2,260 (d) none of these (c) ` 2,115, ` 115 (d) none of these
2. P = 5,000, R = 15, T = 4 ½ using I = PRT/100, I will 12. ` 100 will become after 20 years at 5% p.a
be compound interest amount of
(a) ` 3,375 (b) ` 3,300 (a) ` 250 (b) ` 205
(c) ` 3,735 (d) none of these (c) ` 265.50 (d) none of these
3. If P = 5,000, T = 1, I = ` 300, R will be 13. The effective rate of interest corresponding
to a nominal rate 3% p.a payable half yearly is
(a) 5% (b) 4%
(a) 3.2% p.a (b) 3.25% p.a
(c) 6% (d) none of these
(c) 3.0225% p.a (d) none of these
4. If P = ` 4,500, A = ` 7,200, than Simple interest
i.e. I will be 14. A machine is depreciated at the rate of 20% on
reducing balance. The original cost of the
(a) ` 2,000 (b) ` 3,000 machine was ` 1,00,000 and its ultimate scrap
(c) ` 2,500 (d) ` 2,700 value was ` 30,000. The effective life of the
5. P = ` 12,000, A = ` 16,500, T = 2 ½ years. Rate machine is
percent per annum simple interest will be (a) 4.5 years (appx.) (b) 5.4 years (appx.)
(a) 15% (b) 12% (c) 5 years (appx.) (d) none of these
(c) 10% (d) none of these 15. If A = ` 1,000, n = 2 years, R = 6% p.a compound
interest payable half-yearly, then principal (P)
6. P = ` 10,000, I = ` 2,500, R = 12 ½% SI. The number
is
of years T will be
(a) ` 888.80 (b) ` 885
(a) 1 ½ years (b) 2 years
(c) 800 (d) none of these
(c) 3 years (d) none of these
16. The population of a town increases every year
7. P = ` 8,500, A = ` 10,200, R = 12 ½ % SI, t will be.
by 2% of the population at the beginning of
(a) 1 yr. 7 mth. (b) 2 yrs. that year. The number of years by which the
(c) 1 ½ yr. (d) none of these total increase of population be 40% is
8. The sum required to earn a monthly interest (a) 7 years (b) 10 years
of ` 1,200 at 18% per annum SI is (c) 17 years (app) (d) none of these
(a) ` 50,000 (b) ` 60,000 17. The difference between C.I and S.I on a certain
(c) ` 80,000 (d) none of these sum of money invested for 3 years at 6% p.a is
9. A sum of money amount to ` 6,200 in 2 years ` 110.16. The sum is
and ` 7,400 in 3 years. The principal and rate of (a) ` 3,000 (b) ` 3,700
simple interest are (c) ` 12,000 (d) ` 10,000
(a) ` 3,800, 31.57% (b) ` 3,000, 20% 18. The useful life of a machine is estimated to be
(c) ` 3,500, 15% (d) none of these 10 years and cost ` 10,000. Rate of
depreciationis 10% p.a. The scrap value at the
end of its life is
114 BUSINESS MATHEMATICS, LOGICAL REASONING & STATISTICS (Paper 3) [CA Foundation]
(a) ` 3,486.78 (b) ` 4,383 (a) ` 587.87 (b) ` 587
(c) ` 3,400 (d) none of these (c) ` 578.87 (d) none of these
19. The effective rate of interest corresponding a 28. A = ` 1,200 n = 12 years i = 0.08, V = ?
nominal rate of 7% p.a convertible quarterly is
(a) 7% (b) 7.5% A 1 
Using the formula V = 1- value of
(c) 5% (d) 7.18% i  (1 + i)n 

20. The C.I on ` 16000 for 1 ½ years at 10% p.a v will be


payable half -yearly is (a) ` 3,039 (b) ` 3,990
(a) ` 2,222 (b) ` 2,522 (c) ` 9930 (d) none of these
(c) ` 2,500 (d) none of these 29. a = ` 100 n = 10, i = 5% find the FV of annuity
21. The C.I on ` 40000 at 10% p.a for 1 year when A
the interest is payable quarterly is Using the formula FV = [(1+i)n -1], FV is equal
i
(a) ` 4,000 (b) ` 4,100 to
(c) ` 4,152.51 (d) none of these (a) ` 1,258 (b) ` 2,581
22. The difference between the S.I and the C.I on (c) ` 1,528 (d) none of these
` 2,400 for 2 years at 5% p.a is
30. If the amount of an annuity after 25 years at
(a) `5 (b) ` 10 5% p.a C.I is ` 50,000 the annuity will be
(c) ` 16 (d) ` 6 (a) ` 1,406.90 (b) ` 1,046.90
23. The annual birth and death rates per 1,000 are (c) ` 1,146.90 (d) none of these
39.4 and 19.4 respectively. The number of years
in which the population will be doubled 31. Given annuity of ` 100 amounts to ` 3137.12 at
assuming there is no immigration or 4.5% p.a C. I. The number of years will be
emigration is (a) 25 years (appx.) (b) 20 years (appx.)
(a) 35 years. (b) 30 years. (c) 22 years (d) none of these
(c) 25 years (d) none of these 32. A company borrows ` 10,000 on condition to
repay it with compound interest at 5% p.a by
annual installments of ` 1000 each. The
24. The C.I on ` 4,000 for 6 months at 12% p.a number of years by which the debt will be clear
payable quarterly is is
(a) ` 243.60 (b) ` 240 (a) 14.2 years (b) 10 years
(c) ` 243 (d) none of these (c) 12 years (d) none of these
25. The present value of an annuity of ` 3000 for 33. Mr. X borrowed ` 5,120 at 12 ½ % p.a C.I. At the
15 years at 4.5% p.a CI is end of 3 yrs, the money was repaid along with
(a) ` 23,809.41 (b) ` 32,218.63 the interest accrued. The amount of interest
paid by him is
(c) ` 32,908.41 (d) none of these
(a) ` 2,100 (b) ` 2,170
26. The amount of an annuity certain of ` 150 for
(c) ` 2,000 (d) none of these
12 years at 3.5% p.a C.I is
34. Mr. Paul borrows ` 20,000 on condition to repay
(a) ` 2,190.28 (b) ` 1,290.28
it with C.I. at 5% p.a in annual installments of `
(c) ` 2,180.28 (d) none of these 2000 each. The number of years for the debt to
27. A loan of ` 10,000 is to be paid back in 30 equal be paid off is
instalments. The amount of each installment (a) 10 years (b) 12 years
to cover the principal and at 4% p.a CI is (c) 11 years (d) none of these
BASIC CONCEPTS OF PERMUTATIONS AND COMBINATIONS 115
35. A person invests ` 500 at the end of each year 45. A man purchased a house valued at ` 3,00,000.
with a bank which pays interest at 10% p. a C.I. He paid ` 2,00,000 at the time of purchase and
annually. The amount standing to his credit agreed to pay the balance with interest at 12%
one year after he has made his yearly per annum compounded half yearly in 20 equal
investment for the 12th time is. half yearly instalments. If the first instalment
(a) ` 11,764.50 (b) ` 10,000 is paid after six months from the date of
(c) ` 12,000 (d) none of these purchase then the amount of each instalment
is
36. The present value of annuity of ` 5,000 per
annum for 12 years at 4% p.a C.I. annually is [Given log 10.6 = 1.0253 and log 31.19 = 1.494]
(a) ` 46,000 (b) ` 46,850 (a) ` 8,719.66 (b) ` 8,769.21
(c) ` 15,000 (d) none of these (c) ` 7,893.13 (d) none of these.
37. A person desires to create a fund to be 46. The difference between compound and
invested at 10% CI per annum to provide for a simple interest at 5% per annum for 4 years on
prize of ` 300 every year. Using V = a/I find V ` 20,000 is ` ________
and V will be (a) 250 (b) 277
(a) ` 2,000 (b) ` 2,500 (c) 300 (d) 310
(c) ` 3,000 (d) none of these 47. The compound interest on half-yearly rests on
38. A = ` 5,200, R = 5% p.a., T = 6 years, P will be ` 10,000 the rate for the first and second years
(a) ` 2,000 (b) ` 3,880 being 6% and for the third year 9% p.a. is
(c) ` 3,000 (d) none of these ‘.____________.
39 If P = 1,000, n = 4 years., R = 5% p.a then C. I will (a) 2,200 (b) 2,287
be (c) 2,285 (d) 2,291
(a) ` 215.50 (b) ` 210 48. The present value of ‘ 10,000 due in 2 years at
(c) ` 220 (d) none of these 5% p.a. compound interest when the interest
40 The time in which a sum of money will be is paid on yearly basis is ` ________.
double at 5% p.a C.I is (a) 9,070 (b) 9,000
(a) ` 10 years (b) 12 years (c) 9,061 (d) None
(c) 14.2 years (d) none of these 49. The present value of ` 10,000 due in 2 years at
41. If A = ` 10,000, n = 18yrs., R = 4% p.a C.I, P will 5% p.a. compound interest when the interest
be is paid on half-yearly basis is ` ________.
(a) ` 4,000 (b) ` 4,900 (a) 9,070 (b) 9,069
(c) ` 4,500 (d) none of these (c) 9,060 (d) None
42. The time by which a sum of money would triple 50. Johnson left ` 1,00,000 with the direction that
itself at 8% p. a C. I is it should be divided in such a way that his
(a) 14.28 years (b) 14 years minor sons Tom, Dick and Harry aged 9, 12 and
(c) 12 years (d) none of these 15 years should each receive equally after
43. The present value of an annuity of ` 80 a years attaining the age 25 years. The rate of interest
for 20 years at 5% p.a is being 3.5%, how much each son receive after
(a) ` 997 (appx.) (b) ` 900 getting 25 years old?
(c) ` 1,000 (d) none of these (a) 50,000 (b) 51,994
44. A person bought a house paying ` 20,000 cash (c) 52,000 (d) None
down and ` 4,000 at the end of each year for 25 51. In how many years will a sum of money double
yrs. at 5% p.a. C.I. The cash down price is at 5% p.a. compound interest?
(a) ` 75,000 (b) ` 76,000 (a) 15 years 3 months
(c) ` 76,375 (d) none of these. (b) 14 years 2 months

116 BUSINESS MATHEMATICS, LOGICAL REASONING & STATISTICS (Paper 3) [CA Foundation]
(c) 14 years 3 months 58. A machine costs ` 5,20,000 with an estimated
(d) 15 years 2 months life of 25 years. A sinking fund is created to
52. In how many years a sum of money trebles at replace it by a new model at 25% higher cost
5% p.a. compound interest payable on after 25 years with a scrap value realization of
halfyearly basis? ` 25000. what amount should be set aside
every year if the sinking fund investments
(a) 18 years 7 months
accumulate at 3.5% compound interest p.a.?
(b) 18 years 6 months
(a) 16,000 (b) 16,500
(c) 18 years 8 months
(c) 16,050 (d) 16,005
(d) 22 years 3 months
59. Raja aged 40 wishes his wife Rani to have ` 40
53. A machine depreciates at 10% of its value at lakhs at his death. If his expectation of life is
the beginning of a year. The cost and scrap another 30 years and he starts making equal
value realized at the time of sale being ‘ 23,240 annual investments commencing at the end
and ‘ 9,000 respectively. For how many years of each year at 3% compound interest p.a. how
the machine was put to use? much should he invest annually?
(a) 7 years (b) 8 years (a) 84,448 (b) 84,450
(c) 9 years (d) 10 years (c) 84,449 (d) 84,077
54. A machine worth ` 4,90,740 is depreciated at 60. Appu retires at 60 years receiving a pension of
15% on its opening value each year. When its 14,400 a year paid in half-yearly installments
value would reduce to ` 2,00,000? for rest of his life after reckoning his life
(a) 4 years 6 months expectation to be 13 years and that interest at
(b) 4 years 7 months 4% p.a. is payable half-yearly. What single sum
(c) 4 years 5 months is equivalent to his pension?
(d) 5 years 7 months approximately (a) 1,45,000 (b) 1,44,900
55. A machine worth ` 4,90,740 is depreciated at (c) 1,44,800 (d) 1,44,700
15% of its opening value each year. When its 61. An investor intends purchasing a three year
value would reduce by 90%? ` 1,000 par value bond having nominal interest
(a) 11 years 6 months rate of 10%. At what price the bond may be
(b) 11 years 7 months purchased now if it matures at par and the
(c) 11 years 8 months investor requires a rate of return of 14%?
(d) 14 years 2 months approximately (a) ` 907.125 (b) ` 1007.125
56. Alibaba borrows ` 6 lakhs Housing Loan at 6% (c) ` 807.125 (d) none of these.
repayable in 20 annual installments 62. A person opened an account on 1st April, 2011
commencing at the end of the first year. How with a deposit of ` 800. The account paid 6%
much annual payment is necessary. interest compounded quarterly. On October 1
(a) 52,420 (b) 52,419 2011 he closed the account and added enough
(c) 52,310 (d) 52,320 additional money to invest in a 6 month time-
57. A sinking fund is created for redeming deposit for ` 1,000, earning 6% compounded
debentures worth ` 5 lakhs at the end of 25 monthly.
years. (a) How much additional amount did the person
How much provision needs to be made out of invest on October 1, 2011?
profits each year provided sinking fund (b) What was the maturity value of his time
investments can earn interest at 4% p.a.? deposit on April 1, 2012?
(a) 12,006 (b) 12,040 (c) How much total interest was earned?
(c) 12,039 (d) 12,035

BASIC CONCEPTS OF PERMUTATIONS AND COMBINATIONS 117


(a) ` 175.82 (b) ` 1,030.38
(c) ` 54.56 (d) None of these HOME WORK - 1
63. Assuming that the discount rate is 7% per
annum, how much would you pay to receive `
50, growing at 5%, annually, forever? 1
1. The simple interest on a sum of money is th
(a) 2,000 (b) 2,500 16
of the principal and the number of years is
(c) 3,000 (d) none of these. equal to the rate percent per annum. The
64. Z invests ` 10,000 every year starting from number of years is
today for next 10 years. Suppose interest rate (a) 2 yrs (b) 2.5 yrs.
is 8% per annum compounded annually.
(c) 4 yrs (d) 5 yrs.
Calculate future value of the annuity.
2. A sum of money lent out at simple interest
Given that (1 + 0.08)10 = 2.15892500.
amounts to ` 720 after 2 years and to ` 1020
(a) ` 1,56,454.875 (b) ` 1,44,865.625 after a further period of 5 years. The principal
(c) ` 1,56,854.875 (d) none of these. amount is
(a) 400 (b) 500
(c) 600 (d) 480
 3. A person takes a loan of ` 200 at 5% simple
interest. He returns ` 100 at the end of 1 year.
In order to clear his dues at the end of 2 years,
he would pay
(a) ` 110 (b) ` 115
(c) ` 115.50 (d) none
4. The rate of interest on a sum of money is 8%
p.a. for the first year, 6% p.a. for the next 4
years and 4% p.a. for the period beyond 5
years. If the simple interest accursed by the
sum for a total period of 11 years is ` 1120,
what is the sum ?
(a) ` 2000 (b) ` 3000
(c) ` 4000 (d) ` 5000
5. In how many years would a sum be 3 times at
12% rate of Compound interest ?
1
(Given 3 9.7 = 1.12).
1
(a) 12 years (b) years
9. 7
(c) 9.7 years (d) none
6. A company pays to its depositers 15% rate of
interest compounded monthly. The effective
rate of interest is
(a) 15% (b) 14.64%
(c) 16.08% (d) none

118 BUSINESS MATHEMATICS, LOGICAL REASONING & STATISTICS (Paper 3) [CA Foundation]
7. A company pays 16% rate of interest 14. A company requires ` 60,00,000 after 5 years
compounded half-yearly. The effective rate of to redeem debentures. It is decided to create
interest is a sinking fund by investing a fixed amount
every year in securities giving 10% compound
10400 72900
(a) (b) interest. The yearly investment is [Given
625 625 (1.1)5 = 1.6]
5100 10200 (a) 10,00,000 (b) 9,60,000
(c) (d)
625 625 (c) 12,00,000 (d) none
8. The effective rate that is equivalent to a 15. If an investment of ` 60000 and ` 70000
nominal rate of 8% compounded semi - respectively yields an income of `5750 and
annually is `6500 then an investment of `90000 would
(a) 8% (b) 8.08% yield income of `__________.
(c) 8.16% (d) 8.24% (a) 7500 (b) 8000
9. The present value of ` 16047 to be received (c) 7750 (d) 7800
after 4 years at the interest rate of 12%
16. For a 10 year deposit, what interest rate
compounded quarterly is
payable annually is equivalent to 5% interest
(a) 11000 (b) 10000 payable quarterly
(c) 25750 (d) none (a) 5.1% (b) 4.9%
10. How much should a 30 year old man invest now (c) 6.0% (d) None
at 7% annum compounded semi-annually to
17. What is the annual rate of interest
obtain a lump sum of ` 50000 for retirement at
compounded annually doubles an investment
the age of 60.
(a) 3,93,904 ` (b) 6346.7 ` in 2 years. Given that = 1.4142135.
(c) 6130 ` (d) none (a) 46.04125 % (b) 14.142135
11. If ` 100 is deposited at the end of every month, (c) 41.42135 % (d) None
the total amount accumulated after 20 years if 18. Ram is confused whether to invest at 9% p.a.
the rate of interest is 12% compounded compounded monthly or 9.25% p.a. simple
monthly is interest, given that (1+ 0.0075)12 = 1.09380690
(a) 98925 (b) 1,07,600 The student decided to find effective rate of
(c) 96460 (d) 86460 interest which is
12. Nareshbhai purchased a computer and agreed (a) 9% (b) 9.25%
to pay ` 5000 cash and equal installments of (c) 9.38% (d) None
` 3000 quarterly for 4 years. If the rate of
19. If the compound interest on a certain sum for
interest is 12% compounded quarterly then
2 years at 3% be ` 101.50, what would be the
the value of the computer is
S.I.?
(a) 62320 (b) 25770
(a) ` 100 (b) ` 200
(c) 42683 (d) 32560
(c) ` 300 (d) None
13. A machine is available in ` 70000 or by leasing
if for 5 years at an annual rate of 20,000 ` If 20. A sum of money is put at compound interest
money can be borrowed at 14% per annum, for 2 years at 20% p.a. It would earn ` 482 more,
what is the saving if we go for leasing ? if the interest were payable half-yearly than it
were payable yearly; then the sum is
(a) ` 1339 (b) ` 1829
(a) ` 20,000 (b) ` 25,000
(c) ` 1629 (d) none
(c) ` 26,000 (d) None

BASIC CONCEPTS OF PERMUTATIONS AND COMBINATIONS 119


21. A company sets aside a sum of ` 45,000 27. A person deposited a sum of ` 10,000 in a bank.
annually for 9 years to pay off a debenture After 2 years, he withdrew ` 4,000 and at the
issue of ` 5,00,000. If the fund accumulates at end of 5 years, he received an amount of `
6% p.a., find the surplus after full redemption 7,200; then the rate of simple interest is:
of the debenture issue. (a) 6% (b) 5%
(a) ` 15,500 (b) ` 16,500 (c) 10% (d) None
(c) ` 17,109 (d) None 28. If the value of a car gets depreciated by 20%
22. A company may obtain a machine either by per year, estimated value at the end of five
leasing it for 5 years, (useful life) at an annual year is ______ if its present value is ` 24,000.
rent of ` 2,000 or by purchasing it for ` 8,100. If (a) 7864.32 (b) 7684.23
the company can borrow money at 10% p.a.,
(c) 8764.32 (d) 6789.32
which alternative is preferable?
29. Find the present value of an ordinary annuity
(a) Leasing is preferable of 8 quarterly payments of ` 500 each, the rate
(b) Leasing is not preferable of interest being 8% p.a. compound quarterly.
(c) Cannot say (d) None (a) 4275.00 (b) 4725.00
23. A person opened a savings bank account 4 (c) 3662.50 (d) 3266.50
months ago, which has now a balance of `
21,315. If the bank pays 4.5% simple interest,
how much money was deposited by him,
initially? 
(a) ` 21,000 (b) ` 20,000
(c) ` 15,000 (d) None
24. Mohan deposits ` 500 into an account paying
8% simple interest. He makes two more
deposits of ` 1,000 each; first after 3 months ANSWER KEYS
and second after 6 months. How much will be
in his account at the end of the year, if he makes
no other deposits and withdrawals during the
time?
1. (b) 2. (c) 3. (c) 4. (a)
(a) ` 2,600 (b) ` 2,650
(c) ` 2,640 (d) None 5. (c) 6. (c) 7. (a) 8. (c)
25. How long will it take ` 1,200 to amount ` 2,000
at 5% converted quarterly at Compound 9. (b) 10. (b) 11. (a) 12. (c)
Interest?
(a) 10 years 3 months 13. (a) 14. (a) 15. (b) 16. (a)
(b) 10 years 5 months
17. (c) 18. (c) 19. (a) 20. (a)
(c) 10 years
(d) None 21. (c) 22. (a) 23. (a) 24. (c)
26. If the simple interest on ` 1,200 be more the
interest on ` 1,000 by ` 30 in 3 year. Find the 25. (a) 26. (c) 27. (d) 28. (a)
rate percent per annum.
(a) 7% (b) 6% 29. (c)
(c) 5% (d) None

120 BUSINESS MATHEMATICS, LOGICAL REASONING & STATISTICS (Paper 3) [CA Foundation]
8. `2,000 is invested at annual rate of interest of
10% p.a. The amount after two years if
HOME WORK - 2 compounding is done half yearly, is -
(a) ` 2431 (b) `243.10
1. In what time will `85,000 amount to `1,57,675 (c) ` 2341 (d) None
at 4.5% p.a. S.I. ?
9. On what sum will the compound interest at
(a) 20 years (b) 15 years 5% p.a. for 2 yrs compounded annually be
(c) 22 years (d) 19 years `1,640?
2. A Sum of `46,875 was lent out at simple (a) ` 16,000 (b) ` 17,000
interest and at the end of 1 yr and 8 months, (c) ` 18,000 (d) ` 19,000
the total amount was `50,000. Find the rate of
interest. 10. The compound interest on a certain sum for 2
years is `41.60 and the simple interest is `40.
(a) 4% (b) 5%
Find the sum.
(c) 4.5% (d) 6% (a) ` 500 (b) ` 400
3. Kapil deposited some amount in a bank for (c) ` 250 (d) ` 300
71 2 years at 6% SI. Kapil received `1,01,500 at 11. At what rate % will a sum double itself in 7
the end of the term. Compute initial deposit years if the interest is compounded annually.
of Kapil. (a) 7.0% (b) 8.0%
(a) `1,00,000 (b) `70,000 (c) 10.41% (d) 7.9%
(c) `75,000 (d) `86,500 12. For a 10-year deposit, what interest rate
4. What sum of money will produce 28,600 payable annually is equivalent to 5% interest
interest in 3 yrs & 3 months at 2.5% p.a. simple payable quarterly ?
interest ? (a) 5.1 % (b) 4.9%
(a) ` 3,52,000 (b) ` 3,65,000 (c) 6.0% (d) None of these
13. In what time will compound interest on `320
(c) ` 3,25,000 (d) ` 3,56,000
5. Find out the capital required to earn a monthly at 12 1 2 % p.a. compounded annually be `85?
interest of `800 p.m. at 5% at simple interest ?
(a) 4 1 2 years (b) 2 1 2 Yearss
(a) ` 1,87,000 (b) ` 40,000
(c) ` 1,28,000 (d) ` 1,92,000 (c) 2 Years (d) 5 Years
14. In what time will a sum of `800 at 5% p.a.
6. Simple Interest on a certain sum of money
compound interest, amount two Rs 882 ?
2 1 2 years at 3 1 4 % p.a. is `390. (a) 1 years (b) 5 years
(c) 4 years (d) 2 years
(a) ` 4,800 (b) ` 2,100
15. Suppose your mom decides to gift you `10,000
(c) ` 4,700 (d) ` 4,900 every year starting from today for the next
7. A sum of 44,000 is divided into 3 parts such sixteen years. You deposit this amount in a
that the corresponding interest earned after 2 bank as and when you receive and get 8.5%
years, 3 years and 6 years may be equal at the per annum interest rate compounded
rate of simple interest are 6% p.a., 8% p.a., & annually. What is the present value of this
6% p.a. respectively. Then the smallest part of money : Given that P (15, 0.085) = 8.304236
the sum will be : (a) 83042 (b) 90100
(a) ` 4,000 (b) ` 8,000 (c) 93042 (d) 10100
(c) ` 10,000 (d) ` 12,000

BASIC CONCEPTS OF PERMUTATIONS AND COMBINATIONS 121


16. The future value of an annuity of `1,000 made
annually for 5 years at the rate of interest 14%
compound annually is
(a) ` 5610 (b) ` 6610
(c) ` 6160 (d) ` 5160 ANSWER KEYS
17. In _____ first payment/receipt takes place at
the end of first period.
(a) Annuity immediate 1 (d) 2 (a) 3 (b) 4 (a)
(b) Annuity regular
(c) Annuity due (d) Annuity special 5 (d) 6 (a) 7 (b) 8 (a)
18. A person invested money in bank paying 6%
Compounded semi annually. If the person 9 (a) 10 (c) 11 (c) 12 (a)
expects to receive 8,000 in 6 years, what is the
present value of investment ? 13 (c) 14 (d) 15 (c) 16 (b)
(a) ` 5,000 (b) ` 4,611.03
17 (b) 18 (c) 19 (b)
(c) ` 5,611.03 (d) None
19. The simple interest of P % for P years will be
Rs. P on a sum of :
P 100
(a) Rs. (b) Rs.
100 P

 P   100 
(c) Rs.  1 (d) Rs.   1
 100   P 

---0---0---

122 BUSINESS MATHEMATICS, LOGICAL REASONING & STATISTICS (Paper 3) [CA Foundation]
CHAPTER-5
BASIC CONCPETS OF PERMUTATIONS AND COMBINATIONS

INTRODUCTION
We will learn problem of arranging and grouping of certain things, taking particular number of things at
a time. It should be noted that (p, q) and (q, t) are two different arrangements, but they represent the
same group. In case of arrangements, the sequence or order of things is also taken into account.
The manager of a large bank has a difficult task of filling two important positions from a group of five
equally qualified employees. Since none of them has had actual experience, he decides to allow each
of them to work for one month in each of the positions before he makes the decision. How long can the
bank operate before the positions are filled by permanent appointments?
Solution to above - cited situation requires an efficient counting of the possible ways in which the
desired outcomes can be obtained. A listing of all possible outcomes may be desirable, but is likely to
be very tedious and subject to errors of duplication or omission. We need to devise certain techniques
which will help us to cope with such problems. The techniques of permutation and combination will
help in tackling problems such as above.

PERMUTATIONS
Definition: The ways of arranging or selecting smaller or equal number of persons or objects from a
group of persons or collection of objects with due regard being paid to the order of arrangement or
selection, are called permutations.

COMBINATIONS
Definition : The number of ways in which smaller or equal number of things are arranged or selected
from a collection of things where the order of selection or arrangement is not important, are called
combinations.

BASIC CONCEPTS OF PERMUTATIONS AND COMBINATIONS 123


Formulas :

n!
(1)
nPr =
 n- r !
(2) nPn =n!
(3) o!=1
(4) Two articles never come together
(n-2)x (n-1)!
n!
nC
(5) r=  n - r !r!
nC
(6) n =1
nC
(7) O =1
(8) nC  nC
r n-r
(9) nC  nC  n+1 C
r r-1 r
nC n
(10) r1  Cr2

 
r1=r2 r1+r2=n

n Pr
(11) = nC r
r!
(12) Number of triangles = nC3

(13) Number of straight lines = nC2

(14) Number of chords = nC


2
(15) Number of Quadrilateral = nC4

(16) Number of hand shake= nC2

(17) Number of Gifts = nP2

124 BUSINESS MATHEMATICS, LOGICAL REASONING & STATISTICS (Paper 3) [CA Foundation]
(18) nC  nC  ......  nC
1 2 n

OR

One or more 2n - 1

(19) Number of diagonals = nC2 - n

(20) Round table/Ring /Circular = (n-1)!

n - 1 !
(21) Necklace of different beads=
2

Note:
Polygon  Shape having more than 4 sides

     
Pentagon hexagon septagone octagon nenagon Decagon
5 sides 6 sides 7 sides 8 sides 9 side 10 side

(22) Poker= n C
5
(23) With repetition = nr

(24) Without repetition = n Pr

 With repctition = nr
 Without Repetition = nPr

BASIC CONCEPTS OF PERMUTATIONS AND COMBINATIONS 125


11. Solution of the equation nP3 : (n + 1)P3 = 3 : 4 is
(a) 11 (b) 9
CLASS WORK (c) 6 (d) none
12. Solution of the equation 12nP3 = 5(n + 2)P3 is
1. 0 is a symbol equal to (a) 4 (b) 7
(c) 6 (d) none
(a) 0 (b) 1
13. Solution of the equation nP5 = 240240 is
(c) Infinity (d) none
(a) 10 (b) 9
2. In n Pr , n is always (c) 14 (d) none
14. Solution of the equation 4.nP3 = 5(n – 1)P3 is
(a) an integer (b) a fraction
(a) 14 (b) 15
(c) a positive integer (d) none
(c) 12 (d) none
3. In n Pr the restriction is np np
15. If 4 = 12 x 2 , the n is equal to
(a) n > r (b) nr
(a) -1 (b) 6
(c) n  r (d) none (c) 5 (d) none
4. In n Pr = n (n—1) (n—2)..........(n - r + 1), the 16. If.
np np
3 : 2 = 3. :1, then n is equal too
number of factor is
(a) 7 (b) 4
(a) n (b) r-1
(c) 5 (d) none
(c) n-r (d) r
n P can also written as 17. If 5 pr = 60, then the value of r is
5. r
(a) 3 (b) 2
n n (c) 4 (d) none
(a) (b) 18. In how many different ways five persons can
n-r r n -r
sit in seven chairs arranged in one row ?
r (a) 2520 (b) 4320
(c) (d) none (c) 720 (d) none
nr
19. Two prizes are to be given among ten persons.
6. The value of 11P3 is
If both the prizes are not to be given to one
(a) 40320 (b) 990
person, the number of ways in which the prizes
(c) 1260 (d) none
can be given is
7. The value of 8P8 is
(a) 90 (b) 100
(a) 40320 (b) 990
(c) 110 (d) none
(c) 1260 (d) none
20. Two prizes are to be given among ten persons.
8. The value of 6P2 ´ 7P2 is
If both the prizes can be given to one person.
(a) 40320 (b) 990 the number of ways in which the prizes can be
(c) 1260 (d) none given is
9. Solution of the equation 14Px = 2184 is (a) 90 (b) 80
(a) 4 (b) 2 (c) 100 (d) none
(c) 3 (d) none
21. For a group photo a principal, 4 professors, 2
10. Solution of the equation 840 . x! = 7! is girls and 4 boys are to be arranged in chairs, in
(a) 4 (b) 3 a row. If the principal is to occupy the middle
(c) 2 (d) none chair, the boys to occupy the last two chairs,
on either side and if the girls do not want to sit
126 BUSINESS MATHEMATICS, LOGICAL REASONING & STATISTICS (Paper 3) [CA Foundation]
with the boys, the number of ways in which together to the right of gents.
the arrangements can be made is (a) 40320 (b) 360
(a) 6912 (b) 1564 (c) 576 (d) none
(c) 1444 (d) none 30. How many different arrangements can be
22. In how many ways a chairman, a vice– made by using all the letters of the word
chairman, a secretary and a treasurer can be a “COURAGE” if all vowels are to be together
chosen out of 9 members of a working and all consonant are to be together?
committee? (a) 720 (b) 360
(a) 3024 (b) 4032 (c) 288 (d) none
(c) 1008 (d) none 31. How many words can be formed from the
23. There are 6 cages to keep 6 lions in a circus. letters of the word FATHEPUR when the three
One cage is so small that any of the four big letters PUR occur together?.
lions cannot be kept in it, the number of ways (a) 3600 (b) 4320
in which six lions can be kept in 6 cages is (c) 2880 (d) none
(a) 300 (b) 240 32. How many words can be formed from the
(c) 720 (d) none letters of the word FATHEPUR when vowels
24. In how many ways 5 boys and 3 girls can be occur at even places?
arranged in a row so that no two girls are (a) 3600 (b) 4320
together? (c) 2880 (d) none
(a) 14400 (b) 40320 33. In how many ways can the letters of the word
(c) 10080 (d) none STRANGE be arranged so that the vowels are
25. There are 6 seats (including the driver’s seat) never separated?
in a motorcar, 2 persons out of 8 know car (a) 3600 (b) 1440
driving. In how many ways 6 persons out of 8 (c) 5040 (d) none
can be seated in the car?
34. The number of ways the letters of the word
(a) 720 (b) 40320 COMPUTER can be rearranged is
(c) 5040 (d) none (a) 40320 (b) 40319
26. How many different words using all the letters (c) 40318 (d) none
of the word MAUNISH can be formed in which
35. The number of arrangements of the letters in
vowels are always together?
the word FAILURE, so that vowels are always
(a) 120 (b) 360 coming together is
(c) 720 (d) none (a) 576 (b) 575
27. Seven persons are to be arranged in a row. The (c) 570 (d) none
number of arragement in which a particular
36. If 12 school teams are participating in a quiz
man always occupy the middle seat is
contest, then the number of ways the first,
(a) 720 (b) 120 second and third positions may be won is
(c) 240 (d) none (a) 1230 (b) 1320
28. Four couples are to occupy 8 seats in a row. In (c) 3210 (d) none
how many different ways these four couples
37. The number of ways the letters of the word
can take their seats, if each couple to remain
“Triangle” to be arranged so that the word
together?
‘angle’ will be always present is
(a) 40320 (b) 384
(a) 20 (b) 60
(c) 576 (d) none
(c) 24 (d) 32
29. Four couples are to occupy 8 seats in a row. In
how many different ways these four couples
can take their seats, if all ladies remain
BASIC CONCEPTS OF PERMUTATIONS AND COMBINATIONS 127
38. If the letters word ‘Daughter ’ are to be 46. 5 letters are written and there are five letter-
arranged S0 that vowels occupy the odd places, boxes. The number of ways the letters can be
then number of different words are dropped into the boxes, are in each
(a) 576 (b) 2880 (a) 119 (b) 120
(c) 625 (d) 524 (c) 121 (d) none
39. The number of Ways in which new words the 47. The total number of sitting arrangements of 7
letters of the word DOGMATIC can be arranged persons in a row if 3 persons sit together in a
is particular order is _________.
(a) 40319 (b) 40320 (a) 5! (b) 6!
(c) 40321 (d) none (c) 2! × 5! (d) None
48. The total number of sitting arrangements of 7
40. Mr. X and Mr. Y enter into a railway
persons in a row if 3 persons sit together in
compartment having six vacant seats. The
any order is _________.
number of ways in which they can occupy the
seats is (a) 5! (b) 6!
(c) 2! × 5! (d) None
(a) 25 (b) 31
49. How many different words begin with S and
(c) 32 (d) 30
end with Y can be formed by using all the letters
41. The number of ways in which 6 man can be
of the word SUNDAY ?
arranged in a row so that three particular man
(a) 12 (b) 24
are sitting together is
(c) 120 (d) none
(a) 4P4 (b) 4P4 x 3P3
50. How many new words can be formed by using
(c) ( 3 )2 (d) none the letters of the word QUALITY in such a way
42. There are 5 speakers A, B, C, D and E. The that the vowels always occupy the second, the
number of ways in which A will speak always third and the fifth place?
before B is (a) 144 (b) 720
(a) 24 (b) (c) 143 (d) none
4X 2
51. The number of arrangements in which the
(c) 5 (d) none letters of the word MONDAY be arranged so
43. There are 10 trains plying between Calcutta that the words thus formed begin with M and
and Delhi. The number of ways in which a do not end with N is
person can go from Calcutta to Delhi and return (a) 720 (b) 120
by a different train is (c) 96 (d) none
(a) 99 (b) 90 52. How many different words can be permuted
(c) 80 (d) none by using all the letters of the word
44. The number of ways in which 8 sweets of COMMITTEE?
different sizes among 8 persons of different (a) 3286600 (b) 40320
ages so that the largest sweets always goes to (c) 45360 (d) none
be younger assuming that each one of them 53. The letters of the words CALCUTTA and
gets a sweets is AMERICA are arranged in all possible ways. The
ratio of the number of there arrangements is
(a) 8 (b) 5040
(a) 1:2 (b) 2:1
(c) 5039 (d) none (c) 2:2 (d) none
45. The number of ways in which the letters of 54. The number of arrangement of the letters of
the word MOBILE be arranged so that the word COMMERCE is
consonants always occupy the odd places is
(a) 36 (b) 63 (a) 8 (b) 8 / ( 2 2 2)
(c) 30 (d) none (c) 7 (d) none

128 BUSINESS MATHEMATICS, LOGICAL REASONING & STATISTICS (Paper 3) [CA Foundation]
55. n articles are arranged in such a way that 2 different ways of arranging the persons are
particular articles never come together. The (a) 720 (b) 432
number of such arrangements is (c) 324 (d) none
(a) (n - 2) (n -1) ! (b) (n -1) (n - 2) ! 63. How many numbers of three digits can be
(c) n ! (d) none formed from digits 1, 2, 4, 7, 8, 9 ?
56. 10 examination papers are arranged in such a (a) 24 (b) 720
way that the best and worst papers never come (c) 120 (d) none
together. The number of arrangements is 64. How many different numbers of six digits can
(a) 9 8 (b) 10 be formed by using the digits 2, 1, 7, 8, 0, 4 ?
(a) 720 (b) 600
(c) 8 9 (d) none (c) 120 (d) none
57. There are 7 papers in an examination. Among 65. How many different numbers of six digits can
them two papers are of Mathematics and other be formed by using the digits 2, 1, 7, 8, 0, 4
papers are of different subjects. The numbers which are divisible by 5 ?
of ways in which the papers can be arranged (a) 120 (b) 600
so that the two Mathematics papers are not (c) 24 (d) none
consecutive are 66. How many different numbers of five digits
(a) 1200 (b) 3600 greater than 50,000 can be formed by using
(c) 7200 (d) none the digits 3, 4, 5. 6 and 7 ?
58. Number of ways in which 8 books can be (a) 120 (b) 60
arranged on a shelf so that the two particular (c) 72 (d) none
books are not together is 67. How many numbers of three digits can be
(a) 12000 (b) 30240 formed with the digits 2, 3, 4, 5 ?
(c) 40320 (d) none (a) 24 (b) 64
59. Number of ways in which 8 books can be (c) 12 (d) none
arranged on a shelf so that the two particular 68. How many numbers greater than a million can
books are together is be formed using all the digits 4, 5, 5, 0, 4, 5, 3?
(a) 30240 (b) 40320 (a) 300 (b) 360
(c) 10080 (d) none (c) 80 (d) none
(c) 80 (d) none
60. Six papers are set in an examination of which
two are mathematical. In how many different 69. Using the digits 1, 2, 3, 4, 5 once only how many
orders can the papers be arranged so that the numbers of five digits can be formed such that
two mathematical papers are together? the numbers are divisible by 2 ?
(a) 48 (b) 36
(a) 720 (b) 240
(c) 72 (d) none
(c) 480 (d) none
70. Using the digits 1, 2, 3, 4, 5 once only how many
61. Six papers are set in an examination of which numbers of five digits can be formed such that
two are mathematical. In how many different the numbers are greater than 23000?
orders can the papers be arranged so that the
(a) 72 (b) 96
mathematical papers are not together?
(c) 90 (d) none
(a) 480 (b) 360
71. How many numbers greater than 65000 can be
(c) 288 (d) none formed by using the digits 5, 6, 7, 8, 9 once
62. There are 8 persons. Of them 3 are Indians, 3 only?
are Russians and the remaining 2 are (a) 76 (b) 90
Americans. They have to stand in a line so that (c) 96 (d) none
the 3 Indians are together, the 3 Russians are
together and so also the 2 Americans. The
SEQUENCE AND SERIES 129
72. How many numbers between 3000 and 4000 81. A number of 4 different digits is formed using
can be formed by using the digits 1, 2, 3, 4, 5, 6 the figures 1, 2, 3, 4, 5, 6, 7. How many of them
once only’? are greater than 3,400?
(a) 360 (b) 300 (a) 560 (b) 840
(c) 80 (d) none (c) 420 (d) none
73. How many different numbers of six digits can 82. The number of 4 digit numbers greater than
be formed with the digits 3, 1, 7, 0, 9, 5? 5000 can be formed out of the digits 3,4,5,6
(a) 720 (b) 600 and 7(no. digit is repeated). The number of
such is
(c) 120 (d) none
(a) 72 (b) 27
74. How many different numbers of six digits can
be formed with the digits 3, 1, 7, 0, 9, 5 in which (c) 70 (d) none
the numbers have ‘0’ in ten’s place? 83. 4 digit numbers to be formed out of the figures
(a) 600 (b) 360 0, 1, 2, 3, 4 (no digiti is repeated) then number
of such numbers is
(c) 120 (d) none
(a) 120 (b) 20
75. How many different numbers of six digits can
be formed with the digits 3, 1, 7, 0, 9, 5 in which (c) 96 (d) none
the numbers have ‘5’ in ten’s place? 84. The number of numbers lying between 100 and
1000 can be formed with the digits 1,2,3, 4, 5,
(a) 600 (b) 360
6, 7 is
(c) 120 (d) none (a) 210 (b) 200
76. How many other numbers of six digits can be (c) 110 (d) none
formed by using the digits of the number 85. The number of numbers lying between 10 and
530498? 1000 can be formed with the digits 2,3,4,0,8,9
(a) 600 (b) 240 is
(c) 599 (d) none (a) 124 (b) 120
77. How many numbers of six digits less than
(c) 125 (d) none
500000 can be formed by using the digits of
the number 530498? 86. The total number of 9 digit numbers of
different digits is
(a) 600 (b) 240
(c) 720 (d) none (a) 10 9 (b) 8 9
78. How many four digit numbers greater than (c) 9 9 (d) none
3,000 can be formed out of the digits, 1, 2, 3, 5,
7, 8, 9 if no digit is repeated in any number ? 87. The number of even numbers greater than 300
(a) 600 (b) 800 can be formed with the digits 1, 2, 3, 4, 5
without repetion is
(c) 840 (d) none
(a) 110 (b) 112
79. How many numbers between 300 and 3,000
can be formed with the digits 0, 1, 2, 3, 4 and 5, (c) 111 (d) none
(m + n)
no digit being repeated in any number? 88. What is the value of m if P2 = 56 and (m – n)P2=
(a) 160 (b) 100 12 ?
(a) 3 (b) 4
(c) 180 (d) none
80. Find the numbers that can be formed by the (c) 6 (d) none
digits 1, 2, 3, 4, 3, 2, 1 by placing the odd digits 89. m + n p2 = 56, m - n p2 = 30, then
at odd places. (a) m =6, n = 2 (b) m = 7, n= 1
(a) 16 (b) 18 (c) m=4,n=4 (d) None
(c) 10 (d) none

130 BUSINESS MATHEMATICS, LOGICAL REASONING & STATISTICS (Paper 3) [CA Foundation]
90. If n1+n2p2 =132,n1-n2p2 = 30 then, 100. If 10C3 + 2(10C4) + 10C5 = 12Cx find x.
(a) n1 = 6, n2 = 6 (b) n1 = 10, n2 = 2 (a) 6 (b) 4
(c) n1 = 9, n2 = 3 (d) none (c) 7 (d) none
12 12
91. If all the permutations of the letters of the 101. The value of C4 + C3 is
word “VRUNDA” be written down as in (a) 715 (b) 710
dictionary, what is the rank of the word (c) 716 (d) none
VRUNDA?
102. The solution of 19Cx + 2 = 19C2x – 1 is
(a) 696 (b) 124
(a) 2 (b) 5
(c) 712 (d) none
(c) 3 (d) none
92. If all the permutations of the letters of the 2n n n
103. The value of C2 if C4 = C3 is
word “chalk” are written in a dictionary the
rank of this word will be ____________. (a) 91 (b) 42
(a) 30 (b) 31 (c) 21 (d) none
9 9 10 11
(c) 32 (d) None 104. The value of C5 + C6 + C7 + C8 is
93. The value of n, if four times the number of (a) 240 (b) 205
permutations of n things taken three together (c) 495 (d) none
12 12
is equal to five times the number of 105. For what value of x, Cx + 3 = Cx + 5 is true ?
permutations of (n – 1) things taken 3 together (a) 0 (b) 4
is (c) 2 (d) none
(a) 20 (b) 5 106. What is the value of r if Cr – 2 = 12Cr ?
12

(c) 15 (d) none (a) 10 (b) 7


94. In a group of boys the number of arrangement (c) 6 (d) none
of 4 boys is 12 times the number of 107. 18 18
If Cr = Cr +2 the value of C5 is r

arrangements of 2 boys. The number of boys (a) 55 (b) 50


in the group is
(c) 56 (d) none
(a) 10 (b) 8 n n 25
108. If C10= C14 then Cn is
(c) 6 (d) none
(a) 24 (b) 25
95. What is the value of n if Pn : 12Pn = 3 : 4?
11
(c) 1 (d) none
(a) 4 (b) 3
109. If 24C2r = 24C2r-4 then r then the value of r is
(c) 8 (d) none
n–1 n –1
(a) 7 (b) 5
96. Pr + r. Pr – 1 =
(c) 6 (d) none
(a) n– 1Pr (b) n
Pr 1,000 999 x
n +1
110. If C98 = C97 + C901; fmd x.
(c) Pr (d) none
(a) 99 (b) 998
10 (c) 999 (d) none
97. The value of  r rPr is 111. 500 499 n
If C92 = C407 + C91 then n is
r=1
(a) 501 (b) 500
(a) 11P11 (b) 11P11-1
(c) 502 (d) 499
(c) 11P11+1 (d) none
8 8
112. For what value of x, Px = 24 ´ 7Cx is true?
7
98. The values of C4 + C3 is
(a) 6 (b) 4
(a) 120 (b) 126
(c) 7 (d) none
(c) 150 (d) none n n
10 10 10 12
113. lf Cn – 4 = 35, the value of P2 is
99. If C3 + 2( C4) + C5 = Cx find x.
(a) 36 (b) 24
(a) 6 (b) 5
(c) 42 (d) none
(c) 4 (d) none

SEQUENCE AND SERIES 131


114. The solution of 2xC3 = xP4 is 125. From a group of 7 boys and 3 girls a committee
(a) 6 (b) 5 of four students is to be formed. The number
(c) 7 (d) none of ways in which it can be formed so as to
n+1 n include exactly 2 girls is
115. Find n if C6 : C5 = 11 : 6.
(a) 72 (b) 56
(a) 12 (b) 10
(c) 63 (d) none
(c) 16 (d) none
126. In how many ways a committee of 4 members
116. If 2nC2 + nC2 = 55, find n. can be formed out of 4 professors, 6 boys and
(a) 5 (b) 2 2 girls so as to include I professor and at least 1
(c) 6 (d) none girl?
117. n n
Value of Cr+ Cr –1 is (a) 172 (b) 144
(a) n +1Cr (b) n–1
Cr (c) 160 (d) none
n+1
(c) Cr + 1 (d) none 127. A committee of 6 ministers is to be formed
118. nC1 + nC2 + nC3 + ..... + nCn equals from 16 cabinet ministers. In how many ways
(a) 2n-1 (b) 2n can it be formed so as to include prime
minister and finance minister?
(c) 2n +1 (d) none
n n
(a) 1000 (b) 1220
119. If Pr = 360 and Cr = 15, n and r are respectively
(c) 760 (d) none
(a) 6,4 (b) 4,6
(c) 6,6 (d) none 128. An urn contains 4 black, 3 white and 5 blue
balls. In how many way 2 balls of the same
120. If nPr = 336 and nCr = 56, then nand r will be
colours can be selected from it?
(a) (3, 2) (b) (8, 3)
(a) 10 (b) 12
(c) (7, 4) (d) none (c) 19 (d) none
121. In how many ways a committee of 6 persons 129. An urn contains 4 black, 3 white and 5 blue
can be formed from 6 professors and 8 students balls. In how many way 2 balls of different
so as to include at least one and at the most 3 colours can be selected from it?
professors?
(a) 47 (b) 24
(a) 2406 (b) 2506
(c) 76 (d) none
(c) 2606 (d) none
130. In how many ways 3 cards of the same suit can
122. A person has 10 friends including 4 females. In be drawn out of 52 cards?
how many ways can he invite 5 friends so as to
(a) 1224 (b) 1144
include at least 2 females?
(c) 1414 (d) none
(a) 186 (b) 180
(c) 190 (d) none 131. A bag contains 8 rupee coins 6 fifty paise coins
123. A person wants to invite 4 friends out of 7. If and 4 twenty paise coins. In how many ways
two particular friends do not wish to come selection of 3 coins can be made so that one is
together, in how many ways can he invite of each denomination?
them? (a) 56 (b) 192
(a) 35 (b) 20 (c) 120 (d) none
(c) 25 (d) none 132. A bag contains 8 rupee coins 6 fifty paise coins
124. A cricket team of 17 players consists of 3 fast and 4 twenty paise coins. In how many ways
bowlers, 4 spinners, 2 wicketkeepers and 1 selection of 3 coins can be made so that none
captain. In how many ways 11 players can be is a rupee coin?
selected out of them so that captain, 2 fast
(a) 56 (b) 192
bowlers, 1 wicket-keeper and 2 spinners are
(c) 120 (d) none
included?
(a) 720 (b) 756 133. A box contains 7 red, 6 white and 4 blue balls.
(c) 760 (d) none How many selections of three balls can be

132 BUSINESS MATHEMATICS, LOGICAL REASONING & STATISTICS (Paper 3) [CA Foundation]
made so that all three balls are red balls? (a) 400 (b) 440
(a) 35 (b) 120 (c) 441 (d) none
(c) 168 (d) none 142. A question–paper includes 12 questions in all.
134. A box contains 7 red, 6 white and 4 blue balls. If 7 questions including at least one from the
How many selection of three balls can be made first three questions are to be attempted, the
so that there is one ball of each colour? number of ways of answering the paper is
(a) 35 (b) 120 (a) 156 (b) 124
(c) 168 (d) none (c) 756 (d) none
143. The question–paper of CAIT contains 10
135. A box contains 7 red, 6 white and 4 blue balls.
questions divided into two sections of 5
How many selection of three balls can be made
questions each. In how many ways can an
so that none is a red ball
examinee answer 6 questions taking at least 2
(a) 35 (b) 120 questions from each section?
(c) 168 (d) none (a) 600 (b) 400
136. In how many ways a committee of 3 ladies and (c) 200 (d) none
4 gentlemen be appointed from a group of 8 144. A candidate is required to answer 6 out of 12
ladies and 7 gentlemen if Mrs. X refuses to questions which are divided into two groups
serve on a committee if Mr. Y is a member? containing 6 questions in each group. He is not
(a) 1960 (b) 1540 permitted to attempt more than four from any
(c) 1560 (d) none group. The number of choices are.
137. From a group of 7 boys and 3 girls a committee (a) 750 (b) 850
of four students is to be formed. The number (c) 800 (d) none
of ways in which it can be formed so as to 145. In how many ways a committee of 2 professors
include at least one girl is and 4 students can be formed from 3
(a) 175 (b) 176 professors and 10 students if one particular
(c) 180 (d) none professor is to be included ?
138. In how many ways can a committee of 5 (a) 660 (b) 420
persons be formed out of 6 boys and 7 girls (c) 630 (d) none
such that each committee must include at least 146. In how many ways a committee of 2 professors
one boy? and 4 students can be formed from 3
(a) 1266 (b) 1820 professors and 10 students if one particular
(c) 1366 (d) none student is not to be included in the
139. Of 10 electric bulbs, three are defective but it committee?
is not known which are defective. In how many (a) 630 (b) 420
ways, three bulbs can be selected so as to (c) 378 (d) none
include at least one defective bulb? 147. The number of arrangements of 10 differcnt
(a) 60 (b) 100 things taken 4 at a time in which one particular
(c) 80 (d) none thing always occurs is
140. Five bulbs of which 3 are detective, are to be (a) 2015 (b) 2016
tried in two light–points in a dark room. In how (c) 2014 (d) none
many trials the room shall be lighted ? 148. The number of permutations of 10 different
(a) 6 (b) 7 things taken 4 at a time in which one particular
(c) 8 (d) none thing never occurs is
141. Out of 7 gents and 4 ladies a committee of 5 is (a) 3020 (b) 3025
to be formed. The number of committees such (c) 3024 (d) none
that each committee includes at least one lady
is

SEQUENCE AND SERIES 133


149. In a certain examination a student has to secure members can be formed from these, if each
a certain percentage of marks in each of the committee is to include at least 3 ladies.
five subjects. In how many ways can he fail? (a) 600 (b) 120
(a) 63 (b) 32 (c) 336 (d) none
(c) 31 (d) none 158. The total number of ways in which six ‘+’ and
150. A question-paper consists of six questions. In four ‘-’ signs can be arranged in a line such that
how many ways can a candidate attempt one no two ‘ _’ signs occur together is
or more questions?
(a) 7/ 3 (b) 6X 7/ 3
(a) 64 (b) 32
(c) 35 (d) none
(c) 63 (d) none
159. In how many ways can 5 persons be seated at a
151. A person has 8 friends. The number of ways in
round table conference?
which he may invite one or more of them to a
(a) 120 (b) 12
dinner in.
(c) 24 (d) none
(a) 250 (b) 255
160. The number of ways in which 7 girls form a
(c) 200 (d) none ring is :
152. The number of ways in which a person can (a) 700 (b) 710
chose one or more of the four electrical
(c) 720 (d) none
appliances : T.V, Refrigerator, Wasning
161. The number of ways in which 7 boys sit in a
Machine and a cooler is
round table so that two particular boys may sit
(a) 15 (b) 25
together is
(c) 24 (d) none
(a) 240 (b) 200
153. The number of ways a person can contribute
(c) 120 (d) none
to a fund out of 1 ten-rupee note, 1 fiverupee
162. If 50 different jewels can be set to form a
note, 1 two-rupee and lone rupee note is
necklace then the number of ways is
(a) 15 (b) 25
(c) 10 (d) none 1 1
(a) 50 (b) 49
154. A question paper contains 6 questions, each 2 2
having an alternative.
(c) 49 (d) none
The number of ways an examine can answer
one or more questions is 163. 3 ladies and 3 gents can be seated at a round
(a) 720 (b) 728 table so that any two and only two of the ladies
(c) 729 (d) none sit together. The number of ways is
155. Out 10 consonants and 5 vowels, how may (a) 70 (b) 27
different words can be formed each consisting (c) 72 (d) none of these
of 3 consonants and 2 vowels ? 164. 5 persons are sitting in a round table in such
(a) 10C3 x 5C2 (b) 10
C3 x 5C2 x 5! way that Tallest Person is always on the right-
(c) 5! (d) none side of the shortest person; the number of such
156. The number of different words that can be arrangements is -
formed with 12 consonants and 5 vowels by (a) 6 (b) 8
taking 4 consonants and 3 vowels in each word
(c) 24 (d) none
is
165. In how many ways can 6 boys and 6 girls be
(a) 12C4 x 5C3 (b) 17C7
seated around a table so that no 2 boys are
(c) 4950 x 7 (d) none adjacent?
157. There are 8 gentlemen and 4 ladies. Find the (a) 4! × 5! (b) 5! × 6!
6
number of ways in which the committee of 7 (c) P 6 (d) 5 x 6P6

134 BUSINESS MATHEMATICS, LOGICAL REASONING & STATISTICS (Paper 3) [CA Foundation]
166. In how many ways can 4 Americans and 4 176. The number of straight lines obtained by
English men be seated at a round table so that joining 16 points on a plane, not more than
no 2 Americans may be together? two of them being on the same line, is
4
(a) 4! × 3! (b) P4 (a) 120 (b) 110
4 4
(c) 3 × P4 (d) C4 (c) 210 (d) none
167. In how many ways 7 men and 8 women sit at a 177. Every person shakes hands with each other in
round table so that no two men are together? a party and the total number of hand shakes is
(a) 5! ÷ 2 (b) 5! 66. The number of guests in the party is
(c) 8 x (7!) 2
(d) 7! (a) 11 (b) 12
168. In how many ways 5 men and 3 women are (c) 13 (d) 14
arranged at a round table if the women never 178. The number of parallelograms that can be
sit together? formed from a set of four parallel lines
(a) 6 × 6! (b) 6! intersecting another set of three parallel lines
(c) 7! (d) None is
(a) 6 (b) 18
169. In how many ways 5 men and 3 women are
arranged at a round table if the women always (c) 12 (d) 9
sit together? 179. 8 points are marked on the circumference of a
circle. The number of chords obtained by
(a) 6 × 6! (b) 6!
joining these in pairs is
(c) 7! (d) None
(a) 25 (b) 27
170. How many words of 4 letters can be formed by (c) 28 (d) none
taking 4 letters form the word GUJARAT in
180. The number of ways in which 12 students can
which G must come?
be equally divided into three groups is
(a) 720 (b) 360 (a) 5775 (b) 7575
(c) 288 (d) none (c) 7755 (d) none
171. The ways of selecting 4 letters from the word
EXAMINATION is 181. The number of ways in which 15 mangoes can
(a) 136 (b) 130 be equally divided among 3 students is
(c) 125 (d) none
172. The number of 4 digit numbers formed with (a) 15 / (5)4 (b) 15 / (5)3
the digits 1, 1, 2, 2, 3, 4 is
(c) 15 / (5)
2 (d) none
(a) 100 (b) 101
(c) 201 (d) none 182. The number of ways in which 9 things can be
173. The sum of all 4 digit number containing the divided into three groups containing 2,3, and
digits 2, 4,6,8, without repetitions is 4 things respectively is
(a) 133330 (b) 122220 (a) 1250 (b) 1260
(c) 213330 (d) 133320 (c) 1200 (d) none
174. The number of diagonals in a decagon is 183. Eight guests have to be seated 4 on each
(a) 30 (b) 35 longest side of a rectangular table 2 particular
(c) 45 (d) none guests desire to sit on one particular side of
175. There are 12 points in a plane of which 5 are the table and 3 on the other side. The number
collinear. The number of triangles is of ways in which the sitting arrangements can
be made is
(a) 200 (b) 211
(a) 1732 (b) 1728
(c) 210 (d) none
(c) 1730 (d) 1278

SEQUENCE AND SERIES 135


184. The number of words that can be made by 192. The solution of 2xC3 = xP4 is
rearranging the letters of the word APURNA (a) 6 (b) 5
so that vowels and consonants appear (c) 7 (d) none
alternate is
193. A team of 12 men is to be formed out of n
(a) 18 (b) 35 persons. Then the number of times 2 men ‘A’
(c) 36 (d) none and ‘B’ are select together is ___________.
185. The results of 8 matches (Win, Loss or Draw) (a) nC 12 (b) n-1
C1
are to be predicted. The number of different (c) n-2
C 10 (d) None
forecasts containing exactly 6 correct results
194. A cricket team of 11 players is to be formed
is
from 16 players include 4 bowlers and 2 wicket-
(a) 316 (b) 214 keepers.
(c) 112 (d) none In how many different ways a team can be
186. The number of different factors the number formed so that the team contains exactly 3
75600 has is bowlers and 1 wicket-keeper’?
(a) 120 (b) 121 (a) 960 (b) 2500
(c) 119 (d) none (c) 2472 (d) none
187. How many telephones connections may be 195. A cricket team of 11 players is to be formed
allotted with 8 digits form the numbers 0 1 2 from 16 players including 4 bowlers and 2
…….9? wicket-keepers. In how many different ways
(a) 108 (b) 10! a team can be formed so that the team contains
10
(c) C 8 (d) 10
P8 at least 3 bowlers and at least I wicket-keeper?
188. In how many different ways 3 rings of a lock (a) 960 (b) 2500
can not combine when each ring has digits 0 1 (c) 2472 (d) none
2……9 leading to unsuccessful events?
(a) 999 (b) 103
(c) 10! (d) 997 
189. The total number of numbers less than 1000
and divisible by 5 formed with 0 1 2…..9 such
that each digit does not occur more than once
in each number is
(a) 150 (b) 152
(c) 154 (d) None
190. 2n can be written as

(a) 2n {1.3.5 .... (2n-1)} n


(b) 2n n
(c) {1.3.5 ..... (2n -1) }
(d) none
191. At an election there are 5 candidates and 3
members are to be elected. A voter is entitled
to vote for any number of candidates not
greater than the number to be elected. The
number of ways a voter choose to vote is
(a) 20 (b) 22
(c) 25 (d) none

136 BUSINESS MATHEMATICS, LOGICAL REASONING & STATISTICS (Paper 3) [CA Foundation]
9. Out of 6 teachers and four boys, a committee
of eight is to be formed. In how many ways
HOME WORK - 1 can this be done when there should not be
less than four teachers in the committee.
(a) 45 (b) 55
1. Eleven students are participating in a race. In (c) 30 (d) 50
how many ways the first 5 prizes can be won? 10. Find the number of even numbers greater than
(a) 44550 (b) 55440 100 that can be formed with the digits 0, 1, 2,
(c) 120 (d) 90 3?
2. A box contains 7 red, 6 white and 4 blue balls. (a) 10 (b) 15
How many selections of three balls can be (c) 20 (d) None
made so that none is red? 11. How many words can be formed with the
(a) 90 (b) 120 letters of the word “UNIVERSITY”, the vowels
(c) 48 (d) 24 always remaining together?
3. A letter lock has three rings each marked with (a) 60480 (b) 60482
10 different letters. In how many ways it is (c) 60000 (d) None
possible to make an unsuccessful attempt to 12. Six boys and five girls are to be seated in a row
open the lock ? such that no two girls and no two boys sit
(a) 1000 (b) 999 together. Find the number of ways in which
(c) 5040 (d) None this can be done.
4. In how many ways can the letters of words (a) 86,400 (b) 85,000
“ACCOUNTANT” be arranged if vowels always (c) 85,400 (d) None
occur together 13. There are 3 copies each of two books and two
(a) 7560 (b) 7650 copies each of five books. In how many ways
(c) 7660 (d) 7550 can a book seller arrange the 16 books in a shelf
5. If in a party every person gives a gifts to each so that the copies of the same book are never
other and total number of gift taken is 132. separated?
The number of guests in the party is (a) 5040 (b) 5000
(a) 11 (b) 12 (c) 5030 (d) None
(c) 13 (d) 14 14. How many words can be formed with the
6. 6 seats of articled clerks are vacant in a letters of the world “PARALLEL” so that all L’s
‘Chartered Accountant firm’. How many do not come together?
different batches of candidates can be chosen (a) 2000 (b) 3000
out of 10 candidates if one candidate is always (c) 4000 (d) None
selected. 15. Find the number of different poker hands in a
(a) 124 (b) 125 pack of 52 playing cards.
(c) 126 (d) None (a) 2598960 (b) 1506210
7. In how many ways can the letters of the word (c) 5298216 (d) None
PENCIL be arranged so that N is always next to 16. In an examination a candidate has to pass in
E each of the 4 papers. In how many different
(a) 60 (b) 40 ways can be failed?
(c) 720 (d) 120 (a) 14 (b) 16
8. 7 distinct things are to be divided in 3 groups, (c) 15 (d) None
consisting of 2, 2 and 3 things respectively, no. 17. A boat is to be manned by 8 men, 4 on each of
of ways this can be done is equal to the 2 sides, of which 3 can row only one side
___________ and 2 only on the other. In how many ways can
(a) 110 (b) 105 the crew be arranged?
(c) 100 (d) None (a) 1720 (b) 1700
(c) 1728 (d) None

SEQUENCE AND SERIES 137


18. Mr. X has 8 children of which he takes 3 at a 27. How many three digit numbers are there, with
time to the circus. Find, how many times a distinct digits, with each digits odd
particular child goes to the circus? (a) 120 (b) 60
(a) 20 (b) 30 (c) 30 (d) 15
(c) 21 (d) None 28. n n+1
If P13 : P12 = 3 : 4 then value of n is
19. If 12C5 + 2 12C4 + 12C3 = 14Cx then the value of x is:
(a) 15 (b) 14
(a) 5 (b) 9
(c) 5 or 9 (d) None (c) 13 (d) 12
20. A gentlemen invites 6 of his friends to a party. 29. How many numbers greater than 2000 can be
In how many different arrangements they formed with the digits 1, 2,3,4,5?
along with the wife of the gentleman can sit at (a) 216 (b) 120
a round table for a dinner if the host and his (c) 24 (d) 240
wife always sit side by side?
(a) 1440 (b) 144 30. If = then the value of n is
(c) 1445 (e) None
21. Five balls of different colours are to be placed ______.
in three boxes of different sizes. Each box can (a 0 (b) –2
hold all the five balls. In how many different (c) 8 (d) None
ways can we place the balls so that no box
remains empty?
(a) 100 (b) 120
(c) 150 (d) None ANSWER KEYS
22. How many numbers greater than 1000 can be
formed with the digits of the number 23416; if
the digits are not repeated in the same number. 1 (b) 2 (b) 3 (b) 4 (a)
(a) 120 (b) 200
(c) 240 (d) None
5 (b) 6 (c) 7 (d) 8 (b)
23. How many numbers can be formed with the
digits of the number 112321 that are greater
than one lakh? 9 (a) 10 (c) 11 (a) 12 (a)
(a) 60 (b) 80
(c) 70 (d) None 13 (a) 14 (b) 15 (a) 16 (c)

17 (c) 18 (c) 19 (c) 20 (a)


24. Evaluate :
21 (c) 22 (c) 23 (a) 24 (a)
(a) 249900 (b) 24990
(c) 249000 (d) None 25 (a) 26 (a) 27 (b) 28 (a)
25. In how many ways can 12 different things be
equally distributed among 4 groups?
(a) 15,400 (b) 15,000 29 (a) 30 (c)
(c) 14,400 (d) None
26. There are 7 routes from station X to station
Y. In how many ways one may go from X to Y
and return if for returning one makes a choice
of any of the routes?
(a) 49 (b) 17
(c) 42 (d) 35
138 BUSINESS MATHEMATICS, LOGICAL REASONING & STATISTICS (Paper 3) [CA Foundation]
9. If nPr = 720 and nCr = 120 then r is
(a) 4 (b) 5
HOME WORK - 2 (c) 3 (d) 6
10. A bag contains 4 red, 3 black and 2 white balls.
In how many ways 3 balls can be drawn from
1. The Sum of all the 4 digits’ numbers that can this bag so that they include at least one black
be formed with the digits 3,4,5,5 is ball?
(a) 18887 (b) 33333 (a) 46 (b) 64
(c) 38887 (d) 56661 (c) 86 (d) None
2. There are 12 points in a plane which are
11. If ncr–1 = 56, ncr = 28 and ncr+1 = 8, then r is equal
collinear no three points is a straight lie,
to
number of triangular that can be formed with
the vertices as there points are: (a) 8 (b) 6
(a) 216 (b) 220 (c) 5 (d) none of these
(c) 110 (d) 108 12. The Supreme Court has given a 6 to 3 decision
3. In how many different ways can four persons upholding a lower court; the number of ways
stand in a line for a group photograph. it can give a majority decision reversing the
(a) 24 (b) 16 lower court is
(c) 8 (d) 64 (a) 256 (b) 276
4. A Company wishes top simultaneously (c) 245 (d) 226
promotes three of its 8 department assistant
mangers. In how many ways these promotions
can take place?
(a) 336 (b) 56
ANSWER KEYS
(c) 8 (d) 1680
5. From a committee of 8 persons, in how many 1 (d) 2 (b) 3 (a) 4 (b)
ways can we choose a chairman and a vice
chairman assuming one person cannot hold
more than one position? 5 (b) 6 (b) 7 (c) 8 (a)
(a) 50 (b) 56
(c) 62 (d) none of these 9 (c) 10 (b) 11. (b) 12. (a)
6.
(a) 7 (b) 8
(c) 9 (d) 6
7. The number of words from the letters of the
word BHARAT, in which B and H will never
come together, is
(a) 120 (b) 360
(c) 240 (d) None
1 1 N
8. The value of N in + = is
7! 8! 9!
(a) 81 (b) 64
(c) 78 (d) 89

SEQUENCE AND SERIES 139


CHAPTER-6
SEQUENCE AND SERIES

INTRODUCTION
1.0 SEQUENCE
An arrangement of numbers in a definite order according to some rule is called a sequence.
The various numbers occuring in a sequence are called its ems we denote the terms of a
sequence by a1, a2, a3 ... etc. The nth term an is also called general term.
A swquence is said to be finite or infinite according as it has finite or infinite terms.
Examples :
1. 1, 3, 5, 7, .... is an infinite sequence whose nth term is given by the formula an = 2n – 1, where n
is a natural number.

2. 1,
1 , 1 , 1 , ... is an infinite sequence where n th term is given by the formula
2 3 4

an = 1 , where n is a natural number..


n
3. 2, 4, 6, 8, 10, 12 is a finite sequence in which each term is obtained by adding 2 to the
previous term.
2.0 ARITHMETIC PROGRESSION (A.P.)
1. General Term If a is the first term and d is the common difference
of An A.P. of an A.P. then its nth term tn is given by -
tn = a + (n – 1)d
2. Sum of n Terms The sum Sn of first n terms of an A.P. with first term
of an A.P. a and common difference d is given by -

Sn =
n [2a + (n – 1)d]
2

or, Sn =
n [a + l], where l is the last term a + (n – 1)d.
2

TUTORIAL NOTES

1. If the sum Sn of first n terms of a sequence is

140 BUSINESS MATHEMATICS, LOGICAL REASONING & STATISTICS (Paper 3) [CA Foundation]
given, then the n th term, a n of he sequence can be
determined by
a n= S n – S n – 1
2. If he sum of first n terms of an A.P. is of he form
An 2 + Bn, where A and B are constant, then
common difference is 2A.
3. Properties of A.P. 1. If a constant is added or subracted from each term
of an A.P., then the resulting sequence is also in
A.P., with same common difference.

2. If each term of an A.P. is multiplied or divided by a


non-zero constant k, then the resulting sequence is

also an A.P. with common difference kd or


d,
k
where d is he common difference of the given A.P.

3. If a1, a 2, a3, ... and b1, b2, b3, ... are two arithmetic
progessions, then the sequence
a1 + b1, a2 + b2, a3 + b3, ... is also an A.P.

4. In a finite A.P. he sum of the terms equidistant


from the beginning and end is always same and is
equal to the sum of first and last term.

i.e. ak + an–(k–1) = a1 + an, k = 1, 2, 3,... n – 1

5. Three numbers a, b, c are in A.P. if 2b = a + c.

4. Sum of n Arithmetic =
n (a + b)
2
means between a and b.

3.0 GEOMETRIC PROGRESSION (G.P.)

General Term If a is the first term and r is the common ratio of

of G.P. G.P. then the terms of G.P. are

a, ar, ar2, ar3 ...

and the nth term tn is given by -

tn = ar(n–1)

Sum of n Terms of a G.P. the sum of first n terms of a G.P. with first term a and

SEQUENCE AND SERIES 141


common ratio r is given by

 a ( r n 1)
 if r  1
Sn =  r  1
 na if r  1

Sum of an Infinite G.P. The sum of an infinite G.P. with first term a and common
ratio r (|r| < 1) is

a
S¥ = 1  r

Geometric Mean (a) Single Geometric Mean : A number G is said to be


the geometric between two non-zero numbers a
and b if a, G, b are in G.P. i.e. if
G2 = ab.
For Example : Since G 2, 4, 8 are in G.P., therefore 4
is G.M. at between 2 and 8.
(b) n-Geometric Mean : The number, G1, G2, .... Gn are
said to be n-geometric means between two non-
zero numbers a and b if a, G1, G2, .... Gn, b are in
G.P.

TUTORIAL NOTE

The product of the n geometric means is

n
(ab)n/2 =  ab  .

4.0 HARMONIC PROGRESSION (H.P.)


A sequence of non-zero numbers a1, a2, a3, ... is said to be a harmonic progression if the
sequence

1 1 1
a1 , a2 , a3 , ... in an A.P.
.P.

1 1 1
For Exmaple, The sequence 1, , , , ... is a H.P. because the sequence 1, 3, 5, 7,.... is an
3 5 7
A.P.

142 BUSINESS MATHEMATICS, LOGICAL REASONING & STATISTICS (Paper 3) [CA Foundation]
1 1 1
A general H.P. is , a  d , a  2d , ...
a

5.0 SOME SPECIAL SEQUENCES

n ( n 1)
1. The Sum of the first n  n = 1 + 2 + ... + n = 2

Natural Numbers

n( n 1)  2 n 1
2 The sum of the Squares of  n2 = 12 + 22 + ... + n2 =
6

first n Natural Numbers

2
 n( n  1) 
3 The sum of the Cubes of  n3 = 13 + 23 + ... + n3 =  2 
 

first n Natural Numbers

4 If the nth term of a  n =  Tn = A  n3 + B  n2 + C  n + nD

sequeance is
Tn = An3 + Bn2 + Cn + D
Then the sum of n terms
is given by

SEQUENCE AND SERIES 143


ARITHMETIC & GEOMETRIC
A.P. G.P.
Series having equal Difference Series having equal ratio is known as G.P.
is known A.P.
3,7,11,15,19,23 1,4,16,64

T2 T3 T4 Tn
d= T2-T1=T3-T2=T4-T3=........=Tn-Tn-1 R= T = T = T = T
1 2 5 n-1

a=3 d=4 a=1 r=4


A.P. G.P.
T1=a T1=a
T2=a+d T2=a.r
T3=a+2d T3= a.r2
T4=a+3d T4=a.r3
T
5
=a+4d T5=a.r4
Tn=a+(n-1)d Tn=a.rn-1

 Rules of A.P.G.P.
(1) nthterm =last term
Tn = l
Tn= a+(n-1)d
T10=a+9d
T5= a+4d
T100= a+99d
(2) Sum of first ‘n’ term (Sn)

n
sn= [2a+(n-1)d]
2
 when last term(l) is given in question

n
sn= (a+l)
2
(3) Find out value of ‘n’
When a=d When a  d

l l-a
n= n= +1
a d
Proof:

144 BUSINESS MATHEMATICS, LOGICAL REASONING & STATISTICS (Paper 3) [CA Foundation]
l= a+(n-1)d

l-a
+1 = n
d

l-a
n= +1
d

l-a
n= +1
a

l
n=
a
(4) Sum of 1st ‘n’ odd no. = n2
1+3+5+7+9.....’n’ terms
a=1 d=2
Proof:

n
Sn= [2a+(n-1)d]
2

n
= [2+(n-1)2]
2

n
= [2+2n-2]
2

n
 2n 
2
Sn = n2
(5) Sum of 1st ‘n’ even no. = n (n+1)
2+4+6+8+....... 2n
a=2 l=2n
Proof :

Sn= n 2  a + l 

= n 2 ×  2 + 2n 

= n 2 × 2  1 + n

Sn = n (n+1)

SEQUENCE AND SERIES 145


(6) Sum of 1st ‘n’ natural no.
n(n + 1)
n = 2
1+2+3..... +n
Proof :

n
Sn =  a + l
2
a= 1 l=n

n
 1 + n
2

n  n + 1
n =
2
(7) Sum of squares of 1st ‘n’ natural No.
ex: 12+22+32........+n2

2 n  n + 1  2n + 1 
n =
6
(8) Sum of cubes of 1st ‘n’ natural No.
2 2 2
n n + 1  n n + 1  2
 n3 = =  =   n
4  2 

(9) Sm = n Sn =m
S m+n = -(m+n)
eg :
(1) S40=20 S20=40
S60=-60
(2) S100=40 S40=100 S140=-140

Sn= n 2 (2a + 99d)

40 = 50(2a + 99d)
2a+99d=0.8......(i)
100=20 (2a+39d)
2a+39d=5.....(ii)
From equation (i) & (ii )

60d=-4.2

146 BUSINESS MATHEMATICS, LOGICAL REASONING & STATISTICS (Paper 3) [CA Foundation]
d=-0.07
2a+39(-0.07)=5
2a=5+2.73
a=3.865

140
S 140 = [2(3.865)+139(-0.07)]
2
=70[7.73-9.73]
=70 (-2)
= -140
(10) Tm=n Tn=m
Tr=m+n-r
eg. Tx=m+n-x
Tp=m+n-p
eg. T30=20 T20=30
T15=30+20-15=35
or
eg. T60=20 T20=60
T40=20+60-40=40
Proof:
a+59d=20.....(i)
a+19d=60.....(ii)
40d=-40
d=-1
a+59(-1)=20
a-59=20
a=20+59
a=79
T40=a+39d
= 79+39(-1)
=40
 Tm=n Tn=m
Tm+n=m+n-(m+n)
Tm+n = 0

 Tm-n = m+n-(m-n)
= m+n-(m+n)
Tm+n = 2n

SEQUENCE AND SERIES 147


1 1
(11) Tm= Tn=
n m

1
Smn= (mn+1)
2
Tmn=1

(12) When Sn is given & we find out Tn


Tn =Sn-Sn-1
(13) When Tn is given & we find out Sn
Sn =  T n

T1+T2+T3.........Tn= Sn=  Tn
(14) Arithmethic Mean

a+b
A.m.=
2
eg: 1,3,5,7 are in A.P.

1+ 5 3+7
3= 5=
2 2
3=6/2 5=10/2
3=3 5=5
(15) Geometric Mean

G.m= ab
or
( G.m)2 = ab
eg. 1,2,4,8,16 are in G.P.

8= 4 × 16

(2)2=1x4 8= 64
4=4 8=8

(16) When ratio of Sn is given & we find Ratio of Tn


n = 2 (term) -1
(17) Sum of 3 no. in A.P.= 3a [a-d,a,a+d]
(18) Sum of 4 no. in A.P.= 4a [ a-3d,a-d,a+d,a+3d]
(19) Sum of 5 no. in A.P.= 5a [a-2d,a-d,a,a+d,a+2d]

148 BUSINESS MATHEMATICS, LOGICAL REASONING & STATISTICS (Paper 3) [CA Foundation]
(20) Sum of squares of 3no. in A.P.= (3a2+2d2)
(21) Sum of Squares of 4 no. in A.P.= (4a2 +20d2)
(22) Sum of squares of 5.no. in A.P.= (5a2+10d2)
Note: When we have to find out numbers, always use trial & error method.
Otherwise always use actual method.

GEOMETRIC PROGRESIONS

(1) Last term = nth term. Tn= l = a.rn-1


T10 = a.r9
T3 = a.r2
T8 = a.r7
(2) Sum of ‘n’ term
When When Where
r= 1 r> 1 r< 1

 rn - 1  1 - r 
n

Sn = n.a Sn =a Sn =a
r -1 1-r
eg. 5+5+5...... 100 Terms
a= 5 r= 1
Sn= n.a
= 100x 5
=500
(3) Sum of Infinite terms in G.P.
T1+T2+T3......... 
a+ar+ar 2........... 

a
S 3
 = When -1 < r < 1
1-r
 r > 1  r = 
r= 2 21 = 2 210 = 1024
23=4 2 = 
23=8
 When r < 1  r  = 0
1
r= 1 2  1 2  =0.50

SEQUENCE AND SERIES 149


2 
1 1
  = 0.25    =0
2 2
(4) Sum of squares of Infinite terms

T12+ T22+T33......... T 2

a2+a2r2+a2r4..... 

2
a2
S = 2 When -1 < r < 1
1-r
(5) Sum of cubes of Infinite terms

T13+T23+T33........ T 3

3 a3
S  When -1 < r < 1
1 r3
(6) Ta= x Ta+d= y Ta+2d= z
a, a+d, a+2d are in A.P.
then
x,y,z are in G.P
y2= xz
eg. 1,2,4,8,16,32,64,128
T2=2 T5=16 T8=128
2,5,8 in A.P
2,16,128, in G.P.
eg. T5=x T10= y T15= z
5,10,15 in A.P.
x,y,z in G.P.
y2=xz
(7) Tp+q= m Tp-q= n
1
TP= mn = (mn) 2 = (mn)0.5

Proof: T1 , T2 , T3 , T4 , T5 , T6 , T7 , T8 ...
1, 3, 9, 27, 81,243,729
T4 +3 = T7 = 729 T4-3 = T1 = 1

Tp+q = m Tp-q = n

150 BUSINESS MATHEMATICS, LOGICAL REASONING & STATISTICS (Paper 3) [CA Foundation]
T4 = mn = 729 × 1 = 27
(8) S = Sum of first ‘n’ Terms of G.P.
P = Product of first ‘n’ terms of G.P.
R = Sum of Reciprocals of first ‘n’ terms of G.P.

P2 Rn = Sn

Proof : S= T1 + T3 + T3 ..... +Tn

P= T1  T2  T3 .......x Tn

1 1 1 1
R= T + T + T ........ T
1 2 3 n

P2 Rn = Sn

a
(9) 3 No in G.P. ,a, ar
r

a a 3
(10) 4 No. in G.P. 3 , ,ar,ar
r r

a a 2
(11) 5 No. in G.p. 2 , ,a,ar,ar
r r
Note: Product of No. must be given in question.
 Power Series
 Series not being A.P & G.P is known as power Series
 Always use trial & error Method in case of Power Series
 Always take n=2
 A.m  G.m.(2 positive no.)
a=b a b
A.m.=G.m. A.m.> G.m.

SEQUENCE AND SERIES 151


8. The mth term of an A. P. is n and nth term is m.
The rth term of it is
CLASS WORK
(a) m + n +r (b) n + m – 2r
(c) m + n + r/2 (d) m + n – r
1. The nth element of the sequence 1, 3, 5, 9. The number of the terms of the series 10
7,….…..is
2 1
(a) n (b) 2n – 1 + 9 + 9 + 9 + ......... will amount to 155 is
3 3
(c) 2n +1 (d) none of these
(a) 30 (b) 31
2. The nth element of the sequence –1, 2, –4, 8
….. is (c) a & b both (d) none
(a) ( –1 )n2 n–1 (b) 2 n–1 10. The nth term of the series whose sum to n
(c) 2n (d) none of these terms is 5n2 + 2n is
(a) 3n – 10 (b) 10n – 2
7
(c) 10n – 3 (d) none of these
3.  2i - 1 can be written as
i= 4 11. The 20th term of the progression 1, 4, 7,
10.................is
(a) 7 + 9 + 11 + 13
(a) 58 (b) 52
(b) 2 7 + 2 9 + 2 11 + 2 13 (c) 50 (d) none of these
12. The last term of the series 5, 7, 9,….. to 21 terms
(c) 2 7 + 2 9 + 2 11 + 2 13 is
(d) none of these (a) 44 (b) 43
4. The sum to  of the series –5, 25, –125 , 625,
(c) 45 (d) None of these
….. can be written as
13. The 4 arithmetic means between –2 and 23 are
 k  k
(a)   -5  (b)  5 (a) 3, 13, 8, 18 (b) 18, 3, 8, 13
k=1 k=1
(c) 3, 8, 13, 18 (d) none of these
 k 14. The first term of an A.P is 14 and the sums of
(c)  5 (d) none of these
k=1 the first five terms and the first ten terms are
5. The first three terms of sequence when nth equal in magnitude but opposite in sign. The
term tn is n2 – 2n are 3rd term of the AP is
(a) –1, 0, 3 (b) 1, 0, 2 4
(a) 6 (b) 6
(c) –1, 0, –3 (d) none of these 11
6. Which term of the progression –1, –3, –5, …. Is (c) 4/11 (d) none of these
–39
15. The sum of a certain number of terms of an AP
(a) 21st (b) 20th
series –8, –6, –4, …… is 52. The number of terms
(c) 19th (d) none of these is
7. The value of x such that 8x + 4, 6x – 2, 2x + 7 will (a) 12 (b) 13
form an AP is
(c) 11 (d) none of these
(a) 15 (b) 2
16. The first and the last term of an AP are –4 and
(c) 15/2 (d) none 146. The sum of the terms is 7171. The number
of terms is

152 BUSINESS MATHEMATICS, LOGICAL REASONING & STATISTICS (Paper 3) [CA Foundation]
(a) 101 (b) 100
1 3
(c) 99 (d) none of these 27. The sum of the series +1+ + ........ to 18
3 3
17. The sum of the series 3 ½ + 7 + 10 ½ + 14 + …. to terms is
17 terms is
(a) 530 (b) 535
(a) 9841
1 + 3  (b) 9841
(c) 535 ½ (d) none of these
3
18. The 7th term of the series 6, 12, 24,……is
(a) 384 (b) 834 9841
(c) 438 (d) none of these (c) (d) none of these
3
19. t8 of the series 6, 12, 24,…is
28. The second term of a G P is 24 and the fifth
(a) 786 (b) 768 term is 81. The series is
(c) 867 (c) none of these (a) 16, 36, 24, 54,.. (b) 24, 36, 53,…
20. t12 of the series –128, 64, –32, ….is (c) 16, 24, 36, 54,.. (d) none of these
(a) – 1/16 (b) 16
(c) 1/16 (d) none of these
29. The sum of 3 numbers of a G P is 39 and their
th
21. The 4 term of the series 0.04, 0.2, 1, … is product is 729. The numbers are
(a) 0.5 (b) 1/2 (a) 3, 27, 9 (b) 9, 3, 27
(c) 5 (d) none of these
(c) 3, 9, 27 (d) none of these
22. The last term of the series 1, 2, 4,…. to 10 terms
30. In a G. P, the product of the first three terms
is
27/8. The middle term is
(a) 512 (b) 256
(a) 3/2 (b) 2/3
(c) 1024 (d) none of these
(c) 2/5 (d) none of these
23. The last term of the series 1, –3, 9, –27 up to 7
terms is 31. If you save 1 paise today, 2 paise the next day
4 paise the succeeding day and so on, then
(a) 297 (b) 729
your total savings in two weeks will be
(c) 927 (d) none of these
(a) ` 163 (b) ` 183
24. The last term of the series x 2, x, 1, …. to 31
terms is (c) ` 163.83 (d) none of these
(a) x 28 (b) 1/x 32. Sum of n terms of the series 4 + 44 + 444 + … is
(c) 1/x 28
(d) none of these (a) 4/9 { 10/9 ( 10n –1 ) –n }
25. The sum of the series –2, 6, –18, …. to 7 terms is (b) 10/9 ( 10n –1 ) –n
(a) –1094 (b) 1094 (c) 4/9 ( 10n –1 ) –n
(c) – 1049 (d) none of these (d) none of these
26. The sum of the series 243, 81, 27, …. to 8 terms 33. Sum of n terms of the series 0.1 + 0.11 + 0.111 +
is … is
13 (a) 1/9 {n – ( 1– ( 0.1 )n )}
(a) 36 b) 36 (b) 1/9 {n – (1–(0.1)n)/9}
30
(c) n– 1 – (0.1)n/9
1
(c) 36 (d) none (d) none of these
9

SETS, FUNCTIONS AND RELATIONS 153


34. The sum of the first 20 terms of a G. P is 244 44. The sum of the infinite series 1 + 2/3 + 4/9 + .. is
times the sum of its first 10 terms. The (a) 1/3 (b) 3
common ratio is
(c) 2/3 (d) none of these
(a) ± 3 (b) ±3 45. The sum of the first two terms of a G.P. is 5/3
(c) 3 (d) none of these and the sum to infinity of the series is 3. The
common ratio is
35. Sum of the series 1 + 3 + 9 + 27 +….is 364. The
number of terms is (a) 1/3 (b) 2/3
(a) 5 (b) 6 (c) – 2/3 (d) (b) and (c) both
(c) 11 (d) none of these 46. If p, q and r are in A.P. and x, y, z are in G.P. then
xq–r. y r–p. zp–q is equal to
36. The product of 3 numbers in G P is 729 and the
sum of squares is 819. The numbers are (a) 0 (b) –1
(a) 9, 3, 27 (b) 27, 3, 9 (c) 1 (d) none of these
(c) 3, 9, 27 (d) none of these 47. The sum of three numbers in G.P. is 70. If the
37. The sum of the series 1 + 2 + 4 + 8 + .. to n term two extremes are multiplied each by 4 and the
mean by 5, the products are in AP. The numbers
(a) 2n –1 (b) 2n – 1
are
n
(c) 1/2 – 1 (d) none of these
(a) 12, 18, 40 (b) 10, 20, 40
38. The sum of the infinite GP 14, – 2, + 2/7, – 2/49,
(c) 40, 20, 10 (d) (b) and (c) both
+ … is
48. The sum of 3 numbers in A.P. is 15. If 1, 4 and 19
1 1 be added to them respectively, the results are
(a) 4 (b) 12
12 4 is G. P. The numbers are
(c) 12 (d) none of these (a) 26, 5, –16 (b) 2, 5, 8
39. The sum of the infinite G. P. 1 - 1/3 + 1/9 - 1/27 (c) 5, 8, 2 (d) (a) and (b) both
+... is
(a) 0.33 (b) 0.57
49. Given x, y, z are in G.P. and xp = yq = , then
1/p , 1/q, 1/  are in
(c) 0.75 (d) none of these
40. The number of terms to be taken so that 1 + 2 + (a) A.P. (b) G.P.
4 + 8 + will be 8191 is (c) Both A.P. and G.P. (d) none of these
(a) 10 (b) 13 50. If the terms 2x, (x+10) and (3x+2) be in A.P.,
(c) 12 (d) none of these the value of x is
41. Four geometric means between 4 and 972 are (a) 7 (b) 10
(a) 12, 36, 108, 324 (b) 12, 24, 108, 320 (c) 6 (d) none of these
(c) 10, 36, 108, 320 (d) none of these 51. If A be the A.M. of two positive unequal
42. Three numbers are in AP and their sum is 21. If quantities x and y and G be their G. M, then
1, 5, 15 are added to them respectively, they (a) A<G (b) A>G
form a G. P. The numbers are
(c) AG (d) A  G
(a) 5, 7, 9 (b) 9, 5, 7
52. The A.M. of two positive numbers is 40 and
(c) 7, 5, 9 (d) none of these their G. M. is 24. The numbers are
43. The sum of 1 + 1/3 + 1/32 + 1/33 + ....…  is (a) (72, 8) (b) (70, 10)
(a) 2/3 (b) 3/2 (c) (60, 20) (d) none of these
(c) 4/5 (d) none of these

154 BUSINESS MATHEMATICS, LOGICAL REASONING & STATISTICS (Paper 3) [CA Foundation]
53. Three numbers are in A.P. and their sum is 15. 61. T4 of a G.P. is x, T10 = y and T16 = z. Then
If 8, 6, 4 be added to them respectively, the (a) x2 = yz (b) z2 = xy
numbers are in G.P. The numbers are
(c) y2 = zx (d) none of these
(a) 2, 6, 7 (b) 4, 6, 5
62. If x, y, z are in G.P., then
(c) 3, 5, 7 (d) none of these
(a) y2 = xz
54. The sum of four numbers in G. P. is 60 and the (b) y ( z2 + x2 ) = x ( z2 + y2 )
A.M. of the first and the last is 18. The numbers
are (c) 2y = x+z
(d) none of these
(a) 4, 8, 16, 32 (b) 4, 16, 8, 32
63. The sum of all odd numbers between 200 and
(c) 16, 8, 4, 20 (d) none of these
300 is
55. A sum of ` 6240 is paid off in 30 instalments (a) 11,600 (b) 12,490
such that each instalment is ` 10 more than
the proceeding installment. The value of the (c) 12,500 (d) 24,750
1st instalment is 64. The sum of all natural numbers between 500
and 1000 which are divisible by 13, is
(a) ` 36 (b) ` 30
(a) 28,405 (b) 24,805
(c) ` 60 (d) none of these
(c) 28,540 (d) none of these
56. The sum of 1.03 + ( 1.03 )2 + ( 1.03 )3 + …. to n
65. If unity is added to the sum of any number of
terms is
terms of the A.P. 3, 5, 7, 9,…... the resulting
(a) 103 {(1.03)n – 1} sum is
(b) 103/3 {(1.03 )n – 1} (a) a perfect cube
(c) (1.03)n –1 (b) a perfect square
(d) none of these (c) a number (d) none of these
66. The sum of all natural numbers from 100 to
57. If x, y, z are in A.P. and x, y, (z + 1) are in G.P.
300 which are exactly divisible by 4 or 5 is
then
(a) 10,200 (b) 15,200
(a) (x – z)2 = 4x (b) z2 = (x – y)
(c) 16,200 (d) none of these
(c) z = x – y (d) none of these
67. The sum of all natural numbers from 100 to
58. The numbers x, 8, y are in G.P. and the numbers
300 which are exactly divisible by 4 and 5 is
x, y, –8 are in A.P. (x  y). The value of x and y
are (a) 2,200 (b) 2,000
(a) (–8, –8) (b) (16, 4) (c) 2,220 (d) none of these
(c) (8, 8) (d) none of these 68. A person pays ` 975 by monthly instalment
each less then the former by ` 5. The first
59. The nth term of the series 16, 8, 4,…. in 1/217.
instalment is ` 100. The time by which the
The value of n is entire amount will be paid is
(a) 20 (b) 21 (a) 10 months (b) 15 months
(c) 22 (d) none of these (c) 14 months (d) none of these
60. The sum of n terms of a G.P. whose first terms 69. A person saved ` 16,500 in ten years. In each
1 and the common ratio is 1/2 , is equal to year after the first year he saved ` 100 more
127 than he did in the preceding year. The amount
1 The value of n is of money he saved in the 1st year was
128
(a) ` 1000 (b) ` 1500
(a) 7 (b) 8
(c) ` 1200 (d) none of these
(c) 6 (d) none of these

SETS, FUNCTIONS AND RELATIONS 155


70. At 10% C.I. p.a., a sum of money accumulate to 80. The sum of natural numbers upto 200 excluding
` 9625 in 5 years. The sum invested initially is those divisible by 5 is ________.
(a) ` 5976.37 (b) ` 5970 (a) 20,100 (b) 4,100
(c) ` 5975 (d) ` 5370.96 (c) 16,000 (d) None
71. The population of a country was 55 crose in 81. The sum of all natural numbers between 200
2005 and is growing at 2% p.a C.I. the and 400 which are divisible by 7 is ______.
population is the year 2015 is estimated as (a) 7,730 (b) 8,729
(a) 5705 (b) 6005 (c) 7,729 (d) 8,730
(c) 6700 (d) none of these 82. If a, b, c be the sums of p, q, r terms
72. If a, b, c are in A.P. as well as in G.P. then – respectively of an A.P. the value of (a/p) (q - r)
(a) They are also in H.P. (Harmonic Progression) + (b/q) (r - p) + (c/r) (p - q) is ______.
(b) Their reciprocals are in A.P. (a) 0 (b) 1
(c) Both (a) and (b) are true (c) –1 (d) None
(d) Both (a) and (b) are false 83. If S1 S 2 S3 be the sum of n, 2n, 3n terms of A.P.
73. If a, b, c be respectively pth,qth and rth terms respectively, then value of S3 ÷(S2 -S1 ) is given
of an A.P. the value of an A.P. the value of a(q- by ______.
r )+b(r-p)+c(p-q) is (a) 1 (b) 2
(a) 0 (b) 1 (c) 3 (d) None
(c) –1 (d) None 84. The sum of n terms of two A.P.s are in the ratio
74. If the pth term of an A.P. is q and the qth term of(7n-5)/(5n+17) . Then the _______ term of
is p the value of the rth term is_________. the two series are equal.
(a) p – q – r (b) p + q – r (a) 12 (b) 6
(c) p + q + r (d) None (c) 3 (d) None
75. `If the pth term of an A.P. is q and the qth term 85. Find three numbers in A.P. whose sum is 6 and
is p the value of the (p + q)th term is_______. the product is –24
(a) 0 (b) 1 (a) –2, 2, 6 (b) –1, 1, 3
(c) –1 (d) None (c) 1, 3, 5 (d) 1, 4, 7
76. The sum of first n natural number is _______. 86. Find three numbers in A.P. whose sum is 6 and
(a) (n/2)(n+1) (b) (n/6)(n+1)(2n+1) the sum of whose square is 44.
(a) –2, 2, 6 (b) –1, 1, 3
(c) [(n/2)(n+1)]² (d) None
(c) 1, 3, 5 (d) 1, 4, 7
77. The sum of square of first n natural number is
__________. 87. If a b c are in A.P. then (b + c), (c + a), (a + b) are
in ________.
(a) (n/2)(n+1) (b) (n/6)(n+1)(2n+1)
(a) A.P. (b) G.P.
(c) [(n/2)(n+1)]² (d) None
(c) H.P. (d) None
78. The sum of cubes of first n natural number is
__________. 88. The sum of n terms of an A.P. is 2n + 3n2. Find
the nth term.
(a) (n/2)(n+1) (b) (n/6)(n+1)(2n+1)
(a) 4n + 1 (b) 4n - 1
(c) [(n/2)(n+1)]² (d) None
(c) 2n + 1 (d) 6n - 1
79. If Sn the sum of first n terms in a series is given
by 2n2 + 3n the series is in ______. 89. If S1, S2, S3 be the sums of n terms of three
A.P.s the first term of each being unity and the
(a) A.P. (b) G.P. respective common differences 1, 2, 3 then
(c) H.P. (d) None (S1 + S3) / S2 is ______.
156 BUSINESS MATHEMATICS, LOGICAL REASONING & STATISTICS (Paper 3) [CA Foundation]
(a) 1 (b) 2 98. The sum of n terms of the series 1  2  3 + 2  3  4
(c) –1 (d) None + 3  4  5 + …….is
90. If a, b, c, d are in A.P. then (a) (n/4)(n+1)(n+2)(n+3)
(a) a2 – 3b2 +3c2 – d2=0 (b) (n/3)(n+1)(n+2)(n+3)
(b) a2 +3b2 +3c2 +d2 =0 (c) (n/2)(n+1)(n+2)(n+3)
(c) a2 + 3b2 + 3c2 – d2 (d) None
(d) None 99. The sum of n terms of the series
1  2+3  2²+5  2³+7  24+....... is
91. If a, b, c, d, e are in A.P. then
(a) (n–1)2n+2–2n+1 +6
(a) a – b – d + e = 0 (b) a – 2c + e = 0
(b) (n+1)2n+2–2n+1 +6
(c) b – 2c + d = 0 (d) all the above
(c) (n–1)2n+2–2n+1 –6
92. The four numbers in A.P., whose sum is 20 and
the sum of their squares is 120, are _______. (d) None
(a) 3, 5, 7, 9 (b) 2, 4, 6, 8 100. The sum of n terms of the series 2  32 +5  42
+8  52 + ........is
(c) 5, 9, 13, 17 (d) None
(a) ( n/12)(9n3 +62n2 +123n +22)
93. The sum of n terms of the series 1²/1+(1²+2²)/
2+(1²+2²+3²)/3+.......is (b) (1/12)(9n3 -62n2 +123n -22)
(a) (n/36)(4n² +15n+17) (c) ( n/6)(9n3 +62n2 +123n +22)
(d) None
(b) (n/12)(4n²+15n+17)
101. 24n -1 is divisible by
(c) (n/12)(4n² +15n+17)
(a) 15 (b) 4
(d) None (c) 6 (d) 64
94. The sum of n terms of the series 2  4  6 + 4  6  8 102. n
3 -2n -1 is divisible by
+ 6  8  10 + ………. is (a) 15 (b) 4
(a) 2n(n³+6n²+11n+6) (c) 6 (d) 64
(b) 2n(n³–6n²+11n–6) 103. If a, b, c are the p , q and rth terms of a G.P.
th th

(c) n(n³+6n²+11n+6) respectively the value of a q-r ,b r-p ,C p-q is


________
(d) n(n³+6n²+11n– 6)
(a) 0 (b) 1
95. The sum of n terms of the series 1  32 +4  42
(c) –1 (d) None
+7  52 +10  62 +........ is
104. If a, b, c are in A.P. and x, y, z in G.P. then the
(a) (n/12)(n+1)(9n²+49n+44)–8n
value of xb-c .yc-a .za-b is ________
(b) (n/12)(n+1)(9n²+49n+44)+8n (a) 0 (b) 1
(c) (n/6)(2n+1)(9n²+49n+44)–8n (c) –1 (d) None
(d) None 105. If a, b, c are in A.P. and x, y, z in G.P. then the
96. The sum of n terms of the series 4 + 6 + 9 + 13 value of (xb .yc .za )÷(xc .ya .zb ) is ____
…….. is (a) 0 (b) 1
(a) (n/6)(n²+3n+20) (b) (n/6)(n+1)(n+2) (c) –1 (d) None
106. The sum of n terms of the series 7 + 77 + 777 +
(c) (n/3)(n+1)(n+2) (d) None
…… is
97. The sum of n terms of the series 1  2 + 2  3 +
(a) (7/9)[(1/9)(10n+1-10)-n]
3  4 + ……. Is
(b) (9/10)[(1/9)(10n+1-10)-n]
(a) (n/3)(n+1)(n+2) (b) (n/2)(n+1)(n+2)
(c) (10/9)[(1/9)(10n+1 -10)-n]
(c) (n/3)(n+1)(n– 2) (D) None (d) None
SETS, FUNCTIONS AND RELATIONS 157
107. If ‘S’ be the sum, ‘P’ the product and ‘R’ the
sum of the reciprocals of n terms in a G.P. then
‘P’ is the _______ of Sn and R-n.
(a) Arithmetic Mean
(b) Geometric Mean
(c) Harmonic Mean
(d) None
108. If 1+a+a2 +.......  = x and 1+b+b2 +.......  = y
then 1 + ab + a2 b2 +.......  = ? is given by
__________.
(a) (xy)/(x+y-1) (b) (xy)/(x-y-1)
(c) (xy)/(x+y+1) (d) None
109. The sum of first eight terms of G.P. is five times
the sum of the first four terms. The common
ratio is __________.
(a) 2 (b) - 2
(c) both (d) None

158 BUSINESS MATHEMATICS, LOGICAL REASONING & STATISTICS (Paper 3) [CA Foundation]
9. The sum of progression (a+b), a, (a–b)........n
term is
HOME WORK - 1
(a)
1. The sum of a certain number of terms of an AP
series –8, –6, –4, …… is 52. The number of terms
is
(b)
(a) 12 (b) 13
(c) 11 (d) None
2. Which term of the progression –1, –3, –5, …. is (c)
–39?
(a) 21st (b) 20th
(c) 19th (d) None (d)
3. If the AM and GM for two numbers are 6.50
and 6 respectively then the two numbers are 10. The sum of the following is ________ .
(a) 6 and 7 (b) 9 and 4 1+3–5+7+9–11+13+15-17............................ 3n
terms
(c) 10 and 3 (d) 8 and 5
(a) 2n2+3 (b) 5n2+2
4. The sum of square of first n natural number is
__________. (c) 3n2–4n (d) 3n2
(a) (n/ 2) (n +1) 11. The common ratio of the G.P 2,–6, 18, –54 is
(b) (n/ 6) (n +1) (2n +1) (a) 3 (b) –3
(c) [(n/ 2) (n +1)]2 (c) 4 (d) –4
(d) None 12. Determine the first term of an A.P. with
common difference 3 & 7th term being 11
5. The sum of a series in A.P. is 72 the first term
being 17 and the common difference –2. the (a) –7 (b) 7
number of terms is (c) 6 (d) 5
(a) 6 (b) 12 13. 6 term of series ab, a b , a3b5 …… is equal to
th 2 3

(c) 6 or 12 (d) 10 (a) a6 b11 (b) a5 b10


6. Find the four numbers in A.P. with the sum of (c) a6 b 6 (d) a5b11
second and third being 22 and the product of
the first and fourth being 85. 1 1
14. Which term of series 3 , 1, ….. is 243
3
(a) 3, 5, 7, 9 (b) 2, 4, 6, 8
(a) 13 (b) 14
(c) 5, 9, 13, 17 (d) None
7. The sum of n terms of the series 2 + 6 + 10 + ……. (c) 15 (d) 12
is 15. Sum of three numbers in A.P. is 12 and the sum
(a) 2n2 (b) n2 of their cube is 408. The numbers are
(c) n2 / 2 (d) 4n 2 (a) 3, 4, 5 (b) 1, 4, 7
8. Find the sum of 10 terms G.P with first term (c) 2, 4, 6 (d) None
and common ratio being 8 and 3 respectively 16. Find the three numbers in G.P whose sum is
(a) 2, 63, 291 (b) – 2, 36, 192 52 and the sum of their product in pairs is 624.
(c) 2, 19, 631 (d) 2, 36, 192 (a) 4, 12, 36 (b) 10, 16, 26
(c) 5, 17, 30 (d) None

SETS, FUNCTIONS AND RELATIONS 159


17. Three numbers are in A.P. of whose sum is 15 9 1
and whose product is 105, then numbers are: (a) 10
(b) 10
(a) 3, 5, 7 (b) 2, 5, 8
10
(c) 0, 5, 10 (d) None (c) 9
(d) None
18. The nth term of series whose sum to n terms is 26. The sum of the series 1-1+1-1+1-1+...... to 100
5n2+2n is terms is equal to
(a) 5n + 3 (b) 10n – 2 (a) 1 (b) –1
(c) 5n + 2 (d) 10n – 3 (c) 0 (d) 50
19. Find the sum of first twenty five terms of AP 27. If the sum of first n terms of an A.P.’s Zero,
then the sum of next m terms, where a is the
series whose nth term is . first term of the A.P. is

(a) 105 (b) 115


(a) (b)
(c) 125 (d) 135

20. The sum of the series 1 ,..... to is (c) (d) None

1 1
28. If mth term of an A.P. is n and nth term is m ,
(a) (b)
then sum of mn terms is –
1
(a) mn + 1 (b) mn-1
2
(c) (d) None
1
(c) mn+1 (d) None
2
21. If a, b, c are in A.P., then 2b = _____ 29. If the pth, qth, rth and sth terms of an A.P. are in
(a) a – c (b) a + c G.P.; then p–q, q–r and r–s are in
(a) A.P.
(c) (d) (b) G.P.
(c) Cannot determine
22. If a, b, c are in G.P., the b2 = _____
(d) None
(a) ac (b) –ac
30. If (P+1)th term of A.P. is twice the (q+1)th term;
(c) a+ b (d) a – c then the ratio of (P+q+1)th term and (3P+1)th
23. The sum 1 + 2 + 3 + 4 + ... + 102 is equal to
2 2 2 2
term is:
(a) 385 (b) 386 (a) 1:2 (b) 2:1
(c) 384 (d) None (c) 1:3 (d) None
24. The sum 13 + 23 + 33 +...+ 203 is equal to 31. If the common difference of an A.P. equals to
(a) 4410 (b) 4410000 the first term, then the ratio of its mth term
and nth term is:
(c) 44100 (d) None
(a) n:m (b) m:n
25. The sum of the series 1+10-1 +10-2 +10-3.... to 2
(c) m : m 2
(d) None
is

160 BUSINESS MATHEMATICS, LOGICAL REASONING & STATISTICS (Paper 3) [CA Foundation]
1 (c) cannot calculated
32. If the mth term of A.P. is n and the nth term is
(d) None
1 39. Find the sum of n terms of the series 1 + 9 + 24
, then its mnth term is:
m + 46 + 75 + .................
(a) 1 (b) –1
(c) 0 (d) None (a)

1 , 1 , 1
33. If x + y 2y y +z are in A.P., then x, y, z
(b)
are in
(a) A.P.
(b) G.P. (c)
(c) Cannot determined
(d) None (d) None
34. The ratio of the sum and the difference of two 40. The sum of the first two terms of an infinite
numbers is 7:1. Find the ratio of those two geometric series is 15 and each term is equal
numbers. to the sum of all the terms following it; then
(a) 5:3 (b) 4:3 the sum of the series is
(c) 4:5 (d) None (a) 20 (b) 15
35. The 6th term from the end of the G.P. 8, 4, 2, 1, (c) 25 (d) None
41. Find the sum of the series. 243 + 324 + 432 +
1
.................., is …………………. to n terms
1, 024

1
(a) 64
(b) 32 (a)

1
(c) 32
(d) None

36. In a finite G.P., the product of two terms equi


distant from the beginning and from the end (b)
is equal to the product of the first and the last
term of the G.P. This statement is
(a) True (b) False
(c) Cannot say (d) None (c)
37. If a, b, c are in G.P., a, x, b and b, y, c are both in
1 1 (d) None
A.P., the value of x + y is
42. Which term of the sequence,
(a) 1/b (b) 2 / b
-9 -7
(c) - 2 / b (d) None , -2, , ................ is zero.
4 4
38. If Sn be the sum of n terms of an A.P.; the value
(a) 9th term (b) 10th term
of Sn – 2Sn–1 + Sn–2 is
(c) 12th term (d) None
(a) d
(b) d2

SETS, FUNCTIONS AND RELATIONS 161


43. The product of n G.M.s between the two given
numbers is equal to the n power of the single
G.M. between them. This statement is –
(a) True (b) False
(c) Cannot say (d) None
44. The first term of an A.P. is 100 and the sum of
whose first 6 terms is 5 times the sum of the
next 6 terms, then the c.d. is –
(a) –10 (b) 10
(c) 5 (d) None
45. The sum of n terms of an A.P. is 3n2+n; then its
pth term is
(a) 6P + 2 (b) 6P – 2
(d) 6P – 1 (d) None

ANSWER KEYS

1. (b) 2. (b) 3. (b) 4. (b)


5. (c) 6. (c) 7. (a) 8. (d)
9. (b) 10. (c) 11. (b) 12. (a)
13. (a) 14. (d) 15. (b) 16. (a)
17. (a) 18. (d) 19. (b) 20. (b)
21. (b) 22. (a) 23. (a) 24. (c)
25. (c) 26. (c) 27. (b) 28. (c)
29. (b) 30 (a) 31. (b) 32 (a)
33. (b) 34 (b) 35. (c) 36 (a)
37. (b) 38. (a) 39. (a) 40. (a)
41. (a) 42 (b) 43. (a) 44 (a)
45. (b)

162 BUSINESS MATHEMATICS, LOGICAL REASONING & STATISTICS (Paper 3) [CA Foundation]
(c) 6 or 12 (d) None
10. The number of terms in the A.P. 7,13,19,,.....97
HOME WORK - 2 is
(a) 97 (b) 17
1. Two A, Ps has the same common difference. If (c) 16 (d) 15
the difference between their 100 th terms is 3p - 1
111222333 then the difference between their 11. The pth term of an AP is ,the sum of the
millionth terms is 6
first n terms of the AP is
(a) 123 (b) 112233
(c) 111222333 (d) 112333 n
2. If a,b,c are in A.P., then 2b = _______ (a) n(3n +1) (b) (3n +1)
12
(a) a-c (b) a+c
n
a+ c a-c (c) (3n-1) (d) None of these
(c) (d) 12
2 2
12. The sum of n terms of two APs are in the ratio
3. Which term of the A.P.
7n - 5
3 4 5 17 of , then the _____ term of the two
, , ,..... is ? 5n + 17
7 7 7 7 series are equal
(a) 13 (b) 14 (a) 12 (b) 6
(c) 15 (d) 16 (c) 3 (d) None of these
4. If the 10th term of an A.p. is twice the 4th term, 13. th
If the n terms of two APs are in the ratio
and the 23rd term is ‘k’ times the 8th term, (3n+1):(n+4) the ratio of the fourth term is
then the value of ‘k’ is (a) 2 (b) 3
(a) 2.5 (b) 3 (C) 4 (D) None of these
(c) 3.5 (d) 4
14. A person employed in a company at ` 3000
5. The A. M between 2 & 4 is
per month and he would get anincrease of `
(a) 2 (b) 4 100 per year. find the total amount which he
(c) 3 (d) 6 receives in 25years and the monthly salary in
6. The arithmetic mean between 8 & 20 is- the last year
(a) 6 (b) 12 (a) 1380000 and 6200
(c) 14 (d) 18 (b) 930000 and 5400
7. Three No’s a,b,c are in A. P. find a-b+c (c) 1480000 and 7200
(a) a (b) -b
(d) 1570000 and 4800
(c) b (d) c
15. A person received the salary for the 1st Year is
8. The sum 1 + 2 + 3 + 4.....+70 is equal to
`5,00,000 per year and he received an
(a) 2484 (b) 2485 increment of ` 15,000 per year then the sum
(c) 2845 (d) None of these of the salary he taken in 10 years
9. The sum of a series in ap is 72 the first term (a) ` 56,75,000
being 17 and the common difference -2 the (b) ` 72,75,000
number of terms is ______
(c) ` 63,75,000
(a) 6 (b) 12
(d) None of these

SETS, FUNCTIONS AND RELATIONS 163


16. Find the Numbers whose GM is 5 and AM is 7.5
1 1 
(a) 12 and 13 (b) 13.09 and 1.91 b = 1 -  + + ........to   ,then b is
2 4 
(c) 14 and 11 (d) 17 and 19
(a) 0 (b) 1
17. The number of terms in 6,18 ,54,.....1458 is
(a) 5 (b) 7 1 1
(c) (d)
(c) 8 (d) 6 2 2
18. If the fifth term of a G.P. is 34and the second 25. t1= n, t2= n + 1 , t3= n + 2 and so on, then tn=
term is 3(2)3then the first term is (a) n (b) 2n-1
(a) 23 (b) 32 (c) 2n+1 (d) 2n
26. The sum of n terms of the series 22+52 + 82 +
(c) 43 (d) 24
....... is
19. In a GP series , the product of the first three
n 2
terms
27
,the middle term is
(a)
2

  6n + 3n - 1 
8
n 2
(a)
3
(b)
2 (b) 
  6n - 3n - 1
2

2 3
n 2
(c)
2
(d) None of these (c)
2

  6n + 3n + 1 
5
20. If A be the A.M of two positive unqual (d) None of these
quantities x and y and G be their G.M,then 27. 1 + 11 + 111 + ............ n terms.
(a) [10n + 1 - 9n-10]
(a) A < G (b) A > G
(b) [10n + 1 - 9n -10]
(c) A>G (d) A<G (c) [10n +1 - 9n -10]
b c (d) none of these
21. If a, b-a,c-a are in GP and a = = then a,b, c
3 5 1 + 3 + 5 +..... +n terms 2
28. If  , then the value
are in 2 + 4 + 6 +......50 terms 51
(a) AP (b) GP of ‘n’
(c) HP (d) None (a) 9 (b) 10
(c) 12 (d) 13
22. The sum of four numbers in GP is 60 and the
AM of the 1st and the last is 18 . The numbers ANSWER KEY
are 1. (c) 2. (b) 3. (c) 4. (a)
(a) 4,8,16,32 (b) 4,16,8,32
5. (c) 6. (c) 7. (c) 8. (b)
(c) 16,8,4,20 (d) None of these
1 1 1 9. (c) 10. (c) 11. (b) 12. (b)
23. The value of A 2 .A 4 , A 8 to infinity is -
13. (d) 14. (b) 15. (a) 16. (b)
(a) Zero (b) Infinity
17. (d) 18. (d) 19. (a) 20. (b)
A0 21. (a) 22. (a) 23. (d) 24. (a)
(c) (d) A
2
25. (b) 26. (a) 27. (d) 28. (b)
24. A radioactive sample decays and the remaining
sample at infinite time is given by

164 BUSINESS MATHEMATICS, LOGICAL REASONING & STATISTICS (Paper 3) [CA Foundation]
CHAPTER-7
SETS, FUNCTIONS & RELATIONS

INTRODUCTION

SETS
A set is defined to be a collection of well-defined distinct objects thought of as a whole. Each object is called
an element of a Set.
A. Representation of Sets - Methods of describing a set are:
1. Roster Method (Tabular Form): In this method a set is described by listing all its elements, eparated by
commas (,) with in the braces { }. Ex: A ={1, 2, 3, 4, 5, 6}
2. Set-Builder Method: In this method a set is described by stating a property p(x) which is satisfied by all
its elements. In such a case it is described by S = { x | P(x) holds } or S = { x: P(x) holds} or S = { x | x has the
property P(x)} .
B. Types of Sets:
1. Finite Set: A set is said to be a finite set if the number of distinct elements in it is Finite.
S = {a, e, i, o, u} is a Finite Set.
2. Infinite Set: If the number of elements in a set is not finite then the set is known as an infinite set.
C. Other Terms
1. Elements: The members of set are usually called elements, In A = (a, e, i, o, u}, a is an element and
we write a  A i.e. a belongs to A. But 3 is not an element of B = {2,4, 6, 8, 10} and we write 3  B. i.e.
3 does not belong to B.
2. Cardinal number (Order of a set): The number of distinct elements in a set is called the order of the
set. Example: A = {3, 4, 5, 6, 7}. Cardinal Number is 5.
3. Equivalent Set: Two sets A and B are said to be Equivalent if their cardinal numbers are same, i.e.
n(A) = n(B).
4. Subset: A set X is said to be a subset of set Y if every element of the set X is also an element of the
set Y. Ex: If A = {l, 2,3,4,5} and B = {3,5}, then B is a subset of A and A is the Super Set. The Subsets of
a set include the null set and the Entire Set itself. A set containing n elements has 2n subsets.
5. Proper Subset: A set X is called a proper subset of Y if X  Y; and Y  X and it is denoted byXcY, i.e.
X is said to be a proper subset of Y if every element of X belongs to Y but there is at least one
element of Y which is not in X. A Set containing n elements has 2n - 1 proper subsets. Example: The

Sets, Functions and Relations 165


set {3,4,5}, has {3} , {4 } , {5}, {3,4} , {3,5}, {4,5}, { } , ie 23- 1 = 7 proper subsets.
6. Equal Sets: Two sets A and B are said to be equal if every element of A is an element of B and every
element of B is an element of A and it is denoted by A=B. Thus, A = B  A  B and B  A.
7. Null Set or Empty Set: A set having element is called a null set or a void set or an empty set and is
denoted by  .
8. Singleton Set: A set consisting of only one element as its member is known as Singleton Set.
9. Countable and Uncountable sets: A set whose elements can be put in one-to-one correspondence with
the elements of the set N, the set of positive integers is called countable or enumerable set otherwise
it is Matching sets.
10. Comparable sets: Two sets A and B are said to be comparable if either A  B or B  A. If neither A  B
nor B  A , then A and B are called non-comparable sets.
11. Disjoint Sets: Two sets A and B are said to be disjoint sets if they have no element in common, i.e., their
intersection is null set, i.e., if A  B = 
12. Family of sets or set of sets: If the elements of a set are sets themselves, then this set is called the family
of sets or a class of sets or a set of sets.
13. Power Set: It is the set of all possible subsets of the given set A and is denoted by P(A), i.e., P(A) = {B|
B  A}. Thus a power set is a family of sets.
14. Union of Two Sets: Let A and B be two given sets. Then the union of the two sets A and B is the set of all
those elements x such that x belongs to A or x belongs to B or x belongs to both A and B, and is denoted
by A  B, to be read as ‘A union B’ or A cup B. i.e., A  B = {x | x  A or x  B or x  A as well ass x  B}.
15. Intersection of Two Sets: Let A and B be two sets. Then the intersection of two sets A and B is the set of
al those elements which belong to A as well as to B and is denoted by A  B, to be read as A intersection
B or A cap B. i.e., A  B = {x | x  A and x  B}.
16. Difference of two sets: Let A and B be two sets is the set of all those elements x such that x belongs to
A and x does not belong to B and is denoted by A - B to be read as ‘A difference B’ or simply ‘A minus B’
i.e., A-B = {x|x  A and x  B}
17. Complement of a Set: Let U be the universal set and A  U, then the complement of set A with respect
to the universal set U is the difference of the universal set U and the set A. It is denoted by A’ or AC be
read as A Complement, i.e., Ac = U -A ={x|x  U and x  A} or Ac = {x|x  A}
18. Symmetric difference: The symmetric difference of two sets A and B is the set (A-B)  (B-A)and is
denoted by A  B . i.e., A  B = (A-B)  (B-A) .
19. Ordered Pair: Two elements a and b, listed in a specific order, form an ordered pair, denoted by (a,b).
20. Cartesian Product: If A and B are two non-empty sets, then the set of all ordered pairs (a, b) such that a
belongs to A and b belongs to B, is called the Cartesian product of A and B, is to be denoted by A x B.
Thus, in @ x B = φ if A or B = φ . Example: If A = {1, 7}, B={3, 6, 8}. Then A x B ={(1, 3) (1, 6) (1, 8) (7, 3) (7,
6) (7, 8)}
D. De-Morgan’s law: If A, B, C are finite sets, then.
n (A  B) = n(A) + n(B) - n(A  B) [If A & B are not Disjoint sets]
n (A  B) = n(A) + n(B) [If A & B are Disjoint sets]

166 BUSINESS MATHEMATICS, LOGICAL REASONING & STATISTICS (Paper 3) [CA Foundation]
n (A - B) = n(A) - n(A  B), i.e., n(A) = n(A - B) + n(A  B)
n (A’  B’) = n [(A  B)’] = n(U) - n(A  B)
n (A’  B’) = n [(A  B)’] = n(U) - n(A  B)
If A, B and C are three finite sets, then
n (A  B  C) = n(A) + n(B) + n(C) - n(A  B) - n(A  C) - n(B  C) + n(A  B  C) [If A & B are not Disjoint
sets]
n (A  B  C) = n(A) + n(B) + n(C) [If A & B are Disjoint sets]
E. Other Results:
1. Number of elements which belong to exactly one of A or B .. (A  B) = n(A) + n(B) -2n (A  B)
2. Number of elements in exactly two sets A, B, C = n(A  B) + n(A  C) + n(B  C) - 3n(A  B n  )
3. Number of elements inexactly one set of A, B, C = n(A) + n(B) + n(C) -2n(A  B) -2n(A  C) -2n(B n C)
+ 3n (A  B  C)
F. Venn Diagram: A Venn diagram is constructed with a collection of simple closed curves drawn in the
plane. Venn diagrams normally consist of overlapping circles. The principle of these diagrams is that
sets be represented by regions to represent all possible relations in the same diagram.

Functions
1. Relation and Function: Any subset of the product set XY is said to define a relation from X to Y and any
relation from X to Y in which no two different ordered pairs have the same first element is called a
function. A Function is represented as f: x  Y..
Example: f: X - Y = {(1,2), (3,4} is a valid function between X and Y, whereas f: X  Y = {(4,2),(4,-2)} is not
a valid function between X and Y since a single element of X (4) is mapped to two elements in Y(2 and -
2).
Note: One more essential feature of a function is that every element in X should be mapped to Y. But the
reverse need not be true.

2. Image : The element y is called the image of x under the f and is denoted by f(x), i.e., y = f(x), and x is
called the pre-image of y.
3. Domain & Co-domain: If f: A  B, A is called the Domain and set B is called the Co-Domain of f..

Sets, Functions and Relations 167


4. Range: The set of all images of the elements of A under the mapping f is called the range of f and is
denoted by f(X) or Rf, where f(X) = {f(x)  X} . In general, f(X)  Y..
Example : We consider the rule f(x) = X2. Let A={1,2,3,4...}. Then f(1) = 1; f(2) = 4; f(3) = 9; f(4) = 16. Then
clearly each element in A has a unique image in B. Here domain (f) = {1 2,3,4} and Range (f) = {1, 4, 9,16}
5. Types of functions:
(a) One to One Function (lnjective) : In f : A  B. If different eiements in A have different images in B,
then f is said to be a one-one or an Injective Function or Mapping.
Example: Let A = {1, 2} and B = {2, 4, 6}. Let us consider f : A  B: f(x) = 2x. f(1) = 2; f(2) = 4; f(3) = 6. f
is function from A to B such that different elements in A have different images in B and hence is
one - one.
(b) Into Function: If in A  B, there exists even a single element in B having no pre-image in A, then
f is said to be an Into function.
Example: Let A = {2, 3, 5, 7} and B = {0, 1, 3, 5, 7}. Let us consider f: A  B; F(x) = x - 2. Then f(2) = 0;
f(3) = 1; f(5) = 3 & f(7) = 5. It is clear that f is a function from A to B. Here there exists an element 7
In B, having no pre-mage in A. So, if is an into function.
(c) Onto Function (Surjective): A function f defined from the set X to the set Y is said to be an onto
function if every element in the co-domain is mapped to by some element in its domain.
Example: A = {1,2,3} , B = {2,4,6}, then F(x) = 2x gives, f(1) = 2, f(2) = 4, f(3) = 6 therefore F(x) is an onto
function, since all the images in B have a pre-image in A.
(d) Bijection (One-One Onto): A mapping which is both injective and surjective is called a bijection. In
other words, a mapping f: X  Y is called one-one onto (Bijection) if the following conditions are
satisfied:
• Each element in X is mapped to a different element of Y.
• Given any element of y  Y, there exists an element x  X and such that y = f(x), i.e., everyy
element of Y has a pre-image.
• Example: A = {1,2,4} , B = {2,4,6}, then F(x) = 2x gives, f(1) = 2, f(2) = 4, therefore F(x) is a one to
onto function, since all the images in B have a pre-image in A and all the images are mapped
to only one image.
(e) Many to One: A mapping f from the set X to the set Y is said to be many-one if f(a) = f(b) even if a
 b, a, b  X, f(a), f(b)  Y. In other words, if a given element of Y may have more than one pre-image e
but no element of X can have more than one f-image, then mapping is said to be many-one.
Example: Let f: A  B: be {(-2,5),(2,5),(3,6)}. The same is a many to one function since, there are
two pre— images with the same image.
(f) Identity Mapping: A mapping f defined from the set X to the set X is said to be identity mapping if
every element x  X is mapped to itself, i.e., in case of identity mapping f(x) = x,  x  X, f(x)  X.
The identity mapping is always one-one and onto, i.e., it is always a Bijection.
(g) Constant Mapping: Let f: A  B, defined in such a way that all the elements in A have the same
image in B, then f is said to be a constant function. In other words, a function f: X  Y is constant
function if the range of f is a Singleton Set.
Example: Let A = {1, 2, 3} and B = {5, 7, 9}. Let f: A  B: f(x) = 5 for all x  A. Then, all the elements in
A have the same image namely 5 in B.
(h) Inverse Function: Let f be a function defined from the set X to the set Y, i.e., f: X  Y and g be a
function defined from the set Y to the set X, i.e., g: Y  X; then the function g is said to be inverse
of f if f{f(x)} = x,  x  X and the function g is denoted by f-1. A function g which possesses an
inverse is called invertible.

168 BUSINESS MATHEMATICS, LOGICAL REASONING & STATISTICS (Paper 3) [CA Foundation]
(i) Composite function: Let f: A  B and g: B  C; then the composite of the functions f and g, denoted
by gof or fog is a mapping gof: A  C such that (gof) (x) = g f(x)],  x  A and g[f(x)]  C.

(j) Even Function: A function y = f(x) is called an even function of x if f(x) = f(-x) for all the values of x.
Example: y = x2 is an even function as: (-x)2 = x2
(k) Odd function: A function y = f(x) is said to be an odd function of x if f(-x) = f(x) for all the values of
x. Example: f(x) = x3 is an odd function of x as: (-x)3 = x3 = f(x).
(I ) Equality of two functions: Two functions f and g are said to be equal if they are defined on the same
domain X and if f(x) = g(x),  x  X.
6. Other Terms
(a) Inverse image of an element: Let f be a function defined from the set X to the set Y, then the
inverse image of an element b  Y under f is denoted by f-1(b)to be read as f inverse b and f _1(b) =
{X|X  Y and f(x) = b}. i.e., f-1(b) is the set of those elements in X which have b as their f-image.
(b) Inverse image of a subset: Let f be a function defined from the set X to the set Y and B be a subset
of Y, i.e., B  Y , then the inverse of B under f is given by f-1(B) = {x |x  X and f(x)  B}. Obviously,,
f_1(B) is a subset of X and and f-1 [f(X)] = X.
Relations
A. Relation: Let A and B be two sets, then a relation R from A to B is a subset of A x B. A relation R consists
of the following:
• Two sets say A and B
• An open sentence P(x, y) in which P(a, b) is either true or false for any ordered pair {a, b}  AxB ,
then R is called a relation from A to B and is denoted by R = {A, B, P (x,y)}
B. Types of Relation:
1. Equivalence Relation: Let S = {a, b, c, d,..} be any set then the relation R is a subset of the product
set SxS. A relation R on a set S is said to be an equivalence relation on S if it is -
(a) Reflexive: A relation on a set S is said to be reflexive if every element of S is related to itself.
Thus, R is reflexive if R contains all ordered pairs of the form (a, a) in SxS.
(b) Symmetric:A relation R on set S is said to be symmetric if (a, b) (a,b)  R  (b, a)  for all a,
b  R.

Sets, Functions and Relations 169


(c) Transitive : A relation R on set S is said to be transitive relation if (a, b)  R and
(b, c)  R,  (a, c)  R for a, b, c  R .
2. Universal Relation: Relation A X A  A X A is called the universal relation on A.
3. Identity Relation: IA = {(a, a): a  A} on set A is called the identity relation on A. Example: Let A = {1,
2, 3} then I = {(1,1), (2, 2), (3, 3)}
4. Inverse Relation: If R be a relation on A, then the relation R-1 on A, defined by R-1= {(b, a):(a,b)  R}
is called an inverse Relation on A.
C. Domain and Range of a Relation: If R is a relation form A to B. Then the set of all first co-ordinates of
elements of R is called the domain of R, while the set of all second co-ordinates of elements of R is
called the range of R.
Ex: Relation R= {(1,3) (2,5) (7,9)}. Domain (R) = {1,2,7} Range (R) = {3,5,9}

RELATION
Relation Reflexive Symmetrix Transitive
Is equal to   
Less then / Greater than   
Parallel   
Perpendicular   
Reciprocal   
Divide/ Multiply   
Father / Mother   
Brother / Sister   
equal to, parallel, Father - Mother- Reflexive, symmetric and Transitive
less than / Greater than , Brother / Sister, - Only Transitive,
Prependicular, Reciprocal - Only symmetric,
Divide / Multiply - Reflexive and Transitive but not symmetric

170 BUSINESS MATHEMATICS, LOGICAL REASONING & STATISTICS (Paper 3) [CA Foundation]
9. If X and Y are two sets, then (Y  X)’  X
equals :
CLASS WORK (a)  (b) x
(c) y (d) None
SET THEORY : 10. If A & B are two sets and U is the universal set
such that n (U) = 700, n (A) = 200, n (B) = 300 and
n (A  B) = 100, then n (A’  B’) is equal to :
1. The number of subsets of the sets {6, 8, 11} is:
(a) 200 (b) 300
(a) 9 (b) 6
(c) 400 (d) 600
(c) 8 (d) None
11. A and B are two disjoint sets containing 3 and 6
2. If the universal set E = {x / x is a positive integer elements respectively. The number of
< 25), A = {2, 6, 8, 14, 22}, B = {4, 8, 10, 14}, then: elements in A  B is :
(a) (A  B) = A   B (a) 3 (b) 6
(c) 2 (d) 9
(b)  A  B  = A   B 12. If a  N such that aN = {ax : x  N}. The set 3N
 7N is equal to :
(c)  A   B  =  (d) None (a) 7 N (b) 3 N
(c) 21 N (d) None
3. If the set P has 3 elements, Q has four elements
and R has two elements, then the set P x Q x R 13. For any natural number a, we define aN = (ax :
contains : x € N}. If b, c, d € N such that bN  cN = dN,
then d is equal to:
(a) 9 elements (b) 20 elements
(c) 24 elements (d) None (a) b (b) c
4. If A and B are two sets, then (A  B)  __= B (c) bc (d) None
equals DIRECTION : For Q.No. 14 to Q.no.18.
(a) B (b) A U = {1, 2, ........9} to be the universal set, A = {1,
2, 3, 4} and B = {2, 4, 6, 8}. Then answer the
(c) A  B (d)  following questions
5. Let A = { x / x is a multiple of 2} and B = { x / x is 14. The A  B is equal to :
a muItjple of 3}. Then A  Bis given by : (a) {1, 2, 3,4, 6, 8} (b) {2, 4}
(a) {3, 6, 9, ...} (b) {2, 4, 6, 8, ... } (c) {5, 6, 7, 8, 9} (d) {5, 7,9}
(c) {6, 12, 18, ... } (d) None 15. The set equal to set A  B is :
6. If A = {1,2,3,4}, B = {2, 4, 5, 6) and C = {1, 3, 4, 6, (a) {1,2,3,4,6,8} (b) {2, 4}
8}, then the set (A  B)  (A  C) equal to
(c) {5, 6, 7, 8, 9} (d) (5,7,9)
(a) {1, 2, 3, 4} (b) {1, 2, 3, 4, 5, 6}
16. The set equal to set A’ is :
(c) {2, 4, 5, 6} (d) None
(a) {1, 2, 3, 4, 6, 8} (b) {2, 4}
7. Let A = {1, 3, 5, 7, 8, 9} and B= {3, 5, 8), thenA
(c) {5, 6, 7, 8, 9} (d) {5,7,9}
 B is : 17. The set (A  B)’ is :
(a) {1,7, 9} (b) {3, 5, 8}
(a) {1, 2, 3, 4, 6, 8} (b) {2, 4}
(c)  (d) None (c) {5, 6, 7, 8, 9} (d) {5, 7, 9}.
8. If A = (1, 3, 5, 7, 11, 13, 15, 17}, B = {2, 4, 6 ...18} 18. The set (A  B)’ is equal to :
and N is the universal set, then (a) {1, 2, 3, 4, 6, 8} (b) {2, 4}
A’  [(A  B)  B’] is equal to: o: (c) {5, 6, 7, 8, 9} (d) {1, 3, 5, 6, 7, 8, 9}.
(a) N (b) A’
(c) B (d) None
Sets, Functions and Relations 171
DIRECTION : For Q.No. 19 to Q.No.22. (a) 6000 (b) 8000
A = {2, 3} B = {4, 5} C = {5, 6} (c) 4000 (d) None
19. Then A x (B  C) is equal to 26. If A has 32 elements, B has 42 elements and A
(a) {(2, 4), (2,5), (2,6), (3,4), (3, 5), (3, 6)} u B has 62 elements find the number of
(b) {(2, 5), (3,5)} elements in A  B.
(c) {(2, 4), (2, 5), (3, 4), (3, 5) (4, 5), (4, 6), (5,5) (a) 10 (b) 12
(5, 6)} (c) -12 (d) None
(d) None 27. A town has a total population of 50000 persons
20. The set A x (B  C) is equal to and of them 28000 read, “Gujarat Samachar”
and 23000 read “Sandesh” while 4000 read both
(a) {(2,4), (2,5), (2,6), (3,4), (3, 5), (3, 6)}
the papers. Indicate how many read neither
(b) {(2,5), (3,5)} “Gujarat Samachar” nor “Sandesh”?
(c) {(2, 4), (2, 5), (3, 4), (3, 5) (4, 5), (a) 300 (b) 1300
(4, 6), (5,5) (5, 6)} (c) 3000 (d) None
(d) None 28. 2
If A = {x / x - 17x + 60 = 0}
21. The set (A x B)  (B x C) is equal to B = {x / x2 - 7x+ 12=0} find A  B and.
(a) {(2,4), (2, 5), (2, 6), (3,4), (3, 5), (3, 6)} (a) {3, 4, 5, 12} (b) {-3, 4, 5, -12} {3}
(b) {(2, 5), (3,5)} (c) {3, -4, 5, 12}{12} (d) None
(c) {(2, 4), (2, 5), (3, 4), (3, 5) (4,5), (4, 6), (5, 5) 29. In a college, there are at least 500 girls and of
(5, 6)} them 300 have taken Economics and 250 have
(d) None taken Mathematic. How many of them have
22. Given A {2, 3}, B = {4, 5}, C = {5, 6}, then A x (B  taken both the subjects ?
C) is equal to (a) 40 (b) 50
(a) {(2, 5), (3, 5)} (b) {(5, 2), (5, 3)} (c) -50 (d) None
(c) {(2, 3), (5, 5)} (d) None DIRECTION : For Q.No. 30 to Q.No.31.
23. After qualifying out of 400 professionals, 112 Let the universal set U = {x / 3 < x < 13, x  N}
joined industry, 120 started practice and 160
A = { y / (2  y  7), y  N}
joined as service. There were 32 who were in
both practice and service, 40 in both practice B = {3, 5, 7, 9},
and industry and 20 in both industry and
service. There were 12 who did all the three. 30. Find A’
How many could not get any of these? (a) { 7, 8, 9, 10, 11, 12, 13}
(a) 88 (b) 244 (b) {3, 6, 8, 9, 10, 11, 12, 13}
(c) 122 (d) None (c) {4, 5, 6, 10, 11, 12, 13}
24. In a town of 20,000 families it was found that (d) None
40% families buy newspaper A, 20% families 31. Find (A  B)’
buy newspaper B and 10% families buy (a) {8, 10, 11, 12, 13}
newspaper C, 5% families buy A and B, 3% buy (b) {3, 4, 5, 11, 12, 13}
B and C and 4% buy A and C. If 2% families buy (c) {3, 4, 5, 8, 9, 10, 11, 12, 13}
all the three newspapers, then the number of (d) None
families which buy A only is : DIRECTION : For Q.No. 32 to Q.No.34.
(a) 6600 (b) 6300 If U = (a, b, c, 1, 2, 3) is a universal set,
(c) 5600 (d) 600 A = (a, b, c}, B = (1,2,3), C = (a, 1,2), D = (a, b, 3)
25. In the above questions(24), the number of then
families which buy none of A, B and C is :

172 BUSINESS MATHEMATICS, LOGICAL REASONING & STATISTICS (Paper 3) [CA Foundation]
32. Find the following sets (A — B)  (B — A) 41. How many students play only foot-ball?
(a)  (b) {a, b, 1} (a) 45 (b) 54
(c) {1, 2, 3} (d) None (c) 34 (d) None
33. Find the following sets (C  D)’ 42. How many students play only hockey?
(a) {a} (b) {b} (a) 62 (b) 16
(c) {c} (d) None (c) 36 (d) None
34. Find the following sets (A — D)  (A  D). 43. In a class, 85 students pass in at least one of
(a) {1, 2, 3} (b) {a, b, 2} the three subjects Mathematics, Statistics and
(c)  (d) None Economics. The number of students passing in
35. If A = {2, 3} then find A 2 each of these three subjects is same. Also the
number of students passing in both
(a) (2, 2) (2,3) (3, 2) (3,3) (b) {(4, 9)}
Mathematics and Statistics is 20; passing in
(c) {(2, 2) (3, 3) } (d) None
both Statistics and Economics is 25 and passing
DIRECTION : For Q.No. 36 to Q.No.37. in both Economics and Mathematics is 35. The
If the universal set is X = { x / x  N, 1 < x < 12] and number of students passing in all the three
A= {1,9,10], B = {3,4,6, 11, 12},C = {2,5,6} are subsets subjects is 15. Then find the number of
of X students passing in each of the three subjects.
36. Find the sets A  (B  C) (a) 50 (b) 100
(a) {1, 6, 9, 10} (b) {1, 6}
(c) 25 (d) None
(c) {1, 6, 10} (d) None
44. A class of 100 students appeared for F. Y. B.B.A.
37. Find the sets
examination. Out of 100 students, 40 passed
(A  B)  (A  C). in Mathematics, 36 passed in Management, 60
(a) {1, 6, 9, 10) (b) {1, 2, 5, 6} passed in Accountancy, 8 students passed in
(c) {1, 2, 5, 6, 9, 10} (d) None Mathematics and Management, 17 passed in
DIRECTION : For Q.No. 38 to Q.No.39. Management and Accountancy 16 passed in
If A = {2,4}, B = {2,4,6} Mathematics and Accountancy 5 passed in all
38. Find A x B the three subjects find : How many students
(a) {(2,2), (2,4), (2,6), (4,2), (4,4,) ,(4,6)} passed exactly in one subject?
(b) {(2,2), (2,4), (4,2), (4,4)} (a) 69 (b) 136
(c) {(2,2), (2,4), (2,6), (4,2), (4,4), (4,6), (c) 56 (d) None
(6,2), (6,4), (6,6)} Function :
(d) None 45. If f(x) = x + 3, g(x) = x2, then f(x) g(x) is :
39. Find A x A (a) (x + 3)2 (b) x2 + 3
(a) {(2,2), (2,4), (2,6), (4,2), (4,4,) ,(4,6)} (c) x3 + 3x2 (d) None
(b) {(2,2), (2,4), (4,2), (4,4)} 46. The range of the function f (x) = log (1 + x) for
(c) {(2,2), (2,4), (2,6), (4,2), (4,4), (4,6), the domain of real values of x when x  [0, 9]
(6,2), (6,4), (6,6)} (d) None is
DIRECTION : For Q.No. 40 to Q.No.42. (a) (0, -1) (b) (0, 1, 2)
There are 120 students in a class of a college. 30 (c) [0, 1] (d) None
students do not have interest in games and they do 47. 2
If f(x) = x + 3, g(x) = x , then fog(x) is :
not take part in the games. 64 students of the class (a) x2 + 3 (b) x2 + x + 3
play foot-ball and 36 students play hockey.
(c) (x + 3)2 (d) None
40. How many students play both the games?
48. If f(x) = 1/(1 - x), then f(-1) is :
(a) 5 (b) 10
(a) 0 (b) 1/2
(c) -5 (d) None
(c) 0 (d) None

Sets, Functions and Relations 173


49. The domain of {(1, 7), (2, 6)} is : 60. 1ff: Z  Z,f(x) = x2 + x for all x  z, thenf is :
(a) (1, 6) (b) (7, 6) (a) Many-one (b) One-One
(c) {1, 2} (d) (6, 7) (c) Onto (d) None
50. The range of the function f: A  R, f(x) = x2 + 1, 61. If f :Z  Z, f(x) = 3x + 2 for all x  Z, then f is :
where A = {-1, 0, 2,4} is : (a) Onto (surjective) (b) One-One
(a) {1, 2, 5, 17} (b) {2, 5,17} (c) Injective (d) None
(c) {5, 17} (d) None 62. 3
If f : R  R, f(x) = 3x + 5 for all x  R, then f is :
51. The range of the functiong :A  N, g(x) = 2x, (a) Into
where A = {x  N, x  5) is : (b) One-One onto (bijections)
(a) {1, 2, 3, ...., 10) (b) {2, 4, 6, 8, 10} (c) One-One into (d) None
(c) {1, 3, 5, 7} (d) None 63. 2
If f : R  R; f(x) = x , and g : R  R; g(x) = 2x + 1,
52. Let f : R  R be a function given by f(x) = x2 + 1, then fog is :
then f -1 (-5) is equal to (a) 2x2 + 1 (b) (2x + 1)2
(a) {- 5} (b)  (c) 4x2 + 1 (d) None
(c) {2. 3} (d) None 64. If the function f and g are given by f = {(1, 2), (3,
53. The domain for which the functions f(x) = 2x2 - 5), (4, 1)} and g = {(2, 3), (5, 1), (1, 3)}, then gof
1 and g(x) = 1 - 3x are equal is : is:
(a) {- 2, 1/2} (b) {1/2, 2} (a) {(1, 3), (3, 1), (4, 3)}
(c) { - 2, -1/2} (d) None (b) {(2, 5), (5, 2), (1, 5)}
54. Let f = {(1, 1), (2, 3), (0, -1), (-1, -3)} be a function (c) {(1, 3), (2, 5), 3, 5} (d) None
defined as f(x) = ax + b for some integers a and 65. If f: R  R, f(x) = x2 - 3x + 2, then f(f(x)) =
b. Then the value of (a, b) is : (a) x4 + 10x2 - 3x
(a) (2, -1) (b) (1,2) (b) x4 - 6x3 + 10x2 - 3x
(c) (1, -2) (d) (-1, -2) (c) x4 + 6x4 - 10x2 + 3x (d) None
55. x
Let f: R  R, f(x) = 2 , then the range off is : 66. If f, g : R  R, f(x) = x2 + 3x + 1, g (x) = 2x -3, for all
(a) N (b) R+ x  R, then fog is :
(c) R (d) 1 (a) 4x2 - 6x + 1 (b) 4x2 + 6x - 1
56. If f(x) = x + 3, g(x) = x2, then go f(x) is equal to (c) 4x2 + 6x + 1 (d) None
(a) (x + 3)2 (b) x2 + 3 67. If g : R  R, g(x) = 2x - 3, for all x  R, then gog
(c) x2(x + 3) (d) None is :
57. Let A = {a, b}. Set of subsets of A is called power (a) 4x + 9 (b) 4x - 9
set of A denoted by P(A). Now n(P(A)) is (c) 4x + 7 (d) None
(a) 2 (b) 4 68. If f : R  R, f(x) = 2x + 7 then the inverse off is:
(c) 3 (d) None (a) f -1 (x) = (x - 7)/2 (b) f-1(x) = (x + 7)/2
58. 3
If f R  R, f(x) = x + 2 for all, then f is : (c) f 1(x) = (x - 3)/2 (d) None
(a) One-One onto (b) Onto 69. If f : R  R is a bijection given by f(x) = x3 + 3,
(c) Into (d) None then f-1 (x) is :
59. 2
If f : A  B, f(x) = x , A = {-1, 1, -2, 2}, and B = (a) f-1(x) = (x - 3)1/3 (b) f-1(x) = (x - 3)-1/3
{1,4, 9, 16}, then f is : (c) f-1(x) = (x + 3)1/3 (d) None
(a) One-One (b) Many-one Into 70. Let f: R  R be such that f (x) = 2x, then f(x + y)
(c) Into (d) None equals
(a) f(x) + f(y) (b) f(x) . f(y)
(c) f(x) ÷ f(y) (d) None

174 BUSINESS MATHEMATICS, LOGICAL REASONING & STATISTICS (Paper 3) [CA Foundation]
Relation : 78. If R is the relation “less than or equal to” from
71. The range of the relation {(x, y) : X  N, y  N, A = {1, 3, 4, 5} to B = {2, 3, 5},
and x + y = 10) is : then number of elements in R-1 oR is :
(a) {1,2,3,4,5,6,7, 8,9} (a) 7 (b) 6
(b) {9, 8, 7, 6, 5, 4, 3,2, 1} (c) 8 (d) 9
(c) {1, 2, 3, 4, 5, 7} (d) None 79. A = {1, 2, 3} R = {1, 1), (2, 2), (3, 3), (1, 2), (2,1),
(2, 3), (3, 2)}  A x A, the R is :
72. The domain of there relation {(x, y) : y = x -1
(a) reflexive, transitive but not symmetric
, x  Z and x  3} is : (b) transitive, reflexive and symmetric
(c) reflexive, symmetric nut not transitive
(a) {-3, -2, -1, 0, 1, 2, 3} (b) {4, 3, 2, 1, 0}
(d) reflexive, but not symmetric and not
(c) {6, 1, 2, 3, 4} (d) None
transitive
73. The range of the relation R, where R = {(x, x3);
80. The relation R on the set Z of all integers
x is a prime number less than 10) is :
defined by (x, y)  R  x - y divisible by n is :
(a) {2, 3, 5, 7} (b) {8, 27, 125, 343}
(a) reflexive, transitive and not symmetric
(c) {8, 27, 125, 243} (d) None
(b) not reflexive, transitive and not
74. If f is relation from set A = {2, 3, 5} to set B = {2,
symmetric
3, 6, 8, 10} defined by xfy  x divides y, then
(c) reflexive, not symmetric and not
f=
transitive
(a) {(2, 6), (2, 8), (2, 10), (3, 6), (5, 10)}
(d) an equivalence relation
(b) {(2, 2), (2, 6), (2, 8), (2, 10), (3, 6), (3, 3), (5,
81. The domain of the relation (x, y) : x, y  N, and
10)}
x + y = 10} is :
(c) {(2, 2), (2, 6), (2, 8), (2, 10), (3, 6), (5, 10)}
(a) {1, 2, 3, 4, 5, 6, 7, 8, 9}
(d) none
(b) {9, 8, 7, 6, 5, 4, 3, 2, 1}
75. A = {1, 2, 3} R = {(1, 1), (2, 2), (3, 3)}  A x A.
(c) {7, 6, 5, 4, 3, 2} (d) none
Then R is :
(a) not reflexive but transitive and 82. The range of the relation {(x, y) : y = x  1 ; x
symmetric
(b) not transitive but reflexive and  Z, and x < 3} is :
symmetric (a) {4, 3, 2, 1, 0} (b) {0, 1, 2, 3, 4}
(c) an equivalence relation (c) {1, 2, 3, 4} (d) none
(d) not an equivalence relation 83. Let A = {1, 2, 3, ........, 13, 14}, R = {x, y) : 3x - y =
76. If A = {x / x  N, 1 < x < 8} and R = {(x, y) : x  A 0, x, y  A}, then co-domain of R is
and x + 2y = 9} is a relation in A, (a) {1, 2, 3, 4, 5, 6, 7, 8, 9, 10, 11, 12, 13, 14}
then dom R-1 = (b) {3, 6, 9, 12}
(a) {1, 2, 3, 4} (b) {1, 3, 5, 7 } (c) {12, 9, 6, 3}
(c) {1, 2, 3, 5} (d) {1, 2, 3, .......8}. (d) none
77. A = {1, 2, 3} . R = {(1, 1), (1, 2), (2, 2), (3, 3)} is a 84. The domain of the relation R, where R = {(x, x3)
relation which is : x is a prime number less than 10} is :
(a) reflexive, symmetric, transitive (a) {2, 4, 6, 8} (b) {2, 3, 5, 7}
(c) {2, 3, 5, 7, 9} (c) none
(b) reflexive, symmetric but not transitive
85. The domain of the relation R defined by R =
(c) symmetric, not reflexive and not
{(x, x + 5) : x  {0, 1, 2, 3, 4, 5} } is :
transitive
(a) {0, 1, 2, 3, 4, 5} (b) {5, 4, 3, 2, 1, 0}
(d) reflexive and transitive but not
(c) {5, 6, 7, 8, 9, 10} (d) none
symmetric.
Sets, Functions and Relations 175
86. The range of the relation R, where R = {(x, x + 96. The set of cubes of the natural number is
5) : x  {0, 1, 2, 3, 4, 5}} is : (a) a finite set (b) an infinite set
(a) {0, 1, 2, 3, 4, 5} (b) {5, 6, 7, 8, 9, 10} (c) a null set (d) none of these
(c) {1, 9, 8, 7, 6, 5} (d) none 97. x
The set {2 |x is any positive rational number}
87. The domain of the relation R, where R = {(-3, is
1), (-1, 1), (1, 0), (3, 0)} is :
(a) {-3, -1, 1, 3} (b) { -1, -3, 1, 3} (a) an infinite set (b) a null set
(c) {0, 1} (d) none (c) a finite set (d) none of these
88. The range of the relation R, where
98. {1– (–1)x} for all integral x is the set
R = {(x, y) : x is a multiple of 3 and y is a multiple
of 5} is : (a) {0} (b) {2}
(a) {3n, : n  Z} (b) {5n, : n  Z} (c) {0, 2} (d) none
(c) {15n, n  Z} (d) none 99. E is a set of positive even number and O is a
89. The range of the relation R, where R = {(x, x2) :
set of positive odd numbers, then E  O is a
x is a prime number less than 15} is :
(a) {2, 3, 5, 7, 11, 13} (a) set of whole numbers (b) N
(b) {4, 9, 25, 49, 121, 169} (c) a set of rational number
(c) {7, 9, 25, 49, 121, 169, 225} (d) none (d) none
90. The co-domain of the relation R, where R = {x,
100. If R is the set of positive rational number and E
y) : y = x + 1}, and R : A  A, where A = {1, 2, 3,
is the set of real numbers then
4, 5, 6} is :
(a) R  E (b) R  E
(a) {1, 2, 3, 4, 5, 6}
(b) {1, 2, 5, 9, 10} (c) E  R (d) none
(c) {1, 2, 3, 4, 5} (d) none 101. If N is the set of natural numbers and I is the
91. Let A = {1, 2} and B = {3, 4}. The number of set of positive integers, then
relation from A into B is : (a) N = I (b) N  I
(a) 16 (b) 12
(c) N  I (d) none
(c) 4 (d) 8
92. The range of the relation R, where R = {(4x + 3, 102. If I is the set of isosceles triangles and E is the
1 - x) : x < 4, x  N} is : set of equilateral triangles, then
(a) {0, -1, -2, -3} (a) I  E (b) E  I
(b) {2, 5, 10, 17, 26} (c) E = I (d) none
(c) {7, 11, 15, 19} (d) none 103. If R is the set of isosceles right angled triangles
93. The domain of the relation R, where R = {x + 4, and I is set of isosceles triangles, then
2x (a) R = I (b) R  I
) : 4 < x < 6, x  N} is :
2x (c) R  I (d) none
(a) {8, 9, 10} (b) {-3, -7/3, - 2} 104. {n(n+1)/2 : n is a positive integer} is
(c) {7, 8, 9} (d) none (a) a finite set (b) an infinite set
94. The range of the relation R, where R = {1 + x, 1
(c) is an empty set (d) none
+ x2) : x < 5, x  N} is :
(a) {2, 3, 4, 5, 6} (b) {2, 5, 10, 17, 26} 105. If A = {1, 2, 3, 5, 7}, and B = {x2 : x  A}
(c) {2, 3, 4, 6, 10} (d) none (a) n(b) = n(A) (b) n(B) > n(A)
95. The domain of the relation R, where R = {(1 + x, (c) n(A)= n(B) (d) n(A)<n(B)
1 + x2) : x < 5, x  N} is : 106. A  A is equal to
(a) {2, 3, 4, 5, 6} (b) {1, 2, 3, 4, 5}
(a) A (b) E
(c) {3, 4, 5, 6, 7} (d) none
(c)  (d) none

176 BUSINESS MATHEMATICS, LOGICAL REASONING & STATISTICS (Paper 3) [CA Foundation]
112. As per question No.(16) express the following
sets in set notation: –
107. A  A is equal to
(a)  (b) A (i) {x|x is a family with two or more sets or
income of ` 11000/- to ` 15999/-}
(c) E (d) none
(ii) {x|x is a family with no set}
108. If A  B = (A–B)  (B–A) and A = {1, 2, 3, 4}, B =
{3,5,7} than A  B is
(a) (A  E) (b) (A  B)’
(a) {1, 2, 4, 5, 7} (b) {3}
(c) {1, 2, 3, 4, 5, 7} (d) none (c) Both (d) None
109. If A={a, b, c}, B={a, b}, C = {a, b, d}, D={c, d} and 113. If A = {a, b, c, d} list the element of power set P
E = {d} state which of the following statements (a)
are correct: – (i) B  A (ii) D  C (iii) C  E (iv) (a) {  {a} {b}({c} {d} {a, b} {a, c} {a, d} {b, c}
D  E (v) D  B (vi) D = A (vii) B  C (viii) E 
{b, d} {c, d}
A (ix) E  B (x) a  A (xi) a  A (xii) {a}  A
(b) {a, b, c} {a, b, d} {a, c, d} {b, c, d}
(xiii) {a}  A
(a) (i) (ii) (iii) (ix) (x) (xiii) only are correct (c) {a, b, c, d}
(b) (ii) (iii) (iv) (x) (xii) (xiii) only are correct (d) All the above elements are in P (a)
(c) (i) (ii) (iv) (ix) (xi) (xiii) only are correct 114. If four members a, b, c, d of a decision making
(d) None body are in a meeting to pass a resolution
110. State whether the following sets are finite, where rule of majority prevails list the wining
infinite or empty: – (i) X = {1, 2, 3, …..500} (ii) Y coalitions. Given that a, b, c, d own 50%, 20%,
= {y: y ƒ­ a2 ; a is an integer} (iii) A = {x:x is a 15%, 15% shares each.
positive integer multiple of 2} (iv) B = {x:x is an
(a) {a, b} {a, c} {a, d} {a, b, c} {a, b, d} {a, b, c, d}
integer which is a perfect root of 26 < x < 35}
(a) finite, infinite, infinite, empty (b) {b, c, d}
(b) infinite, infinite, finite, empty (c) {b, c} {b, d} {c, d} {a, c, d} {b, c, d} {a} {b} {c}
(c) infinite, finite, infinite, empty
{d}  (d) None
(d) None
111. A sample of income group of 1172 families was 115. Out of a total population of 50,000 only 28,000
surveyed and noticed that for income groups read Telegraph and 23,000 read Times of India
< `6000/-, 6000/- to `10999/-, `11000/-, to while 4,000 read the both. How many do not
`15999/-, `16000 and above No. TV set is read any paper?
available to 70, 50, 20, 50 families one set is (a) 3,000 (b) 2,000
available to 152, 308, 114, 46 families and two (c) 4,000 (d) None
or more sets are available to 10, 174, 84, 94
116. Out 2000 staff 48% preferred coffee 54% tea
families.
and 64% cocoa. Of the total 28% used coffee
If A = {x|x is a family owning two or more sets},
and tea 32% tea and cocoa and 30% coffee and
B = {x|x is a family with one set,
cocoa. Only 6% did none of these. Find the
}C = {x|x is a family with income less than `
number having all the three.
6000/-}, D = {x|x| is a family with income
` 6000/- to ` 10999/-}, E = {x|x is a family with (a) 360 (b) 280
income ` 11000/- to ` 15999/-}, find the (c) 160 (d) None
number of families in each of the following 117. Complaints about works canteen had been
sets (i) C  B (ii) A  E about Mess (M) Food (F) and Service (S). Total
(a) 152, 580 (b) 152, 20 complaints 173 were received as follows: –
(c) 152, 50 (d) 152, 496 n(M) = 110, n(F) = 55, n(S) = 67,
n(M  F  S’) = 20, n(M  S  F’) = 11

Sets, Functions and Relations 177


and n(F  S  M’) = 16. Determine the If S M L T I denote short medium long terms skilled
complaints about all the three. and indirect workers respectively find the number
(a) 6 (b) 53 of workers in set M.
(c) 35 (d) None (a) 42 (b) 8
(c) 10 (d) 43
118. Out of total 150 students 45 passed in Accounts
50 in Maths. 30 in Costing 30 in both Accounts 123. Out of 1000 students 658 failed in the
and Maths. 32 in both Maths and Costing 35 in aggregate, 166 in the aggregate and in group-I
both Accounts and Costing. 25 students passed 434 in aggregate and in group-II, 372 in group-
in all the three subjects. Find the number who I, 590 in group-II and 126 in both the groups.
passed at least in any one of the subjects. Find out how many failed in all the three.
(a) 63 (b) 53 (a) 106 (b) 224
(c) 206 (d) 464
(c) 73 (d) None
124. If A = {x, y, z}, B = {p, q, r, s} which of the relation
119. After qualifying out of 400 professionals, 112 on A to B are function.
joined industry, 120 started practice and 160
(a) {n, p), (x, q), (y, r), (z, s)},
joined as service. There were 32 who were in
both practice and service, 40 in both practice (b) {( x, s), (y, s), (z, s)}
and industry and 20 in both industry and (c) {(y, p), (y, q), (y, r),(z, s),
service. There were 12 who did all the three.
(d) {(x, p), (y, r), (z, s)}
How many could not get any of these?
(a) 88 (b) 244 125. {(x, y)|x+y = 5} where x, y  R is a
(c) 122 (d) None (a) not a function
120. On a survey of 100 boys it was found that 50 (b) a composite function
used white shirt 40 red and 30 blue. 20 were (c) one-one mapping
habituated in using both white and red shirts
15 both red and blue shirts and 10 blue and (d) none of these
white shirts. Find the number of boys using all 126. {( x, y)|x = 4} where x, y  R is a
the colours. (a) not a function
(a) 20 (b) 25 (b) function
(c) 30 (d) None (c) one-one mapping
121. A survey of 1000 customers revealed the (d) none of these
following in respect of their buying habits of
127. {(x, y), y=x2} where x, y  R is
different grades:
A grade A and C C grade A grade but A grade C and B None (a) not a function (b) a function
only grades not B grade grades
(c) inverse mapping (d) none
180 80 480 230 360 80 140
128. {(x, y)|x<y} where x, y  R is
How many buy B grade?
(a) not a function (b) a function
(a) 280 (b) 400
(c) one-one mapping (d) none
(c) 50 (d) None
129. The domain of {(1, 7), (2, 6)} is
122. Consider the following data: –
(a) (1, 6) (b) (7, 6)
Skilled Unskilled & Skilled & Unskilled &
& Direct Direct Indirect Indirect (c) (1, 2) (d) {6, 7}
Worker Worker Worker Worker
130. The range of {(3, 0), (2, 0), (1, 0), (0, 0)} is
Short Term 6 8 10 20
Medium Term 7 10 16 9 (a) {0, 0} (b) {0}
Long Term 3 2 8 0 (c) {0, 0, 0, 0} (d) none

178 BUSINESS MATHEMATICS, LOGICAL REASONING & STATISTICS (Paper 3) [CA Foundation]
131. The domain and range of {(x,y) : y = x2} where 142. Which of the diagram is graph of a function
x, y  R is
(a) (reals, natural numbers)
(b) (reals, positive reals)
(c) (reals, reals) (d) none
(a)
132. Let the domain of x be the set {1}. Which of
the following functions are equal to 1
(a) f(x) = x2, g(x) = x
(b) f(a) = x, g(x) = 1–x
(c) f(x) = x2 + x + 2, g(x) = (x+ 1)2
(d) none
133. If f(x) = 1/1–x, f(–1) is (b)
(a) 0 (b) ½
(c) 0 (d) none
134. If g(x) = (x–1)/x, g(–½) is
(a) 1 (b) 2
(c) 3/2 (d) 3
(c)
135. If f(x) = 1/1–x and g(x) = (x–1)/x, than fog(x) is
(a) x (b) 1/x
(c) –x (d) none
136. If f(x) = 1/1–x and g(x) = (x–1)/x, then g of(x) is
(a) x–1 (b) x
(c) 1/x (d) none
x
137. The function f(x) = 2 is
(d)
(a) one-one mapping (b) one-many
(c) many-one (d) none
138. The Inverse function f–1 of f(x) = 2x is
143. If f(x) = x + 3, g(x) = x2, then g of (x) is
x
(a) 1/2x (b) (a) (x + 3)2 (b) x2 + 3
2 (c) x2 (x + 3), (d) none
(c) 1/x (d) none 144. If f(x) = 1/1–x, then f-1(x) is
139. If f(x) = x+3, g(x) = x2, then (a) 1–x (b) (x–1)/x
f(x).g(x) is (c) x/(x–1) (d) none of these
(a) (x + 3)2 (b) x2 + 3 145. "Is smaller than" over the set of eggs in a box
(c) x3 + 3x2 (d) none is
–1
140. The Inverse h when h(x) = log10 x is (a) Transitive (T) (b) Symmetric (S)
(a) log 10 x (b) 10x (c) Reflexive (R) (d) Equivalence (E)
(c) log 10(1/x) (d) none 146. "has the same father as" …… over the set of
141. For the function h(x) = 101+x the domain of real children
values of x where 0 <x <9, the range is (a) R (b) S
(a) 10 < h (x) < 1010 (b) 0 < h (x) < 1010
(c) T (d) All of the above
(c) 0 < h(x) < 10 (d) none
147. "Is the square of" over n set of real numbers is
(a) R (b) S
(c) T (d) None
---0---0---0---

Sets, Functions and Relations 179


10. If P is a set of natural number then P P’ is
(a)  (b) Sample Space.
HOME WORK-1 (c) 0 (d) (P  P’)’
11. If A = {1, 2, 3, 4}, B = {5, 6, 7} then cardinal number
1. Which of the following pairs of events are of A X B is
mutually exclusive? (a) 4 (b) 7
(a) A : The student reads in a school. B : He (c) 12 (d) None
studies Philosophy. 12. The set of squares of positive integers is
(b) A : Raju was born in India. B : He is a fine (a) A finite set (b) Null set
Engineer. (c) An infinite set (d) None
(c) A : Ruma is 16 years old. B : She is a good 13. If B is any set then B B is
singer. (a) Null Set (b) B
(d) A : Peter is under 15 years of age. B : Peter (c) Whole set (d) None
is a voter of Kolkata. 14. If B is any set then B B is
(a) B (b) Null set
2. If y= f(x) = then f(y) is ______ . (c) Whole set (d) None
15. “Is equal to” is a
(a) –x (b) 2x (a) Symmetric relation
(c) x (d) x2 (b) Reflexive relation
3. If g(x) = (x–1)/x, g(–½) is (c) Transitive relation
(a) 1 (b) 2 (d) Equivalence relation
(c) 3/2 (d) 3 16. If f(x) = x2 + 2, then the given function is
4. If the set P has 3 elements, Q four and R two (a) odd function (b) even function
then the set P×Q×R contains (c) Neither odd nor even function
(a) 9 elements (b) 20 elements (d) None
(c) 24 elements (d) None 17. For the function f(x) = 121+x, the domain of real
5. In a group of 20 children, 8 drink tea but not values of x where 0 < x < 9 the range is
coffee and 13 like tea. The number of children (a) 12 < f(x) < 1210 (b) 0 < f(x) < 1210
drinking coffee but not tea is (c) 0 < f(x) < 12 (d) None
(a) 6 (b) 7 18. “Is greater than” over the set of real number s
(c) 1 (d) None is
6. If A has 32 elements, B has 42 elements and (a) Transitive relation
A B has 62 elements, the number of (b) Symmetric relation
elements in A B is (c) Reflexive relation
(a) 12 (b) 74 (d) Equivalence relation
(c) 10 (d) None 19. If f(x) = x2+3x then f(2) – f(4) is equal to
7. A A’ is equal to o (a) –15 (b) –18
(a) A. (b) Sample Space. (c) 18 (d) 12
(c) (d) None
8. If A has 70 elements, B has 32 elements and A
 B has 22 elements then A  B is 20. If f (x)= –x,f is
(a) 60 (b) 124
(c) 80 (d) None (a) 3/2 (b) 2/3
9. If A = {1,2,3,5,7} and B= {1,3,6,10,15} then (c) 1 (d) 0
cardinal number of A-B is 21. If f (x+1) = 2x + 7 then f ( – 2) is
(a) 3 (b) –4 (a) 1 (b) 2
(c) 6 (d) None (c) 3 (d) 4
180 BUSINESS MATHEMATICS, LOGICAL REASONING & STATISTICS (Paper 3) [CA Foundation]
22. If f(x) = 2x + 3 then f(2x) – 2f(x)+3 is equal to 32. A town has a total population of 50,000. Out of
(a) 1 (b) 0 it 28,000 read the newspaper X and 23000 read
(c) –1 (d) None Y while 4000 read both the papers. The number
2
23. If f’(x) = 3x + 2 then find f(2). of persons not reading X and Y both is
(a) 8 (b) 10 (a) 2000 (b) 3000
(c) 12 (d) None (c) 2500 (d) None
33. The number of subsets of the set {1,2,3,4} is
24. If f(x)=x 2 –1 and g(x) = then (a) 13 (b) 12
(c) 16 (d) 15
34. The number of subset of set {2, 4, 6} is
(a) 12 (b) 8
is (c) 6 (d) None
35. The number of subset of a set containing n
(a) 5/4 (b) 4/5 element is:
(c) 3/5 (d) 5/3 (a) 2n (b) 2n
-n
25. 2
If f(x) = 2x+5 and g(x) = x +1 the fog = ? (c) 2 (d) None
(a) 2x2 + 7 (b) 2x + 1 36. The set of cubes of the natural number is
(c) x2 + 5 (d) None (a) A finite set (b) An infinite set

(c) As null set (d) None


26. If f(x) = then value of f(x) – f (1/x) is
37. If A = {4,5}, B={2,3}, C={5,6} then AX (B  C) is
equal to (a) {(2,5), (3,5)} (b) {(4,2), (4,6)}
(a) 0 (b) 1 (c) {(4,3), (4,2) (d) None

(c) x3 + (d) None ANSWER KEYS


1. (d) 2 (c) 3 (d) 4 (c)
27. A function f(x) is an even function if
(a) –f(x) = f(x) (b) f(–x) = f(x) 5 (b) 6 (a) 7 (b) 8 (c)
(c) f(–x) = –f(x) (d) None 9 (a) 10 (a) 11 (c) 12 (c)
28. If a function in x is defined by f(x) = 13 (b) 14 (a) 15 (d) 16 (b)
17 (a) 18 (a) 19 (b) 20 (a)
X R then f(1/x) = ________
21 (a) 22 (b) 23 (c) 24 (b)
(a) f(x) (b) f(–x) 25 (a) 26 (a) 27 (b) 28 (a)
(c) –f(x) (d) 0 29 (a) 30 (b) 31 (a) 32 (b)
29. In a class 30 students, 20 students like maths,
18 like science and 12 like both the subject. 33 (c) 34 (b) 35 (b) 36 (b)
F ind the number of student who like no 37 (d)
subject.
(a) 4 (b) 5
(c) 8 (d) None
30. The number of subsets of the set {2, 3, 5} is
(a) 3 (b) 8
(c) 6 (d) None
31. Given A = {2, 3}, B = {4, 5}, C = {5, 6} then
A × (B  C) is
(a) {(2, 5), (3, 5)} (b) {(5, 2), (5, 3)}
(c) {(2, 3), (5, 5)} (d) None

Sets, Functions and Relations 181


Determine the compliants about all the three.
(a) 6 (b) 43
(c) 35 (d) None
HOME WORK - 2
10. If f(x + 1) = 2x + 7 then f(0) is equal to
(a) 5 (b) 4
1. The null set is represented by (c) 3 (d) 0
(a)  (b)  11. If f(x) = x3 - x2 + x + 1 then the value of
[f(1) + f(-1)]/2 will be
(c)  (d) None
(a) 5 (b) 2
2. If A = {a, b, c}, then n (p (A)) is (c) 0 (d) -2
(a) 3 (b) 8
(c) 7 (d) 1
3. Equal sets are -
ANSWER KEYS
(a) equivalent (b) null
(c) disjoint (d) None
4. The range set of a constant function is a - 1 (c) 2 (b) 3 (a) 4 (b)
(a) Disjoint set (b) Singleton set
(c) Void set (d) Infinite set 5 (a) 6 (a) 7 (c) 8 (c)
5. If P is a set of natural number then P  P’ is
(a)  (b) Sample Space 9 (a) 10 (a) 11 (c)
(c) 0 (d) (P  P’)
6. If A x b = {(3, 2), (3, 4), (5, 2), (5, 4)}, then find A
and B.
(a) A = {3, 5} and B = {2, 4}
(b) A = {3, 4} and B = {2, 5}
(c) A = {3, 2} and B = {5, 4}
(d) A = {5, 4} and B = {2, 3}
7. If A = {1, 2, 3, 4}, B = {5, 6, 7} then cardinal number
of the set A x B is
(a) 7 (b) 1
(c) 12 (d) None
8. A survey shows that 74% of the Indians like
grapes, whereas 68% like bananas. What % of
Indians like both grapes and bananas ?
(a) 32% (b) 26%
(c) 42% (d) 50%
9. Complaints about works canteen had been
about Mess (M) Food (F) and Services (S). Total
complanits 173 were received as follows - n(M)
= 110; n (F) = 55; n (S) = 67; n(M  F  S’) = 20;
n(M  S  F’) = 1; n (F  S  M’) = 16.

182 BUSINESS MATHEMATICS, LOGICAL REASONING & STATISTICS (Paper 3) [CA Foundation]
CHAPTER-8
BASIC CONCEPTS OF DIFFERENTIAL AND INTEGRAL CALCULUS

UNIT-I : DIFFERENTIAL CALCULUS

INTRODUCTION
Differentiation is very important operations in calculus. Its theory primarily depends on the idea of limit and
continuity of function.
To express the rate of change in any function we introduce concept of derivative which involves a very small
change in the dependent variable with reference to a very small change in independent variable.
Thus differentiation is the process of finding the derivative of a continuous function. It is defined as the
limiting value of the ratio of the change (increment) in the function corresponding to a small change
(increment) in the independent variable (argument) as the later tends to zero.

DEFINITION
Let y = f(x) be a function. If h (or  x) be the small increment in x and the corresponding increment in y or f(x)
be  y = f(x+h) – f(x) then the derivative of f(x) is defined

d
This is denoted as f’(x) or dy/ dx or f(x).
dx

Basic Concepts of Differential and Integral Calculus 183


Differentiation Formula
Some standard results (formulas)

d
1. (xn) = nxn-1
dx

d x
2. (e ) = ex
dx

d
3. (ax) = ax loge a
dx

d
4. (constant) = 0
dx

d ax
5. (e ) = aeax
dx

d 1
6. (log x) = x
dx

d 1
7. (loga x) = loga e
dx x

d x x
8. x : x (1+ logx)
dx

184 BUSINESS MATHEMATICS, LOGICAL REASONING & STATISTICS (Paper 3) [CA Foundation]
Rules in Differentiation :
S.No Function Derivative

d (constant)
1 0
dx
d {c f(x)} C d { f(x)}
2
dx dx
d {f(x)  g(x)} d {f(x)}  d {g(x)}
3
dx dx dx
d {f(x)g(x)} f(x) d {g(x)} + g(x) d {f(x)}
4
dx dx dx

d { f(x) } g(x) d {f(x)} - f(x) d {g(x)}


5 dx g(x) dx dx
{g(x)}2

dy dy/dt
6 If x =  (t) and y =  (t), then
dx dx/dt

du du/dx
7 If u and v are the functions of x, then
dv dv/dx

UNIT-II INTEGRAL CALCULUS


INTRODUCTION
Integration is the reverse process of differentiation.

Basic Concepts of Differential and Integral Calculus 185


We know

Integration is the inverse operation of differentiation and is denoted by the symbol  .


Hence, from equation (1), it follows that

i.e. Integral of xn with respect to variable x is equal to

Thus if we differentiate we can get back xn

Again if we differentiate + c and c being a constant, we get back the same xn .

i.e. + c and this c is called the constant of integration.

Integral calculus was primarily invented to determine the area bounded by the curves dividing the entire
area into infinite number of infinitesimal small areas and taking the sum of all these small areas.

186 BUSINESS MATHEMATICS, LOGICAL REASONING & STATISTICS (Paper 3) [CA Foundation]
BASIC Formulas

x
(vii)  x (1 + logx) = xx + K
Note: In the answer for all integral sums we add +c (constant of integration) since the differentiation of
constant is always zero.
Elementary Rules:

Product Rule :

Basic Concepts of Differential and Integral Calculus 187


 du 
 uvdx = u vdx - 
   dx  vdx  dx
u and v are decided as per LATE Rule
where,
L = logarithmic terms
A = algebraic terms
T = Trigonometric terms
E = Exponential terms

Some Standard Results :


S.No. Function Integral

1 Kf(x)dx K  f(x)dx

2  f(x)  g(x) dx  f(x)dx   g(x)dx


n+1
n 1  ax+b
3 (ax+b) dx a n+1
+C

1 1
4  ax +b dx a
log ax +b + c

1 abx+c
5 a
bx+c + c,b > 0,b  1
b logea

bx+c 1 bx+c
6  e dx e +c
b

188 BUSINESS MATHEMATICS, LOGICAL REASONING & STATISTICS (Paper 3) [CA Foundation]
SOME SPECIAL INTEGRALS

dx 1  xa 
1.  x2  a2 = 2a log  x  a  (x > a)
dx 1  a x 
2. a 2
x 2 = log 
2a  a  x  (x < a)

dx
3.  2
x a 
2 2
2 = log  x  x – a 

dx
4.  x
x  a 2 = log 
2 x2 a2 

2 2 x a2 
5.  x  a dx = x  a  log  x  x2  a2 
2 2
2 2

2 2 x a2
6.  x a dx = x a  log x  x2  a2 
2 2
2 2  
x
7. e [ f (x) f '(x)]dx = e f(x) x

Concept of Definite Integrals :


1. Meaning : The summation of the values of function f(x) in the interval [a,b] of the variable x is called as
b

the Definite Integral  f' (x)dx


a
= f(b) - f(a), where f’(x) is the given integrand and f(x) is the indefinitee

Integral of f’(x).
2. Properties of Definite Integrals :
b a

Property 1.  f(x)dx    f(x)dx


a b

b c b

Property 2.  f (x)dx   f(x)dx +  f(x)dx;for a < c < b.


a a c

Basic Concepts of Differential and Integral Calculus 189


b b

Property 3.  f (x)dx   f (a +b - x)dx


a a

a a

Property 4.  f (x)dx   f (a - x)dx


0 0

2a a a

Property 5.  f (x)dx   f (x) dx +  f (2a - x) dx


0 0 0

2a a

Property 6. (a)  f (x)dx  2 f(x)dx, if f(2a - x) = f(x)


0 0

2a a

(b)  f (x)dx  0 f(x)dx, if f(2a- x) = f(x)


0 0

a a

Property 7. (a)  f (x)dx  2 f(x)dx, if f is an even function


-a 0

(b)  f (x)dx  0, if f is an odd function


-a

na a

Property 8.  f (x)dx =n f(x) dx, If f(x) = f(x + a)


0 0

190 BUSINESS MATHEMATICS, LOGICAL REASONING & STATISTICS (Paper 3) [CA Foundation]
7. The gradient of the curve y – xy + 2px + 3qy = 0
-2
UNIT : 1 DIFFERENTIATION at the point (3, 2 ) is The values of p and q are
3
(a) (1/2, 1/2) (b) (2, 2)
(c) (–1/2, –1/2) (d) (1/2, 1/6)
CLASS WORK 2 3
8. The curve y = ux + v passes through the point
1. The gradient of the curve y = 2x3 – 3x2 – 12x + 8 dy
at x = 0 is P(2, 3) and = 4 at P. The values of u and v
dx
(a) –12 (b) 12 are
(c) 0 (d) none of these (a) (u = 2, v = 7) (b) (u = 2, v =– 7)
2. The gradient of the curve y = 2x3 –5x2 – 3x at x = (c) (u = –2, v =– 7) (d) (0, -1)
0 is 9. The gradient of the curve y + px + qy = 0 at (1, 1)
(a) 3 (b) –3 is 1/2 . The values of p and q are
(c) 1/3 (d) none of these (a) (–1, 1) (b) (2, –1)
3. The derivative of y = X + 1 is (c) (1, 2) (d) (0, -1)
10. If xy = 1 then y2 + dy/dx is equal to
(a) 1/ X +1 (a) 1 (b) 0
(b) -1 / X + 1 (c) –1 (d) none of these

(c) 1/2 X +1 11. The derivative of the function X + X is.


(d) none of these
1 1
2 (a) (b) 1+
4. If f(x) = eax +bx+c the f ’(x) is 2 x+ x 2 x

(a) eax2 +bx+c 1  1 


(c) 1 +  (d) none of these
2
(b) e ax +bx+c  2ax + b  2 x+ x  2 x 

(c) 2ax +b dy
12. Given e-xy – 4xy = 0, can be proved to be
dx
(d) none of these
(a) – y /x (b) y/x
2
X +1 (c) x/y (d) none of these
5. If f(x) = then f’(x) is
X2 - 1 x2 y 2 dy
(a) –4x / (x2 – 1)2 (b) 4x / (x2 – 1)2 13. If 2 - 2 = 1, can be expressed as
a a dx
(c) x / (x2 – 1)2 (d) none of these
x x
dy (a) (b)
6. If y = x (x –1 ) (x – 2) then is y 2 2
dx x -a

(a) 3x2 – 6x +2 (b) –6x + 2 1


(c) 3x2 + 2 (d) none of these x2
(c) -1 (d) none of these
a2

Basic Concepts of Differential and Integral Calculus 191


dy 20. The slope of the tangent to the curve y = 2
14. If log (x / y) = x + y, may be found to be 4-X
dx at the point, where the ordinate and the abscissa
are equal, is
y 1 - x  y
(a) (b) ((a) –1 ((b) 1
x 1 + y  x ((c) 0 ((d) ((a) and ((b) both
1- x 21. The slope of the tangent to the curve y = x2 –x at
(c) (d) none of these the point, where the line y = 2 cuts the curve in
1+ y
the Ist quadrant, is
dy (a) 2 (b) 3
15. If f(x, y) = x3 + y3 – 3axy = 0, can be found (c) –3 (d) none of these
dx
2 2
out as 22. For the curve x + y + 2gx + 2hy = 0, the value
dy
ay - x2 ay - x2 of at (0, 0) is
(a) (b) dx
y2 + ax y2 - ax
(a) -g/h (b) g/h
ay + x2 (c) h/g (d) none of these
(c) (d) none of these
y2 + ax e3x - e2x dy
23. If y = 3x 2x , then is equal to
dy e +e dx
16. Given x = at2, y = 2at; is calculated as
dx (a) 2e5x (b) 1/(e5x + e2x )2
(a) t (b) –1/t (c) e5x/(e5x + e2x ) (d) none of these
(c) 1/t (d) none of these dy
24. If xy . yx = M, where M is constant then is
dy dx
2
17. Given x = 2t + 5, y = t – 2; is calculated as
dx equal to
(a) t (b) –1/t -y  y + x log y 
-y
(c) 1/t (d) none of these (a) (b)
X x  y log x + x 
1 dy
18. If y = then is equal to y + x log y
x dx
(c) (d) none of these
y log x + x
1 -1
(a) (b) dy
2x x x x
25. Given x = t + t–1 and y = t – t–1 the value of
dx
-1 at t = 2 is
(c) ((d) none of these
2x x (a) 3/5 (b) –3/5
(c) 5/3 (d) none of these
dy
19. If x = 3t2 –1, y = t3 –t, then is equal to
dx dy
26. If x3 –2x2 y2 + 5x +y –5 =0 then at x = 1, y =
dx
3t2 - 1 1 is equal to
(a) (b) 3t2–1
6t (a) 4/3 (b) – 4/3
3t - 1 (c) 3/4 (d) none of these
(c) (d) none of these
6t
192 BUSINESS MATHEMATICS, LOGICAL REASONING & STATISTICS (Paper 3) [CA Foundation]
27. The derivative of x2 log x is a+1+2x
(a) 1+2log x (b) x(1 + 2 log x) a + x 
33. If f(x) =   the value of f’(0) is
(c) 2 log x (d) none of these 1 + x 
3 - 5x (a) aa+1
28. The derivative of
3 + 5x  1 - a2 
(b) aa+1  + 2log a 
(a) 30/(3 +5x)2 (b) 1/(3 +5x)2  a 
(c) –30/(3 +5x)2 (d) none of these
(c) 2 log a
dy (d) none of these
29. Let y = 2x + 32x then is equal to
dx
 dy 
2x 34. If x = at2, y = 2at then   is equal to
(a)  
1 / 2x + 2.3 loge 3  dx  t=2
(a) 1/2 (b) –2
(b) 1 / 2x (c) –1/2 (d) none of these
(c) 2.32x loge3
2
(d) none of these  1 
35. Let f(x) =  x +  then f’(2) is equal to
 x
  x - 2 3/4 
30.
 x
The derivative of log e  x + 2 
 is (a) 3/4 (b) 1/2
    (c) 0 (d) none of these
 
36. If f(x) = x2 – 6x+8 then f’(5) – f’(8) is equal to
x2 + 1 x2 - 1 (a) f’(2) (b) 3f’(2)
(a) (b)
x2 + 4 x2 - 4 (c) 2f’(2) (d) none of these

1 n
(c) 2
x -4
(d) none of these 37.  2
If y = x + x + m
2
 then dy/dx is equal to

2
31. The derivative of e3x -6x+2 is (a) ny (b) ny / x2 + m2
5 5
(a) 30(1 –5x) (b) (1–5x )
2 (c) -ny / x2 + m2 (d) none of these
(c) 6  x – 1  e3x -6x+2 (d) none of these
38. If y = + dy - x + m
x /m + m / x then 2xy
ex + 1 dy dx - m + x
32. If y = x then is equal to is equal to
e -1 dx
(a) 0 (b) 1
x x (c) –1 (d) none of these
-2e 2e
(a) 2 (b) 2
 ex - 1   ex - 1 x2 x3 xn
39. If y = 1 + x + + + .. +
2! 3! n!

-2 dy
then – y is proved to be
(c) 2 (d) none of these dx
 ex - 1  (a) 1 (b) –1
(c) 0 d )none of these

Basic Concepts of Differential and Integral Calculus 193


40. If f(x) = xk and f’(1) = 10 the value of k is 48. f(x) = x 2 /e x then f ’(1) is equal to
(a) 10 (b) –10 _____________.
(c) 1/10 (d) none of these (a) – 1/e (b) 1/e
41. If y = 2 2 (c) e (d) none of these
x + m then y y1 (where y1 = dy/dx) is
equal to m
(a) –x (b) x
49.  2
If y = x + x - 1 
(c) 1/x (d) none of these
2 2 2 2
42. If y = ex + e–x then
dy
- y2 - 4 is equal to
 
then x - 1  dy / dx  - m y is proved to be
dx (a) –1 (b) 1
(a) 1 (b) –1 (c) 0 (d) none of these
(c) 0 (d) none of these
4 - 2x
43. The derivative of (x2–1)/x is 50. If f(x) = then the values of x for
2 2
2 + 3x + 3x2
(a) 1 + 1/x (b) 1 – 1/x
2
which f’(x) = 0 is
(c) 1/x (d) none of these
44. The differential coefficients of (x2 +1)/x is  5
(a) 1 + 1/x2 (b) 1 – 1/x2 ((a) 2  1 ±


3 
((b) 1 ± 3 
(c) 1/x2 (d) none of these
((c) 2 ((d) none
dy
45. If y = e 2x then is equal to
dx

e 2x
(a) (b) e 2x
2x

e 2x
(c) (d) none of these
2x

46. If y = x......  then dy is equal to


x dx
_____________.

y2 y2
(a) (b)
2 - y log x x  2 - y log x 

y2
(c) (d) none of these
log x
47. If x = (1 – t2 )/(1 + t2) y = 2t/(1 + t2) then dy/dx
at t =1 is _____________.
(a) 1/2 (b) 1
(c) 0 (d) none of these

194 BUSINESS MATHEMATICS, LOGICAL REASONING & STATISTICS (Paper 3) [CA Foundation]
7
(a) 3 + 6x +
(2  x )2
HOME WORK - 1 7
(b) 2 + 6x –
(2  x )2
5 3 1
1. If f (x) = x 2  x 2  x 2  7 then f '(x) is 7
(c) 2 + 6x +
5 3 2 3 12 1  1 2 (2  x )2
(a) x  x  x
2 2 2 (d) none
5 3 2 3 12 1  12
(b) x  x  x 7
2 2 2 dy
7. If (2x + 1) (y + 3) = 12 then is
5 3 2 3 12 1  1 2
dx
(c) x  x  x
2 2 2 24  24
(a) (b)
(d) none 2x  1 (2 x  1)2
2. If f (x) = (2x2 + 5x – 3) (4x + 1) then f '(x) is 24
(a) 24x2 + 44x + 7 (b) 24x2 + 44x – 7 (c) (d) none
(2 x  1)2
(c) 8x2 + 22x – 7 (d) none
dy
2 x2  3 dy 8. If xy + 3x + 5y – 1 = 0 then is
3. If y = 2 then is dx
x 7 dx
 16 16
(a) (b)
34 x 34 ( x  5)2 ( x  5)2
(a) (b)
x2  7 ( x  7)2
2
 16
(c) (d) none
34 x ( x  5)2
(c) (d) none
( x  7)2
2
dy
9. If y = log (ex . xn.3x) then is
dx
3 dy 1 n
4. If y = x + then is (a) 1 + nlogex + (b) 1+ + loge3
2x  1 dx 3x x
n–1
5x  1 (c) 1 + nx + loge3 (d) none
6
(a) (b) 1–
(2 x  1)2 (2 x  1)2 dy
10. If y = log (xex) then is
dx
4 x2  2 x  5
(c) (d) none 1
(2 x  1)2 (a) (b) 1 + ex
xe x
11 dy
5. If y = then is 1
( x  1) ( x  2) dx (c) 1 + (d) none
x
 11  11 (2 x  3) 7 dy
(a)
( x  1)2 ( x  2)2
(b)
( x 2  3x  2)2 11. If y = ( x 3  11x 2  9) 2 then is
dx
(c) 11 (d) none 7x 3 5
(a) ( x  11x 2  9) 2 (3x + 22)
7 2
6. If f (x) = 1 + 2x + 3x2 + the f '(x) is
2x 7 5
(b) 3 2 2 (3x2 + 22x + 9)
2 ( x  11x  9)

Basic Concepts of Differential and Integral Calculus 195


7 1 x
(c) (3x 3  22 x ) 2 (c) (d)
2y  1 2y  1
(d) none
2 dy
12. If y = e5x  9 x  1 then is
dx 18. If y = x 2  x 2  x 2  ..................  then
2
5x  9 x  1
(a) e dy
is
(b) 10x – 9 dx
2
(c) e5x  9 x  1 .(10x – 9) x 2x
(a) (b)
2 1  2y 2y  1
(d) e5 x  9 x  1 .(10x – 9) . log(5x2 – 9x + 1)
dy x2
13.
2
If y = 32 x  7 x  1 then is (c) (d) none
dx 2y  1
2
(a) 32 x  7 x  1
2 19. If y = 3 x 2  3 x 2  3 x 2  ...................... 
(b) 32 x  7 x  1 .loge3.(4x + 7)
2
(c) 32 x  7 x  1 .(4x + 7) (d) none dy
then is
14. 2
If f (x) = log (11x + 7x – 5) then f '(x) is dx
2x 2x
log (11x 2  7 x  5) (a) (b)
(a) 1  3 y2 1  3y
22 x  7
2x
11x 2  7 x  5 (c) (d) none
(b) 3 y2  1
22 x  7  5
22 x  7
(c) x ...................... dy
11x 2  7 x  5 20. If y = x x then is
dx
(d) none
15. Derivative of x2 log x of with respect to e3x + 4 is
y y2
(a) (b)
x (1  y log x ) x (1  log x )
(a) 3e3x + 4 (b) x + 2x.log x
x  2 x log x 3 e3x  4 y2
(c) (d) (c) (d) none
3 e3x  4 x  2 x log x x (1  y log x )

y 1 dy
16. If x = then is dy
y 1 dx 21. If y = log xa  x  b then is
dx
2 2
(a) (b) 1 1
( x  1)2 ( x  1)2 (a) (b)
x  a  x b ( x  b) ( x  b)
1 1
(c) (d) 1
( x  1)2 ( x  1)2 (c) 2 ( x  a) ( x  b) (d) none
dy 22. If f (x) = e5x + e –5x then f ''(x) is
17. If y = x  x  x  ..................  then is
dx (a) 5 f (x) (b) f (x)
1 x (c) 25 f (x) (d) none
(a) (b)
1  2y 1  2y
196 BUSINESS MATHEMATICS, LOGICAL REASONING & STATISTICS (Paper 3) [CA Foundation]
23. If f (x) = 3e–3x + 2e3x then f ''(x) is dy
(a) 6 f (x) (b) 3 f (x) 29. If x2ey + 4 log x = 0 then is
dx
(c) 9 f (x) (d) none
3 2
24. If f (x) = 2x + 7x – 5x + 12 and f ''(x) = 38 then
(a) (b)
value of x is
(a) 2 (b) 3
(c) 1 (d) none
3 2
25. If f (x) = 3x – 5x + 2x – 8 then f ''(1) is (c) (d) None
(a) 18 (b) 8
(c) 10 (d) none
dy
dy 30. Find of
26. If x = y log (xy), then is equal to: dx
dx
(a) –b2x / a2y (b) –b2y / a2x
x+y (c) –b2 / a2 (d) 0
(a)
x (1 +logxy)
dy
31. If y = x 5 + e2x + log3x then is
x-y dx
(b)
x (1 +logxy)
(a)
x+y
(c)
x (logx +logy)
(b)
x-y
(d)
x (logx +logy) (c)
27. Differentiate e(xx ) : (d) None
(a) (1 + log x) (b) xx(1 + log x)
xx xx 32. if y =
(c) e (1 + log x)xx (d) e (1 + log x)

2
d y
28. lf Y = XX then 2 = _______
dx
(a)
dy d
(a) (1 + log x) + y (1 + log x)
dx dx
dy d
(b) (1 + log x) + (1 + log x) (b)
dx dx
dy d
(c) (1 + log x) - y (1 + log x)
dx dx
(c)
dy d
(d) (1 + log x) - (1 + log x)
dx dx

(d) None

Basic Concepts of Differential and Integral Calculus 197


dy
33. if y3 . x5 = (x+y)8, then is
dx

(a) (b)

ANSWER SHEET
(c) (d) None
1. (c) 2. (b) 3. (c)
dy
34. if y = x log (log x); then is
dx 4. (b) 5. (b) 6. (c)

7. (b) 8. (a) 9. (b)


(a)
10. (c) 11. (a) 12. (c)

13. (b) 14. (c) 15. (c)

(b) 16. (b) 17. (c) 18. (b)

19. (c) 20. (c) 21. (c)

22. (c) 23. (c) 24. (a)


(c)
25. (b) 26. (b) 27. (c)
(d) None
28. (a) 29. (c) 30. (a)

31. (a) 32. (b) 33. (a)


35. If
34. (a) 35. (c)

(a) (b)

(c) (d) None

198 BUSINESS MATHEMATICS, LOGICAL REASONING & STATISTICS (Paper 3) [CA Foundation]
dy
8. If y = ax3 + bx2 + cx + d then is equal to
HOME WORK - 2 dx
(a) 3ax2 + 2bx + c

ax2 bx 3 cx2
(b)   + dx
dy 4 4 2
1. If y = 5x2 then is
dx (c) 0 (d) None
(a) 10x (b) 5x 9. The gradient of the curve
(c) 2x (d) None y = 2x3 - 5x2 - 3x at x = 0 is
dy (a) 3 (b) -3
2. If y = x3 then is
dx (c) 1/3 (d) 1
x4 x4 10. The derivative of e0 is -
(a) (b) -
4 4 (a) 0 (b) 1
(c) e (d) 
(c) 3x 2 (d) - 3x2
3. If f (x) = xk and f’ (1) = 10 the value of k is d 2logx
11. e is equal to
dx
(a) 10 (b) - 10
(c) 1/10 (d) none of these (a) 2 (b) 2x
(c) x2 (d) 0
d
4. (x -1)(x -2) is equal to
dx dy
12. If y = x10 + 5 log 3x + 6e2x + 10 then is
(a) 2x - 3 (b) 3x -2 dx
(c) 1 (d) None of these (a) 10x9 + 15x + 12e2x
8 10 dy (b) 10x9 + 5
x + 12e
2x
5. y = 9x4-7x3+ 8x2- + 3 then is
x x dx
(a) 36x3 - 21x2 + 16x + 8x-2 - 30x-4 (c) 10x9 + 5
x + 6e
2x
(d) None
(b) 36x3 - 21x2 + 16x + 8x-2 + 30x-4
d
(c) 36x3 + 21x2 + 16x + 8x-2 + 30x-4 13. (x log x) is equal to
dx
(d) None
1
2 3
dy (a) (1 + log x) (b)
6. If y = (3x + 1) (x + 2x) then is log x
dx
(a) 15x4 + 21x2 + 2 (b) 15x3 + 21x2 + 2 x
(c) log x (d)
(c) 3
15x + 21x + 2 (d) None log x

dy 14. The derivative of x2 log X is


2 2
7. If x - y + 3x - 5y = 0 the is
dx (a) 1 + 2 log x (b) x (1 + 2 log x)
(a) (2x + 3) (2y + 5)-1 (c) 2 log x (d) None
(b) (2x + 3) (2y - 5)-1 15. Differentiate ex logx with respect to x.
(c) (2x - 3) (2y + 5)-1 (d) None ex ex
(a) (b)  log x
x x

ex
(c)
x
 (1 +log x) (d) elogx

Basic Concepts of Differential and Integral Calculus 199


dy dy
16. If x3 - xy2 + y2 + 2 = 0 then is 22. If y = (3x3 - 5x2 + 8)3 then is
dx dx
(a) 3 (3x3 - 5x2 + 8)2 (9x2 - 10x)
(y2 - 3x2 ) (y2 - 3x2 )
(a) (b) (b) 3 (3x3 - 5x2 + 8)2 (9x2 + 10x)
2y(1 - x) 2y(x -1)
(c) 3 (3x3 - 5x2 + 8)2 (10x2 - 9x)
2 2
(y2-3x2) (y2-3x2) (y - 3x )
(c) (d) dy
2y(1+x) 2y(1-x) (1 + x) 23. y = et and x = log t, then =
dx
dy
17. Find for x2y2 + 3xy + y = 0 1
dx (a) (b) t.e t
t
(2xy + y) (2xy2 + 3y) 1
(a) (b) - 2 (c) - (d) None
(x + 2x) (2x y + 3x +1) t2

(2x2y - 3y) 24. The gradient of curve y = -2x3 +3x + 5 at x = 2 is-


x2y2 - 2y
(c) (d) - (a) - 20 (b) 27
2xy (x2y + 3x)
(c) - 16 (d) - 21
3 d2p 25. The gradient of curve y = x3 - x2 at (0, 0)
18. Let p = x log x, so what is the value of 2
dx (a) 1 (b) 0
(a) 5x + 6x log x (b) 5x2 + log x2 (c) -1 (d) None
(c) 5x2 + 6x log x (d) None
dy
19. Find value of if y = xx
dx
ANSWER KEYS
(a) xx ( 1 + log x) (b) 1 + log x
(c) y. log x (d) None
1 A 11 B 21 A
1/2 dy 2 C 12 B 22 A
20. If y = 3x 2 - 7
  then is
dx
3 A 13 A 23 B
(a) 3x (3x2 - 7)-1/2 (b) 6x(3x2 - 7)-1/2
4 A 14 B 24 D
(c) 3x (3x2 + 7)-1/2 (d) 5 A 15 C 25 B
dy 6 A 16 A
21. If y = (6x5 - 7x3 + 9)-1/3 then is
dx 7 A 17 B
 1 5 8 A 18 A
(a)  -  (6x - 7x3 + 9)-4/33 (30x4 - 21x2)
 3 9 B 19 A
10 A 20 A
1 5
(b)   (6x - 7x3 + 9)-4/33 (30x4 - 21x2)
3

 1 5
(c)  -  (6x - 7x3 + 9)4/33 (30x4 - 21x2)
 3
(d) None

200 BUSINESS MATHEMATICS, LOGICAL REASONING & STATISTICS (Paper 3) [CA Foundation]
5.  1 - 3x 1 + x  dx is equal to
UNIT - 2 INTEGRATION
(a) x – x2 – x3 (b) x3 – x2 + x

CLASS WORK (c) x – x2 – x3 + k (d) none

6.   x - 1 / x  dx is equal to

2 3/2
-2 x1/2 + k
2
1. Evaluate 5x dx:
 a) ×
3
3
5x3
a) 5 / 3x + k b) +k
3 2
b) x -2 x +k
c) 5x3 d) none of these 3
2. Integration of 3 – 2x – x4 will become
1 1
2 5 c) + +k
a) –x –x /5 2 x 2x x
x5
b) 3x - x2 - +k d) none of these
5
7. The integral of px3 + qx2 + rk + w/x is equal to
x5 a) px2 + qx + r + k
c) 3x - x2 + +k
5 b) px3/3 + qx2/2 + rx
d) none of these c) 3px + 2q – w/x2
d) none of these
3. Given f(x) = 4x3 + 3x2 – 2x + 5 and  f  x  dx is: 8. Use method of substitution to integrate the function
a) 4 3
x + x – x + 5x 2 f(x) = ( 4x + 5 )6 and the answer is
1
(b) x4 + x3 – x2 + 5x + k (a) ( 4x + 5 )7 + k
28
c) 12x2 + 6x – 2x2
1
(b) ( 4x + 5 )7 + k
(d) none of these 7

2 2 (c) ( 4x + 5 )7 + k
4. Evaluate  x -1  dx
(d) none of these
5 3
a) x /5 – 2/3 x + x + k 9. Use method of substitution to evaluate
5
2
(b)
x3
- x +k
 
x x +4  dx and the answer is
3
(a) (x2 + 4 )6 + k
c) 2x (d) none 1
(b) ( x2 + 4 )6 + k
12
(c) ( x2 + 4 )6/ + k
(d) none of these

Basic Concepts of Differential and Integral Calculus 201


10. Integrate ( x + a )n and the result will be
14.  logx dx is equal to
n+1
a)
 x + a +k a) x logx + k (b) x logx – x2 + k
n +1
(c) x logx + k (d) none of
n+1
b)
 x + a these
n+1 x
15.  xe dx is
n+1
c)  x + a (a) (x – 1)ex + k (b) (x – 1) ex
(c) x ex + k (d) none of these
d) none of these 2
3
16.  log x  dx and the result is
11.  8x2 / x 3 + 2 dx is equal to
  (a) x (logx)2 – 2x logx + 2x + k
a) – 4/3(x3 + 2)2 + k
(b) x (logx )2 – 2x + k
4
-
2
+k (c) 2x logx – 2x + k
b) 3 x3 + 2
  (d) none of these]
17. Using method of partial fraction to evaluate
4
+k 2
2
c) 3 x3 + 2
   x + 5  dx /  x + 1  x + 2  we get

(d) none of these (a) 4 log (x + 1) – 4 log (x + 2) + 3/x + 2


12. Using method of partial fraction the integration +k
1 (b) 4 log (x + 2) – 3/x + 2 + k
of f(x) when f(x) = 2 2 and the answer is
x -a (c) 4 log (x + 1) – 4 log (x + 2)
a
a) log x - +k (d) none of these
x+a
b) log (x – a) – log (x + a) + k 1
2 3
c)
1
2a
 x -a 
log  +k 18. Evaluate   2x -x  dx and the value is
 x+a 0
d) none of these
(a) 4/3 + k(b) 5/12
2 3x
13. Use integration by parts to evaluate x e dx  (c) – 4/3 (d) none of these
(a) x2 e3x/3 – 2x e3x/9 + 2/27 e3x + k 4
2
(b) 2
x e3x – 2x e + 2e + k 3x 3x 19. Evaluate   3x - 2 dx and the value is
2
(c) e3x/3 – x e3x/9 + 2e3x + k (a) 104 (b) 100
(d) none of these (c) 10 (d) none of these

202 BUSINESS MATHEMATICS, LOGICAL REASONING & STATISTICS (Paper 3) [CA Foundation]
1 (c) ex + k
x
20. Evaluate xe dx dx and the value is
 (d) none of these
0
a
(a) –1 (b) 10
25.  f  x  + f  -x  dx is equal to.
(c) 10/9 (d) +1 0 a

21.
x
 x 1 + logx  dx is equal to
(a)  2 f  x  dx
0
(a) xx logx + k (b) ex2 + k a

c)
x 2
+ k d) xx + c
(b)  f  x  dx
-a
2 (c) 0
22. If f(x) = 1+x
2 then  f  x  dx is
a
2 2 3/2 (d)   f  -x  dx
(a)
3
× 1+x  +k -a
xex
x 1 
(b) 1 + x2 + log  x + x2 +1  +k 26.
 x + 1
2 dx is equal to
2 2  

2 3/2
4
2 ex
(c)
3
× 1+x  +k (a)  (2x  5)dx
1
(b)
x
+k

(d) none of these (c) ex + k


2 2
23.  dx 
+ 1 / x + 2 is equal to (d) none of these

(a)
x
2
 x2 + 2 + k 27.  x
4 +3/x
 dx is equal to
(a) x5/5 + 3 log x
(b) 2 x2 + 2 + k (b) 1/5 x5 + 3 log x + k
(c) 1/5 x5 + k
3/2
(c) 1 / x2 + 2   +k (d) none of these
3
(d) none of these
28. Evaluate the integral  1 - x  / x dx and the
answer is equal to
2 x -x
24.  ex + e-x 
e - e dx is  a) log x – 3x + 3/2x2 + k
(b) logx – 2 + 3x2 + k
3x + 5
-e
(a) + c (c) logx + 3x2 + k
3
(d) none of these
1 2
(b)
2
 ex + e-x  +k

Basic Concepts of Differential and Integral Calculus 203


1
29. The equation of the curve in the form y = f(x) if a) 51 (b) 48/5
the curve passes through the point (1, 0) and f’(x) 5 7
= 2x – 1 is c) 48 d) 55
15
(a) y = x2 – x (b) x = y2 – y 36. The equation of the curve which passes through
the point (1, 3) and has the slope 4x – 3 atany
(c) y = x2 (d) none point (x, y) is
(a) y = 2x3 – 3x + 4
4

30. Evaluate  (2 x  5)dx (b) y = 2x2 – 3x + 4


1

and the value is (c) x = 2y2 – 3y + 4

(a) 3 (b) 10 (d) none of these

(c) 30 (d) none of these 3 3

2
37. The value of  f  5 - x  dx - f  x  dx is
2x 2 2
31.  2 dx is equal to (a) 1 (b) 0
11+x

(a) loge (5/2) (b) loge5 – loge2 (c) –1 (d) none

(c) loge (2/5) (d) none of these x / x2


38.   x - 1 e dx equal to
4
(a) ex/x + k
32.  3x + 4 dx is.
0 (b) e–x/x + k
(a) 9/112 (b) 112/9 (c) – ex/x + k
(c) 11/9 (d) none of these (d) none of these
2 x
x+2 e  xlog x + 1 
33.  x +1 dx is 39.  x
dx equal to
0
a) 2 + loge2 (b) 2 + loge3 (a) ex logx + k (b) ex + k
(c) loge3 (d) none of these (c) logx + k (d) none of these

e2 2
dx 40.  log x dx is equal to
34. Evaluate  2 and the value is
1 x  1 + log x  (a) x (log x – 1) + k

(a) 2/3 (b) 1/3 (b) 2x (log x – 1) + k


(c) 26/3 (d) 1/2 (loge5) (c) 2 (log x – 1) + k
4 x +1 x + 4 (d) none of these
  
35.  x
dx equal to
0

204 BUSINESS MATHEMATICS, LOGICAL REASONING & STATISTICS (Paper 3) [CA Foundation]
2 (d) none of these
41.  xlog xdx is equal to 2
1 47.  logx  xdx is equal to
(a) 2 log 2 (b) – 3/4
x2  2 1
(c) 2 log 2 – ¾ (d) none (a) logx  - logx +  + k
2  2
2  2  x+ 1
x -1 x dx. 2 1
42. Evaluate  2  e The value is (b) logx  -log x + +k
1 x  2
x2  2 1
(a) e 2
 e -1  (c)  log x  +  + k
2  2
2
(b) e  e -1 +k
 
(d) none of these
c) e2 e
(d) none of these  ex - e-x 
48. Evaluate   x -x  dx and the value is
 e +e 
2
2 ex + e-x
43.  3x dx is (a) loge x -x
1 e +e

a) 7 (b) –8
(b) loge ex + e-x + k
(c) 8 (d) none of these

 2 - x  ex (c) loge ex - e-x + k


44. Evaluate  2 dx and the value is
1 - x  (d) none of these

ex 3x
(a) +K (b) ex + k 49. By the method of partial fraction  2
1- x x 
- x - 2 dx is

1 (a) 2loge x - 2 + loge x + 1 + k


(c) +K (d) none of these
1- x (b) 2loge x - 2 -loge x + 1 + k
3 (c) 2loge x - 2 + loge x + 1 + k
45. Using integration by parts x log xdx 
(a) x4/16 + k (d) none of these
50. If f’(x) = x – 1, the equation of a curve y = f(x)
(b) x4/16 ( 4 log x – 1 ) + k
passing through the point (1, 0) is given by
(c) 4 log x – 1 + k
(a) y = x2 – 2x + 1
(d) none of these
(b) y = x2/2 – x + 1
46.  log logx  / xdx is (c) y = x2/2 – x + 1/2 (d) none
(a) log (log x – 1) + k
(b) log x – 1 + k

(c) [ log (log x – 1) ] log x + k

Basic Concepts of Differential and Integral Calculus 205


1
6. Value of  5  4x dx is
HOME WORK -1
1
(a) log (5 – 4x) +k
4
(b) log (5 – 4x) +k
1 2
1. Integration of x – x+ is 1
2 x
(c) log (5 – 4x) +k
1
5
2 3 1
(a) x 2 – x2 + 4 x 2 + c 1
3 4 (d) – log (5 – 4x) +k
1
4
3 3
(b) x 2 – x2 + x 2 + c 1
2 7. Integration of (x2 – 3x + 3 x + 7) is
1
x
2 3
(c) x 2 – x2 + 2 x 2 + c
3 3 2
x  x  3x  3 x 4 3  7x 
(d) none (a)   +k
2 3 2 4 
2. Value of  ( 3 x + e2x) dx is x3 1
(b) – 2x2 + 3 x 3 + 7x +k
4 4 3 4 3 e2 x
3
(a) x 3 + 2e2x + c (b) x + +c 5
3 4 2 5
2x 2 3 6x 6
(c) – 2x 2 + + 14x1/2 +k
1 2 3 5 5
(c) x + 2e2x + c (d) none
3 (d) none
3. Value of  3 x 5 2 
x dx is 8. Value of
x2
 x  2 dx is
9 3
(a) x 2 + 5x + 2logx + c x 2 x3
2 (a) + +c
2 6
3 2
(b) 2x 2 + 5x – 2 + c
x x2
(b) – 2x + 4log (x + 2) + c
3 2
(c) 2 x 2 + 5x + 2logx + c (c) 2x log (x + 2) + c
(d) none (d) none
4. Value of  (1 – x)3 dx is
9. Value of
x3
 x  1 dx is
1 1
(a) – (1 – x)4 +k (b) (1 – x)4 +k 1 3 1 2
4 4 (a) x + x + x + log (x – 1) + c
(c) 3 (1 – x)2 +k (d) none 3 2
x4
5. Value of  (5 + 2x)2 dx is (b)
4
+C

1 1 (c) 3x2.log (x – 1)
(a) (5 + 2x)3 +k (b) (5 + 2x)3 +k
3 6 (d) none
5 3x  4
(c) (5 + 2x)3 +k (d) none 10. Value of  6x  7 dx is
3

206 BUSINESS MATHEMATICS, LOGICAL REASONING & STATISTICS (Paper 3) [CA Foundation]
1 3 2x
(a)
6
log (6x + 7) + c (b)
2
 e  1 3
2
+c

1 (c)  e2x  1 + c
1
2
(b) x + log (6x + 7) + c
12
(d) none
x 1
(c) + log (6x + 7) + c 1
2 12 2
15. Value of  3x  5) dx is
2
(d) none  (2x  3)(x
3 2 
8 2 2 3
11. Value of  (3x  5)  x  5x  7  dx is (a) ( x  3x  5) 2 + c
 2  3
9
1 1
1 3 2  (b) ( x 2
 3 x  5) 2 +c
(a) x  5x  7  + c 2
9  2 
2 2 3
9 (c) ( x  3x  5) 2 + c
3 2  3
(b)  2 x  5x  7  + c (d) none
 
2
3 2  16. Integration of x. ex with respect to x is
(c)  2 x  5x  + c
  2 2
(a) ex + c (b) x. ex + c
(d) none
1 x2
12. 2 3

1/ 2
 (c) e +c (d) none
x x 5 dx  2
2 3 3 4

(a) ( x  5) 2 + c 17. Value of  4x3 e x dx is


3
4 1 x4
3 3 3
( x  5) 2 + c (a) 12x2 ex + c (b) e +c
(b) 4
2
4
2 3 3 (c) ex + c (d) none
(c) ( x  5) 2 + c
9
3x 2  2x  1
(d) none 18. Value of  x3  x 2  x  5
dx is
x2
13. Value of  (x 3
 2)
1
4
dx is (a) x3  x 2  x  5 + c

4 3 3 1
(a) ( x  2) 4 + c (b) +c
3 x  x2  x  5
3

4 3 3
(b) ( x  2) 4 + c (c) 2 x 3  x 2  x  5 + c
9
3 3 (d) none
(c) 4 ( x  2) 4 + c
(d) none 5x 4  3x 2
19. Value of  x5  x3  1 dx is
e2 x
14. Value of  dx is (a) log (x5 + x3 + 1) + c
e2x  1 (b) log (5x4 + 3x2) + c
2 2x (c) log (x5 + x3) + c
(a)
3
 e  1 + c 3
2

(d) none

Basic Concepts of Differential and Integral Calculus 207


1
ex dx
20.  ex  5 dx = 26. Value of  (1  x)(2  x) is
0

(a) log ex – log 5 + c (a) log 4 3 (b) log 2 3


(b) log ex – 5 + c
(c) log (ex – 5) + c (c) log 2 3 (d) none
(d) none
27. dx is equal to
1
21. Value of   x  1 x  2  dx is
(a) 9/112 (b) 112/9
 x 2 (c) 11/9 (d) None
(a) log   +c
 x 1

 x 1 28. dx is equal to


(b) log   +c
 x 2
e3x+5 e3x
1  x 2 (a) +c (b) +c
(c) log   +c 3 5
2  x 1
(d) none -e3x+5
(c) +c (d) None
1
1
3
22. Value of  (x  2)(x  3) dx is
0 29. The value of x x dx is

(a) log 9 (b) log 9 8  


(a) (b)
 
(c) log 8 9 (d) none

5
(c) (d) None
x
23. Value of 
2 x  7x
dx is
ex
30.  e x + 1 dx is equal to
(a) 5 2 (b) 7
2 (a) log (1+ex) + k (b) (ex+1)–2 + k
(c) 3 2 (d) none
8
8x
(c) +k (d) None
24. Value of 
0 8x  x
dx is

(a) 8 (b) 4 31. Evaluate


(c) 2 (d) none
1
x 2  4x  3
25. Value of  x 1
dx is
0 (a)
(a) 7 2 (b) 1
2
(c) 7 (d) none

(b)

208 BUSINESS MATHEMATICS, LOGICAL REASONING & STATISTICS (Paper 3) [CA Foundation]
35. Evaluate
(c)
(a) log10 (x log x - x) + c
(d) None (b) log 10 e (x log x - x) + c
(c) log10 e(x log x - 10) + c
32. Evaluate (d) None

36. Evaluate
(a) (a) x log (x+1) – x + log (x+1) + c
(b) x2 log (x +1) - x2 + log (x2 +1) + c
(c) x log (x +1) - x2 + log (x +1) + c
(b) (d) None

x2 - x+2
(c)
37. Evaluate  x3 +x2 - 2x
(a) –log x + 3/2 log (x–1) + 4/3 log (x2 + 2) + log c
(d) None
(b) –log x + 2/3 log (x2–1) + 3/4 log (x2 +2) + log c
33. Evaluate (c) –log x + 2/3 log (x–1) + 4/3 log (x + 2) + log c

(d) None
(a)
38. Evaluate
-1
(b)  log x + 1 + c
x
(a)

(c)

(d) None
(b)

34. Evaluate

(c)

(d) None
(a)

(b) log (log x ) + c


(c) log (log x2 ) + c
(d) None

Basic Concepts of Differential and Integral Calculus 209


39. Evaluate: ANSWER KEYS

1. (a) 2. (b) 3. (c)


(a)
4. (a) 5. (b) 6. (d)

7. (c) 8. (b) 9. (a)

(b) 10. (c) 11. (a) 12. (c)

13. (b) 14. (c) 15. (a)

16. (c) 17. (c) 18. (c)


(c)
19. (a) 20. (c) 21. (a)

(d) None 22. (b) 23. (c) 24. (b)

25. (a) 26. (a) 27. (b)

40. Evaluate: 28. (a) 29. (b) 30. (a)

31. (a) 32. (c) 33. (b)


(a) 1 (b) 0
(c) –1 (d) None 34. (b) 35. (a) 36. (a)

37. (c) 38. (d) 39. (a)

40. (a) 41. (a) 42. (b)


41. Evaluate

(a) 1 (b) 0
(c) 2 (d) None

42. The value of is equal to

(a) log

(b) log +k

(c) log

(d) None

210 BUSINESS MATHEMATICS, LOGICAL REASONING & STATISTICS (Paper 3) [CA Foundation]
1
HOME WORK -2 5.  1+ x
is equal to

2
(a) +K (b) (1+ x)1/2 +K
12 (1 + x)1/2
1. Evaluate  (x + ) dx
x (c) 2(1 + x)1/2 + K (d) None of these
x3 3x 1
(a) + 2x + c (b) - +c ax
2 2 x 6. e dx

x3 1 x2 2 eax
(c) + 2x - + c (d) - +c (a) ex (b)
3 x 3 x a
1
x (x 3 + 2x - 3) dx. (c) log x (d)
2. Evaluate  e-ax
5 2x
7 7. The value of  (6x + 3e + 5)dx is equal to
(a) x2 3x 2
  8x + c
5 7 3 2x
(a) x6 + e + 5x + k
2
9 5
3
2x 2 4x 2 (b) 30x4 + 6e2x
(b)   2x 2 + c
9 5 3 2x
(c) e x6 +
7 3 2
3
(c) 3x 2 x 2 (d) None of these
  2x 2 + c
9 5 x
8. Find  3 dx.
5 3
5
(d) 2x 2 x 2 ex
  2x 2 + c (a) loge 3 + C (b) log3 + C
7 9 3
2
3.  (7x - 3x + 8 - x -1/2 + x -1 + x -2 ) dx 3x
(c) C (d) 3x + C
log e 3
3 1 2
(a) x - x + 4 x + C
2 4
3 3 2 2 1 x 2
(b) x - x + 8x - x1/2 +log x + x -1 9. Integrate x- +
7 3 2 2 x
7 3 3 2 2 1
(c) x  x + 8x - 2x1/2 + log x + x -1 (a) x x - x2  4 x + c
3 2 3 4
(d) None 3 1
3 2 (b) x - x2 + 4 x + c
4. Integrate 4x + 3x - 2x + 5 2 4
(a) x4 + x3 - x2 + 5x + k 2 1 1
3 (c) x x - x2 - x +c
4 2 3 2 2
(b) x - x + x - 5x + k
(c) x4 + x3 + x2 - 5 + k (d) None (d) None

Basic Concepts of Differential and Integral Calculus 211


3
10. Integrate + 4x2 - 3x + 8 15. Evaluate  x logx dx.
x
x
4 3 (a) xlog x + c (b) x log x - +c
(a) 3logx - x3 + x2 - 8x + c 3
3 2
x2 x2 1
4 3 (c) logx - +c (d) +c
(b) 3logx + x3 - x2 - 8x + c 2 4 xlogx
3 2 16. The equation of the curve which passes
4 3 3 2 through the point (1, 3) and has the slope 4x-3
(c) 3logx + x + x + 8x + c at any point (x y)
3 2
(d) None (a) y = 2x3 - 3x + 4
(b) y = 2x2 - 3x + 4
 b c  2
11. Integrate  ax + 3 + 7  x (c) x = 2y2 - 3y + 4
 x x 
(d) None of these
1 4 1 1
(a) ax + blogx - CX -4 + k x
4 4 17. The value of 
0 x  1 x
dx is
(b) 4ax4 + blogx - 4CX-4 + k (a) 1/2 (b) 1
1 4 1 (c) 2 (d) 0
(c) ax + blogx + CX -4 + k
4 4 3
3
(d) None 18. Evaluate x
-3

+x dx dx

5
x2 (a) 0 (b) 3
12. 0 x2 + (5 - x)2 dx is equal to (c) -3 (d) 1
3
(a) 5 (b) 5/2 3

(c) 1 (d) None


19. Evaluate the value of  3x
0
+ 5x + 2 dx
1 (a) 55 (b) 57
13. Integrate 2 2 is (c) 55.5 (d) 56
x -a
x -a
(a) log +k ANSWER KEYS
x+a
(b) log (x-a) - log (x+a)
x -a 1 (c) 2 (b) 3 (a) 4 (a)
1
(c) log +k
2a x+a 5 (c) 6 (b) 7 (a) 8 (c)

1 x -a 9 (a) 10 (b) 11 (a) 12 (b)


(d) log +k
2 x +a 13 (c) 14 (b) 15 (c) 16 (b)
17 (a) 18 (a) 19 (c)
14. Integrate x3logx
(a) x4 / 16 + k
(b) x4 / 16 (4logx -1) + k
(c) 4log x - 1 + k
(d) None of these

212 BUSINESS MATHEMATICS, LOGICAL REASONING & STATISTICS (Paper 3) [CA Foundation]
CHAPTER-9
NUMBER SERIES, CODING DECODING AND ODD MAN OUT SERIES

INTRODUCTION

SERIES
Series Classified into Two Types, Namely
A. Number Series
B. Alphabet Series

A. NUMBER SERIES
Case 1: Missing terms of the series
In this type the questions we have to identify the missing term of the series real according to a specific
pattern of the series rule to form its code. The students are required to detect the missing number of
the series and answer the questions accordingly.
B. ALPHABET SERIES
Alphabet series consists of letters of the alphabet placed in a specific pattern. For example, the series
are in the following order of the numbers.

C. LETTER SERIES:
This type of question usually consist of a series of small letters which follow a certain pattern.
However some letters are missing from the series. These missing letters are then given in a proper
sequence as one of the alternatives.

NUMBER SERIES, CODING AND DECODING AND ODD MAN OUT 213
CODING AND DECODING
Before transmitting, the data is encoded and at receiver side encode data is decoded in order to obtain
original data by determining common key in encoded data.
The Coding and Decoding is classified into seven types according to the on what way it is doing.
They are type
Type 1: Letter Coding
Type 2: Number Coding

Type 1: Letter Coding


In this type the real alphabets in a word are replaced by certain other alphabets according to a specific rule
to form its code. The candidate is required to detect the common rule and answer the questions accordingly

Case1: To form the code for another word


Case 2: To find the word by analysing the given code (DECODING)

Type 2: Number Coding


In these questions, either numerical code values are assigned to a word or alphabetical code letters are
assigned to the numbers. The candidate is required to analyse the code as per the directions.

Case 1: When a numerical code values are assigned to words.


Case 2: Number to letter coding.

ODD MAN OUT


Classification means ‘to assort the items’ of a given group on the basis of a certain common quality they
possess and then spot the stranger or ‘odd one out’.

These questions are based on words, letters and numerals. In these types of problems, we consider the
defining quality of particular things. In these questions, four or five elements are given, out of which one
does not belong to the group. You are required to find the ’odd one’.

214 BUSINESS MATHEMATICS, LOGICAL REASONING & STATISTICS (Paper 3) [CA Foundation]
12. 28, 33, 31, 36, 34 ?
CLASS WORK (a) 38 (b) 39
(c) 40 (d) 42
13. 120, 80, 40, 45, ?, 5
1. 6, 11, 21, 36, 56 ? (a) 15 (b) 20
(a) 42 (b) 51 (c) 25 (d) None
(c) 81 (d) 91 14. 2, 15, 41, 80, 132 ?
2. 10 , 100, 200, 310 ? (a) 184 (b) 144
(a) 400 (b) 410 (c) 186 (d) 197
(c) 420 (d) 430 15. 6, 17, 39, ?, 116
3. 11, 13, 17, 19, 23, 25 ... ? (a) 72 (b) 75
(a) 33 (b) 29 (c) 85 (d) 80
(c) 40 (d) 48 16. 1, 4, 10, 22, ?, 94
4. 6, 12, 21, 33 ? (a) 46 (b) 48
(a) 33 (b) 27 (c) 49 (d) 47
(c) 29 (d) 48 17. 4, 9, 25, 49, ? , 169, 289, 361
5. 2, 5, 9, 14, ? , 27 (a) 120 (b) 121
(a) 20 (b) 36 (c) 122 (d) 164
18. 4, 12, 36, ? , 324
(c) 91 (d) 51
which one does not belong to the group. You
6. 6, 11, 21, ? , 56, 81
are required to find the ’odd one’.
(a) 42 (b) 36 (a) 107 (b) 109
(c) 18 (d) 24 (c) 108 (d) 110
7. 10, 18, 28, 40, 54, ?, 88 19. 1, 1, 4, 8 , 9, ? , 16, 64
(a) 70 (b) 36 (a) 27 (b) 28
(c) 45 (d) 40 (c) 32 (d) 40
8. 120, 99, ?, 63, 48, 35 20. 5760, 960, 192, ? 16, 8
(a) 80 (b) 86 (a) 47 (b) 48
(c) 87 (d) 98 (c) 52 (d) 50
9. 22, 24, 28, 36, ? , 84 21. 1, 2, 6, 7, 21, 22, 66, ? , 201
(a) 44 (b) 52 (a) 69 (b) 68
(c) 38 (d) 54 (c) 67 (d) 69
10. 4832, 5840, 6848, 7856 ? 22. 48, 24, 96 , ? 192
(a) 8864 (b) 8815 (a) 48 (b) 47
(c) 8846 (d) 8887 (c) 44 (d) 54
11. 10, 100, 200, 310, 430 ? 23. 165, 195, 255, 285, ?, 435
(a) 560 (b) 540 (a) 345 (b) 390
(c) 550 (d) 590 (c) 335 (d) 395

NUMBER SERIES, CODING AND DECODING AND ODD MAN OUT 215
24. 2, 3, 3, 5, 10, 13, 39, ?, 172, 177 36. If DELHI is coded 73541 and CALCUTTA as
(a) 42 (b) 44 82589662, How can CALICUT be coded?
(c) 43 (d) 40 (a) 5279431 (b) 5978213
25. 7, 26, 63, 124, 215, ?, 511 (c) 8251896 (d) 8543962
(a) 342 (b) 343 37. If CLOCK is coded 34235 and TIME is 8679, what
will be code of MOTEL?
(c) 441 (d) 421
(a) 72894 (b) 77684
26. 3, 7, 15, 31, ? 127
(c) 72964 (d) 27894
(a) 62 (b) 63
38. If PALE is coded as 2134 and EARTH is coded as
(c) 64 (d) 65
41590, how is PEARL is code?
27. 8, 28, 116, 584, ?
(a) 29530 (b) 24153
(a) 1752 (b) 3502
(c) 25430 (d) 254313
(c) 3504 (d) 3508
39. If LOSE is coded as 1357 and GAIN is coded as
28. 6, 13, 28, 59, ? 2468, what do figure 82146 stands for?
(a) 122 (b) 114 (a) NGLAI (b) NGLIA
(c) 113 (d) 112 (c) GNLIA (d) GNLIA
29. 2, 7, 27, 107, 427, ? 40. If MEKLF is coded as 91782 and LLLJK as 88867,
(a) 1707 (b) 4027 how can IHJED is coded as?
(c) 4207 (d) 1207 (a) 97854 (b) 64512
30. 5, 2, 7, 9, 16, 25, 41, ? (c) 54610 (d) 75632
(a) 65 (b) 66 41. If in a certain code language NAME is written
(c) 67 (c) 68 as 4258 then what is coded as MEAN ?
31. In a certain language, MADRAS is coded (a) 2458 (b) 5842
NBESBT, how DELHI is coded in that code? (c) 8524 (d) 5824
(a) EMMJI (b) EFMIJ 42. If GOLD is written as IQNF, how WIND can be
(c) EMFIJ (d) JIFEM written as code?
32. If RAMAN is written as 12325 and DINESH as (a) YKPF (b) VHCM
675489 how HAMAM is written? (c) XJOE (d) DNIW
(a) 92323 (b) 92233 43. If ROSE is written as TQUG, how BISCUIT can be
(c) 93233 (d) 93292 written in that code?
33. If RED is coded as 6720 then GREEN would be (a) DKUEWKV (b) CJTDVJU
coded as (c) DKVEWKV (d) DKUEWKY
(a) 9207716 (b) 167129 LETTER: C Z N V R S W F D
(c) 1677209 (d) 1672091 CODE DIGIT: 8 6 4 7 2 9 3 5 1
34. If A = 1, FAT = 27, FAITH = ? (Q. No. 44-46) In each of the following
questions find out the correctly coded
(a) 44 (b) 45
alternative from amongst the given four
(c) 46 (d) 36 alternatives (a), (b), (c), (d).
35. If BROTHER is coded 2456784, SISTER coded as 44. ZDRCVF
919684, what is coded for BORBERS?
(a) 612875 (b) 619875
(a) 2542849 (b) 2542898
(c) 612845 (d) 612835
(c) 2454889 (d) 2524889

216 BUSINESS MATHEMATICS, LOGICAL REASONING & STATISTICS (Paper 3) [CA Foundation]
45. WNCSZV 55. 1, 4, 9, 16, 24, 25, 36
(a) 348267 (b) 318267 (a) 9 (b) 24
(c) 348957 (d) 348967 (c) 25 (d) 36
46. RDNFVS 56. 41, 43, 47, 53, 61, 71, 73, 75
(a) 21679 (b) 216549 (a) 75 (b) 73
(c) 214579 (d) 218579 (c) 71 (d) 53
47. If DELHI is coded as CCIDD, how would you 57. 16, 25, 36, 73, 144, 196, 225
encode BOMBAY? (a) 36 (b) 73
(a) AJMTVT (b) AMJXVS (c) 196 (d) 225
(c) MJXVSU (d) WXYZAX 58. 1, 4, 9, 16, 19, 36, 49
48. In a certain code, RIPPLE is written as 613382 (a) 19 (b) 9
and LIFE is written as 8192. How is PILLER
(c) 49 (d) 16
written in that code?
59. 1, 5, 14, 30, 49, 55, 91
(a) 318826 (b) 318286
(a) 49 (b) 30
(c) 618826 (d) 338816
(c) 55 (d) 91
49. If PALAM could be given the code number 43,
what code number can be given to 60. 835, 734, 642, 751, 853, 981, 532
SANTACRUZ? (a) 751 (b) 853
(a) 123 (b) 85 (c) 981 (d) 532
(c) 120 (d) 125 61. 4, 5, 7, 10, 14, 18, 25, 32
50. Directions: The number in each question below (a) 7 (b) 14
is to be codified in the following code: (c) 18 (d) 33
Digit 7 2 1 5 3 9 8 6 4 62. 52, 51, 48, 43, 34, 27, 16
Letter W L M S I N D J B (a) 27 (b) 34
51. 184632 (c) 43 (d) 48
(a) MDJBSI (b) MDJBIL
(c) MDJBWL (d) MDBJIL
52. In a certain code ‘256’ means ‘you are good’,
‘637’ means ‘we are bad’ and ‘358’ means ‘good

and bad’. Which of the following represents
‘and’ in that code?
(a) 2 (b) 5
(c) 8 (d) 3
Directions: Find odd man out of the following
(51-60):
53. 3, 5, 7, 15, 17, 19
(a) 15 (b) 17
(c) 19 (d) 7
54. 10, 14, 16, 18, 23, 24, 26
(a) 26 (b) 23
(c) 24 (d) 18

NUMBER SERIES, CODING AND DECODING AND ODD MAN OUT 217
11 If PAINT is coded as 74128 and EXCEL is coded
as 93596, then how would you encode
HOME WORK - 1 ANCIENT ?
(a) 18339 (b) 19338
1. Find the missing term of the series 2, 7, 16, (c) 4251928 (d) None
______ , 46, 67, 92 12. In a certain code, 2 is coded as P, 3 as N, 9 as Q,
(a) 39 (b) 28 5 as R, 4 as A and 6 as B. How is 423599 coded in
(c) 27 (d) 29 that code?
2. Find the wrong terms of the series 9, 29, 65, (a) APNRQQ (b) APNQRR
126, 217, 344 (c) APQNRR (d) APQNAA
(a) 27 (b) 29 13. Odd man Out
(c) 30 (d) None January, May, July, November
3. Find the missing term of the series 1,9, 25, 49, (a) January (b) May
81, 121, .............
(c) July (d) November
(a) 167 (b) 168
14. 10, 14, 16, 18, 23, 24 and 26
(c) 169 (d) None
(a) 26 (b) 17
4 Find the next term of the series BKS, DJT, FIU,
HHV? (c) 23 (d) 9
(a) JGW (b) GJW 15. 6, 9, 15, 21, 24, 26, 30
(c) WQJ (d) None (a) 9 (b) 26
5. aab, ____, aaa, bba, ____ (c) 24 (d) 30
(a) baa (b) abb 16. 1, 5, 14, 30, 51, 55, 91
(c) bab (d) aab (a) 5 (b) 55
6. If in a certain language MYSTIFY is coded as (c) 51 (d) 91
NZTUJGZ, how is MENESIS coded in that 17. 16, 25, 36, 62, 144, 196, 225
language?
(a) 36 (b) 62
(a) NFOFTJT (b) NNFQTJT
(c) 196 (d) 144
(c) NQFTFJT (d) None
7. If TAP is coded as SZO, then how is FRIEND
coded?
(a) EGQMDC (b) EGQDMC ANSWER KEYS
(c) EGMCQD (d) EQGDMC
8 In a certain code, MENTION is written as (1) (d) (2) (b) (3) (c) (4) (a)
LNEITNO. How is PRESENT written in that code?
(a) QERESNT (b) QERESTN (5) (a) (6) (a) (7) (d) (8) (b)
(c) Both (d) None
9. If in a certain language CARROM is coded as (9) (a) (10) (c) (11) (c) (12) (a)
BZQQNL, which word will be coded as HORSE?
(a) IPSTF (b) IPSFT (13) (d) (14) (c) (15) (b) (16) (c)
(c) SFTIP (d) IPSIF
10. If in a certain language A is coded as 1, B is (17) (b)
coded as 2, and so on, how is AICCI is coded in
that code?
(a) 19933 (b) 19393
(c) 19339 (d) 19939

218 BUSINESS MATHEMATICS, LOGICAL REASONING & STATISTICS (Paper 3) [CA Foundation]
12. Find out the next number in the following
series 7,11,13,17,19,23,25,29,?
HOME WORK - 2
(a) 33 (b) 30
(c) 32 (d) 31
13. If HONEY is coded as JQPGA, which word is code
1. 6, 11, 21, 36, 56 ?
as VCTIGVU?
(a) 42 (b) 51
(a) CARPETS (b) TRAPETS
(c) 81 (d) 91
(c) UMBRELU (d) TARGETS
2. Find the oddman out
14. Find odd man out of the following series
(a) January (b) April
15,21,63,81,69
(c) July (d) October
3. Find the oddman out of the series 5,10, 17, 25 (a) 15 (b) 21
(a) 5 (b) 10 (c) 81 (d) 63
(c) 17 (d) 25 15. Find odd man out of the following series
4. In a certain language MADRAS is coded as 7, 9, 13, 17, 19.
NBEST, how BOMBAY is coded in that language? (a) 9 (b) 7
(a) CPNCBX (b) CPNCBZ (c) 13 (d) 19
(c) CPOCBZ (d) CQOCBZ
5. 10, 18, 28, 40, 54, ?, 88
ANSWER KEYS
(a) 70 (b) 86
(c) 87 (d) 98
6. 18, 24, 21, 27, ?, 30, 27 (1) (c) (2) (b) (3) (c) (4) (b)
(a) 33 (b) 30
(c) 24 (d) 21
(5) (b) (6) (d) 7 (d) (8) (d)
7. If F=6, MAT=34, then how much is CAR?
(a) 21 (b) 22
(9) (d) 10 (b) (11) (b) 12 (d)
(c) 25 (d) 28
8. If in a certain language NAME is written as 4258
13 (d) 14 (d) 15 (d)
then what is coded as MEAN?
(a) 2458 (b) 5842
(c) 8524 (d) 5824
9. 52, 51, 48, 43, 34, 27, 16
(a) 27 (b) 34
(c) 43 (d) 48
10. 1, 4, 9, 16, 24, 25, 36
(a) 9 (b) 24
(c) 25 (d) 36
11. If PLAY is coded as 8123 and RHYME is coded as
49367. What will be code of MALE?
(a) 6285 (b) 6217
(c) 6395 (d) 6198

NUMBER SERIES, CODING AND DECODING AND ODD MAN OUT 219
CHAPTER-10
DIRECTION TEST

INTRODUCTION
After reading this chapter, students will be able to understand:
 In this test, the questions consist of a sort of direction puzzle. A successive follow-up of direction is
formulated and the students is required to ascertain the final direction. The test is meant to judge then
ability to trace and follow correctly and sense the direction correctly.
 The adjoining figure shows the four main directions (North N, South S, East E, and West W) and four
cardinal directions (North East (NE), North West (NW), South East (SE), South West (SW) to help the
students know the directions.

` `

Always Remember:
Left + left Down
Left + right Up
Right + left Up
Right + right Down
Up + left Left
Up + right Right
Down + left Right
Down + right Left

220 BUSINESS MATHEMATICS, LOGICAL REASONING & STATISTICS (Paper 3) [CA Foundation]
6. A rat run 20 towards East and turns to right
runs 10 and turns to right runs 9 and again turns
to left runs 5 and then turns to left runs 12 and
CLASS WORK finally turns to left and rusn 6. Now what
direction is the rat facing?
(a) East (b) North
1. Mohan starts from point A and walks 1 km
(c) West (d) South
towards south, turns left and walks 1km.
7. A driver left his village and drove North for 20
Then he turns left again and walks 1 km. Now
km, after which he stopped for breakfast.
he is facing.
Then he turned left and drove another 30 km,
(a) East (b) West
when he stopped for lunch. After some rest,
(c) North (d) South-west he again turned left and drove 20 kms before
2. Suresh starts from a point, walks 2 miles stopping for evening tea. Once more he turned
left and drove 30 kms to reach the town where
towards south, turns right and walks 1 1 2 he had supper. After evening tea in which
miles,turns left and walks ½ miles and then he direction did he drive?
turns back. What is the direction he is facing (a) West (b) East
now?
(c) North (d) South
(a) East (b) West
8. A man is facing East, then he turns left and
(c) South (d) North goes 10 m, then turns right and goes 5 m then
3. A man starts from a point, walks 4 miles goes 5 m to the South and from there 5 m to
towards north and turns left and walks 6 miles, West. In which direction is he from his original
turns right and walks for 3 miles and again turns place?
right and walks 4 miles and takes rest for 30 (a) East (b) West
minutes. He gets up and walks straight 2 miles
in the same direction and turns right and walks (c) North (d) South
one mile. What is the direction he is facing? 9. From her home Prerna wishes to go to school.
(a) North (b) South From home she goes towards North and then
turns left and then turns right, and finally she
(c) South-east (d) West turns left and reaches school. In which
4. Arun started from point A and walked 10 km direction her school is situated with respect
East to point B, then turned to North and to her home?
walked 3 km to point C and then turned West (a) North-East (b) North-West
and walked 12 kms to point D, then again
turned South and walked 3 kms to point E. In (c) South-East (d) South-West
which direction is he from his starting point? 10. A child walks 25 feet towards North, turns right
(a) East (b) South and walks 40 feet, turns right again and walks
45 feet. He then turns left and walks 20 feet.
(c) West (d) North He turns left again walks 20 feet.
5. A starts from a point and walks 5 kms north, Finally, he turns to his left to walks another 20
then turns left and walks 3 kms. Then again feet. In which direction is the child from his
turns left and walks 5 km. Point out the starting point?
direction in which he is going now.
(a) North (b) South
(a) North (b) South
(c) West (d) East
(c) East (d) West

DIRECTION TESTS 221


11. Raju facing North and moves 20 km, then he (a) North (b) South
turned to his right and moves 20 km and then (c) West (4) East
he moves 10 km in North-East, then he turned
16. One evening, Raja started to walk toward the
to his right and moves 20 km and then he
Sun. After walking a while, he turned to his
turned to his right and moves 20 km and again
right and again to his right. After walking a
he turned to his left and moves 20 km. Now in
while, he again turned right. In which direction
which direction Rahu is facing?
is he facing?
(a) South-East (b) North-East (a) South (b) East
(c) South-West (d) North-West (c) West (d) North
12. K is a place which is located 2 km away in the 17. Five boys A, B, C, D, E, are sitting in a park in a
north-west direction from the capital P. R is circle. A is facing South-West, D is facing South-
another place that is located 2 km away in the East, B and E are right opposite A and D
south-west direction from K. M is another place respectively and C is equidistant between D
and that is located 2 km away in the north- and B. Which direction is C facing?
west direction from R. T is yet another place
(a) West (b) South
that is located 2 km away in the south-west
direction from M. In which direction is T (c) North (d) East
located in relation to P? 18. If a man on a moped starts from a point and
(a) South-west (b) North-west rides 4 km South then turns left and rides 2 km
and turn again to the right to ride to go more
(c) West (d) North towards which direction is he moving ?
13. Babu is Rahim’s neighbour and his house is 200 (a) North (b) West
meters away in the north-west direction.
(c) East (d) South
Joseph is Rahim’s neighbour and his house is 19. A man starts from a point, walk 8 km towards
located 200 meter away in the south-west North, turns right and walks 12 km, turns left
direction. Gopal is Joseph’s neighbour and he and walks 7 km turns and walks 20 km towards
stays 200 meters away in the south-east South, turns right and walks 12 km.
direction. Roy is Gopal’s neighbour and his
house is located 200 meters away in the north- In which direction is he from the starting point?
east direction. Then where is the position of (a) North (b) South
Roys’ house in relation to Babu’s ? (c) West (d) East
(a) South-east (b) south-west 20. Daily in the morning the shadow of Gol Gumbaz
(c) North (d) North-east falls on Bara Kaman and in the evening the
14. A tourist drives 10 km towards west and turns shadow of Bara Kaman falls on Gol Gumbaz
to left and takes a drive of another 4 km. exactly. So in which direction is Gol Gumbaz to
Bara Kaman?
He then drives towards east another 4 km and
then turns to his right and drives 5 km . (a) Easter side (b) Western side
Afterwards he turns to his left and travels 6 (c) Northern side (d) Southern side
km. In which direction is he from the starting 21. Ashok went 8 km South and turned West and
point? walked 3 km again he turned North and walked
(a) North (b) East 5 kms. He took a final turn to East and walked 3
(c) West (d) South kms . In which direction was Ashok from the
starting point?
15. A man started walking West. He turned right,
then right again and finally turned left. (a) East (b) North
Towards which direction was he walking now? (c) West (d) South

222 BUSINESS MATHEMATICS, LOGICAL REASONING & STATISTICS (Paper 3) [CA Foundation]
22. If X stands on his head with his face towards 29. A man starts from a point, walks 15 metres
south, to which direction will his left hand towards East, turns left and walks 10 metres,
point ? turns right again and walks. Towards which
(a) East (b) West direction is he now waking?
(c) North (d) South (a) North (b) East
23. I drove East for 5 miles then drove North 3 (c) West (d) South
miles, then turned to my left and drove for 2 30. A boy starts walking towards West, he turns
miles and again turned to my left. Which right and again he turns right and then turns
direction am I going now? left at last. Towards which direction is he
(a) South (b) North walking now?
(c) West (d) North-west (a) West (b) North
24. If A stands on his head with his face towards (c) West (d) East
north. In which direction will his left hand point 31. I stand with my right hand extended side-ways
? towards South. Towards which direction will
(a) North-East (b) North my back be ?
(c) East (d) North-West (a) North (b) West
25. A car travelling from south covers a distance (c) East (d) South
of 8 km, then turns right and runs another 9 32. If a person moves 4 km towards west, then
kms and again turns to the right and was turns right and moves 3 km and then turns right
stopped. Which direction does it face now? and moves 6 km, which is the directions in
(a) South (b) North which he is now moving ?
(c) West (d) East (a) East (b) West
26. A taxi driver commenced his journey from a (c) North (d) South
point and drove 10 km toward north and turned 33. If Mohan sees the rising sun behind the temple
to his left and drove another 5 km. After and the setting sun behind the railway station
waiting to meet a friend here, he turned to his from his house, what is the direction of the
right and continued to drive another 10 km. temple from the railway station?
He has covered a distance of 25 km so far, but (a) South (b) North
in which direction would he be now?
(c) East (d) West
(a) South (b) North
34. Laxman went 15 km to North then he turned
(c) East (d) South-east West and covered 10 kms. Then he turned
27. A walks 3 kms northward and then he turns south and covered 5 kms. Finally turning to
left and goes 2 km. He again turns left and goes East he covered 10 kms. In which direction he
3 km. He turns right and walks straight. In which is from his house?
direction is he walking now? (a) East (b) West
(a) East (b) West (c) North (d) South
(c) North (d) South 35. A man starts from a point, walks 4 miles North,
28. A walks southeards, then turns right, then left turns to his right and walks 2 miles, again turns
and then right. In which direction is he from to his right and walks 2 miles, again turns to
the starting point? his right and walks 2 miles. In which direction
(a) South (b) East would he be now from starting point ?
(c) West (d) North (a) North (b) South
(c) East (d) West
DIRECTION TESTS 223
36. I started walking down a road in the morning 43. Roy walks 2 km to East, then turns North-West
facing the Sun. After walking for sometime I and walks 3 km. Then he turns South and walks
turned to my left. Then I turned to my right. In 5 km. Then again he turns West and walks 2
which direction was I going then ? km. Finally he turns North and walks 6 km. In
(a) East (b) West which direction, is he from the starting point?
(c) North (d) South (a) South-West (b) South-East
(c) North-West (d) North-East
37. Lakshmi walked 2 furlongs north from her
house and took a turn to left and continued to 44. Seeta starts from a point, walks 2 km towards
walk another one kilometre and finally she north, turns towards her right and walks 2 km,
turned left and reached the school. Which turns right again and walks. What is the
direction is she facing now? direction she is facing now?
(a) West (b) North (a) East (b) West
(c) South (d) East (c) South (d) North
45. Shyam was facing East. He walked 5 km forward
38. You are going straight, first eastwards, then
and then after turning to his right walked 3
turn to the right, then right again, then left. In
km. Again he turned to his right and walked 4
which direction would you be going now?
km. After this he turned back.
(a) East (b) West
Which direction was he facing at that time?
(c) South (d) North (a) East (b) West
39. If Ahmed travels towards North from his house, (c) North (d) South
then to left, then to South covering equal
46. Raju is standing facing north. He goes 30 metres
distances in each direction to reach Sohan’s
ahead and turns left and goes for 15 metres.
house, in which direction is Ahmed’s house
Now he turns right and goes for 50 metres and
now?
finally turns to his right and walks. In which
(a) East (b) South direction is he heading?
(c) North (d) West (a) North (b) East
40. You go North, turn right, then right again and (c) South (d) West
then go to the left. In which direction are you 47. Sanmitra starts from his house and walks 3 km
now? towards north. Then he turns right and walks 2
(a) South (b) East km and then turns right and walks 5 km, then
(c) West (d) North turns right and walks 2 km and then again turns
right and walks 2 km. Which direction is he
41. Roopa starts from a point and walks 15 metre
facing now?
towards west, turns left and walks 12 metre,
turns right again and walks. What is the (a) North (b) South
direction she is now facing? (c) West (d) East
(a) South (b) West 48. Raju is Ramu’s neighbour and he stays 100
metres away towards southeast. Venu is Raju’s
(c) East (d) North
neighbour and he stays 100 metres away
42. A man starts his journey facing the sun early towards southwest. Khader is Venu’s
morning. He then turns right and walks 2 km. neighbour and he stays 100 metres away
He then walks 3 km after turning right again. towards, north-west. Then where is the
Which is the direction he is facing now? position of Khader ’s home in relation to
(a) North-East (b) North Ramu’s?
(a) South-East (b) South-West
(c) West (d) South
(c) North-West (d) East

224 BUSINESS MATHEMATICS, LOGICAL REASONING & STATISTICS (Paper 3) [CA Foundation]
49. Ramesh walked 3 km, towards West and 56. Ramu walks 5 kms starting from her house
turned to his left and walked 2 km. He, then towards west then turns right and walks 3 km.
turned to his right and walked 3 km. Finally, he Thereafter she takes left turn and walks 2 km.
turned to his right again and walked another 2 Further, she turn left and walks 3 km. Finally,
km. In which direction is Ramesh from his she turns right and walks 3 kms. In what
starting point now? direction she is now from her house?
(a) East (b) West (a) West (b) North
(c) North (d) South (c) South (d) East
50. Deepa starts walking north towards and after 57. Gopal started walking 2 km straight from his
a while she turns to her right. After walking school. Then he turned right and walked 1 km.
some distance, she turns to his left and walks Again he turned right and walked 1 km to reach
a distance of 1 km. She then urns to her left his house. If his house is sourth-east from his
again. In which direction she moving now? school, then in which direction did Gopal start
(a) North (b) West walking from the school?
(c) East (d) South (a) East (b) West
51. Raman starts walking in the morning facing the (c) South (d) North
Sun. After sometime, he turned to the left later 58. A man starts from a point, walks 2 km towards
again he turned to his left. At what direction is north, turns towards his right and walks 2 km,
Raman moving now? turns right again and walks. What is the
(a) East (b) West direction now he is facing?
(c) South (d) North (a) South (b) East
52. A starts walking towards North turns left, again (c) North (d) West
turns left, turns right, again turns right once 59. Janki started from her house and walked 2 km
again turns left. In which direction is A walking towards North. Then she took a right turn and
now? covered one kilometre. Then she took again a
(a) East (b) South right turn and walked for 2 kms. In what
(c) West (d) South-East direction is she going?
53. X walks southwards and then turns right, then (a) North (b) East
left and then right,. In which direction is he (c) South (d) West
moving now?
(a) South (b) North ----0---0---
(c) West (d) South-West
54. A man started to walk East. After moving a
distance, he turned to his right. After moving
a distance, he turned to his right again. After
moving a little he turned in the end to his left.
In which direction was he going now.?
(a) North (b) South
(c) East (d) West
55. A man starts from a point and walks 2 km
towards North, turns towards his right and
walks 2 km, turns right again and walks. What
is the direction now be is facing?
(a) South (b) South-East
(c) North (d) West

DIRECTION TESTS 225


(a) 5 kilometres West
(b) 5 kilometres North-east
HOME WORK - 1 (c) 7 kilometres Eas
(d) 7 kilometres West
7 A man leaves for his office from hishouse.He
1. A man is fecing south. He turns 135o in the house.He walks towards East.After moving a
anticlockwise direction and then 180o in the distance of 20 m. he turns towards South and
clockwise direction. Which direction is he walks 10 m. Then he walks 35 m towards the
facing now ? West and further 5 m towards the North.He
(a) North-east (b) North-west then turns towards East and walks 15 m. what
is the straight distance in metres between his
(c) South-east (d) soukth-west initial and final positions?
2. Aman is facing north-west.He turns 90o in the (a) 0 (b) 5
clockwise direction and the n 135 o anti
(c) 10
clockwise, then where he is facing now?
(d) Cannot be determined
(a) East (b) West
8 Gaurav walks 20 metres towards North.He then
(c) North (d) South turns left and walks 40 metres .He again turns
3 A man is facing north-west.He turns 90o in the left and walks 20 metres .Further,he moves 20
clockwise direction,then 180 o in the metres after turning to the right.How far is he
anticlockwise direction and then another 90o from his original position?
in the same direction.which direction is he (a) 20 metres (b) 30 metres
facing now?
(c) 50 metres (d) 60 metres
(a) South (b) South-west
9. Radha moves towards South-eastb a distance
(c) West (d) South-east. of 7 km. then she moves towards West and
4. I am facing east.I turn 100o in the clockwise travels a distance of 14 m From here.she moves
direction and then 145o in the anticlockwise towards North-west a distance of 7 m and
direction.which direction am I facing now? finally she moves a distance of 4 m towards
(a) East (b) North-east East and stood at that point.How far is the
(c) North (d) South-west starting point from where she stood?
5. Deepak starts walking straight towards east (a) 3 m (b) 4 m
east.After walking 75 metres. he turns to the (c) 10 m (d) 11 m
left and walks 25 metres straight.Again he turns 10. Gopal starts from his house towards West.After
to the left. walks a distance of 40 metres walking a distance of 30 metres he turned
staight,again he turns to the left and left and towards right and walked 20 metres .He then
walks a distance of 25 metres .How far is he turned left and moving a distance of 10 metres
from the starting point? ,turned to his left again and walked 40 metres.
(a) 25 metres (b) 50 METRES He now turns to the left and runs 6. Now which
(c) 115 ,MERES (d) None of these direction is her at facing?
6 Kishenkant walks 10 killometres towards (a) East (b) West
North.Fromthere.he walks 6 kilometres (c) North (d) South
towards south. Then he walks 3 km towards 11. Arat runs 20’ towards East and turns to
east. How far and in which direction is he with right,runs 10’ and turns to right.runs 9’ and
reference to his starting point? again turns to left ,runs 5’ and then turns to
left,runs 12’ and finally turns to left and runs
6’ ,Now ,which direction is the rat facing?
226 BUSINESS MATHEMATICS, LOGICAL REASONING & STATISTICS (Paper 3) [CA Foundation]
(a) East (b) West 17. Rohit walked 25 metres towards South.Then
(c) North (d) South he turneed to his left and walked 20 metres.
12. A girl leaves from her home .she first walks 30 He then turned to his leftand walked 25
metres in North -West direction and then 30 metres.He again turned to his right and walked
metres in South-west direction.Next,she 15 metres.At what distance is he from the
walks 30 metres in South-east starting point and in which direction?
direction.F ibnally,ahe turns towards her (a) 50 metres south
house,In which direction is she moving? (b) 50 metres north
(a) North-east (b) North-west (c) 35 meters east
(c) South-west (d) None of these (d) 30 meters north
13. Sanjeev walks 10 metres towards the 18. Starting from a point P,Sachin walked 20 metres
South.Turning to the left ,he walks 20 metres towrds South.He turned left and walked 30
and then moves to his right .After moving a metres He then turned left and walked 20
distance of 20 metres .he turns to the right metres.He ahgain turned left and walked 40
and walks20 metres ,Finally ,he turns to the metres and reached a point Q. how far and in
right and moves a distance of 10 metres.How which direction is the point Q from the point
far and in which direction is he from the P?
starting point?
(a) 20 metres West (b) 10 metres East
(a) 10 metres -North
(c) 10 metres West (d) 10 metres south
(b) 20 metres South
19. Ramakant walks northwards .After a while he
(c) South east
turns to hi s right and a little further to his
(d) 10 metres South left.Finally,after walking a distance of one
14. Kashish goes 30 metres North ,then turns right kilometre he turns to his left again.In which
and walks 40 metres,then again turns right and direction is he moving now ?
walks 20 metres,then aggain turns right and (a) North (b) South
walks 40 metres.How many metres is he from
his original position? (c) East (d) West
(a) 0 (b) 10 20. A man walks 1 km towards East and thehn he
(c) 20 (d) 40 turns to South and walks 5 km Again he turns
to East and walks 2 km after this he turns to
15. I am facing South.Iturn right and walk 20 m.
North and walks 9 km. Now,how far is he from
Then I turn right again and walk 10 m. and then
his from his starting point?
turning right walk 20 m. Then I turn right again
and walk 60 m In which direction am I from the (a) 3 km (b) 4 km
staring ppint? (c) 5 km (d) 7 km
(a) North (b) North-west 21. Raj travelled from a point X straight to Y at a
(c) East (d) South distance of 80 metres .He turned right and
16. A man walks 30 metres towards walked 50 metres,then again turned right and
South.Then,turning to his right.he walks 30 walked 70 metres. Finally,he turned right and
metres. Then,turning to his left, he walks 20 walked 50 metres.How far is he from the
metres.Again ,he turns to his left and walks 30 starting point?
metres.How far is he from his initial position? (a) 10 metres (b) 20 metres
(a) 50 metres south (c) 50 metres (d) 70 metres
(b) 50 metres north 22. Laxman went 15 kms to the west from my
(c) 30 meters south house.then turned left and walked 20 kms.He
(d) 30 meters north then turned East andc walked 25 kms and

DIRECTION TESTS 227


finally turning left covered 20 kms. How far East.Fromthe back side of his house,he walks
was he from his house? straight 50 metres ,then turns to the right and
(a) 5 kms (b) 10 kms walks 50 metres again.Finally ,he turns
towards left and stops after walking 25
(c) 40 kms (d) 80 kms
metres.Now,Aditya is in which direrction from
23. From his house,lokesh went 15 kms to the the starting point?
North .Then he turned West and coverd 10
(a) South-east (b) North-east
kms.Then,he turned South and coverd 5
kms.F inally,turning to East,he covered 10 (c) South-west (d) North-west
kms.In which direction is he from his hoouse?
(a) East (b) West
ANSWER KEYS
(c) North (d) South
(1) D (2) B (3) D
24. Going 50 m to the South of her house,Radhjika
(4) B (5) D (6) B
turns left and goes another 20 m Then,turning
to the North,the goes 30 m and then starts (7) B (8) D (9) C
walking to her house.In which direction is she (10) A (11) C (12) A
waking now? 13) B (14) B (15) D
(a) North-west (b) North (16) A (17) C (18) C
(c) South-east (d) East (19) D (20) C (21) A
25. A walks 10 metres in front and 10 metres to
(22) B (23) C (24) A
the right.Then every time turning to his left.he
walks 5,15 and 15 metres respectively.How far (25) A (26) D (27) B
is henow from his starting point? (28) D
(a) 5 metres (B) 10 metres 
(c) 15 metres (d) 20 metres
26. Rasik walks 20 m North.Then he turns right and
walks 30 m Then he turns right and walks 35 m
Then he turns left and walks 15 m. Then he
again turns left and walks 15 m In which
direction and how many metres away is he
from his original position?
(a) 10 metres West (b) 30 metres East
(c) 30 metres West (d) 45 metres East
27 A child is lokking for his father .He went 90
metres in the East before turning to his
right.He went 20 metres before turning to his
right again to look for his father at his uncle’s
place 30 metres from this point.His fatherwas
not there.From here he went 100 metres to
the North before meeting his father in a
street. How far did the son meet his father
from the starting point?
(a) 80 metres (b) 100 metres
(c) 140 metres (d) 260 metres
28 The door of Aditya’s house faces the

228 BUSINESS MATHEMATICS, LOGICAL REASONING & STATISTICS (Paper 3) [CA Foundation]
7. A man started to walk East. After moving a
certain distance, he turns to his right. After
HOME WORK - 2 moving some distance, he turns to his right
again. After moving a little he turns now to his
left currently, he is going in ……………….
Direction.
1. Mohan Starts from Point A and walks 1 km (a) North (b) East
towards south, turns left and walks 1 km. Then (c) West (d) South
he turns again left and walks 1 km. Now he is
8. Manu wants to go to the market. He starts from
facing.
his house towards North reaches at a crossing
(a) East (b) West after 30m. He turns towards East, goes 10m till
(c) North (d) South- West the second crossing and turns again, moves
2. Arun Started from Point A and Walked 10 kms towards South straight for 30m where
East to Point B , then turned to North and marketing complex exits. In which direction is
walked 3 kmsB to point C and then turned West the market from his house?
and walked 12 kms to point D , then again
(a) North (b) West
turned South and walked 3 kms to point E . In
which direction is he form his starting point? (c) South (d) East
(a) East (b) South 9. Anoop Starts walking towards South after
(c) West (d) North walking 15 meters he turns towards North.
3. I Stand with my right hand extended side-ways After walking 20 meters he turns towards East
towards South. Towards which direction will and walks 10 meters. He then turns towards
my back be? south and walks 5 meters. In which direction
(a) North (b) West is he from the original position.
(c) East (d) South (a) East (b) South
4. A man is facing East, then he turns left and (c) West (d) North
goes 10 m, then turns right and goes 5 m then
goes 5 m to the South and from there 5 m to
West. In which direction is to be from his
original place? ANSWER KEYS
(a) East (b) West 1 C 2 C 3 B 4 C
(c) North (d) South 5 B 6 D 7 D 8 D
5. A rat run 20 feet towards East and turns to right 9 A
runs 10 feet and turns to right runs 9 feet and
again turns to left runs 5 feet and then turns to
left runs 12 feet and finally turns to left and
runs 6 feet . Now what direction is the rat
facing.
(a) East (b) North
(c) West (d) South
6. Rahim started from point X and walked straight
5 km. West, then turned left and walked
straight 2 km and again turned left and walked
straight 7 km. In which direction is he from the
point X?
(a) North-East (b) South-East
(c) South-West (d) North-West

DIRECTION TESTS 229


CHAPTER-11
SEATING ARRANGEMENTS

INTRODUCTION
LEARNING OBJECTIVES
 To understand the Logical statements involved in the Seating Arrangements.
 To understand the types of Seating Arrangements.
The process of making a group of people to sit as per a prefixed manner is called Seating Arrangement
these questions, some conditions are given on the basis of which students are required to arrange
objects, either in a row or in a circular order.
INTRODUCTION
1 BASED ON VARIOUS PATTERN OF SITTING ARRANGEMENTS
ARE CLASSIFIED INTO
1) Linear Arrangements
2) Circular Arrangements
3) Polygon Arrangements
Here we are limited to our topic linear and circular arrangements only. While making arrangements, it
should be noted that all the conditions given are compiled with. These type of questions generally
involve five to eight individuals arranged in a certain manner or pre-conditions. They may have to be
arranged in a Circle or in a row accordingly.
Sometimes these questions are made more difficult by allowing an individual to a particular position
with some conditions.
General instructions to Solve Seating Arrangement Questions are as follows.
1) First of all take a review on the given information. After performing this step, you would get an
idea of the situation of people or objects.
2) Next, determine the usefulness of each information’s and classify them accordingly into ‘definite
information’, ‘comparative information’ and ‘negative information’.
3) When the place of any objects or persons is definitely mentioned then we say that it is a definite
information, X is sitting on the right end of the bench.
4) When the place of any object or person is not mentioned definitely but mentioned only in the
comparison of another person or object, then we say that it is a comparative information.
2 TYPE-1 LINEAR ARRANGEMENT
In this type of arrangement, we arrange objects or persons in a line or row. The arrangement is done
only on one ‘axis’ and hence, the position of persons or objects assumes importance in terms of order

230 BUSINESS MATHEMATICS, LOGICAL REASONING & STATISTICS (Paper 3) [CA Foundation]
like positions. In this type of arrangement, we take directions according to our left and right.
Steps to Solve the Linear Arrangements:
(a) Identify the number of objects and their names.
(b) Use pictorial method to represent the people or objects and their positions.
(c) Arrange the information with relevant facts and their positions and try to find out the solution.
(d) Answer the questions based on the arrangement having made.
There are few words which must be paid adequate attention, i.e., ‘between’ means sandwiched,
‘immediate left’ is different from ‘to the left’. To understand it let us see some pictorial representation.
When direction of face is not clear, then we take One Row Sequence
(A) When direction of face is not clear, then we take based on diagram will be as follows:

From the above diagram, it is clear that


(i) Q, R, S, T are right of P but only Q is the immediate right of P.
(ii) S, R, Q, P are left of T but only S is the immediate left of T.
(iii) R, S, T are right of Q only R is the immediate right of Q.
(iv) R, Q, P are left of S but only R is the immediate left of S.
(v) S and T are right of R but only S is the immediate right of R.
(vi) Q and P are left of R but only Q is the immediate left of R.
(vii) A is the immediate left of Q while T is the immediate right of S.
(B) When direction of face is towards you, then the diagram will be as follows:

From the above diagram, it is clear that


(i) Left of P = P, R, S and T
(ii) Right of T = S, R, Q and P
(iii) Q is immediate left of P; R is immediate left of Q; S is immediate left of R and T is immediate left
of S.
(iv) S is immediate right of T; R is immediate right of S; Q is immediate right of R; and P is immediate
right of Q.
Two Rows Sequence
Let us see 6 persons seating in two rows.

SEATING ARRANGMENTS 231


From the above diagram, it is clear that
(i) A is sitting opposite D
(ii) B is sitting opposite E
(iii) C is sitting opposite F
(iv) D and C are sitting at diagonally opposite positions
(iv) S and R are sitting at diagonally opposite positions.

232 BUSINESS MATHEMATICS, LOGICAL REASONING & STATISTICS (Paper 3) [CA Foundation]
6. Five boys are standing in a row facing East.
Pavan is left of Tavan, Vipin and Chavan to the
CLASS WORK left of Nakul. Chavan is between Tavan and
Vipin. Vipin is fourth from the left, then how
far is Tavan to the right?
1. Five boys A, B, C, D and E are sitting in a row A (a) First (b) Second
is to the right of B and E is to the left of B but to
the right of C. A is to the left of D. Who is (c) Third (d) Fourth
second from the left end? 7. Six persons M, N, O, P, Q and R are sitting in
(a) D (b) A two row with three persons in each row. Both
the row are in front of each other. Q is not at
(c) E (d) B the end of any row. P is second the left of R. O
2. There are five different houses, A to E, in a is the neighbour of Q and diagonally opposite
row. A is to the right of B and E is to the left of to P. N is the neighbour of R. Who is in front N?
C and right of A, B is to the right of D. Which of
(a) R (b) Q
the houses is in the middle?
(c) P (d) M
(a) A (b) B
(c) C (d) D 8. Six persons A, B, C, D, E and F are sitting in two
row, three in each row.
3. Five friends P, Q, R, S and T are sitting in a row
facing North. Here, S is between T and Q and (I) E is not at the end of any row
Q is to the immediate left of R. P is to the (II) D is second to the left of F
immediate left of T. Who is in the middle? (III) C, the neighbor of E, is sitting diagonally
(a) S (b) T opposite to F.
(c) Q (d) R (IV) B is the neighbor of F.
4. Six children A, B, C, D, E and F are standing in a Which of the following are in one of the two
row. B is between F and D. E is between A and rows?
C. A does not stand next to F or D. C does not
(a) D, B and F (b) C, E and B
stand next to D. F is between which of the
following pairs of children? (c) A, E and F (d) F, B
(a) B and E (b) B and C 9. Direction (Q.No.9): Read the following
(c) B and D (d) B and A information carefully and answer that question
that follows.
5. There are eight books kept one over the other.
Two books are on Organisation Behaviour, two Five boys A1, A2, A3, A4and A5 are sitting in a
books on TQM, three books on Industrial stair in the following way.
Relations and one book is on Economics. I. A5 is above A1
Counting from the top, the second, fifth and II. A4 is under A2
sixth books are on Industrial Relations. Two
books on Industrial Relations are between two III. A2 is under A1
books on TQM. One book of Industrial IV. A 4 is between A2 and A3.
Relations is between two books on Who is at the lowest position of the stair?
Organizational Behaviour while the book
above the book of Economics is a book of TQM. (a) A1 (b) A3
Which book is the last book from the top? (c) A5 (d) A2
(a) Economics (b) TQM
10. Five children are sitting in a row. S is sitting
(c) Industrial Relations next to P. K is sitting next to R, who is sitting on
(d) Organizational Behaviour the extreme left and T is not sitting next to K.

SEATING ARRANGMENTS 233


Who is/are adjacent to S? 16. What is the position of H with respect to F?
(a) K and P (b) R and P (a) Third to the left
(c) Only P (d) P and T (b) Immediate right
(c) Second to right
11. F ive senior citizens are living in a multi-
storeyed building. Mr. Muan lives in a flat (d) Fourth to left
above Mr. Ashokan, Mr. Lokesh in a flat below 17. How many persons are seated between A and
Mr. Gaurav, Mr. Ashokan lives in a flat above E?
Mr. Gaurav and Mr. Rakesh lives in a flat below (a) One (b) Two
Mr. Lokesh. Who lives in the topmost flat? (c) Three (d) Four
(a) Mr. Lokesh (b) Mr. Gaurav Directions (Q. No. 18-22)
(c) Mr. Muan (d) Mr. Rakesh Study the following information carefully to
answer the given questions.
12. In a gathering seven members are sitting in a
row. ‘C’ is sitting left to ‘B’ but on the right to Ten students are A to J are sitting in a row facing
‘D’. west.
I. B and F are not sitting on either of the
‘A’ is sitting right to ‘B’, ‘F; is sitting right to ‘E’
edges.
but left to ‘D’. ‘H’ is sitting left to ‘E’. Find the
person sitting in the middle II. G is sitting left of D and H is sitting to the
right of J.
(a) C (b) D
III. There are four persons between E and A.
(c) E (d) F IV. I is the north of B and F is the south of D.
Directions (No: 13-17): Study the following V. J is between A and D and G is in E and F.
information carefully to answer the given VI. There are two persons between H and C.
questions.
18. Who is sitting at the seventh place counting
A to H are seated in straight line facing North. C from left?
sits fourth left of G. D sits second to right of G. (a) H (b) C
Only two people sit between D and A. B and F are (c) J (d) Either H or C
immediate neighbours of each other. B is not an 19. Who among the following is definitely sitting
immediate neighbour of A. H is not neighbour of at one of the ends?
D.
(a) C (b) H
13. Who amongst the following sits exactly in the (c) E
middle of the persons who sit fifth from the
(d) Cannot be determined
left and the person who sit sixth from the
right? 20. Who are immediate neighbours of I?
(a) BC
(a) C (b) H
(b) BH
(c) E (d) F
(c) AH
14. Who amongst the following sits third to the (d) Cannot determined
right of C?
21. Who is sitting second left of D?
(a) B (b) F (a) G (b) F
(c) A (d) E (c) E (d) J
15. Which of the following represents persons 22. If G and A interchange their positions, then
seated at the two extreme ends of the line? who become the immediate neighbours of E?
(a) C, D (b) A, B (a) G and F (b) Only F
(c) B, G (d) D, H (c) Only A (d) J and H

234 BUSINESS MATHEMATICS, LOGICAL REASONING & STATISTICS (Paper 3) [CA Foundation]
Directions (Q. Nos. 23-24) Read the following 28. Four girls A, B, C, D are sitting around a circle
information carefully and then answer the facing the centre. B and C infront of each other,
questions which of the following is definitely true ?
that follow. (a) A and D infront of each other
A group of singers, facing the audience, are (b) A is not between B and C
standing in line on the stage as follows. (c) D is left of C
I. D is not right to C (d) A is left of C
II. F is not standing beside G.
III. B is not left of F 
IV. E is not left of A
V. C and B have one person between E and
F
VI. There are two persons H and C.
23. Who is on the Second extreme right?
(a) D (b) F
(c) G (d) E
24. If we start counting from the left, on which
number is B ?
(a) 1st (b) 2nd
(c) 3rd (d) 5th
Directions (Q. No. 25- 27): Study the following
information carefully to answer the given
questions.
Eight persons P to W are sitting in front of one
another in two rows. Each row has four persons.
P is between U and V and facing North. Q, who is
to the immediate left of S is facing W. R is
between T and S and W is to the immediate right
of V.
25. Who is sitting in front of R?
(a) U (b) Q
(c) V (d) P
26. Who is to the immediate right of R?
(a) S (b) U
(c) M or S (d) None of these
27. In which of the following pairs, persons are
sitting in front of each other?
(a) SV (b) RV
(c) TV (d) UR

SEATING ARRANGMENTS 235


7. (Q Nos. 1 to 3) Study the following Question
carefully and answer the given questions.
HOME WORK-1 Four ladies & A, B, C and D and Four Gentlemen
E, F, G and H are sitting in a circle around a
table
1. Four Children’s are sitting in arrow. A is
facing each other .
occupying seat next to B but not next to C. If C
is not sitting next to D? Who is occupying seat I. No two ladies or gentlemen are sitting
next to adjacent to D. side by side.
(a) B (b) B and A II. C, who I sitting between G and E, is facing
(c) Impossible to tell (d) A D.
2. P, Q, R, S, T, U, V and W are sitting in a row III. F is between D and A and facing G.
facing North. IV. H is to the right of B.
P is fourth to the right of T
(1) Who is sitting left of A?
W is fourth to the left of S
(a) E (b) F
R and U, which are not at the ends, are
neighbours of Q and T respectively. (c) G (d) H
W is next to the left of P and P is the neighbour (2) E is facing whom?
of Q, who are sitting at the extreme ends. (a) F (b) B
(a) P,Q (b) Q,R (c) G (d) H
(c) T,V (d) None
(3) Who is immediate neighbour of B?
3. There are Five houses P, Q , R, S, T . P is right of
(a) G and H (b) E and F
Q and T is left of R and right of P . Q is right of
S. Which house in the middle. (c) E and H (d) F and H
(a) P (b) Q 8. Eight persons A, B, C, D, E, F, G and H are sitting
(c) R (d) T around the circle as given in the figure.
4. Friends are sitting on a bench. A is to the left They are facing the direction opposite to
of B but on the right of C, D is to the right of B centre. If they move upto three places anti-
but one the left of E. Who are at the extremes? clockwise, then.
(a) A, B (b) A, D
(c) C, E (d) B, D
5. In a college party, 5 girls are sitting in a row. F
is to the left of M and to the right of O. R is
sitting to the right of N but to the left of O.
Who is sitting in the middle?
(a) O (b) R
(c) P (d) M
6. Five boys A, B, C, D and E are standing in a row.
D is on the right of E, B is on the left of E but on
the right of A. D is one the left of C, who is
standing on the extreme right. Who is standing
in the middle?
(a) B (b) C
(c) D (d) E

236 BUSINESS MATHEMATICS, LOGICAL REASONING & STATISTICS (Paper 3) [CA Foundation]
(a) B will face west (5) Who amongst the following is sitting third to
(b) E will face East the right of A?
(a) F (b) B
(c) H will face North-West
(d) A will face South (c) H (d) C

9. Five People A, B, C, D and E are seated about a


round table. Every chair is spaced equidistant ANSWER KEYS
from adjacent chairs.
I. C is seated next to A .
II. A is seated two seats from D. 1. (d) 2. (c) 3. (a) 4. (c)

III. B is not seated next to A. 5. (a) 6. (d)

Which of the following must be true? 7. (1) (b) (2) (d) 3 (a)

I. D is seated next to B. 8. (a) 9. (c)

II. E is seated next to A. 10. (1)(c) (2)(d) 3(d) 4(c) (5) (d)

Select the correct from the options given


below:
(a) Only I (b) Only II ---0---0---
(c) Both I and II (d) Neither I nor II
10. Eight friends A, B, C, D, E, F, G and H are sitting
in a circle facing the centre, not necessarily in
the same order. D sits third to the left of A. E
sits to the immediate right of A. B is third to
left of D. G is second to the right of B. C is
neighbour of B. C is third to left of H.
(1) Who amongst the following is sitting
exactly between F and D?
(a) C (b) E
(c) H (d) A
(2) Three of the following four are alike in a
certain way based on the information
given above and so form a group. Which
is does not belong to that group.
(a) DC (b) AH
(c) EF (d) DF
(3) Who amongst the following second to
the left of H?
(a) E (b) B
(c) A (d) Noe of these
(4) Who amongst the following are
immediate neighbours of G?
(a) CA (b) AF
(c) DC (d) DF
SEATING ARRANGMENTS 237
7-10 Six persons P, Q, R, S, T and U are sitting in two
rows, three in each.
T is not at the end of any row
HOME WORK-2 S is the second to the left of U
R the neighbour of T, is sitting diagonally opposite
to S.
Q is the neighbour of U
1. There are Five houses A, B, C, C, D, E, A is the
right of B and E is left of C and right of A, B is 7. Which of the following are sitting diagonally
right of D, which house is middle opposite to each other?
(a) U and R (b) S and P
(a) A (b) B
(c) P and R (d) P and U
(c) C (d) D
(e) P and Q
2. Girls are sitting on a bench, Q is the left of R
but on the right of P. S is to be right of R and 8. Which of the following are in the same row?
left of T. Who are the extremes. (a) P and T (b) T and S
(a) P,T (b) P,S (c) R and Q (d) P and Q
(c) Q,T (d) Q,S (e) R and T
3. Five friends P, Q, R, S and T are sitting in a row 9. Which of the following are in one of the two
facing North. Here, S is between T and Q and Q rows?
is tothe immediate left of R. P is to the (a) UQR (b) RTQ (c) SQU
immediate left of T. Who is in the middle? (d) PTU (e) PQU
(a) S (b) T 10. After interchanging seat with T, who will be
the neighbours of S in the new position?
(c) Q (d) R
(a) R and P (b) U and Q
4-5. Read the following information and answer the
Questions that follows. (c) Only Q (d) Only P
(e) Only R
(1) Six friends A, B, C, D, E and F are sitting in a
closed circle facing the center. 11. Five students A, B, C, D and E are standing in a
row. D is on the right of E, B is on the left of E
(2) E is to the left of D. but on the right of A. D is next to C on his left.
(3) C is between A and B. The student in middle is
(4) F is between E and A. (a) B (b) A
4. Who is to the left of B? (c) E (d) C
(a) A (b) C 12. Six flats on a floor in two rows facing North
and South are allotted to P, Q, R, S, T and U. If Q
(c) D (d) E
gets a North facing flat and is not next to S. S
5. Who is to the right of C? and U get diagonally opposite flat. R next to U
(a) A (b) B gets a South facing flat and T gets a North facing
(c) D (d) E flat. Whose flat is between Q and S?
6. In a march past, seven persons are standing in (a) P (b) T
a row. Q is standing left to R but right to P. O is (c) R (d) U
standingright to N and left to P. Similarly, S is 13. Eight persons A, B, C, D, E, F, G and H are sitting
standing right to R and left to T. Find out who in a line. E Sits second right to D. H sits fourth
is standing in middle? left to D. C and F are immediate neighbors,
(a) P (b) Q but C is not immediate neighbor of A. G is not
(c) R (d) O neighbor of E.

238 BUSINESS MATHEMATICS, LOGICAL REASONING & STATISTICS (Paper 3) [CA Foundation]
Only two persons sit between A and E. The
persons on left and right end respectively are
(a) G and B (b) G and E
(c) H and E (d) B and E
14. Six children A, B, C, D, E and F are sitting in a
row. B is between F and D. E is between A and
C.
However, A does not sit next to F or D. C does
not sit next to D. Then, F is sitting between.
(a) B and D (b) B and C
(c) E and C (d) None

ANSWERS KEYS

1. (a) 2. (a) 3. (a) 4. (c)

5. (a) 6. (b) 7. (d) 8. (a)

9. (c) 10. (a) 11. (c) 12. (d)

13. (b) 14. (b)

SEATING ARRANGMENTS 239


CHAPTER-12
BLOOD RELATIONS

INTRODUCTION
DEFINITION
A person who is related to another by birth rather than by marriage.
Prerequisites:
To remember easily the relations may be divided into two sides as given below:
(i) Relations of Paternal side:
Father’s father  Grandfather
Father’s mother  Grandmother
Father’s brother  Uncle
Father’s sister  Aunt
Children of uncle  Cousin
Wife of uncle  Aunt
Children of aunt  Cousin
Husband of aunt  Uncle
(ii) Relations of Maternal side:
Mother’s father  Maternal grandfather
Mother’s mother  Maternal grandmother
Mother’s brother  Maternal uncle
Mother’s sister  Aunt
Children of maternal uncle  Cousin
Wife of maternal uncle  Maternal aunty

Relations:
1. Grandfather’s son • Father or Uncle
2. Grandmother’s son • Father or Uncle
3. Grandfather’s only son • Father

240 BUSINESS MATHEMATICS, LOGICAL REASONING & STATISTICS (Paper 3) [CA Foundation]
4. Grandmother’s only son • Father
5. Mother’s or father’s mother • Grandmother
6. Son’s wife • Daughter-in-Law
7. Daughter’s husband • Son-in-Law
8. Husband’s or wife’s sister • Sister-in-Law
9. Brother’s son • Nephew
10. Brother’s daughter • Niece
11. Uncle or aunt’s son or daughter • Cousin
12. Sister’s husband • Brother-in-Law
13. Brother’s wife • Sister-in-Law
14. Granson’s or grand daughter’s daughter • Great grand Daughter
The efficiency in doing the problems of blood relations depends upon the knowledge of the blood relations.
Some of the important relations are given below:
(a) My mother’s or father’s son is my Brother.
(b) My mother’s or father’s daughter is my Sister.
(c) My mother’s or father’s father is my Grandfather.
(d) My mother’s or father’s sister is my Aunt.
(e) My mother’s or father’s brother is my Uncle.
(f) My son’s wife is my Daughter-in-law.
(g) My daughter’s husband is my Son-in-law.
(h) My brother’s son is my Nephew.
(i) My brother’s daughter is my Neice.
(j) My sister’s husband is my Brother-in-law.
(k) My brother’s wife is my Sister-in-law.
(l) My husband’s wifer’s sister is my Sister-in-law.
(m) My husband’s or wife’s brother is my Brother-in-law.
(n) My uncle’s or aunt’s son or daughter is my Cousin.
(o) My wife’s father or husband’s father is my Father-in-law.
(p) My wife’s mother or husband’s mother is my Mother-in-law.
(q) My father’s wife is my Mother.
(r) My mother’s husband is my Father.
(s) My son’s or daughter’s son is my Grandson.
(t) My son’s or daughter’s daughter is my Grand-daughter.

BLOOD REALATION 241


8. A reads a book and find the name of the author
familiar. The author ‘B’ is the paternal uncle of
C. C is the daughter of A. How is B related to A?
CLASS WORK
(a) Brother (b) Sister
(c) Father (d) Uncle
9. A’s mother is sister of B and she has a daughter
1. A is B’s brother. C is A’s mother. D is C’s father, C who is 21 years old. How is B related to C?
E is B’s son. How is D related to A? (a) Uncle (b) Maternal Uncle
(a) Son (b) Grandson (c) Niece (d) Daughter
(c) Grandfather 10. A is B’s brother. C is A’s mother. D is C’s father. F
(d) Great Grandfather is A’s son. How is F related to D?
2. A is B’s brother. C is A’s father. D is C’s sister and (a) Son (b) Grandson
E is D’s mother. How is B related to E?
(c) Grand-grandson
(a) Grand-daughter
(d) Grand-daughter
(b) Great grands daughter
11. A is B’s brother. C is A’s mother. D is C’s father. E
(c) Grandaunt
is B’s son. How is B related to D?
(d) Daughter
(a) Son
3. A is B’s Sister. C is B’s Mother. D is C’s Father. E is
D’s Mother. Then how is A related to D? (b) Grand-daughter
(a) Grandmother (b) Grandfather (c) Grandfather
(c) Daughter (d) Great grandfather
(d) Grands-daughter 12. A is B’s brother. C is A’s mother. D is C’s father. F
4. A is the father of B. C is the daughter of B. D is is A’s son. How is B related to F’s child?
the brother of B. E is the son of A. What is the (a) Aunt (b) Cousin
relationship between C and E?
(c) Nephew (d) Grandfather
(a) Brother and sister (b) Cousins
13. A is B’s daughter. B is C’s mother. D is C’s brother.
(c) Niece and uncle
How is D related to A?
(d) Uncle and aunt
(a) Father (b) Grandfather
5. If P is the husband of Q and R is the mother of
S and Q. What is R to P? (c) Brother (d) Son
(a) Mother (b) Sister 14. A is D’s brother. D is B’s father. B and C are
sisters. How is C related to A?
(c) Aunt (d) Mother-in-law
6. P and Q are brothers. R and S are sister. P’s son (a) Cousin (b) Niece
is S’s brother. How is Q related to R? (c) Aunt (d) Nephew
(a) Uncle (b) Brother 15. A is B’s brother. C is A’s mother, D is C’s father.
(c) Father (d) Grandfather E is B’s son. How is D related to E ?
7. X is the husband of Y. W is the daughter of X. Z (a) Grandson
is husband of W. N is the daughter of Z. What is (b) Great Grandson
the relationship of N to Y?
(c) Great Grandfather
(a) Cousin (b) Niece
(d) Grandfather
(c) Daughter
(d) Grand-daughter

242 BUSINESS MATHEMATICS, LOGICAL REASONING & STATISTICS (Paper 3) [CA Foundation]
16. X and Y are the children of A. A is the father of 24. A and B are brothers. E is the daughter of F. F is
X but Y is not his son. How is Y related to A? the wife of B. What is the relation of E to A?
(a) Sister (b) Brother (a) Sister (b) Daughter
(c) Son (d) Daughter (c) Niece (d) Daughter
17. A is B’s brother. C is A’s mother. D is C’s father. E 25. M and F are a married couple. A and B are
is B’s son. How is E related to A? sisters. A is the sister of F. Who is B to M?
(a) Cousin (b) Nephew (a) Sister (b) Sister-in-law
(c) Uncle (d) Grandson (c) Niece (d) Daughter
26. If A is the mother of D. B is not the son of C. C is
18. Based on the statements given below, fi nd
the father of D, D is the sister of B, then how is
out who is the uncle of P?
A related to B?
(i) K is the bother of J
(a) Mother (b) Brother
(ii) M is the sister of K (c) Step son (d) Sister
(iii) P is the brother of N 27. A and B are brother and sister respectively. C
(iv) N is the daughter of J is A’s father. D is C’s sister and E is D’s mother.
(a) K (b) J How is B related to E?
(c) N (d) M (a) Grand-daughter
19. A and B are sisters. A is mother of D. D has a (b) Great grand-daughter
daughter C who is married to F. G is the husband (c) Aunt
of A. How is C related to D? (d) Daughter
(a) Daughter (b) Niece 28. Q is the son of P. X is the daughter of Q. R is the
(c) Aunt (d) Sister-in-law aunty (Bua) of X and L is the son of R, then
what is L to P?
20. R and S are brothers. X is the sister of Y and X is
mother of R. What is Y to S? (a) Grandson
(b) Grand-daughter (c) Daughter
(a) Uncle (b) brother
(d) Nephew
(c) Father (d) Mother
29. P and Q are brothers. R and S are sisters. P’s
21. A is B’s brother. C is A’s mother. D is C’s father. B son is S’s brother. How is Q related to R?
is D’s grand-daughter. How is B related to E.
(a) Uncle (b) Brother
Who is A’s son?
(c) Father (d) Grandfather
(a) Aunt (b) Cousin
30. A and B are the young ones of C. If C is the
(c) Niece (d) Grandaunt mother of B but A is not the daughter of C,
22. A is the son of B while B and C are sisters to then what is the relationship between C and
one another. E is the mother of C. If D is the A?
son of E, which of the following statements is (a) Nephew and Aunty
correct?
(b) Brother and Sister
(a) D is the maternal uncle of A (c) Mother and son
(b) E is the brother of B (d) Niece and Aunty
(c) D is the cousin of A 31. A is the mother of D and sister of B. B has a
(d) B and D are brothers daughter C who is married to F. G is the husband
23. P is the father of T. T is the daughter of M. M is of A. How is G related to D?
the daughter of K. What is P to K? (a) Uncle (b) Husband
(a) Father (b) Father-in-law (c) Son (d) Father
(c) Brother (d) Son-in-law

BLOOD REALATION 243


32. Pointing towards A, B said “your mother is the 39. Rajiv is the brother of Atul. Sonia is the sister
younger sister of my mother”. How is A related of Sunil. Atul is the son of Sonia. How is Rajiv
to B? related to Sonia?
(a) Uncle (b) Cousin (a) Nephew (b) Son
(c) Nephew (d) Father (c) Brother (d) Father
33. A is B’s wife’s husband’s brother. C and D are 40. Sita is the niece of Ashok. Ashok’s mother is
sisters of B. How is A related to C? Lakshmi. Kalyani is Lakhshmi’s mother.
(a) Brother (b) Sister-in-law Kalyani’s husband is Gopal. Parvathi is the
(c) Wife (d) Sister mother-in-law of Gopal. How is Sita related to
Gopal?
34. A and B are brothers. C and D are sisters. A’s
son is D’s brother. How is B related to C? (a) Great grandson’s daughter
(a) Father (b) Brother (b) Gopal’s Sita’s father
(c) Uncle (d) Son (c) Sita is Gopal’s great grand-daughter
35. A is B’s sister. C is B’s mother. D is C’s father. E is (d) Grand niece
D’s mother. Then how is A related to D?
41. Seema is the daughter-in-law of Sudhir and
(a) Grandmother (b) Grandfather
sister-in-law of Ramesh. Mohan is the son of
(c) Daughter Sudhir and only brother of Ramesh. Find the
(d) Grand-daughter relation between Seema and Mohan.
36. P, Q, R, S, T, U are 6 members of a family in (a) Sister-in-law (b) Aunt
which there are two married couples. T, a
(c) Cousin (d) Wife
teacher is married to a doctor who is mother
of R and U. Q the lawyer is married to P. P has 42. Suresh introduces a man as “He is the son of
one son and one grandson. Of the two married the woman who is the mother of the husband
ladies one is a housewife. There is also one of my mother”. How is Suresh related to the
student and one male engineer in the family. man?
Which of the following is true about the grand- (a) Uncle (b) Son
daughter of the family?
(c) Cousin (d) Grandson
(a) She is a lawyer
43. Pointing to a lady in a photograph. Meera said.
(b) She is an engineer “Her father’s only son’s wife is my mother-in-
(c) She is a student law “How is Meera’s husband related to that
(d) She is a doctor lady in the photo?
37. Six members of a family namely A, B, C, D, E (a) Nephew (b) Uncle
and F are travelling together. ‘B’ is the son of C (c) Son (d) Father
but C is not the mother of B. A and C are
44. Pointing to a photograph Vikas said “She is the
married couple. E is the brother of C. D is the
daughter of my grandfather’s only son”. How
daughter of A. F is the brother of B. How many
is the related to Vikas in the photograph?
male members are there in the family?
(a) 3 (b) 2 (a) Father (b) Brother
(c) 4 (d) 1 (c) Sister (d) Mother
38. A’s mother is sister of B and has a daughter C. 45. Suresh’s sister is the wife of Ram. Ram is Rani’s
How can A be related to B from among the brother. Ram’s father is Madhur. Sheetal is
following? Ram’s grandmother. Rema is Sheetal is
daughter-in-law. Rohit is Rani’s brother’s son.
(a) Niece (b) Uncle
Who is Rohit to Suresh?
(c) Daughter (d) Father

244 BUSINESS MATHEMATICS, LOGICAL REASONING & STATISTICS (Paper 3) [CA Foundation]
(a) Brother-in-law (b) Son 52. Ravi’s father has a son Rohit who has an aunt
(c) Brother (d) Nephew Laxmi who has a husband Rao whose father-
in-law is Mohan. What is the relation of Mohan
46. Vinod introduces Vishal as the son of the only to Ravi?
brother of his father’s wife. How is Vinod
related to Vishal? (a) Nephew (b) Grandfather
(c) Son (d) Uncle
(a) Cousin (b) Brother
53. Vijay says, Ananda’s mother is the only
(c) Son (d) Uncle
daughter of my mother”. How is Ananda
47. Among her children, Ganga’s favourites are relation to Vijay?
Ram and Rekha. Rekha is the mother of Sharat, (a) Brother (b) Father
who is loved most by his maternal uncle
Mithun. The head of the family is Ram Lal, who (c) Nephew (d) Grandfather
is succeeded by his sons Gopal and Mohan. 54. Introducing a man, a woman said, “His wife is
Gopal and Ganga have been married for 35 the only daughter of my mother.” How is the
years and have 3 children.What is the relation woman related with the man?
between Mithun and Mohan? (a) Sister-in-law (b) Wife
(a) Uncle & Nephew (b) Son (c) Aunt (d) Mother-in-law
(c) Brother (d) No relation 55. A prisoner introduced a boy who came to visit
48. Rahul and Robin are brothers. Promod is him to the jailor as “Brothers and sisters I have
Robin’s father. Sheela is Pramod’s sister. Prema none, he is my father’s son’s son”. Who is the
is Promod’s niece. Shubha is Sheela’s grand- boy?
daughter. How is Rahul related to Shubha? (a) Nephew (b) Son
(a) Brother (b) Cousin (c) Cousin (d) Uncle
(c) Uncle (d) Nephew
49. Preeti has a son, named Arun. Ram is Preeti’s
brother. Neeta too has a daughter named
Reema. Neeta is Ram’s sister. What is Arun’s
relationship to Reema?
(a) Brother (b) Nephew
(c) Cousin (d) Uncle
50. There are 2 film stars. One is the father of the
other’s son. What is the relationship of the
two with each other?
(a) Grandfather and Grandson
(b) Grandfather and son
(c) Husband and wife
(d) Father and Son
51. Ramu’s mother said to Ramu,“My mother has
a son whose son is Achyut”. How is Achyut
relation to Ramu?
(a) Uncle (b) Cousin
(c) Brother (d) Nephew

BLOOD REALATION 245


(a) A (b) B
HOME WORK-1 (c) C (d) D
9. A is D’ brother. D is B’s father. B and C are sisters.
How is A related to C?
1 A is B’s daughter, B is C’s mother. D is C’s brother. (a) Son (b) Grandson
How is D related to A? (c) Father (d) Uncle
10. A is B’s sister. C is B’s mother. D is C’s father. E is
(a) Father (b) Grandfather
D’s mother, then how A is related to D?
(c) Brother (d) Son (a) Grandfather (b) Daughter
2 P is Q’s brother. R is Q’s mother. S is R’s father. (c) Grandmother
T is S’s mother. How is P related to T? (d) Granddaughter
(a) Grand-daughter 11. (i) F is the brother of A.
(ii) G is the daughter of A.
(b) Great grandson
(iii) K is the sister of F.
(c) Grandson (iv) G is the sister of C.
(d) Grandmother Who is the uncle of G?
3 A is B’s brother. C is D’s father. E is B’s mother. (a) A (b) C
A and D are brothers. How is E related to C? (c) K (d) F
(a) Sister (b) Sister-in-law 12. A is father of C and D is son of B. E is brother of
A. If C is sister of D how is B related to E?
(c) Niece (d) Wife
(a) Sister-in-law (b) Sister
4 A is the sister of B. B is the brother of C. C is the (c) Brother (d) Brother-in-law
son of D. How is D related to A? 13. C is wife of B. E is the son of C A is the brother
(a) Mother (b) Daughter of B and father of D. What is the relationship
(c) Son (d) Uncle of E to D?
5. B is the brother of A. whose only sister is (a) Mother (b) Sister
mother of C. D is maternal grandmother of C. (c) Brother (d) Cousin
How is A related to D? 14. M is the son of P. Q is the grand-daughter of O,
(a) Daughter-in-law (b) Daughter who is the husband of P. How is M related to
(c) Aunt (d) Nephew (d) Nephew O?
(a) Son (b) Daughter
6. A and B are sisters. R and S are brothers. A’s
(c) Mother (d) Father
daughter is R’s sister. What is B’s relation to S?
15. X and Y are brothers. R is the father of Y. S is the
(a) Mother (b) Grandmother
brother of T and maternal uncle of X. What is T
(c) Sister (d) Aunt to R?
7. E is the sister of B. A is the father of C. B is the (a) Mother (b) Wife
son of C. How is A related to E? (c) Sister (d) Brother
(a) Grandfather
(b) Grand-daughter
ANSWER KEYS
(c) Father
(d) Great-grandfather
1. (c) 2. (b) 3. (d) 4. (a)
8. Given that:
A is the mother of B. 5. (b) 6. (d) 7. (a) 8. (a)
C is the son of A. 9. (d) 10. (d) 11. (d) 12. (a)
D is the brother of E. 13. (d) 14. (a) 15. (b)
E is the daughter of B.
Who is grandmother of D? 

246 BUSINESS MATHEMATICS, LOGICAL REASONING & STATISTICS (Paper 3) [CA Foundation]
10. Six Persons are seen together in a group. They
are A, B, C, D, E and F. B is brother of D, but D is
HOME WORK-2 not brother of B. F is brother of B. C and A are
married together. F is son of C, but C is not
mother of F. E is brother of A. The number of
female member in the group is
(a) 1 (b) 2
1 A is the sister of B. B is the brother of C. C is
(c) 3 (d) 4
the son of D. How is D related to A?
11. Ram and Mohan are brothers, Shankar is
(a) Mother (b) Daughter
Mohan’s father. Chhaya is Shankar's sister. Priya
(c) Son (d) Uncle
is shankar's niece. Shubhra is Chhaya's
2. If P is the husband of Q and R is the mother of granddaughter. Then, Ram is Shubhra's
S and Q. What is R to P?
(a) Brother (b) Uncle
(a) Mother (b) Sister
(c) Cousin (d) Nephew
(c) Aunt (d) Mother-in-law
12. If P + Q means P is the mother of Q, P ÷Q
3. P is the father of T. T is the daughter of M. M is means P is the father of Q, P - Q means P is
the daughter of K. What is P to K? the sister of Q.
(a) Father (b) father-in-law Then which of the following relationship shows
(c) Brother (d) Son-in-law that M is the daughter of R?
4. A and B are brothers. E is the daughter of F. F is (a) R÷M+N (b) R+N÷M
the wife of B. What is the relation of E to A?
(c) R-M÷N (d) None
(a) Sister (b) Daughter
(c) Niece (d) Daughter
5. P and Q are brothers. R and S are sister. P’s son
is S’s brother. How is Q related to R?
ANSWER KEYS
(a) Uncle (b) Brother
(c) Father (d) Grandfather 1. (a) 2. (d) 3. (d) 4. (c)
6. A is B’s daughter. B is C’s mother. D is C’s brother. 5. (a) 6. (b) 7. (c) 8. (d)
How is D is related to A?
9. (d) 10. (b) 11. (b) 12. (a)
(a) Father (b) Brother
(c) Son (d) Grandfather
7. A and B are brothers. E is the daughter of F. F is
the wife of B. What is the relation of E to A?
(a) Sister (b) Daughter
(c) Niece (d) Cousin
8. X and Y are the children of A. A is the father of
X but Y is not his son. How is Y related to A?
(a) Sister (b) Brother
(c) Son (d) Daughter
9. Pointing to man in a photograph, a woman
said “the father of his brother is the only son
of my grandfather”, how is the woman related
to the man in the photograph?
(a) Mother (b) Daughter
(c) Aunty (d) Sister

BLOOD REALATION 247


CHAPTER-13
SYLLOGISM

INTRODUCTION
Syllogism is a ‘Greek’ word that means inference or deduction. As such inferences are based on logic, then
these inferences are called logical deduction. These deductions are based on propositions (premise).
Different types of questions covered in this chapter are as follows:
 Two Statements and Two Conclusions
‘Syllogism’ checks basic aptitude and ability of a candidate to derive inferences from given statements
using step by step methods of solving problems.
Proposition
Let us consider the following sentences:

In all the sentences mentioned above, a relation is established between subject and predicate with
the help of quantifi er and copula.
Now, we can de fi ne proposition as under:

248 BUSINESS MATHEMATICS, LOGICAL REASONING & STATISTICS (Paper 3) [CA Foundation]
A proposition or premise is grammatical sentence comprising of four components.
• Quantifier • Subject • Copula • Predicate
Components of Proposition
• Quantifier – The words ‘All’ ‘No’ and ‘Some’ are called quanti fi ers as they specify a quantity.
Keep in mind that ‘All’ and ‘No’ are universal quanti fi ers because they refer to each and every object
of a certain set.
‘Some’ is a particular quanti fi er as it refers to atleast one existing object in a certain set.
• Subject – Subject is the part of the sentence something is said about. It is denoted by S.
• Copula – It is that part of a proposition that denotes the relation between subject and predicate.
• Predicate – It is that part of a proposition which is affi rmed detail about that subject.
Classification of Proposition
A proposition can mainly be divided into three categories

(i) Categorical Proposition: In categorical proposition, there exists a relationship between the subject
and the predicate without any condition. It means predicate is either affi rmation or denial of the
subject unconditionally.
(ii) Hypothetical Proposition: In a hypothetical proposition, relationship between subject and predicate is
asserted conditionally.
(iii) Disjunctive Proposition: In a disjunctive proposition, the assertion is of alteration.
Keeping in view with the existing pattern of Syllogism in competitive examinations, we are concerned
only with the categorical type of proposition.
Venn Diagram Representation of Two Propositions
Types of Venn diagram can be understood by the following diagram:

SYLLOGISM 249
From the above diagram, following things are very much clear:
(i) Universal propositions, Either
(a) completely include the subject (A-type)
or
(b) completely exclude the subject (E-type)
(ii) Particular propositions, Either
(a) partly include the subject (I-type)
or
(b) partly exclude the subject (O-type)
Now, we can summarize the four standard types of propositions (premises) as below:
Format Type
All S are P (Universal Affirmative) A
No S is P (Universal Negative) E
Some S are P (Particular Affirmative) I
Some S are not P (Particular Negative) O
Venn Diagram Representation
(i) A-Type (All S are P)
(ii) E-Type (No S is P)
(iii) E Type (No S is P)
(iv) O Type (Some S are not P)
Hidden Propositions
The type of propositions we have discussed earlier are of standard nature but there are propositions
which do not appear in standard format and yet can be classified under any of the four types.
Let us now discuss the type of such propositions.
I. A-Type Propositions
(i) All positive propositions beginning with ‘every’ and ‘any’ are A type propositions.
(ii) A positive sentence with a particular person as its subject is always an A-type proposition.
(iii) A sentence with a definite exception is A type.
II. E-Type Propositions
(i) All negative sentences beginning with ‘no one’, ‘not a single’ etc., are E-type propositions.
(ii) A negative sentence with a very definite exception is also of E-type proposition.
(iii) When an Introgattive sentence is used to make an assertion, this could be reduced to an E-type
proposition. example: Is there any person who can scale Mount Everest? ? Non can climb Mount
Everest.
(iv) A negative sentence with a particular person as its subject is E-type proposition
III. I-Type Propositions
(i) Positive propositions beginning with words such as ‘most’, ‘a few’ ‘mostly’, ‘generally’, ‘almost’,
‘frequently’, and ‘often’ are to be reduced to the I-type propositions.

250 BUSINESS MATHEMATICS, LOGICAL REASONING & STATISTICS (Paper 3) [CA Foundation]
(ii) Negative propositions beginning with words such as ‘few’ ‘seldom’, ‘hardly’, ‘rarely’, ‘little’ etc.
are to be reduced to the I-type propositions.
(iii) A positive sentence with an exception which is not definite, is reduced to an I-type proposition.
IV. O-Type Propositions
(i) All negative propositions beginning with words such as ‘all’, ‘every’, ‘any’, ‘each’ etc. are to be
reduced to O-type propositions.
(ii) Negative propositions with words as ‘most’, ‘a few’, ‘mostly, ‘generally’, ‘almost’, ‘frequently’ are
to be reduced to the O-type propositions.
(iii) Positive propositions with starting words such as ‘few’, ‘seldom’, ‘hardly’, ‘scarcely’, ‘rarely’, ‘little’,
etc., are to be reduced to the O-type propositions.
(iv) A negative sentence with an exception, which is not defi nite is to be reduced to the O-type
propositions.
e.g.

Exclusive Propositions
Such propositions start with ‘only’, ‘alone’, ‘none but’, ‘none else but’ etc., and they can be reduced
to either A or E or l-type.
Types of Inferences
Inferences drawn from statements can be of two types:
1. Immediate Inference: When an inference is drawn from a single statement, then that inference
is known as an immediate inference.
In the above example, a conclusion is drawn from a single statement and does not require the
second statement to be referred, hence the inference is called an immediate inference.
2. Mediate Inference In mediate inference, conclusion is drawn from two given statements.
In the above example, conclusion is drawn from the two statements or in other words, both the

SYLLOGISM 251
statements are required to draw the conclusion. Hence, the above conclusion is known as mediate
inference.
Method to Draw Immediate Inferences
There are various methods to draw immediate inferences like conversion, obversion, contraposition;
etc. Keeping in view the nature of questions asked in various competitive examinations, we are required
to study only two methods, implications and conversion.
(i) Implications (of a given proposition): Below we shall discuss the implications of all the four types
of propositions. While drawing a conclusion through implication, subject remains the subject and
predicate remains the predicate.
A-Type: All boys are blue.
From the above A-type proposition, it is very ‘clear that if all boys are blue, then some boys will defi nitely
be blue because some is a part of all. Hence, from A-type proposition, we can draw l type conclusion
(through implication).
E-Type: No cars are buses.
If no’ cars are buses, it clearly means that some cars are not buses. Hence, from E-type proposition,
O-type conclusion (through implication) can be drawn.
I-Type: Some chairs are tables.
‘From the above l-type proposition, we cannot draw any valid conclusion (through implication).
O-Type: Some A are not B.
From the above O-type proposition, we can not draw any valid inference (through implication). On
first look, it appears that if some A are not B, then conclusion that some A are B must be true but the
possibility of this conclusion being true can be over ruled with the help of following example:
Case I A = {a, b, c} and B = {d, e, f}
Case II A = {a, b, c} and B ={b, c, d}
The above two cases show the relationship between A and B given by O-type proposition. “Some
A are not B”. Now, in case I, none of the element of set A is the element of set B. Hence, conclusion
“Some A are B” cannot be valid. However, in case II, elements b and c are common to both sets A and B.
Hence, here conclusion “Some A are B” is valid. But for any conclusion to be true, it should be true for
all the cases. Hence, conclusion “Some A are B” is not a valid conclusion drawn from an O-type
proposition.
All the results derived for immediate inference through implication can be presented in the table as
below:
Types of propositions Proposition Types of Inferences Inference
A All S are P I Some S are P
E No S is P O Some S are not P
I Some S are P - -
0 Some S are not P - -
Conversion: Conversion is other way of getting immediate inferences. Unlike implication, in case of
conversion, subject becomes predicate and predicate becomes subject. Let us see

252 BUSINESS MATHEMATICS, LOGICAL REASONING & STATISTICS (Paper 3) [CA Foundation]
Clearly I gets converted into I- type.
Conversion of O – type.
O type propositions cannot be converted.
Now we can make a conversion as follows:

SYLLOGISM 253
Types of propositions Gets Converted intoA
A A
E E
I I
0 Never gets Converted
Immediate Inference Table
Types of Proposition Valid immediate Types of Immediate Method
propositions inference inference
A All S are P Some S are P I Implication
Some P are S II Conversion
E No S is P Some S are not P E Implication
No P is S E Conversion
I Some S are P No Valid Inference - Implication
Some P are S I Conversion
O Some S are not P No Valid Inference - Implication
No Valid Inference - Conversion
Venn Diagram Representation of Immediate Inferences:
Immediate inferences are drawn from each type of Propositions (A, E, I , O)
1) A type All S are P
(i) S = { a, b, c } , P = { a, b, c, d, e}

(i) S = { a, b, c} , P = { a, b, c, d, e, f}
(ii) S = { a, b, c} , P = { a, b, c}
The above cases show all the possibilities of two sets S and P showing the relationship by the proposition.
All S are P in both cases.
Some S are P.(Is true from relationship)
Some P are S (Its true)
Some P are not S is not valid because from it is case (i) but false from case (ii)
Inference All (P are S ) is not valid because its true from case (ii) and False from case (i)

254 BUSINESS MATHEMATICS, LOGICAL REASONING & STATISTICS (Paper 3) [CA Foundation]
2) E type – No S Is P

We can draw the inferences as


(i) No P is S
(ii) Some S are not P
(iii) Some P are not S
Any other inference drawn from E- type proposition is not valid.
3) I-type: Some S are P
(i) S = { a, b, c, d} , P = {c, d, e, f}

Some S are P
(ii) S = { a, b, c, d} and P = { a, b}

Set {a, b} is the part of S as well as Set P, hence some set S are P.
(iii) S = {a, b} , P = {a, b, c, d}

SYLLOGISM 255
Set {b} is the part of the set S as well as Set P, hence some S are P.
(iv) S = {a, b, c} , P = {a, b, c}

The above diagrams show the relationship between S and P from I-type relationship. From the possible
combinations, it’s clear that inference (Some P are S) is true. Inference (S are not P) is true from
combinations (i) and (ii) but is not true from combinations (iii) and (iv).
Therefore inference (Some S are not P) is not a valid inference drawn from the above proposition.
Set {a, b} is part of set S and P, hence some s are P.
4) O-Type: Some S are not P
(i) S = {a, b, c, d} , P = {c, d, e, f}

Set {a, b} is part of the set S but not Set P


Hence Some S are not P

256 BUSINESS MATHEMATICS, LOGICAL REASONING & STATISTICS (Paper 3) [CA Foundation]
(ii) S= {a, b, c} and P = {d, e, f}
Set {a, b} is the part of S but not set P Hence the above relation represented by Some S are not P
On the basis of all possible combinations showing relationship between S and P, no valid inference can
be drawn. Inference from Some P are not S) is true combination (i) and (iii) but not true for combination
(iii). Hence it is invalid inference.
Inference (Some P are not S) is true from combination (i) and (ii) but not true for combination (iii),
hence it is also invalid .
Following are the main rules for solving syllogism problems.
1) All + All = All
2) All + No = No
3) `All + Some = No conclusion
4) Some + No = Some Not
5) Some + Some = No conclusion
6) No + All = Some not (Reversed)
7) No + All = Some Not (Reversed)
8) No + Some = Some Not (Reserved)
9) No + No = No conclusion
10) Some Not/ Some not reserved + Anything = No conclusion
11) If all A are B then we can say – Some B are Not A is a possibility
12) If Some B are not A then we can say – All A are B is a possibility
13) If some A are B then we can say All A are B is a possibility. All B are A is a possibility.
14) All  Some not reserved
15) Some  All
16) No  conclusion = Any possibility is true
Implications (In case of Conclusions from Single Statement)
All  Some That means If A are B then Some B is true.
Some  Some that means if Some A are B then Some B are A is true.
No  No that means if No A is B then No B is true.
Examples:
In this type of questions two statements and two conclusions are given. Its required to check.

SYLLOGISM 257
Solution
(b) Statement I is an I-type proposition which distributes neither the subject nor the predicate.
Statement II is an A-type propositions which distributes the subject ‘Engineers’ only.
Since, the Engineers is distributed in Conclusion I without being distributed in the premises. So,
Conclusion I cannot follow. In second conclusion, where it is asked that some boys are Engineers but
from Statement I nit is clear that some boys are not students. These boys may not be Engineers.
Solution: (c)
Here, the fi rst premise is an A-type proposition and so the middle term ‘Lotus’ forming the subject is
distributed. The second premise is an E proposition and so the middle term ‘Lotus’ forming the predicate
is distributed. Since, the middle term is distributed twice, so the conclusion cannot be universal.
Solution: (a) Now, taking conclusion I, it is clear that all D’s are also C’s but taking conclusion II, we cannot say
that some D’s are not A’s because from Statement I it is clear that all D’s are A’s.
Hence, only Conclusion I follows.
The sentences are already aligned. From the above given Table, A + A = A. Hence the conclusion is of
type-A whose subject is the subject of the fi rst proposition and the predicate is the predicate of the
second proposition?
So the conclusion is All balls are Stumps.
This pair is not properly aligned because the subject of both the sentences is ‘Professors’.
Since both the sentences are of type-A, we may convert any of them. So the aligned pair is Some
readers are Professors.
All Professors are writers.
Here the conclusion will be of type - I
because I + A = I.
The conclusion is Some readers are writers.
The subject of both the sentences is the same. By the rule of IEA, we convert the
I - type statement. So the aligned pair is,
Some Sweets are Mangos.
All Mangos are Fruits
I+A=I. So the conclusion is
Some Sweets are Fruits.
By changing the order of the statements itself.
We can align the sentences. The aligned pair is
No balls are lights.
All lights are bats.
E + A = O*.
So the conclusion is,
Some bats are not balls.
Here the common term is ‘blue’ which is the predicate of both the sentences. By the rule of IEA, we
convert the I-type statement. After conversion, the given pair becomes,

258 BUSINESS MATHEMATICS, LOGICAL REASONING & STATISTICS (Paper 3) [CA Foundation]
Some blue are caps.
No clip is blue.
Now by changing the order of the statements, we can align the sentences. So the aligned pair is, No clip
is blue.
Some blue are caps.
The conclusion is of type O* since, E + I = O*. Hence the conclusion is Some caps are not clips.
We know that either some vans will be trucks or some vans will not be trucks.
Hence either (i) or (ii) is true. Such pair of statements are called complementary pairs. So in a
complementary pair, at least one of the two statements is always true. We can call a pairs a
complementary pair if i) The subject and predicate of both the sentences are the same.
ii) They are an I + O - type pair or an A + O type pair or an I + E - type pair.
Some complementary pairs are given below.
i) All birds are Pigeons.
Some birds are not Pigeons.
ii) Some Chairs are watches.
Some Chairs are not watches.
iii) Some kids are cute.
No kids are cute.
Note: The steps to be followed to do a syllogism problem by analytical method are mentioned below.
i) Align the sentences properly.
ii) Draw conclusion using the table.
iii) Check for immediate inferences.
iv) Check for complementary pair if steps ii and iii fail.

SYLLOGISM 259
7. Statement: All names are dogs.
No dogs are foxes.
CLASS WORK
Conclusions: I. All names are foxes.
II. No dogs are names.
8. Statement: All pens are dogs.
Directions (Qs. 1 - 25) : Each of the following
questions contains two statements followed by two Some pens are lights.
conclusions numbered I and II. You have to consider Conclusions: I. Some dogs are lights.
the two statements to be true, even if they seen to
II. Some lights are not dogs
be at variance at the commonly known facts. You
have to decide which of the given conclusions defi 9. Statement: Some animals are clouds.
nitely follows from the given statements. Horse is a animal.
Give answer (a) if only I follows; (b) if only Conclusions: I. Some clouds are animal.
conclusion II follows; (c) if either I or II follows; (d) II. Hen is not a cloud.
if neither I nor II follows and (e) if both I and II
follow. 10. Statement: All tables are rats.
1. Statement: Some Chairs are glasses. Some Rats are chairs.
All trees are Chairs. Conclusions: I. All rats are tables
Conclusions: I. Some trees are glasses II. Some chairs are not rats.
II. Some glasses are trees. 11. Statement: All tigers are birds.
Some birds are cows.
2. Statement: No man is a lion.
Conclusions: I. Some cows are birds.
Ram is a man.
II. Some tigers are cows.
Conclusions: I. Ram is not a lion.
12. Statement: All papers are pens.
II. All men are not Ram.
All pens are erasers.
3. Statement: All boys are Fathers.
Conclusions: I. Some erases are papers.
All Fathers are Mothers. II. Some pens are no papers.
Conclusions: I. All Fathers are boys. 13. Statement: Some trees are monkeys.
II. All boys are Mothers. Some ships are trees.
4. Statement: All pens are cups. Conclusions: I. Some Monkeys are ships.
All cups are bowls. II. Some trees are neither ships
Conclusions: I. All pens are bowls. nor monkeys.
II. All cups are pots. 14. Statement: All glasses are mirrors.
5. Statement: All students are boys. Some mirrors are Black.
Conclusions: I. All mirrors are glasses.
No boy is dull
II. Some glasses are black.
Conclusions: I. There are no girls in the class
15. Statement: Some dogs are monkeys.
II. No student is dull.
No monkey is black.
6. Statement: Some cats are kittens.
Conclusions: I. Some dogs are black.
All Rats are kittens.
II. Some monkeys are dogs.
Conclusions: I. Some cats are Rats.
II. Some Rats are cats.

260 BUSINESS MATHEMATICS, LOGICAL REASONING & STATISTICS (Paper 3) [CA Foundation]
16. Statement: All roads are poles. 25. Statement: All scientists are hard working.
No poles are Bungalows. No scientists are superstitious.
Conclusions: I. Some roads are Bungalows. Conclusions: I. No scientists are
superstitious.
II. Some Bungalows are poles.
II. All superstitious are not
17. Statement: Many actors are directors. scientists.
All Directors are dancers.
Conclusions: I. Some actors are dancers.
II. No director is an actor. 
18. Statement: Only dogs are animals.
No historian is an animal.
Conclusions: I. Some dogs are not
historians.
II. Some historians are not
dogs.
19. Statement: Some chairs are caps.
No cap is red.
Conclusions: I. Some caps are chairs.
II. No Chair is red.
20. Statement: Some cups are belts.
No belt is black.
Conclusions: I. Some cups are black.
II. Some cups are not black.
21. Statement: Some girls are fl owers.
Some fl owers are books.
Conclusions: I. Some girls are books.
II. No books are girls.
22. Statement: Some fi les are rats.
All animals are rats.
Conclusions: I. All fi les are rats.
II. Some rats are animals.
23. Statement: All cricketers are tall.
Rajesh is tall.
Conclusions: I. Rajesh is a cricketer.
II. Rajesh is not cricketer.
24. Statement: Some cats are cows.
All cows are horses.
Conclusions: I. Some horses are cats.
II. Some cats are horses.

SYLLOGISM 261
7. Statements : All planets are moons.
All moons are stars.
Conclusions : I. All moons are planets.
II. All planets are stars.
HOME WORK-1
8. Statements : All men are dogs.
Give answer All dogs are cats.
(a) if only conclusion I follows; Conclusions : I. All men are cats.
(b) if only conclusion II follows; II. All cats are men.
(c) if either I or II follows;
(d) if neither I nor II follows and
9. Statements : All tubes are handles.
(e) if both I and II follow.
All cups are handles.
Conclusions : I. All cups are tubes.
1. Statements : No women can vote.
Some women are politicians. II. Some handles are not cups.
Conclusions : I. Male politicians can vote.
II. Some politicians can vote. 10. Statements : All bags are cakes.
All lamps are cakes.
2. Statements : Some books are toys. Conclusions : I. Some lamps are bags.
No toy is red. II. No lamp is bag.

Conclusions : I. Some books are red. 11. Statements : All flowers are stems.
II. Some books are not red. All stems are roots.
Conclusions : I. All roots are flowers.
3. Statements : Many books are rocks. II. All stems are flowers.
All rocks are clips.
Conclusions : I. Some books are clips.
12. Statements : All locks are keys.
II. No rock is a book.
No key is a spoon.
Conclusions : I. No lock is a spoon.
4. Statements : Most clocks are fans.
Some fans are walls. II. No spoon is a lock.
Conclusions : I. Some walls are fans.
II. Some clocks are walls. 13. Statements : All young scientists are open-
minded.
5. Statements : No man is a donkey. No open-minded men are
Rahul is a man. superstitious.
Conclusions : I. Rahul is not a donkey. Conclusions : I. No scientist is superstitious.
II. All men are not Rahul. II. No young people are
superstitious.
6. Statements : All poles are guns.
Some boats are not poles. 14. Statements : All plants are trees.
Conclusions : I. All guns are boats. No tree is green.
II. Some boats are not guns. Conclusions : I. Some plants are green.
II. Those plants which are not
trees are green.

262 BUSINESS MATHEMATICS, LOGICAL REASONING & STATISTICS (Paper 3) [CA Foundation]
15. Statements : No magazine is cap. 23. Statements : Some men are educated.
All caps are cameras. Educated persons prefer small
Conclusions : I. No camera is magazine. families.
II. Some caps are magazines. Conclusions : I. All small families are
educated.
16. Statements : Some shirts are biscuits. II. Some men prefer small
families.
No biscuit is book.
Conclusions : I. Some shirts are books.
24. Statements : Some nurses are nuns.
II. Some books are biscuits.
Madhu is a nun.
Conclusions : I. Some nuns are nurses.
17. Statements : All puppets are dolls.
II. Some nurses are not nuns.
All dolls are toys.
Conclusions : I. Some toys are puppets.
25. Statements : All lamps are hooks.
II. All toys are puppets.
No hook is coloured.
Conclusions : I. Some lamps are coloured.
18. Statements : All apples are oranges.
II. No lamp is coloured.
Some oranges are papayas.
Conclusions : I. Some apples are papayas.
26. Statements : All windows are doors.
II. Some papayas are apples.
No door is wall.
Conclusions : I No window is wall.
19. Statements : Some players are singers.
II. No wall is door.
All singers are tall.
Conclusions : I. Some players are tall.
27. Statements : All cars are tables.
II. All players are tall.
Some children are tables.
Conclusions : I. Some cars are children.
20. Statements : All coins are crows.
II. Some children are cars.
Some crows are pens.
Conclusions : I. No pen is coin.
28. Statements : All windows are needles.
II. Some coins are pene.
Some trees are windows.
Conclusions : I. Some trees are needles.
21. Statements : All men are married.
II. Some trees are not
Some men are educated.
needles.
Conclusions : I. Some married are
educated.
29. Statements : Some dogs bark.
II. Some educated are
married. All dogs bite.
Conclusions : I. Those dogs who do not
bark, also bite.
22. Statements : Some sticks are bolts.
II. Those dogs who do not
Kite is a stick.
bark, not necessarily bite.
Conclusions : I. Some bolts are sticks.
II. Some kites are bolts.

SYLLOGISM 263
30. Statements : Some fools are intelligent. 37. Statements : All students in my class are
Some intelligent are great intelligent.
Rohit is not intelligent.
Conclusions : I. Some fools are great. Conclusions : I. Rohit is not a student of my
II. All great are intelligent. class.
II. Rohit must work hard.
31. Statements : Some papers are files.
Some files are pens. 38. Statements : All hill stations have a sun-set
point.
Conclusions : I. Some files are not pens.
X is a hill station.
II. Some pens are papers.
Conclusions : I. X has a sun-set point.
II. Places other than hill
32. Statements : Some bottles are pencils.
stations do not have sun-
Some pencils are glasses. set points.
Conclusions : I. No glass is bottle.
II. Some bottles are glasses.
ANSWER KEYS
33. Statements : Some soldiers are famous.
Some soldiers are intelligent. 1. (d) 2. (b) 3. (a) 4. (a)
Conclusions : I. Some soldiers are either 5. (a) 6. (d) 7. (b) 8. (a)
famous or intelligent.
II. Some soldiers are neither 9. (d) 10. (c) 11. (d) 12. (e)
famous nor intelligent. 13. (d) 14. (d) 15. (d) 16. (d)
17. (a) 18. (d) 19. (a) 20. (d)
34. Statements : All boys are honest.
Sachin is honest. 21. (e) 22. (a) 23. (b) 24. (a)
Conclusions : I. Sachin is a boy.
25. (b) 26. (e) 27. (d) 28. (a)
II. All honest persons are boys.
29. (a) 30. (d) 31. (d) 32. (d)
35. Statements : Lawyers married only fair girls. 33. (d) 34. (d) 35. (c) 36. (b)
Shobha is very fair.
37. (a) 38. (a)
Conclusions : I. Shobha was married to a
lawyer.
II. Shobha was not married to
a lawyer.

36. Statements : Sohan is a good sportsman.


Sportsmen are healthy.
Conclusions : I. All healthy persons are
sportsmen.
II. Sohan is healthy.

264 BUSINESS MATHEMATICS, LOGICAL REASONING & STATISTICS (Paper 3) [CA Foundation]
4. Statement: All pens are cups.
HOME WORK-2 All cups are bowls.
Conclusions: I. All pens are bowls.
1. In this questions two statements and two II. All cups are pots.
conclusions are given. Its required to check. (a) If only I follows
Statement (b) If only conclusion II follows
I: Some boys are student. (c) If either I and II follows
II: All students are Engineers. (d) If neither I nor II follows
(e) If both I and II follow
Conclusions:
5. Statement: All tables are rats.
I. All Engineers are students. Some rats are chairs.
II. Some boys are Engineers. Conclusions: I. All rats are tables.
(a) Only I follows II. Some chairs are not rats.
(b) Only II follows (a) If only I follows
(b) If only conclusion II follows
(c) Both I and II follow
(c) If either I and II follows
(d) Neither I nor II follows. (d) If neither I nor II follows
2. A conclusion drawn from two given (e) If both I and II follow
statements, mark the conclusion as 6. Statement: Some cats are kittens.
(a) if it is necessarily following the two All rats are kittens.
statements. Conclusions: I. Some cats are rats.
(b) if it is only far drawn conclusion II. Some rats are cats.
(a) If only I follows
(c) if it is conflict with the two given statements
(b) If only conclusion II follows
(d) if it is doubtful
(c) If either I and II follows
Statement P : The Sum of angles of a triangle ABC is (d) If neither I nor II follows
1800 (e) If both I and II follow
Q : Angle A in the triangle ABC = 900 7. Statement: Some chairs are caps.
No cap is red.
Conclusion : Angle B is 450
Conclusions: I. Some caps are chairs.
3. Given below are two statements, P and Q II. No chair is red.
followed by a conclusion. marks the conclusion (a) If only I follows
as: (b) If only conclusion II follows
(a) If Statements P supports the conclusion. (c) If either I and II follows
(b) If statements Q supports the conclusion. (d) If neither I nor II follows
(c) If Statement P and Q together support the (e) If both I and II follow
conclusion
(d) If neither statement support the conclusion
Statement P: oxygen is essential for survival
Q: There is no atmosphere on Mars
Conclusion: No living being can survive on
Mars.

SYLLOGISM 265
8. Directions (27-29) : Each of the following
question contains two statements followed by
two conclusions number I and II. You have to
decide which of the given conclusions
definitely follows from the given statements. ANSWER KEYS
Statements:
1. Some phones are watches 1. (b) 2. (b) 3. (c) 4. (a)
2. All watches are guns
5. (d) 6. (d) 7. (a) 8. (b)
Conclusions :
I. All guns are watches 9. (c) 10. (a)
II. Some guns are phones.
(a) Only conclusion I follows
(b) Only conclusion II follows
(c) Neither I nor II follows
(d) Either I or II follows
9. Statements:
1. Some books are pens
2. No pen is pencil
Conclusions :
I. Some books are pencil
II. No book is pencil
(a) Only conclusion I follows
(b) Only conclusion II follows
(c) Either I or II follows
(d) Neither I nor II follows
10. Statements:
1. Some players are singers
2. All singers are tall
Conclusions :
I. Some players are tall
II. All players are tall
(a) Only conclusion I follows
(b) Only conclusion II follows
(c) Either I or II follows
(d) Neither I nor II follows

266 BUSINESS MATHEMATICS, LOGICAL REASONING & STATISTICS (Paper 3) [CA Foundation]
CHAPTER-14
STATISTICAL DESCRIPTION OF DATA

INTRODUCTION
1. Croxton and Cowden has defined statistics in singular sense as statistical method.
2. Prof. Horane Serist has defined statistics in plural sense as statistical data.
3. P. C. Mahalanobis is an Indian statistician who has made significant contribution in the development of
statistics.
4. The real giant in the development of the theory of statistics is R. A. Fisher.
5. Kilometers- Tonnes is an example of complex statistical unit.
6. Primary data is collection of original data for the first time.
7. Secondary data is basically compilation of existing data.
8. Primary data can be collexted by Interview Method, Observation Method and Questionaire Method.
9. Data collected on religion from Census Report is secondary data.
10. Questionnaire Method can be used when informants are literates.
11. Where the field of enquiry is wide and the iformants are literate, the suitable method of collecting
primary data is Questionnaires sent through enumerators.
12. Quickest method to collect personal data is Telephone interview.
13. Best method to collect data in case of natural calamity is personal interview.
14. In case of railaccident, the appropriate method of data collection is indirect interview.
15. The term ‘Error’ in statistics refers to bias.
16. Sampling Errors are present only in sampling survey and usually decreases with increase in sample size.
17. Non-sampling errors include both bias and mistakes and may occur in both sample survey and complete
enumeration survey.
18. Marks of a student is a Discrete Variable but Weight of a student is a Continuous Variable.
19. Nationality and Reading habit of a student are attributes.

STATISTICAL DESCRIPTION OF DATA 267


CLASSIFICATION AND TABULATION
1. A table showing Height and Weight is an example of Quantitative classification.
2. A table showing population of Delhi during 1971 to 2001 is an example of Temporal classification.
3. A table showing state-wise wheat production in India is an example of Spatial classification.
4. The methods of presentation of data are textual, tabulation and diagrammatic.
5. Mutually exclusive classification is usually meant for continuous variable.
6. Contrinuous Random Variable is a real valued function on an infinite sample space and Discrete Random
Variable is a real valued function on an finite sample space.
7. Exclusive class limits are suitable for continous variable and Inclusive class limits are suitable for
discrete variable.
8. In exclusive data series the upper limit class is not included.
9. In inclusive data series the uppe limit class is included.
10. The distribution of shares is an example of the frequency distribution of a discrete variable and the
distribution of profits is an example of the frequency distribution of a continuous variable.
11. In tabulation, row designations are called Stubs and column headings are called Captions.
12. The entire upper part of the table is known as Box head.
13. The unit of measurement in tabulation is shown in Box head.
14. In tabulation source of the data, if any, is shown in the Foot Note.
15. Class width is Range/No of classes decided.
16. The midpoint of a class is obtained by adding upper and lower limits and dividing by 2.

Diagrammatic and Graphical Presentation


1. Types of Diagrams : Non-demensional diagrams : Pictograms & Cartograms
1. One Dimensional Diagrams One Dimensional Diagram is a diagram which is pre-
pared on the basis of only one dimension i.e. length. This
type of diagram takes the shape of bars.
(a) Simple Bar Diagrams Simple Bar Diagram is a one dimensional diagram in
which bar is constructed to represent one value of a given
variable. The length of various bars is in the ratio of the
magnitude of the given data.
(b) Multiple Bar Diagrams Multiple Bar Diagram is one dimensional diagram in which
two or more bars adjoining each other are constructed to
represent the values of different variables or the values of
various components of the same variable.

268 BUSINESS MATHEMATICS, LOGICAL REASONING & STATISTICS (Paper 3) [CA Foundation]
(c) Sub-divided Bar Diagrams Sub-divided Bar Diagram is one dimensional diagram in
which one bar is constructed for the total value of the vari
able and this bar is sub-divided in proportion to the values
of the various components of that variable.
2. Two Dimensional Two Dimensional Diagram is a diagram which is pre
Diagrams pared on the basis of two dimensions i.e. length and
(Area Diagrams) width. As the product of length and width indicates the
area, this type of diagram is also called Area Diagram.
(a) Rectangle Diagrams Rectangle Diagram is a two dimensional diagram in
which a rectangle is prepared representing one variable
by length another variable by width and vale of given
variable by area.
(b) Sub-Divided Rectangle Sub-divided Rectangle is a two dimensional diagram in
which one rectangle is constructed for the total value of
the variable and then this rectangle is sub-divided in pro
portion to the values of the various components of that
variable.
(c) Square Diagrams Square Diagram is a two dimensional diagram in which
a square is prepared and the side of square is deter
mined on the basis of minimum square root value of the
variable.
(d) Circle Diagrams Circle Diagram is a two dimensional diagram in which a
circle is prepared and the radius of circle is determined
on the basis of minimum square root value of the variable.
(e) Pie Diagrams Pie Diagram is a circular diagram whose area is propor
tionately divided among the various components of a given
variable.
2 Types of Graphs
1. Time series Graph
(a) One Dependent One Dependent Variable Historigram is graph of time
Variable series data which is prepared to show the value of one
Historigram dependent variable over different periods of time.

STATISTICAL DESCRIPTION OF DATA 269


(b) More Than One More than one Dependent Variable Historigram is a graph
Dependent Variable of time series data which is prepared to show the value of
Historigram more than one dependent variable over different periods
of time.
(c) Mixed Graph Mixed Graph is a type of Historigram which is prepared
to show the two dependent variables with two different
units of measurement.
(d) Range Graph Range Graph is a type of historigram which is prepared
to show the range of the data between two extreme
values of a dependent variable at different points of time.
2. Frequency Distribution
Graphs
(a) Histogram The graphical representation of vertical adjacent rect
angles with class intervals on x-axis and corresponding
simple frequencies on y-axis.
(b) Frequency Polygon The figure obtained by joining the mid-points of tops of
adjacent rectangles of the histogram by straight lines. It is
used to locate mode.
(c) Frequency Curve The free hand smooth curve drawn through the points of
frequency polygon.
(d) Ogive The graphical representation of cumulative frequency dis
tribution with class intervals on x-axis and corresponding
cumulative frequencies on y-axis. It is used to located
median.
3. Historgam is an area diagram.
4. Ogive is a line diagram.
5. Median of a distribution can be obtained from Ogives which are constructed on the basis of cumula-
tive frequencies.
6. Mode of a distribution can be obtained from frequency polygon constructed on the basis of simple
frequencies.
7. In a pie diagram, the sum total of various components of a variable is taken as 360°.

270 BUSINESS MATHEMATICS, LOGICAL REASONING & STATISTICS (Paper 3) [CA Foundation]
7. Marks of a student is an example of
(a) An attribute
CLASS WORK (b) A discrete variable
(c) A continuous variable
(d) None of these.
1. Which of the following statements is false? 8. Nationality of a student is
(a) Statistics is derived from the Latin word (a) An attribute
‘Status’ (b) A continuous variable
(b) Statistics is derived from the Italian word (c) A discrete variable
‘Statista’
(d) (a) or (c).
(c) Statistics is derived from the French word 9. Drinking habit of a person is
‘Statistik’
(a) An attribute
(d) None of these.
(b) A variable
2. Statistics is defined in terms of numerical data
(c) A discrete variable
in the
(d) A continuous variable.
(a) Singular sense
10. Age of a person is
(b) Plural sense
(a) An attribute
(c) Either (a) or (b)
(b) A discrete variable
(d) Both (a) and (b). (c) A continuous variable
3. Statistics is applied in (d) A variable.
(a) Economics 11. Data collected on religion from the census
(b) Business management reports are
(c) Commerce and industry (a) Primary data
(d) All these. (b) Secondary data
4. Statistics is concerned with (c) Sample data
(a) Qualitative information (d) (a) or (b).
(b) Quantitative information 12. The data collected on the height of a group of
(c) (a) or (b) students after recording their heights with a
(d) Both (a) and (b). measuring tape are
5. An attribute is (a) Primary data
(a) A qualitative characteristic (b) Secondary data
(b) A quantitative characteristic (c) Discrete data
(c) A measurable characteristic (d) Continuous data.
(d) All these. 13. The primary data are collected by
6. Annual income of a person is (a) Interview method
(a) An attribute (b) Observation method
(b) A discrete variable
(c) Questionnaire method
(c) A continuous variable
(d) All these.
(d) (b) or (c).

STATISTICAL DESCRIPTION OF DATA 271


14. The quickest method to collect primary data is 21. The accuracy and consistency of data can be
(a) Personal interview verified by
(b) Indirect interview (a) Internal checking
(c) Telephone interview (b) External checking
(d) By observation. (c) Scrutiny
15. The best method to collect data, in case of a (d) Both (a) and (b).
natural calamity, is 22. The mode of presentation of data are
(a) Personal interview (a) Textual, tabulation and diagrammatic
(b) Indirect interview (b) Tabular, internal and external
(c) Questionm (c) Textual, tabular and internal
(d) Government sources. (d) Tabular, textual and external.
16. In case of a rail accident, the appropriate
23. The best method of presentation of data is
method of data collection is by
(a) Textual
(a) Personal interview
(b) Direct interview (b) Tabular
(c) Indirect interview (c) Diagrammatic
(d) All these. (d) (b) and (c).
17. Which method of data collection covers the 24. The most attractive method of data
widest area? presentation is
(a) Telephone interview method (a) Tabular
(b) Mailed questionnaire method (b) Textual
(c) Direct interview method (c) Diagrammatic
(d) All these. (d) (a) or (b).
18. The amount of non-responses is maximum in 25. For tabulation, ‘caption’ is
(a) Mailed questionnaire method (a) The upper part of the table
(b) Interview method (b) The lower part of the table
(c) Observation method
(c) The main part of the table
(d) All these.
(d) The upper part of a table that describes
19. Some important sources of secondary data are the column and sub-column.
(a) International and Government sources 26. ‘Stub’ of a table is the
(b) International and primary sources
(a) Left part of the table describing the
(c) Private and primary sources columns
(d) Government sources (b) Right part of the table describing the
20. Internal consistency of the collected data can columns
be checked when (c) Right part of the table describing the rows
(a) Internal data are given (d) Left part of the table describing the rows.
(b) External data are given 27. The entire upper part of a table is known as
(c) Two or more series are given (a) Caption (b) Stub
(d) A number of related series are given. (c) Box head (d) Body.

272 BUSINESS MATHEMATICS, LOGICAL REASONING & STATISTICS (Paper 3) [CA Foundation]
28. The unit of measurement in tabulation is (b) Two or more related time series when
shown in the variables are expressed in the same
(a) Box head (b) Body unit
(c) Caption (d) Stub. (c) Two or more related time series when
the variables are expressed in different
29. In tabulation source of the data, if any, is unit
shown in the
(d) Multiple variations in the time series.
(a) Footnote (b) Body
36. Multiple axis line chart is considered when
(c) Stub (d) Caption. (a) There is more than one time series
30. Which of the following statements is untrue (b) The units of the variables are different
for tabulation?
(c) (a) or (b)
(a) Statistical analysis of data requires
(d) (a) and (b).
tabulation
37. Horizontal bar diagram is used for
(b) It facilitates comparison between rows
and not columns (a) Qualitative data
(b) Data varying over time
(c) Complicated data can be presented
(c) Data varying over space
(d) Diagrammatic representation of data
requires tabulation. (d) (a) or (c).
31. Hidden trend, if any, in the data can be noticed 38. Vertical bar diagram is applicable when
in (a) The data are qualitative
(a) Textual presentation (b) The data are quantitative
(b) Tabulation (c) When the data vary over time
(c) Diagrammatic representation (d) (a) or (c).
39. Divided bar chart is considered for
(d) All these.
(a) Comparing different components of a
32. Diagrammatic representation of data is done
variable
by
(b) The relation of different components to
(a) Diagrams (b) Charts the table
(c) Pictures (d) All these. (c) (a) or (b)
33. The most accurate mode of data presentation (d) (a) and (b).
is
40. In order to compare two or more related
(a) Diagrammatic method series, we consider
(b) Tabulation (a) Multiple bar chart
(c) Textual presentation (b) Grouped bar chart
(d) None of these. (c) (a) or (b)
34. The chart that uses logarithm of the variable (d) (a) and (b).
is known as 41. Pie-diagram is used for
(a) Line chart (b) Ratio chart (a) Comparing different components and
(c) Multiple line chart their relation to the total
(d) Component line chart. (b) Representing qualitative data in a circle
35. Multiple line chart is applied for (c) Representing quantitative data in circle
(a) Showing multiple charts (d) (b) or (c).

STATISTICAL DESCRIPTION OF DATA 273


42. A frequency distribution (a) A discrete variable
(a) Arranges observations in an increasing (b) A continuous variable
order (c) An attribute
(b) Arranges observation in terms of a (d) Any of these.
number of groups 49. The LCB is
(c) Relaters to a measurable characteristic (a) An upper limit to LCL
(d) All these. (b) A lower limit to LCL
43. The frequency distribution of a continuous (c) (a) and (b)
variable is known as (d) (a) or (b).
(a) Grouped frequency distribution 50. The UCB is
(b) Simple frequency distribution (a) An upper limit to UCL
(c) (a) or (b) (b) A lower limit to LCL
(d) (a) and (b). (c) Both (a) and (b)
44. The distribution of shares is an example of the (d) (a) or (b).
frequency distribution of 51. length of a class is
(a) A discrete variable (a) The difference between the UCB and LCB
(b) A continuous variable of that class
(c) An attribute (b) The difference between the UCL and LCL
(d) of that class
(a) or (c). (c) (a) or (b)
45. The distribution of profits of a blue-chip (d) Both (a) and (b).
company relates to
52. For a particular class boundary, the less than
(a) Discrete variable cumulative frequency and more than
(b) Continuous variable cumulative frequency add up to
(c) Attributes (a) Total frequency
(d) (a) or (b). (b) Fifty per cent of the total frequency
46. Mutually exclusive classification (c) (a) or (b)
(a) Excludes both the class limits (d) None of these.
(b) Excludes the upper class limit but 53. Frequency density corresponding to a class
includes the lower class limit interval is the ratio of
(c) Includes the upper class limit but (a) Class frequency to the total frequency
excludes the upper class limit (b) Class frequency to the class length
(d) Either (b) or (c). (c) Class length to the class frequency
47. Mutually inclusive classification is usually (d) Class frequency to the cumulative
meant for frequency.
(a) A discrete variable 54. Relative frequency for a particular class
(b) A continuous variable (a) Lies between 0 and 1
(c) An attribute (b) Lies between 0 and 1, both inclusive
(d) All these. (c) Lies between –1 and 0
48. Mutually exclusive classification is usually (d) Lies between –1 to 1.
meant for

274 BUSINESS MATHEMATICS, LOGICAL REASONING & STATISTICS (Paper 3) [CA Foundation]
55. Mode of a distribution can be obtained from 64. In general the number of types of tabulation
(a) Histogram are
(b) Less than type ogives (a) two (b) three
(c) one (d) four
(c) More than type ogives
65. A table has
(d) Frequency polygon.
(a) four (b) two
56. Median of a distribution can be obtained from
(c) five (d) none parts.
(a) Frequency polygon
66. The number of errors in Statistics are
(b) Histogram
(a) one (b) two
(c) Less than type ogives
(c) three (d) four
(d) None of these.
67. The number of “Frequency distribution“ is
57. A comparison among the class frequencies is
possible only in (a) two (b) one
(a) Frequency polygon (b) Histogram (c) five (d) four
(c) Ogives (d) (a) or (b). 68. (Class frequency)/(Width of the class ) is
58. Frequency curve is a limiting form of defined as
(a) Frequency polygon (b) Histogram (a) Frequency density
(c) (a) or (b) (d) (a) and (b). (b) Frequency distribution
59. Most of the commonly used frequency curves (c) both
are (d) none
(a) Mixed
69. Tally marks determines
(b) Inverted J-shaped
(a) class width (b) class boundary
(c) U-shaped (d) Bell-shaped.
(c) class limit
60. The distribution of profits of a company
(d) class frequency
follows
70. Cumulative Frequency Distribution is a
(a) J-shaped frequency curve
(a) graph (b) frequency
(b) U-shaped frequency curve
(c) Statistical Table (d) distribution
(c) Bell-shaped frequency curve
71. To find the number of observations less than
(d) Any of these.
any given value
61. Graph is a
(a) Single frequency distribution
(a) Line diagram (b) Bar diagram
(b) Grouped frequency distribution
(c) Pie diagram (d) Pictogram
(c) Cumulative frequency distribution
62. Details are shown by
(d) None is used.
(a) Charts
72. An area diagram is
(b) Tabular presentation
(a) Histogram
(c) both
(b) Frequency Polygon
(d) none
(c) Ogive (d) none
63. The relationship between two variables are 73. When all classes have a common width
shown in
(a) Pie Chart
(a) Pictogram (b) Histogram
(b) Frequency Polygon
(c) Bar diagram (d) Line diagram
(c) both (d) none is used.

STATISTICAL DESCRIPTION OF DATA 275


74. An approximate idea of the shape of frequency 83. Classes with zero frequencies are called
curve is given by (a) nil class (b) empty class
(a) Ogive (c) class (d) none
(b) Frequency Polygon
84. For determining the class frequencies it is
(c) both (d) none necessary that these classes are
75. Ogive is a (a) mutually exclusive
(a) Line diagram (b) Bar diagram
(b) not mutually exclusive
(c) both (d) none
(c) independent
76. Unequal widths of classes in the frequency
distribution do not cause any difficulty in the (d) none
construction of 85. Most extreme values which would ever be
(a) Ogive included in a class interval are called
(b) Frequency Polygon (a) class limits (b) class interval
(c) Histogram (c) class boundaries (d) none
(d) none 86. The value exactly at the middle of a class
77. The graphical representation of a cumulative interval is called
frequency distribution is called (a) class mark (b) mid value
(a) Histogram (b) Ogive (c) both (d) none
(c) both (d) none. 87. Difference between the lower and the upper
78. The most common form of diagrammatic class boundaries is
representation of a grouped frequency (a) width (b) size
distribution is
(c) both (d) none
(a) Ogive (b) Histogram
88. In the construction of a frequency distribution,
(c) Frequency Polygon (d) none it is generally preferable to have classes of
79. Vertical bar chart may appear somewhat alike
(a) equal width (b) unequal width
(a) Histogram
(c) maximum (d) none
(b) Frequency Polygon
89. Frequency density is used in the construction
(c) both of
(d) none
(a) Histogram
80. The number of types of cumulative frequency
(b) Ogive
is
(c) Frequency Polygon
(a) one (b) two
(c) three (d) four (d) none when the classes are of unequal
width.
81. A representative value of the class interval for
the calculation of mean, standard deviation, 90. “Cumulative Frequency“ only refers to the
mean deviation etc. is (a) less-than type (b) more-than type
(a) class interval (b) class limit (c) both (d) none
(c) class mark (d) none 91. For the construction of a grouped frequency
82. The number of observations falling within a distribution
class is called (a) class boundaries (b) class limits
(a) density (b) frequency (c) both (d) none are used.
(c) both (d) none
276 BUSINESS MATHEMATICS, LOGICAL REASONING & STATISTICS (Paper 3) [CA Foundation]
92. In all Statistical calculations and diagrams (a) width (b) size
involving end points of classes (c) range (d) none
(a) class boundaries 101. In Histogram if the classes are of unequal width
(b) class value then the heights of the rectangles must be
(c) both proportional to the frequency densities.
(d) none are used. (a) true (b) false
93. Upper limit of any class is ________ from the (c) both (d) none
lower limit of the next class 102. When all classes have equal width, the heights
of the rectangles in Histogram will be
(a) same (b) different
numerically equal to the
(c) both (d) none
(a) class frequencies
94. Upper boundary of any class coincides with the (b) class boundaries
Lower boundary of the next class.
(c) both
(a) true (b) false
(d) none
(c) both (d) none.
103. Consecutive rectangles in a Histogram have no
95. Excepting the first and the last, all other class space in between
boundaries lie midway between the upper (a) true (b) false
limit of a class and the lower limit of the next
higher class. (c) both (d) none
104. Histogram emphasizes the widths of
(a) true (b) false
rectangles between the class boundaries.
(c) both (d) none
(a) false (b) true
96. The lower extreme point of a class is called (c) both (d) none
(a) lower class limit 105. To find the mode graphically
(b) lower class boundary (a) Ogive
(c) both (d) none (b) Frequency Polygon
97. For the construction of grouped frequency (c) Histogram
distribution from ungrouped data we use
(d) none may be used.
(a) class limits
106. When the width of all classes is same,
(b) class boundaries frequency polygon has not the same area as
(c) class width (d) none the Histogram.
98. When one end of a class is not specified, the (a) True (b) false
class is called (c) both (d) none
(a) closed- end class 107. For obtaining frequency polygon we join the
(b) open- end class successive points whose abscissa represent
(c) both the corresponding class frequency_____
(d) none (a) true (b) false
99. Class boundaries should be considered to be (c) both (d) none
the real limits for the class interval. 108. In representing simple frequency distributions
(a) true (b) false of a discrete variable
(c) both (d) none (a) Ogive (b) Histogram
100. Difference between the maximum & minimum (c) Frequency Polygon (d) both is useful.
value of a given data is called

STATISTICAL DESCRIPTION OF DATA 277


109. Diagrammatic representation of the 117. For overlapping class-intervals the class limit
cumulative frequency distribution is & class boundary are
(a) Frequency Polygon (b) Ogive (a) same (b) not same
(c) Histogram (d) none (c) zero (d) none
110. For the overlapping classes 0–10 , 10–20 , 20– 118. Classification is of
30 etc.the class mark of the class 0–10 is (a) four (b) Three
(a) 5 (b) 0 (c) two (d) five kinds.
(c) 10 (d) none 119. Out of 1000 persons, 25 per cent were
111. For the non-overlapping classes 0–19 , 20–39 , industrial workers and the rest were
40–59 the class mark of the class 0–19 is agricultural workers. 300 persons enjoyed
(a) 0 (b) 19 world cup matches on TV. 30 per cent of the
people who had not watched world cup
(c) 9.5 (d) none matches were industrial workers. What is the
112. Class : 0–10 10–20 20–30 30–40 40–50 number of agricultural workers who had
Frequency : 5 8 1 5 6 4 enjoyed world cup matches on TV?
For the class 20–30, cumulative frequency is (a) 260 (b) 240
(a) 20 (b) 13 (c) 230 (d) 250
(c) 15 (d) 28 120. A sample study of the people of an area
revealed that total number of women were
113. An Ogive can be prepared in _______ different
40% and the percentage of coffee drinkers
ways.
were 45 as a whole and the percentage of male
(a) 2 (b) 3 coffee drinkers was 20. What was the
(c) 4 (d) none percentage of female non-coffee drinkers?
114. The curve obtained by joining the points, (a) 10 (b) 15
whose x- coordinates are the upper limits of (c) 18 (d)
the class-intervals and y coordinates are 20
corresponding cumulative frequencies is 121. Cost of sugar in a month under the heads raw
called materials, labour, direct production and others
(a) Ogive were 12, 20, 35 and 23 units respectively. What
is the difference between the central angles
(b) Histogram
for the largest and smallest components of the
(c) Frequency Polygon cost of sugar?
(d) Frequency Curve (a) 72o (b) 48o
115. The breadth of the rectangle is equal to the (c) 56o (d) 92o
length of the class-interval in
122. The number of accidents for seven days in a
(a) Ogive (b) Histogram locality are given below :
(c) both (d) none No. of accidents: 0 1 2 3 4 5 6
116. In Histogram, the classes are taken
Frequency : 15 19 22 31 19 3 2
(a) overlapping
What is the number of cases when 3 or less
(b) non-overlapping accidents occurred?
(c) both (a) 56 (b) 6
(d) none (c) 68 (d) 87

278 BUSINESS MATHEMATICS, LOGICAL REASONING & STATISTICS (Paper 3) [CA Foundation]
123. The following data relate to the incomes of 86
persons : HOME WORK-1
Income in Rs. : 500–999 1000–1499 1500–1999 2000–2499

No. of persons : 15 28 36 7
1. When the data are classified in respect of
What is the percentage of persons earning more
successive time points, they are known
than Rs. 1500?
as_______.
(a) 50 (b) 45 (a) Chronological data
(c) 40 (d) 60 (b) Geographical data
124. The following data relate to the marks of a (c) Ordinal data (d) Cordinal data
group of students: 2. The number of accidents for seven days in a
Marks : Below 10 Below 20 Below 30 Below 40 Below 50 locality are given below :
No. of No. of accidents : 0 1 2 3 4 5 6
Frequency : 15 19 22 31 9 3 2
students : 15 38 65 84 100
What is the number of cases when 3 or less
How many students got marks more than 30? accidents occurred?
(a) 65 (b) 50 (a) 56 (b) 6
(c) 35 (d) 43 (c) 68 (d) 87
125. Find the number of observations between 250 3. The following data relate to the marks of a
and 300 from the following data : group of students:
Marks : Below 10 Below 20 Below 30 Below 40 Below 50
Value : More than 200 No. of : 15 38 65 84 100
More than 250 More than 300 students
More than 350 How many students got marks more than 30?
No. of observations : 56 (a) 65 (b) 50
38 15 (c) 35 (d) 43
0 4. The following data relate to the incomes of 86
persons :
(a) 56 (b) 23 Income in ` : 500–999 1000–14991500–1999 2000–2499
(c) 15 (d) 8 No. of persons: 15 28 36 7
What is the percentage of persons earning more
than Rs. 1500?
(a) 50 (b) 45
---0---0---
(c) 40 (d) 60
5. Bivariate Data are the data collected for
(a) Two variables.
(b) More than two variables.
(c) Two variables at the same point of time.
(d) Two variables at different points of time.
6. An Ogive can be prepared in ______ different
ways.
(a) 2 (b) 3
(c) 4 (d) 5

STATISTICAL DESCRIPTION OF DATA 279


7. Find the number of observations between 250 (c) Diagrammatic representation.
and 300 from the following data : (d) None of these.
Value : More than More than More than More than 16. Data collected on minority from the census
200 250 300 350
No. of reports are
observation : 56 38 15 0 (a) Primary data.
(a) 56 (b) 23 (b) Secondary data.
(c) 15 (d) 8 (c) Discrete data.
8. The entire upper part of a table is known as (d) Continuous data.
(a) Caption (b) Stub 17. __________ is the upper part of the table,
describing the columns and sub columns.
(c) Box head (d) Body
(a) Box head (b) Stub
9. The following data relate to the marks of a
(c) Caption (d) Body
group of students:
Marks : Below 10 Below 20 Below 30 Below 40 Below 50 18. Refer following table:
No. of Frequency distribution of weights of 16 students
students : 15 38 65 84 100 Weight in kg. No. of students
How many students got marks more than 30? (Class interval) (Frequency)
(a) 65 (b) 50 44 – 48 4
(c) 35 (d) 43 49 – 53 5
10. The unit of measurement in tabulation is 54 – 58 7
shown in Total 16
(a) Box head (b) Body. F ind Frequency density of the second class
interval.
(c) Caption (d) Stub.
(a) 0.80 (b) 0.90
11. Most extreme values which would ever be
(c) 1.00 (d) 1.10
included in a class interval are called
19. A variable is known to be _______ if it can
(a) Class limits (b) Class interval assume any value from a given interval.
(c) Class boundaries. (d) None of these (a) Discrete (b) Continuous
12. Number of petals in a flower is an example of (c) Attribute (d) Characteristic
(a) A continuous variable. 20. ________ is the entire upper part of the table
(b) A discrete variable. which includes columns and sub–column
(c) An attribute. numbers, unit(s) measurement.
(d) All of these. (a) Stub (b) Box–head
13. A Qualitative characteristic is known as (c) Body (d) Caption
(a) An attribute (b) A variable 21. _______ is the left part if the table providing
the description of the rows.
(c) A discrete variable
(a) Caption (b) Body
(d) A continuous variable
(c) Stub (d) Box head
14. Methods that are employed for the collection 22. Data collected on sex ratio from the census
of primary data – reports are ______ .
(a) Interview method. (a) Primary data
(b) Questionnaire method. (b) Secondary data
(c) Observation method. (d) All of these. (c) Discrete data
15. This method presents data with the help of a (d) Continuous data
paragraph or a number of paragraphs. 23. Refer following table:
(a) Tabular presentation. Frequency distribution of weights of 16
(b) Textual presentation. students

280 BUSINESS MATHEMATICS, LOGICAL REASONING & STATISTICS (Paper 3) [CA Foundation]
Weight in kg. No. of students 30. Pie diagram is used for
(Class interval) (Frequency) (a) Comparing different components and
44 – 48 4 their relation to the total
49 – 53 5 (b) Representing qualitative data in a circle
54 – 58 7 (c) Representing quantitative data in a circle.
Total 16 (d) (b) or (c)
Find Relative frequency for the second class 31. To find the median graphically use
interval. (a) Ogive
(a) 1/11 (b) 5/4 (b) Frequency Polygon
(c) 5/16 (d) 1/4 (c) Histogram (d) None
24. _______ may be defined as the minimum value 32. Unequal widths of classes in the frequency
and the maximum value, the class interval may distribution do not cause any difficulty in the
contain. construction of
(a) Class mark (b) Class limit (a) Ogive
(c) Both of the above (d) None (b) Frequency Polygon
25. The colour of a flower is an example of ______. (c) Both (d) None
(a) An attribute (b) A variable 33. Frequency density is used in the construction
(c) A discrete variable of
(d) A continuous variable (a) Histogram (b) Ogive
26. A quantitative characteristic is known as (c) Frequency polygon (d) None
______ . 34. The chart that was logarithm of the variable is
(a) An attribute (b) A variable known as
(c) Both of above (d) None (a) Line chart (b) Ratio chart
27. Refer following table (c) Multiple line chart
Frequency distribution of weights of 16 (d) Component line chart
students 35. The mean salary for a group of 4 male is ` 5200
Weight in kg. No. of students per month and that for a group of 6 female is `
(Class interval) (Frequency) 6800 per month. What is the combined salary?
44 – 48 4 (a) ` 6160 (b) ` 6610
49 – 53 5 (c) ` 6110 (d) None
54 – 58 7 36. A frequency distribution can be presented
graphically by a
Total 16
(a) Pie diagram (b) Histogram
Find Relative frequency for the third class
interval. (c) Pictogram (d) Line diagram.
(a) 7/16 (b) 7/4 37. The width of each of ten classes in a frequency
distribution is 2.5 and the lower class boundary
(c) 16/7 (d) None
of the lowest class is 10.6. Which one of the
28. Representation of data is done by following is the upper class boundary of the
(a) Diagrams (b) Pictures highest class?
(c) Charts (d) All these (a) 35.6 (b) 33.1
29. A statistic is described as (c) 30.6 (d) None
(a) A function of sample observation 38. Let L be the lower class boundary of a class in a
(b) A function of population units frequency distribution and m be the mid point
(c) A characteristic of a population of the class. Which one of the following is the
(d) A part of population
STATISTICAL DESCRIPTION OF DATA 281
higher class boundary of the class?

(a) (b) HOME WORK-2

(c) 2m - L (d) m - 2L 1. For frequency distribution and time series


39. Ogive is used to obtain which from the presentation is rarely used.
(a) Mean (b) Mode (a) Diagrammatic presentation
(c) Quartiles (d) All these (b) Graphic
40. In audit test statistical methods are not used (c) both Diagrammatic and Graphic
(a) True (b) False (d) More information required
(c) Both (d) None of these 2. The technician of graphic presentation is
extremely helpful in which of the following
(a) Analysing the changes at different points
of Time
(b) Analysing cause and effect relationship
ANSWER KEYS (c) Analysing proportional relationship
(d) Analysing the degree of relationship
1 (a) 11 (c) 21 (c) 31 (a) 3. Statistics Analyses:
2 (d) 12 (b) 22 (b) 32 (a) (a) Qualitative (b) Quantitative
(c) Either Qualitative or Quantitative
3 (c) 13 (a) 23 (c) 33 (a)
(d) Quantitative and Qualitative
4 (a) 14 (d) 24 (b) 34 (b) 4. _____ in the entire upper part of the table
5 (c) 15 (b) 25 (a) 35 (a) which includes columns and sub -column
numbers, unit(s) measurement
6 (a) 16 (b) 26 (b) 36 (b)
(a) Stub (b) Box-head
7 (b) 17 (c) 27 (a) 37 (a) (c) Body (d) Caption
8 (c) 18 (c) 28 (d) 38 (a) 5. The following frequency distribution
X : 12 17 24 36 45
9 (c) 19 (b) 29 (a) 39 (c)
Y : 2 5 3 8 9
10 (a) 20 (b) 30 (a) 40 (b) is classified as:
(a) Discrete distribution
(b) Continuous distribution
(c) Cumulative frequency distribution
---0---0--- (d) None of the above
6. Data are said to be _______ if the investigator
himself is responsible for the collection of the
data.
(a) Primary data
(b) Secondary Data
(c) Mixed of primary and secondary data
(d) None

282 BUSINESS MATHEMATICS, LOGICAL REASONING & STATISTICS (Paper 3) [CA Foundation]
7. A suitable graph for representing the
portioning of total into sub parts in statistics is
(a) A pictograph (b) A Pie Chart
(c) An ogive (d) Histogram
8. The number of times a particular items occurs
in a class interval is called its
(a) Mean
(b) Cumulative frequency
(c) Frequency (d) None

ANSWER KEYS

1 (a) 2 (a)
3 (d) 4 (b)
5 (a) 6 (a)
7 (b) 8 (c)

STATISTICAL DESCRIPTION OF DATA 283


CHAPTER-15
MEASURES OF CENTRAL TENDENCY AND DISPERSION

MEASURES OF CENTRAL TENDENCY


INTRODUCTION

1.0 Comparative Study of Measures of Central Tendancy


Arithmetic Median Mode Geometric Harmonic
Mean (AM) Mean (GM) Mean (HM)
1 Meaning It is obtained by It is the central value It is that value in GM of n items HM of various
dividing the sum of the variable that a series which is the nth root items of a series
of values of all divides the series is the greatest of their product the reciprocal of
items of a series into two equal parts frequency. the AM of their
in such a way that reciprocals.
half of the items lie
above this value and
the remaining half
lie below this value.

2 Symbol X Md M0 G.M. H.M.


used
3 Whether Yes No No Yes Yes
based on
All Items
of Series.
4 Can its Yes No No Yes Yes
formula be
extended to
calculated
combined
Average of
two or more
related
series ?
5 Whether it No Yes No No No
requires
arrangement

284 BUSINESS MATHEMATICS, LOGICAL REASONING & STATISTICS (Paper 3) [CA Foundation]
of data in
ascending/
descending
order ?
6 Whether Least Affected more than Affected more Affected more Affected more
affected by than AM than AM than AM than AM
Sampling
Fluctuations
7 Whether Yes No No Yes (gives more Yes (gives largest
affected by weight to small weight to small
extreme item) est item)
values.
8 Suitable Other cases Open-ended Qualitative Average Rate of For Rates and
for distribution data Increase/ Ratio involving
Decrease, Speed, Time,
Average Ratios Distance, Price
/Percentages and Quantity.
9 Can it be No Yes Yes No No
determined
graphically ?
10 Is it Indepen- No No No No No
dent of -
Choice of
Origin ?
11 Is it Indepen- No No No No No
dent of -
Choice of
Scale ?
12 Mathematical 1. Sum of Deviati- The sum of Absolute 1. The product If each value of
Property. ations from AM Deviations from of the values of the variate is
is always zero. Median is minimum. geometric mean replace by
2. The Sum of is substituted for harmonic mean,
Squared Deviati- each individual the total of reci-
ations from AM value. procals of values
is minimum. 2. The sum of the of the variate
devitions of the remains the
logarithms of the same.
original observa-
tions above or
below the logari-
thm of the
geometric mean
is equal.

MEASURES OF CENTRAL TENDENCY AND DISPERSION 285


2. Relationship between Mean, Median and Mode
1. Symmetrical Mean = Median = Mode
2. Positively Skewed Mean > Median > Mode
3. Negatively Skewed Mean < Median < Mode
4. Moderately Skewed Mode = 3 Median - 2 Mean
3. Relationship between AM, GM and HM
1. Symmetrical AM = GM = HM
2. Moderately & Asymmetrical AM > GM > HM
3. For a set of 2 observations GM2 = AM × HM
4. The sum of deviations is zero and sum of squared deviations is minimum when taken from
mean.
5. The sum of absolute deviations from median is minimum.
6. The suitable average in case of open end distribution is median.
7. Ascending/descending arrangement of data is required to compute median and not mean.
8. For a symmetrical distribution, Median = (Q1 + Q3)/2
9. GM < AM because GM gives more weight to small items and less weight to large items.
10. GM is theoretically considered to be the best average in the construction of index number since
it satisfied the time reversal test and gives weight to equal ratio of change.
11. Both GM and HM cannot be computed when there are both positive and negative values in a
series.
12. In a series of values if one value is 0, GM is 0 and HM is indeterminate.
13. Geometric Mean is the geometric mean of arithmetic mean (AM) and Harmonic Mean (HM)
and thus, G.M. = AM  HM
2 2
14. (a + b) - (a – b) = 4ab
15. If weights are multiplied/divided by a constant, weighted arithmetic mean shall remain un-
changed.

286 BUSINESS MATHEMATICS, LOGICAL REASONING & STATISTICS (Paper 3) [CA Foundation]
LIST OF FORMULAE
Individual Series Discrete Series Continuous Series
1. Arithmethic mean Direct Method : Direct Method :
Direct Method :
X fX fm
X = N X = N X = N
Short-cut Method : Short-cut Method : Short-cut Method :
d fd fd
X =A+ N X =A+ N X =A+ N ×i
2. Median
N 1 N 1 N
Size of th item Size of th item Size of th item
2 2 2
N / 2 – c. f .
Med = L + ×i
f

f1 – f 0
3. Mode Either by Inspection or Mo = L +
f1 – f0  f1 – f2 × i
Grouping method deter-
mining that value around
which most of the frequen-
cies are concentrated
Note : Empirical Mode in case of a Moderately Skewed Distribution : Mode = 3 Median – 2 Mean
4. Geometric Mean
  log X   f log X   f log m 
G. M. = AL   G.M. = AL   G.M. = AL  
 N   N   N 
5. Harmonic Mean
N N N
H. M. = H. M. = H. M. =
(1 / X ) ( f / X ) ( f / m)
6. Weighted Arithmetric Weighted Geometric Mean Weighted Harmonic Mean
Mean
W
WX  (W log X ) 
Xw  G.M. = AL   H.M. = AL  1  W    1  W 
W w  W  w  1   2 
a  b 
7. Combined Arithmetic Combined Geometric Mean
Mean

N1 X1  N 2 X 2  N1 log GM1  N 2 log GM2 


X 12  GM = AL  
N1  N 2 12  N1  N 2 

MEASURES OF CENTRAL TENDENCY AND DISPERSION 287


MEASURES OF DISPERSION

INTRODUCTION
1. Comparative Study of Measures of Dispersion
Range Inter-Quartile Mean Standard Deviation
Range (Quartile
Deviation)

1 Meaning It is the difference It is half of the It is the arithmetic It is the square root of the
between the value difference between mean of the absolute arithmetic mean of the
of largest item and upper quartile and deviations of all items squares of deviations of
the value of lowre quartile. from a measure of all items from arithmetic
smallest item. central tendancy. mean.

2 Symbol used R Q.D. M.D. s.d


3 Whether based
on all items
of Series. No No Yes Yes
4 Whether affected Yes Least affected Affected less than Affected more than M.D.
by extreme S.D.
values.
5 Whether No Yes No No
Suitable for
Open-end
distribution
6 Can its formula No No No Yes
be extended to
calculate Combi-
ned Dispersion
of two or more
related series ?
7 Whether affected Yes Yes Affected more than Affected less than M.D.
by Sampling S.D.
Fluctuations.
8 Signs of Deviati- Actual  Signs of De- Actual Signs of Deviations
ons viations are ignored are taken into considerati-
and all devations are on.
taken as positive.
9 Is it Independent Yes Yes Yes Yes
of Choice of
Origin ?
10 Is it Independent No No No No
of Choice of

288 BUSINESS MATHEMATICS, LOGICAL REASONING & STATISTICS (Paper 3) [CA Foundation]
Scale ?
11 Mathematical The Sum of Absolute The Sum of Squared
Property. Deviations from Deviations from AM is
Median is minimum. minimum.
12 In a Symmetri- 1. Highest Value (a) Q1 & Q3 are M.D. = 4/5 s.d 1. S.D. is never less than
cal Distribution = Mean + Half equidistant from M.D./Q.D.
of Range the median. Hence, 2. Mean ± 1 s.dcovers
Lowest Value Med. – Q1 68.27% of items
Mean – = Q3 – Med. 3. Mean ± 2 s.dcovers
Half of Range 95.45% of items
(b) Q.D. = 2/3 s.d 4. Mean ± 3 s.dcovers
99.73% of items
2. Absolute Measures of Dispersion (Range, Q.D., M.D., S.D.) are Independent of Origin and
not of Scale of Measurement. These are independent of units of measurement.
3. Relative Measures of Dispersion (Coefficient of Range, Coefficient of M.D., Coefficient of
Variation) are Independent of Scale and not of Origin.
4. If all observations are same, any absoulte and relative measure of dispersion will be zero.
5. Standard Deviation of one observation is zero.
6. Standard deviation of two obervations = (Largest Item – Smallest Item)/2
7. SD > MD about Mean.
8. Root-Mean square deviation is minimum because deviations are taken from arithmetic
mean.

n2 1 n 1
9. S.D. of n natural number is and Arithmetic Mean of natural number is
12 2

10. S.D. and Variance (  2 ) can never be negative because the S.D. is the square root of the
averages of squares of deviation of the values from their means.
11. The suitable measure of dispersion in case of open end distribution is Quartile Deviation.
12. (a) For a normal distribution, QD = 2/3 s.d and MD = 4/5 s.d
(b) For a symmetrical distribution, Highest Value = Mean + half of range
Lowest Value = Mean – half of range
13. S.D. is computed from A.M., and M.D. is usually computed from median.
14. M.D. about median < M. D. about mean or any other value or MD is minimum when
deviations are taken about median.

MEASURES OF CENTRAL TENDENCY AND DISPERSION 289


7. In case of an even number of observations
which of the following is median?
CLASS WORK (a) Any of the two middle-most value
(b) The simple average of these two middle
1. Measures of central tendency for a given set values
of observations measures (c) The weighted average of these two
(a) The scatterness of the observations middle values
(b) The central location of the observations (d) Any of these
(c) Both (a) and (b) 8. The most commonly used measure of central
tendency is
(d) None of these.
(a) AM (b) Median
2. While computing the AM from a grouped
(c) Mode
frequency distribution, we assume that
(d) Both GM and HM.
(a) The classes are of equal length
9. Which one of the following is not uniquely
(b) The classes have equal frequency
defined?
(c) All the values of a class are equal to the
(a) Mean (b) Median
mid-value of that class
(c) Mode
(d) None of these.
(d) All of these measures
3. Which of the following statements is wrong?
10. Which of the following measure of the central
(a) Mean is rigidly defined
tendency is difficult to compute?
(b) Mean is not affected due to sampling
(a) Mean (b) Median
fluctuations
(c) Mean has some mathematical properties (c) Mode (d) GM
(d) All these 11. Which measure(s) of central tendency is(are)
considered for finding the average rates?
4. Which of the following statements is true?
(a) AM (b) GM
(a) Usually mean is the best measure of
central tendency (c) HM
(d) Both (b) and (c)
(b) Usually median is the best measure of
central tendency 12. For a moderately skewed distribution, which
of he following relationship holds?
(c) Usually mode is the best measure of
central tendency (a) Mean – Mode = 3 (Mean – Median)
(d) Normally, GM is the best measure of (b) Median – Mode = 3 (Mean – Median)
central tendency (c) Mean – Median = 3 (Mean – Mode)
5. For open-end classification, which of the (d) Mean – Median = 3 (Median – Mode)
following is the best measure of central
13. Weighted averages are considered when
tendency?
(a) AM (b) GM (a) The data are not classified
(c) Median (d) Mode (b) The data are put in the form of grouped
frequency distribution
6. The presence of extreme observations does
not affect (c) All the observations are not of equal
importance
(a) AM (b) Median
(d) Both (a) and (c).
(c) Mode (d) Any of these.

290 BUSINESS MATHEMATICS, LOGICAL REASONING & STATISTICS (Paper 3) [CA Foundation]
14. Which of the following results hold for a set of (b) Two distributions may have the identical
distinct positive observations? measures of central tendency but
(a) AM  GM  HM different measures of dispersion.

(b) HM  GM  AM (c) Two distributions may have the different


measures of central tendency but
(c) AM > GM > HM identical measures of dispersion.
(d) GM > AM > HM (d) All the statements (a), (b) and (c).
15. When a firm registers both profits and losses, 22. Dispersion measures
which of the following measure of central
(a) The scatterness of a set of observations
tendency cannot be considered?
(b) The concentration of a set of observations
(a) AM (b) GM
(c) Both (a) and (b)
(c) Median (d) Mode
(d) Neither (a) and (b).
16. Quartiles are the values dividing a given set of
observations into 23. When it comes to comparing two or more
distributions we consider
(a) Two equal parts
(a) Absolute measures of dispersion
(b) Four equal parts
(b) Relative measures of dispersion
(c) Five equal parts
(c) Both (a) and (b)
(d) None of these
(d) Either (a) or (b).
17. Quartiles can be determined graphically using
24. Which one is easier to compute?
(a) Histogram
(a) Relative measures of dispersion
(b) Frequency Polygon (b) Absolute measures of dispersion
(c) Ogive (c) Both (a) and (b)
(d) Pie chart. (d) Range
18. Which of the following measure(s) possesses 25. Which one is an absolute measure of
(possess) mathematical properties? dispersion?
(a) AM (b) GM (a) Range
(c) HM (d) All of these (b) Mean Deviation
19. Which of the following measure(s) satisfies (c) Standard Deviation
(satisfy) a linear relationship between two (d) All these measures
variables? 26. Which measure of dispersion is most usefull?
(a) Mean (b) Median (a) Standard deviation
(c) Mode (d) All of these (b) Quartile deviation
20. Which of he following measures of central (c) Mean deviation
tendency is based on only fifty percent of the (d) Range
central values?
27. Which measures of dispersions is not affected
(a) Mean (b) Median by the presence of extreme observations?
(c) Mode (d) Both (a) and (b) (a) Range
21. Which of the following statements is correct? (b) Mean deviation
(a) Two distributions may have identical (c) Standard deviation
measures of central tendency and (d) Quartile deviation
dispersion.

MEASURES OF CENTRAL TENDENCY AND DISPERSION 291


28. Which measure of dispersion is based on the (a) Positive (b) Negative
absolute deviations only? (c) Zero (d) (a) or (c)
(a) Standard deviation 35. Which measure of dispersion is considered for
(b) Mean deviation finding a pooled measure of dispersion after
(c) Quartile deviation combining several groups?
(d) Range (a) Mean deviation
29. Which measure is based on only the central (b) Standard deviation
fifty percent of the observations? (c) Quartile deviation
(a) Standard deviation (d) Any of these
(b) Quartile deviation 36. A shift of origin has no impact on
(c) Mean deviation (a) Range
(d) All these measures (b) Mean deviation
30. Which measure of dispersion is based on all (c) Standard deviation
the observations? (d) All these and quartile deviation.
(a) Mean deviation 37. The range of 15, 12, 10, 9, 17, 20 is
(b) Standard deviation (a) 5 (b) 12
(c) Quartile deviation (c) 13 (d) 11.
(d) (a) and (b) but not (c) 38. The standard deviation of 10, 16, 10, 16, 10, 10,
31. The appropriate measure of dispersion for 16, 16 is
open-end classification is (a) 4 `(b) 6
(a) Standard deviation (c) 3 (d) 0.
(b) Mean deviation 39. For any two numbers SD is always
(c) Quartile deviation (a) Twice the range
(d) All these measures. (b) Half of the range
32. The most commonly used measure of (c) Square of the range
dispersion is
(d) None of these.
(a) Range
40. If all the observations are increased by 10, then
(b) Standard deviation
(a) SD would be increased by 10
(c) Coefficient of variation
(b) Mean deviation would be increased by
(d) Quartile deviation. 10
33. Which measure of dispersion has some (c) Quartile deviation would be increased by
desirable mathematical properties? 10
(a) Standard deviation (d) All these three remain unchanged.
(b) Mean deviation 41. If all the observations are multiplied by 2, then
(c) Quartile deviation (a) New SD would be also multiplied by 2
(d) All these measures (b) New SD would be half of the previous SD
34. If the profits of a company remains the same (c) New SD would be increased by 2
for the last ten months, then the standard
deviation of profits for these ten months (d) New SD would be decreased by 2.
would be ?
292 BUSINESS MATHEMATICS, LOGICAL REASONING & STATISTICS (Paper 3) [CA Foundation]
42. The number of measures of central tendency 54. G.M is less than H.M
is (a) true (b) false
(a) two (b) three (c) both (d) none
(c) four (d) five 55. The value of the middlemost item when they
43. The words “mean” or “average” only refer to are arranged in order of magnitude is called
(a) A.M (b) G.M (a) standard deviation (b) mean
(c) H.M (d) none (c) mode (d) median
44. —————— is the most stable of all the 56. Median is unaffected by extreme values.
measures of central tendency. (a) true (b) false
(a) G.M (b) H.M (c) both (d) none
(c) A.M (d) none. 57. Median of 2, 5, 8, 4, 9, 6, 71 is
45. Mean is of ———— types. (a) 9 (b) 8
(a) 3 (b) 4 (c) 5 (d) 6
(c) 8 (d) 5 58. The value which occurs with the maximum
46. Weighted A.M is related to frequency is called
(a) G.M (b) frequency (a) median (b) mode
(c) H.M (d) none. (c) mean (d) none
47. Frequencies are also called weights. 59. In the formula Mode = L1 + (d1 x c)/ (d1 + d2)
(a) True (b) false d1 is the difference of frequencies in the modal
(c) both (d) none class & the —————— class.
48. The algebraic sum of deviations of (a) preceding (b) following
observations from their A.M is (c) both (d) none
(a) 2 (b) -1 60. In the formula Mode = L1 + (d1 x c)/ (d1 + d2)
(c) 1 (d) 0
d 2 is the difference of frequencies in the
49. G.M of a set of n observations is the ————
modal class & the ———————— class.
root of their product.
(a) preceding (b) succeeding
(a) n/2 th (b) (n+1)th
(c) both (d) none
(c) nth (d) (n -1)th
61. In formula of median for grouped frequency
50. The algebraic sum of deviations of 8, 1, 6 from
distribution N is
the A.M viz.5 is
(a) total frequency
(a) -1 (b) 0
(b) frequency density
(c) 1 (d) none
(c) frequency
51. G.M of 8, 4,2 is
(d)cumulative frequency
(a) 4 (b) 2
62. When all observations occur with equal
(c) 8 (d) none
frequency ————— does not exit.
52. ——————— is the reciprocal of the A.M of
reciprocal of observations. (a) median (b) mode
(a) H.M (b) G.M (c) mean (d) none
(c) both (d) none 63. —————— of a set of observations is
53. A.M is never less than G.M defined to be their sum, divided by the no. of
(a) True (b) false observations.
(c) both (d) (a) H.M (b) G.M
none (c) A.M (d) none

MEASURES OF CENTRAL TENDENCY AND DISPERSION 293


64. Simple average is sometimes called 72. Frequencies are generally used as
(a) weighted average (a) range (b) weights
(b) unweighted average (c) mean (d) none
(c) relative average (d) none 73. The total of a set of observations is equal to
the product of their number of observations
65. When a frequency distribution is given, the
and the
frequencies themselves treated as weights.
(a) A.M (b) G.M
(a) True (b) false
(c) H.M (d) none
(c) both (d) none
74. The total of the deviations of a set of
66. Each value is considered only once for observations from their A.M is always
(a) simple average (a) 0 (b) 1
(b) weighted average (c) -1 (d) none
(c) both 75. Deviation may be positive or negative or zero
(d) none (a) true (b) false
67. Each value is considered as many times as it (c) both (d) none
occurs for 76. The sum of the squares of deviations of a set
(a) simple average of observations has the smallest value, when
the deviations are taken from their
(b) weighted average
(a) A.M (b) H.M
(c) both
(c) G.M (d) none
(d) none
77. For a given set of positive observations H.M is
68. Multiplying the values of the variable by the less than G.M
corresponding weights and then dividing the
(a) true (b) false
sum of products by the sum of weights is
(c) both (d) none
(a) simple average
78. For a given set of positive observations A.M is
(b) weighted average greater than G.M
(c) both (a) true (b) false
(d) none (c) both (d) none
69. Simple & weighted average are equal only 79. Calculation of G.M is more difficult than
when all the weights are equal. (a) A.M (b) H.M
(a) True (b) false (c) median (d) none
(c) both (d) none 80. ————— has a limited use
70. The word “average “ used in “simple average“ (a) A.M (b) G.M
and “weighted average“ generally refers to (c) H.M (d) none
(a) median (b) mode 81. A.M of 8, 1, 6 is
(c) A.M , G.M or H.M (d) none (a) 5 (b) 6
71. ———— average is obtained on dividing the (c) 4 (d) none
total of a set of observations by their number 82. ————— can be calculated from a frequency
(a) simple (b) weighted distribution with open end intervals
(c) both (d) none (a) Median (b) Mean
(c) Mode (d) none

294 BUSINESS MATHEMATICS, LOGICAL REASONING & STATISTICS (Paper 3) [CA Foundation]
83. The values of all items are taken into 91. H.M is defined when no observation is
consideration in the calculation of (a) 3 (b) 2
(a) median (b) mean (c) 1 (d) 0
(c) mode (d) none 92. When all values occur with equal frequency,
84. The values of extreme items do not influence there is no
the average in case of (a) mode (b) mean
(a) median (b) mean (c) median (d) none
93. ———— cannot be treated algebraically
(c) mode (d) none
(a) mode (b) mean
85. In a distribution with a single peak and
(c) median (d) none
moderate skewness to the right, it is closer to
the concentration of the distribution in case 94. For the calculation of ————— , the data
of must be arranged in the form of a frequency
distribution.
(a) mean (b) median
(a) median (b) mode
(c) both (d) none (c) mean (d) none
86. If the variables x & z are so related that z = ax + 95. ———— is equal to the value corresponding
b for each x = xi where a & b are constants, to cumulative frequency
then z = a x + b (a) mode (b) mean
(c) median (d) none
(a) true (b) false
96. ————— is the value of the variable
(c) both (d) none corresponding to the highest frequency
87. G.M is defined only when (a) mode (b) mean
(a) all observations have the same sign and (c) median (d) none
none is zero 97. The class in which mode belongs is known as
(b) all observations have the different sign (a) median class (b) mean class
and none is zero (c) modal class (d) none
(c) all observations have the same sign and 98. The formula of mode is applicable if classes
one is zero are of ————— width.
(d) all observations have the different sign (a) equal (b) unequal
and one is zero (c) both (d) none
88. ———— is useful in averaging ratios, rates and 99. For calculation of ——— we have to construct
percentages. cumulative frequency distribution
(a) A.M (b) G.M (a) mode (b) median
(c) H.M (d) none (c) mean (d) none
100. For calculation of ——— we have to construct
89. G.M is useful in construction of index number.
a grouped frequency distribution
(a) true (b) false (a) median (b) mode
(c) both (d) none (c) mean (d) none
90. More laborious numerical calculations involves 101. Relation between mean, median & mode is
in G.M than A.M (a) mean - mode = 2 (mean - median)
(a) True (b) false (b) mean - median = 3 (mean - mode)
(c) both (d) none (c) mean - median = 2 (mean - mode)
(d) mean - mode = 3 (mean - median)

MEASURES OF CENTRAL TENDENCY AND DISPERSION 295


101. When the distribution is symmetrical, mean, 111. ——————— quartile is known as Upper
median and mode quartile
(a) coincide (a) First (b) Second
(b) do not coincide (c) Third (d) none
(c) both (d) none 112. Lower quartile is
102. Mean, median & mode are equal for the (a) first quartile (b) second quartile
(a) Binomial distribution (c) upper quartile (d) none
(b) Normal distribution 113. The number of observations smaller than
(c) both lower quartile is the same as the no. lying
(d) none between lower and middle quartile.
103. In most frequency distributions, it has been (a) true (b) false
observed that the three measures of central (c) both (d) none
tendency viz. mean, median & mode, obey the 114. ———— are used for measuring central
approximate relation, provided the tendency, dispersion & skewness.
distribution is
(a) Median (b) Deciles
(a) very skew (b) not very skew (c) Percentiles (d) Quartiles.
(c) both (d) none 115. The second quartile is known as
104. —————— divides the total number of (a) median (b) lower quartile
observations into two equal parts.
(c) upper quartile (d) none
(a) mode (b) mean
116. The lower & upper quartiles are used to define
(c) median (d) none
(a) standard deviation
105. Measures which are used to divide or partition
(b) quartile deviation
the observations into a fixed number of parts
are collectively known as (c) both
(d) none
(a) partition values (b) quartiles
117. Three quartiles are used in
(c) both (d) none
(a) Pearson’s formula
106. The middle most value of a set of observations
is (b) Bowley’s formula
(a) median (b) mode (c) both
(d) none
(c) mean (d) none
118. Less than First quartile, the frequency is equal
107. The number of observations smaller than —
to
—— is the same as the number larger than it.
(a) N /4 (b) 3N /4
(a) median (b) mode
(c) N /2 (d) none
(c) mean (d) none
119. Between first & second quartile, the frequency
109. ———— is the value of the variable
is equal to
corresponding to cumulative frequency N /2
(a) 3N/4 (b) N /2
(a) mode (b) mean
(c) N /4 (d) none
(c) median (d) none
120. Between second & upper quartile, the
110. ——————— divide the total no. frequency is equal to
observations into 4 equal parts.
(a) 3N/4 (b) N /4
(a) median (b) deciles
(c) N /2 (d) none
(c) quartiles (d) percentiles

296 BUSINESS MATHEMATICS, LOGICAL REASONING & STATISTICS (Paper 3) [CA Foundation]
121. Above upper quartile, the frequency is equal 132. 10th percentile is equal to
to (a) 1st decile (b) 10 th decile
(a) N /4 (b) N /2 (c) 9th decile (d) none
(c) 3N /4 (d) none 133. 50th percentile is known as
(a) 50th decile (b) 50th quartile
122. Corresponding to first quartile, the cumulative (c) mode (d) median
frequency is 134. 20th percentile is equal to
(a) N /2 (b) N/4 (a) 19th decile (b) 20th decile
(c) 3N /4 (d) none (c) 2nd decile (d) none
123. Corresponding to second quartile, the
135. (3rd quartile —— 1st quartile)/2 is
cumulative frequency is
(a) skewness (b) median
(a) N/4 (b) 2 N/4
(c) quartile deviation (d) none
(c) 3N/4 (d) none
124. Corresponding to upper quartile, the 136. 1st percentile is less than 2nd percentile.
cumulative frequency is (a) true (b) false
(a) 3N/4 (b) N/4 (c) both (d) none
(c) 2N/4 (d) none 137. 25th percentile is equal to
125. The values which divide the total number of (a) 1st quartile (b) 25th quartile
observations into 10 equal parts are (c) 24th quartile (d) none
(a) quartiles (b) percentiles 138. 90th percentile is equal to
(c) deciles (d) none (a) 9th quartile (b) 90th decile
126. There are ————— deciles. (c) 9th decile (d) none
(a) 7 (b) 8 139. 1st decile is greater than 2nd decile
(c) 9 (d) 10
(a) True (b) false
127. Corresponding to first decile, the cumulative
(c) both (d) none
frequency is
140. Quartile deviation is a measure of dispersion.
(a) N/10 (b) 2N/10
(c) 9N/10 (d) none (a) true (b) false
(c) both (d) none
128. Fifth decile is equal to
(a) mode (b) median 141. To define quartile deviation we use
(c) mean (d) none (a) lower & middle quartiles
129. The values which divide the total number of (b) lower & upper quartiles
observations into 100 equal parts is (c) upper & middle quartiles
(a) percentiles (b) quartiles (d) none
(c) deciles (d) none 142. Calculation of quartiles, deciles ,percentiles
130. Corresponding to second decile, the may be obtained graphically from
cumulative frequency is (a) Frequency Polygon (b) Histogram
(a) N/10 (b) 2N/10 (c) Ogive (d) none
(c) 5N/10 (d) none 143. 7th decile is the abscissa of that point on the
131. There are ———— percentiles. Ogive whose ordinate is
(a) 100 (b) 98 (a) 7N/10 (b) 8N /10
(c) 97 (d) 99 (c) 6N /10 (d) none

MEASURES OF CENTRAL TENDENCY AND DISPERSION 297


144. Rank of median is 155. For grouped frequency distribution ————
(a) (n+ 1)/2 (b) (n+ 1)/4 —— is equal to the value corresponding to
(c) 3(n + 1)/4 (d) none cumulative frequency N /4
145. Rank of 1st quartile is (a) median (b) 1st quartile
(a) (n+ 1)/2 (b) ( n+ 1)/4 (c) 3rd quartile (d) none
(c) 3(n + 1)/4 (d) none
156. For grouped frequency distribution ————
146. Rank of 3rd quartile is
—— is equal to the value corresponding to
(a) 3(n+ 1)/4 (b) ( n+ 1)/4
cumulative frequency 3N /4
(c) (n + 1)/2 (d) none
147. Rank of k th decile is (a) median (b) 1st quartile
(a) (n+ 1)/2 (b) ( n+ 1)/4 (c) 3rd quartile (d) none
(c) (n + 1)/10 (d) k( n +1)/10 157. For grouped frequency distribution ————
148. Rank of k th percentile is —— is equal to the value corresponding to
(a) (n+ 1)/100 (b) k( n+ 1)/10 cumulative frequency kN/10
(c) k(n + 1)/100 (d) none (a) median (b) kth percentile
149. —————— is equal to value corresponding
(c) kth decile (d) none
to cumulative frequency (N + 1)/2 from simple
frequency distribution 158. For grouped frequency distribution ————
(a) Median (b) 1st quartile —— is equal to the value corresponding to
(c) 3rd quartile (d) 4th quartile cumulative frequency kN /100
150. ———— is equal to the value corresponding (a) kth quartile (b) kth percentile
to cumulative frequency (N + 1)/4 from simple (c) kth decile (d) none
frequency distribution
159. In Ogive, abscissa corresponding to ordinate
(a) Median (b) 1st quartile
N/2 is
(c) 3rd quartile (d) 1st decile
151. ———— is equal to the value corresponding (a) median (b) 1st quartile
to cumulative frequency 3 (N + 1)/4 from simple (c) 3rd quartile (d) none
frequency distribution 160. In Ogive, abscissa corresponding to ordinate
(a) Median (b) 1st quartile N/4 is
(c) 3rd quartile (d) 1st decile
(a) median (b) 1st quartile
152. ———— is equal to the value corresponding
to cumulative frequency k (N + 1)/10 from (c) 3rd quartile (d) none
simple frequency distribution 161. In Ogive, abscissa corresponding to ordinate
(a) Median (b) kth decile 3N/4 is
(c) kth percentile (d) none (a) median (b) 3rd quartile
153. ———— is equal to the value corresponding (c) 1st quartile (d) none
to cumulative frequency k(N + 1)/100 from 162. In Ogive, abscissa corresponding to ordinate
simple frequency distribution —————— is kth decile.
(a) kth decile (b) kth percentile
(a) kN/10 (b) kN/100
(c) both (d) none
(c) kN/50 (d) none
154. For grouped frequency distribution ————
163. In Ogive , abscissa corresponding to ordinate
—— is equal to the value corresponding to
—————— is kth percentile.
cumulative frequency N /2
(a) median (b) 1st quartile (a) kN/10 (b) kN/100
(c) kN/50 (d) none
(c) 3rd quartile (d) none

298 BUSINESS MATHEMATICS, LOGICAL REASONING & STATISTICS (Paper 3) [CA Foundation]
164. The average of a series of overlapping 172. Ninth Decile lies in the class interval of the
averages, each of which is based on a certain item
number of item within a series is known as (a) n/9 (b) 9n/10
(a) moving average (c) 9n/20 (d) none item.
(b) weighted average 173. Ninety Ninth Percentile lies in the class
(c) simple average interval of the item
(d) none (a) 99n/100 (b) 99n/10
165. ————— averages is used for smoothening (c) 99n/200 (d) none item.
a time series.
174. ————— is the value of the variable at which
(a) moving average
the concentration of observation is the
(b) weighted average densest.
(c) simple average
(a) mean (b) median
(d) none
(c) mode (d) none
166. Pooled Mean is also called
175. Height in cms:
(a) Mean
(b) Geometric Mean 60–62 63–65 66–68 69–71 72–74
(c) Grouped Mean No. of
(d) none students: 15 118 142 127 18
167. Half of the numbers in an ordered set have Modal group is
values less than the ——————— and half (a) 66–68 (b) 69–71
will have values greater than the ————— (c) 63–65 (d) none
—.
176. A distribution is said to be symmetrical when
(a) mean, median
the frequency rises & falls from the highest
(b) median, median
value in the ———————— proportion.
(c) mode, mean
(a) unequal (b) equal
(d) none.
(c) both (d) none
168. The median of 27, 30, 26, 44, 42, 51, 37 is
177. ——————— always lies in between the
(a) 30 (b) 42
arithmetic mean & mode.
(c) 44 (d) 37
(a) G.M (b) H.M
169. For an even number of values the median is
the (c) Median (d) none
(a) average of two middle values 178. Logarithm of G.M is the ——————— of
(b) middle value logarithms of the different values.
(c) both (a) weighted mean (b) simple mean
(d) none (c) both (d) none
170. In the case of a continuous frequency 179. —————— is not much affected by
distribution, the size of the —————— item fluctuations of sampling.
indicates class interval in which the median (a) A.M (b) G.M
lies. (c) H.M (d) none
(a) (n-1)/2th (b) (n+ 1)/2th 180. The data 1, 2, 4, 8, 16 are in
(c) n/2th (d) none
(a) Arithmetic progression
171. The deviations from median are —————
—— if negative signs are ignored as compared (b) Geometric progression
to other measures of central tendency. (c) Harmonic progression
(a) minimum (b) maximum (d) none
(c) same (d) none

MEASURES OF CENTRAL TENDENCY AND DISPERSION 299


181. ————— & —————— can not be 193. —————— is used when rate of growth or
calculated if any observation is zero. decline required.
(a) G.M & A.M (b) H.M & A.M (a) mode (b) A.M
(c) H.M & G. M (d) None. (c) G.M (d) none
182. ————— & ————— are called ratio 194. In finding ————, the distribution has open-
averages. end classes.
(a) H.M & G.M (b) H. M & A.M (a) median (b) mean
(c) A.M & G.M (d) none (c) standard deviation(d) none
183. —————— is a good substitute to a 195. The cumulative frequency distribution is used
weighted average. for
(a) A.M (b) G.M (a) median (b) mode
(c) H.M (d) none (c) mean (d) none
184. For ordering shoes of various sizes for resale, 196. In ——— the quantities are in ratios.
a —————— size will be more appropriate. (a) A.M (b) G.M
(a) median (b) modal (c) H.M (d) none
(c) mean (d) none 197. ————— is used when variability has also
185. —————— is called a positional measure. to be calculated.
(a) mean (b) mode (a) A.M (b) G.M
(c) median (d) none (c) H.M (d) none
186. 50% of actual values will be below & 50% of 198. ————— is used when the sum of absolute
will be above ————— deviations from the average should be least.
(a) mode (b) median (a) Mean (b) Mode
(c) mean (d) none (c) Median (d) None
187. Extreme values have ———— effect on mode. 199. ————— is used when sampling variability
(a) high (b) low should be least.
(c) no (d) none (a) Mode (b) Median
188. Extreme values have ———— effect on (c) Mean (d) none
median. 200. ————— is used when distribution pattern
(a) high (b) low has to be studied at varying levels.
(c) no (d) none (a) A.M (b) Median
189. Extreme values have ———— effect on A.M. (c) G.M (d) none
(a) greatest (b) least 201. The average discovers
(c) some (d) none (a) uniformity in variability
190. Extreme values have ———— effect on H.M.
(b) variability in uniformity of distribution
(a) least (b) greatest
(c) medium (d) none (c) both
191. —————— is used when representation (d) none
value is required & distribution is asymmetric. 202. The average has relevance for
(a) mode (b) mean (a) homogeneous population
(c) median (d) none
(b) heterogeneous population
192. —————— is used when most frequently
occurring value is required (discrete variables). (c) both
(a) mode (b) mean (d) none
(c) median (d) none

300 BUSINESS MATHEMATICS, LOGICAL REASONING & STATISTICS (Paper 3) [CA Foundation]
203. The correction factor is applied in 213. In measuring dispersion, it is necessary to
(a) inclusive type of distribution know the amount of ———— & the degree of
(b) exclusive type of distribution —————.
(c) both (d) none (a) variation, variation
204. “Mean has the least sampling variability“ (b) variation, median
prove the mathematical property of mean
(c) median, variation
(a) True (b) false
(c) both (d) none (d) none
205. “The sum of deviations from the mean is zero“ 214. The amount of variation is designated as ——
—— is the mathematical property of mean ———— measure of dispersion.
(a) True (b) false (a) relative (b) absolute
(c) both (d) none (c) both (d) none
206. “ The mean of the two samples can be 215. The degree of variation is designated as ——
combined” — is the mathematical property of ———— measure of dispersion.
mean (a) relative (b) absolute
(a) True (b) false (c) both (d) none
(c) both (d) none 216. For purposes of comparison between two or
207. “Choices of assumed mean does not affect the more series with varying size or no. of items,
actual mean”— prove the mathematical varying central values or units of calculation,
property of mean only —————— measures can be used.
(a) True (b) false (a) absolute (b) relative
(c) both (d) none (c) both (d) none
208. “In a moderately asymmetric distribution
217. The relation Relative range = Absolute range/
mean can be found out from the given values
Sum of the two extremes. is
of median & mode“— is the mathematical
(a) True (b) false
property of mean
(a) True (b) false (c) both (d) none
(c) both (d) none 218. The relation Absolute range = Relative range/
209. The mean wages of two companies are equal. Sum of the two extremes is
It signifies that the workers of both the (a) True (b) false
companies are equally well-off. (c) both (d) none
(a) True (b) false 219. In quality control ———— is used as a
(c) both (d) none substitute for standard deviation.
210. —————— in particular helps in finding out (a) mean deviation (b) median
the variability of the data. (c) range (d) none
(a) Dispersion (b) Median 220. —————— factor helps to know the value
(c) Mode (d) None of standard deviation.
211. Measures of central tendency are called (a) Correction (b) Range
averages of the ———order.
(c) both (d) none
(a) 1st (b) 2nd
221. If there are 3 observations 15, 20, 25 then the
(c) 3rd (d) none sum of deviation of the observations from
212. Measures of dispersion are called averages of their AM is
the ———order. (a) 0 (b) 5
(a) 1st (b) 2nd
(c) –5 (d) None of these.
(c) 3rd (d) none

MEASURES OF CENTRAL TENDENCY AND DISPERSION 301


222. What is the median for the following (a) 40% (b) 50%
observations? (c) 60% (d) none of these
5, 8, 6, 9, 11, 4. 232. If there are two groups with 75 and 65 as
(a) 6 (b) 7 harmonic means and containing 15 and 13
(c) 8 (d) None of these observation then the combined HM is given
by
223. What is the modal value for the numbers 5, 8,
6, 4, 10, 15, 18, 10? (a) 65 (b) 70.36
(a) 18 (b) 10 (c) 70 (d) 71.
(c) 14 (d) None of these 233. What is the HM of 1,1/2, 1/3,…………….1/n?
224. What is the GM for the numbers 8, 24 and 40? (a) n (b) 2n
(a) 24 (b) 12 2 n(n + 1)
(c) (d)
(c) 8 × 3 15 (d) 10 (n + 1) 2
225. The harmonic mean for the numbers 2, 3, 5 is 234. An aeroplane flies from A to B at the rate of
500 km/hour and comes back from B to A at
(a) 2.00 (b) 3.33
the rate of 700 km/hour. The average speed of
(c) 2.90 (d)  3 30 the aeroplane is
226. If the AM and GM for two numbers are 6.50 (a) 600 km. per hour
and 6 respectively then the two numbers are (b) 583.33 km. per hour
(a) 6 and 7 (b) 9 and 4 (c) 35 100 km. per hour
(c) 10 and 3 (d) 8 and 5. (d) 620 km. per hour.
227. If the AM and HM for two numbers are 5 and 235. If a variable assumes the values 1, 2, 3…5 with
3.2 respectively then the GM will be frequencies as 1, 2, 3…5, then what is the AM?
(a) 16.00 (b) 4.10
11
(c) 4.05 (d) 4.00. (a) (b) 5
3
228. What is the value of the first quartile for
observations 15, 18, 10, 20, 23, 28, 12, 16? (c) 4 (d) 4.50
236. Two variables x and y are given by y= 2x – 3. If
(a) 17 (b) 16
the median of x is 20, what is the median of y?
(c) 12.75 (d) 12
(a) 20 (b) 40
229. The third decile for the numbers 15, 10, 20, 25,
(c) 37 (d) 35
18, 11, 9, 12 is
237. If the relationship between two variables u
(a) 13 (b) 10.70
and v are given by 2u + v + 7 = 0 and if the AM of
(c) 11 (d) 11.50 u is 10, then the AM of v is
230. If there are two groups containing 30 and 20 (a) 17 (b) –17
observations and having 50 and 60 as
(c) –27 (d) 27.
arithmetic means, then the combined
arithmetic mean is 238. If x and y are related by x–y–10 = 0 and mode of
x is known to be 23, then the mode of y is
(a) 55 (b) 56
(a) 20 (b) 13
(c) 54 (d) 52.
(c) 3 (d) 23.
231. The average salary of a group of unskilled
239. If GM of x is 10 and GM of y is 15, then the GM
workers is `10,000 and that of a group of skilled
of xy is
workers is `15,000. If the combined salary is
`12,000, then what is the percentage of skilled (a) 150 (b) log 10 × log 15
workers? (c) log 150 (d) None of these.

302 BUSINESS MATHEMATICS, LOGICAL REASONING & STATISTICS (Paper 3) [CA Foundation]
240. If the AM and GM for 10 observations are both is
15, then the value of HM is (a) 7.20 (b) 6.80
(a) Less than 15 (c) 20 (d) 18.80.
(b) More than 15 249. If two variables x and y are related by 2x + 3y –
(c) 15 7 =0 and the mean and mean deviation about
(d) Can not be determined. mean of x are 1 and 0.3 respectively, then the
241. What is the coefficient of range for the coefficient of mean deviation of y about its
following wages of 8 workers? mean is
`80, `65, `90, `60, `75, `70, `72, `85. (a) –5 (b) 12
(a) `30 (b) `20 (c) 50 (d) 4.
(c) 30 (d) 20 250. The mean deviation about mode for the
242. If Rx and Ry denote ranges of x and y numbers 4/11, 6/11, 8/11, 9/11, 12/11, 8/11 is
respectively where x and y are related by (a) 1/6 (b) 1/11
3x+2y+10=0, what would be the relation (c) 6/11 (d) 5/11.
between x and y?
251. What is the standard deviation of 5, 5, 9, 9, 9,
(a) Rx = Ry (b) 2 R x= 3 Ry 10, 5, 10, 10?
(c) 3 Rx= 2 Ry (d) Rx= 2 Ry
243. What is the coefficient of range for the 42
(a) 14 (b)
following distribution? 3
Class Interval : 10-19 20-29 30-39 40-49 50-59 (c) 4.50 (d) 8
Frequency: 11 25 16 7 3 252. If the mean and SD of x are a and b respectively,
(a) 22 (b) 50
x -a
(c) 72.46 (d) 75.82 then the SD of is
244. If the range of x is 2, what would be the range b
of –3x +50 ? (a) –1 (b) 1
(a) 2 (b) 6 (c) ab (d) a/b.
(c) –6 (d) 44 253. What is the coefficient of variation of the
245. What is the value of mean deviation about following numbers?
mean for the following numbers? 53, 52, 61, 60, 64.
5, 8, 6, 3, 4. (a) 8.09 (b) 18.08
(a) 5.20 (b) 7.20 (c) 20.23 (d) 20.45
(c) 1.44 (d) 2.23 254. If the SD of x is 3, what us the variance of (5–
246. What is the value of mean deviation about 2x)?
mean for the following observations?
(a) 36 (b) 6
50, 60, 50, 50, 60, 60, 60, 50, 50, 50, 60, 60, 60, 50. (c) 1 (d) 9
(a) 5 (b) 7 255. If x and y are related by 2x+3y+4 = 0 and SD of x
(c) 35 (d) 10 is 6, then SD of y is
247. The coefficient of mean deviation about mean (a) 22 (b) 4
for the first 9 natural numbers is
(c) 5 (d) 9.
(a) 200/9 (b) 80
(c) 400/9 (d) 50. 256. The quartiles of a variable are 45, 52 and 65
248. If the relation between x and y is 5y–3x = 10 respectively. Its quartile deviation is
and the mean deviation about mean for x is (a) 10 (b) 20
12, then the mean deviation of y about mean (c) 25 (d) 8.30.
MEASURES OF CENTRAL TENDENCY AND DISPERSION 303
257. If x and y are related as 3x+4y = 20 and the (a) A.M (b) G.M
quartile deviation of x is 12, then the quartile (c) H.M (d) none
deviation of y is 266. A person purchases 5 rupees worth of eggs
(a) 16 (b) 14 from 10 different markets.You are to find the
(c) 10 (d) 9. average no. of eggs per rupee for all the
258. If the SD of the 1st n natural numbers is 2, then markets taken together. What is the suitable
the value of n must be form of average in this case?
(a) 2 (b) 7 (a) A.M (b) G.M
(c) 6 (d) 5. (c) H.M (d) none
267. You are given the population of India for the
259. If x and y are related by y = 2x+ 5 and the SD
courses of 1981 & 1991. You are to find the
and AM of x are known to be 5 and 10
population of India at the middle of the period
respectively, then the coefficient of variation
by averaging these population figures,
is
assuming a constant rate of increase of
(a) 25 (b) 30 population.
(c) 40 (d) 20. What is the suitable form of average in this
case?
2
260. The mean and SD for a, b and 2 are 3 and (a) A.M (b) G.M
3
(c) H.M (d) none
respectively, The value of ab would be
268. —————— is least affected by sampling
(a) 5 (b) 6 fluctions.
(c) 12 (d) 3. (a) Standard deviation
261. For 899, 999, 391, 384, 590, 480, 485, 760, 111, (b) Quartile deviation
240 (c) both (d) none
Rank of median is 269. “Root –Mean Square Deviation from Mean“ is
(a) 2.75 (b) 5.5 (a) Standard deviation
(c) 8.25 (d) none (b) Quartile deviation
262. For 333, 999, 888, 777, 666, 555, 444 (c) both (d) none
Rank of 1st quartile is 270. Standard Deviation is
(a) 3 (b) 1 (a) absolute measure
(c) 2 (d) 7 (b) relative measure
(c) both (d) none
263. For 333, 999, 888, 777, 1000, 321, 133
271. Coefficient of variation is
Rank of 3rd quartile is
(a) absolute measure
(a) 7 (b) 4
(b) relative measure
(c) 5 (d) 6
(c) both (d) none
272. —————— deviation is called semi-
264. Price per kg.( ‘) : 45 50 35; Kgs.Purchased : 100 interquartile range.
40 60 Total frequency is
(a) Percentile (b) Standard
(a) 300 (b) 100
(c) Quartile (d) none
(c) 150 (d) 200
273. ———————— Deviation is defined as half
265. The length of a rod is measured by a tape 10 the difference between the lower & upper
times. You are to estimate the length of the quartiles.
rod by averaging these 10 determinations.
(a) Quartile (b) Standard
What is the suitable form of average in this
(c) both (d) none
case?

304 BUSINESS MATHEMATICS, LOGICAL REASONING & STATISTICS (Paper 3) [CA Foundation]
274. Quartile Deviation for the data 1, 3, 4, 5, 6, 6, 285. Mean is influenced by extreme values.
10 is (a) true (b) false
(a) 3 (b) 1 (c) both (d) none
(c) 6 (d) 1.5 286. Mean of 6, 7, 11, 8 is
275. Coefficient of Quartile Deviation is (a) 11 (b) 6
(a) (Quartile Deviation x 100)/Median (c) 7 (d) 8
(b) (Quartile Deviation x 100)/Mean 287. The sum of differences between the actual
(c) (Quartile Deviation x 100) /Mode values and the arithmetic mean is
(a) 2 (b) -1
(d) none
(c) 0 (d) 1
276. Mean for the data 6, 4, 1, 6, 5, 10, 3 is
288. When the algebraic sum of deviations from
(a) 7 (b) 5 the arithmetic mean is not equal to zero, the
(c) 6 (d) none figure of arithmetic mean ——————
277. Coefficient of variation = (Standard Deviation correct.
x 100)/Mean (a) is (b) is not
(a) true (b) false (c) both (d) none
(c) both (d) none 289. In the problem
278. If mean = 5, Standard deviation = 2.6 then the No. of shirts: 30–32 33–35 36–38 39–41 42–44
coefficient of variation is No. of persons: 15 14 42 27 18
(a) 49 (b) 51 The assumed mean is
(c) 50 (d) 52 (a) 34 (b) 37
279. If median = 5, Quartile deviation = 1. 5 then (c) 40 (d) 43
the coefficient of quartile deviation is 290. In the problem
Size of items: 1–3 3–8 8–15 15–26
(a) 33 (b) 35
Frequency: 5 10 16 15
(c) 30 (d) 20
The assumed mean is
280. A.M of 2, 6, 4, 1, 8, 5, 2 is (a) 20.5 (b) 2
(a) 5 (b) 3 (c) 11.5 (d) 5.5
(c) 4 (d) none 291. The mean wage in factory A is Nu.6,000
281. Most useful among all measures of dispersion whereas in factory B it is Nu.5,500. It signifies
is that factory A pays more to all its workers than
(a) S.D (b) Q.D factory B.
(c) Mean deviation (d) none (a) True (b) false
282. For the observations 6, 4, 1, 6, 5, 10, 4, 8 Range (c) both (d) none
is 292. Mean of 0, 3, 5, 6, 7, 9, 12, 0, 2 is
(a) 10 (b) 9 (a) 4.9 (b) 5.7
(c) 8 (d) none (c) 5.6 (d) none
293. Median of 15, 12, 6, 13, 12, 15, 8, 9 is
283. A measure of central tendency tries to
estimate the (a) 13 (b) 8
(a) central value (b) lower value (c) 12 (d) 9
294. Median of 0.3, 5, 6, 7, 9, 12, 0, 2 is
(c) upper value (d) none
(a) 7 (b) 6
284. Measures of central tendency are known as
(c) 3 (d) 5
(a) differences (b) averages
295. Mode of 0, 3, 5, 6, 7, 9, 12, 0, 2 is
(c) both (d) none
(a) 6 (b) 0
(c) 3 (d) 5

MEASURES OF CENTRAL TENDENCY AND DISPERSION 305


296. Mode 0f 15, 12, 5, 13, 12, 15, 8, 8, 9, 9, 10, 15 is 307. Standard deviation is denoted by
(a) 15 (b) 12 (a) 2 (b) 
(c) 8 (d) 9
297. Median of 40, 50, 30, 20, 25, 35, 30, 30, 20, 30 is (c)  (d) none
(a) 25 (b) 30 308. The square of standard deviation is known as
(c) 35 (d) none (a) variance
298. Mode of 40, 50, 30, 20, 25, 35, 30, 30, 20, 30 is (b) standard deviation
(a) 25 (b) 30 (c) mean deviation (d) none
(c) 35 (d) none 309. Mean of 25, 32, 43, 53, 62, 59, 48, 31, 24, 33 is
299. ———————— is extremely sensitive to (a) 44 (b) 43
the size of the sample (c) 42 (d) 41
(a) Range (b) Mean 310. For the following frequency distribution
(c) Median (d) Mode Class interval: 10–20 20–30 30–40 40–50 50–60 60–70
300. As the sample size increases, —————— Frequency: 20 9 31 18 10 9
also tends to increase. assumed mean can be taken as
(a) Range (b) Mean (a) 55 (b) 45
(c) Median (d) Mode (c) 35 (d) none
301. As the sample size increases, range also tends 311. The value of the standard deviation does not
to increase though not proportionately. depend upon the choice of the origin.
(a) true (b) false (a) True (b) false
(c) both (d) none. (c) both (d) none
302. As the sample size increases, range also tends 312. Coefficient of standard deviation is
to
(a) S.D/Median (b) S.D/Mean
(a) decrease (b) increase
(c) S.D/Mode (d) none
(c) same (d) none
303. The dependence of range on extreme items
313. The value of the standard deviation will
can be avoided by adopting
change if any one of the observations is
(a) standard deviation changed.
(b) mean deviation (a) True (b) false
(c) quartile deviation (d) none (c) both (d) none
304. Quartile deviation is called 314. When all the values are equal then variance &
(a) semi inter quartile range standard deviation would be
(b) quartile range (a) 2 (b) -1
(c) both (d) none (c) 1 (d) 0
305. When 1st quartile = 20, 3rd quartile = 30, the 315. For values lie close to the mean, the standard
value of quartile deviation is deviations are
(a) 7 (b) 4 (a) big (b) small
(c) -5 (d) 5 (c) moderate (d) none
306. (Q 3 – Q1)/(Q 3 + Q1) is 316. If the same amount is added to or subtracted
(a) coefficient of Quartile Deviation from all the values, variance & standard
(b) coefficient of Mean Deviation deviation shall
(c) coefficient of Standard deviation (a) changed (b) unchanged
(d) none (c) both (d) none

306 BUSINESS MATHEMATICS, LOGICAL REASONING & STATISTICS (Paper 3) [CA Foundation]
317. If the same amount is added to or subtracted 326. In ——— range has the greatest use.
from all the values, the mean shall increase or (a) Time series (b) quality control
decrease by the ———— amount (c) both (d) none
(a) big (b) small 327. Mean is an absolute measure & standard
(c) same (d) none deviation is based upon it. Therefore standard
318. If all the values are multiplied by the same deviation is a relative measure.
quantity, the ————— & ———— also (a) true (b) false
would be multiple of the same quantity. (c) both (d) none
(a) mean, standard deviation 328. Semi-quartile range is one-fourth of the range
(b) mean , median in a normal symmetrical distribution.
(c) mean, mode (a) Yes (b) No
(d) median , deviations (c) both (d) none
319. For a moderately non-symmetrical 329. Whole frequency table is needed for the
distribution, Mean deviation = 4/5 of standard calculation of
deviation (a) range (b) variance
(a) true (b) false (c) both (d) none
(c) both (d) none
330. Relative measures of dispersion make
320. For a moderately non-symmetrical deviations in similar units comparable.
distribution, Quartile deviation = Standard
(a) true (b) false
deviation/3
(a) true (b) false (c) both (d) none
(c) both (d) none 331. Quartile deviation is based on the
321. For a moderately non-symmetrical (a) highest 50% (b) lowest 25%
distribution, probable error of standard (c) highest 25%
deviation = Standard deviation/3 (d) middle 50% of the item.
(a) true (b) false 332. S.D is less than Mean deviation
(c) both (d) none (a) true (b) false
322. Quartile deviation = Probable error of Standard (c) both (d) none
deviation. 333. Coefficient of variation is independent of the
(a) true (b) false unit of measurement.
(c) both (d) none (a) true (b) false
323. Coefficient of Mean Deviation is (c) both (d) none
(a) Mean deviation x 100/Mean or mode 334. Coefficient of variation is a relative measure
(b) Standard deviation x 100/Mean or median of
(c) Mean deviation x 100/Mean or median (a) mean (b) deviation
(d) none (c) range (d) dispersion.
324. Coefficient of Quartile Deviation = Quartile 335. Coefficient of variation is equal to
Deviation x 100/Median (a) Standard deviation x 100 / median
(a) true (b) false (b) Standard deviation x 100 / mode
(c) both (d) none (c) Standard deviation x 100 / mean
325. Karl Pearson’s measure gives (d) none
(a) coefficient of Mean Variation 336. Coefficient of Quartile Deviation is equal to
(b) coefficient of Standard deviation (a) Quartile deviation x 100 / median
(c) coefficient of variation (d) none (b) Quartile deviation x 100 / mean

MEASURES OF CENTRAL TENDENCY AND DISPERSION 307


(c) Quartile deviation x 100 / mode 346. For finding A.M in Step-deviation method, the
(d) none class intervals should be of
337. If each item is reduced by 15 A.M is (a) equal lengths
(a) reduced by 15 (b) unequal lengths
(b) increased by 15 (c) maximum lengths (d) none
(c) reduced by 10 347. The sum of the squares of the deviations of
(d) none the variable is —————— when taken about
A.M
338. If each item is reduced by 10, the range is
(a) maximum (b) zero
(a) increased by 10
(c) minimum (d) none
(b) decreased by 10
348. The A.M of 1, 3, 5, 6, x, 10 is 6 . The value of x is
(c) unchanged (d) none
(a) 10 (b) 11
339. If each item is reduced by 20, the standard
deviation (c) 12 (d) none
(a) increased (b) decreased 349. The G.M of 2 & 8 is
(c) unchanged (d) none (a) 2 (b) 4
340. If the variables are increased or decreased by (c) 8 (d) none
the same amount the standard deviation is 350. (n+1)/2 th term is median if n is
(a) decreased (b) increased (a) odd (b) even
(c) unchanged (d) none (c) both (d) none
341. If the variables are increased or decreased by 351. For the values of a variable 5, 2, 8, 3, 7, 4, the
the same proportion, the standard deviation median is
changes by (a) 4 (b) 4.5
(a) same proportion (c) 5 (d) none
(b) different proportion 352. The abscissa of the maximum frequency in the
(c) both (d) none frequency curve is the
(a) mean (b) median
342. The mean of the 1st n natural no. is (c) mode (d) none
(a) n/2 (b) ( n-1)/2 353.
(c) (n+1)/2 (d) none Varia ble: 2 3 4 5 6 7

343. If the class interval is open-end then it is No. of men: 5 6 8 13 7 4


difficult to find Mode is
(a) frequency (b) A.M (a) 6 (b) 4
(c) both (d) none (c) 5 (d) none
344. Which one is true— 354. The class having maximum frequency is called
(a) A.M = assumed mean + arithmetic mean (a) modal class (b) median class
of deviations of terms (c) mean class (d) none
(b) G.M = assumed mean + arithmetic mean 355. For determination of mode, the class intervals
of deviations of terms should be
(c) Both (d) none (a) overlapping (b) maximum
345. If the A.M of any distribution be 25 & one term (c) minimum (d) none
is 18. Then the deviation of 18 from A.M is 356. First Quartile lies in the class interval of the
(a) 7 (b) -7 (a) n/2th item (b) n/4th item
(c) 43 (d) none (c) 3n/4th item (d) n/10th item

308 BUSINESS MATHEMATICS, LOGICAL REASONING & STATISTICS (Paper 3) [CA Foundation]
357. The value of a variate that occur most often is 367. Coefficient of Standard deviation is equal to
called (a) S.D/A.M (b) A.M/S.D
(a) median (b) mean (c) S.D/GM (d) none
(c) mode (d) none 368. The distribution, for which the coefficient of
358. For the values of a variable 3, 1, 5, 2, 6, 8, 4 the variation is less, is ——— consistent.
median is (a) less (b) more
(a) 3 (b) 5 (c) moderate (d) none
(c) 4 (d) none
359. If y = 5 x - 20 & x = 30 then the value of y is
(a) 130 (b) 140
(c) 30 (d) none
---0---0---
360. If y = 3 x - 100 and x = 50 then the value of y is
(a) 60 (b) 30
(c) 100 (d) 50
361. The median of the numbers 11, 10, 12, 13, 9 is
(a) 12.5 (b) 12
(c) 10.5 (d) 11
362. The mode of the numbers 7, 7, 7, 9, 10, 11, 11,
11, 12 is
(a) 11 (b) 12
(c) 7 (d) 7 & 11
363. In a symmetrical distribution when the 3rd
quartile plus 1st quartile is halved, the value
would give
(a) mean (b) mode
(c) median (d) none

364. In Zoology —————— is used.


(a) median (b) mean
(c) mode (d) none
365. For calculation of Speed & Velocity
(a) G.M (b) A.M
(c) H.M (d) none is used.
366. The S.D is always taken from
(a) median (b) mode
(c) mean (d) none

MEASURES OF CENTRAL TENDENCY AND DISPERSION 309


7. If each observation of a set is divided by 5,
then the mean of new values :
HOME WORK-1 (a) is 5 times the original mean
(b) is decreased by 5
(c) is (1/5)th of the original mean
1. Mean is a measure of :
(d) remains the same
(a) location (central value)
8. The median of the following observations is
(b) dispersion 10, 19, 22, 16, 15, 18, 20, 18, 14, 18, 23.
(c) correlation (a) 17.55 (b) 18
(d) none of the above (c) 15 (d) 16.5
2. Which of the following is a measure of central 9. The mode of the following observations is
value ? 10, 19, 22, 16, 15, 18, 20, 18, 14, 18, 23.
(a) Median (a) 17.55 (b) 18
(b) Variance (c) 15 (d) 16.5
(c) Standard deviation 10. The first quartile of the following observations
is
(d) (b) and (c) both
10, 19, 22, 16, 15, 18, 20, 18, 14, 18, 23.
3. Which of the followings represents median ?
(a) 17.55 (b) 18
(a) First quartile
(c) 15 (d) 20
(b) Fiftieth percentile 11. The third quartile of the following
(c) Sixth decile observations is
(d) None 10, 19, 22, 16, 15, 18, 20, 18, 14, 18, 23.
4. If a constant value 40 is subtracted from each (a) 17.55 (b) 18
observation of a set, the mean of the set is (c) 15 (d) 20
(a) increased by 40 12. The second quartile of the following
observations is
(b) decreased by 40
10, 19, 22, 16, 15, 18, 20, 18, 14, 18, 23.
(c) is not affected (a) 17.55 (b) 18
(d) zero (c) 15 (d) 20
5. If each value of a series is multiplied by 7, the 13. The mean of the following observations is
median of the coded values is : 10, 8, –9, –12, 15, 0, 23, –3, –2, –13, –24, 28, 35,
(a) not affected 42
(a) 4 (b) –2
(b) 7 times the original median value
(c) 7 (d) –9.75
(c) (1/7)th of the original median value
14. The median of the following observations is
(d) increased by 7 10, 8, –9, –12, 15, 0, 23, –3, –2, –13, –24, 28, 35,
6. If each value of a series is multiplied by 8, the 42
mode of the coded values is : (a) 4 (b) –2
(a) not affected (c) 7 (d) –9.75
(b) (1/8)th of the original modal value 15. The mode of the following observations is
(c) 8 times of the original modal value
10, 8, –9, –12, 15, 0, 23, –3, –2, –13, –24, 28, 35,
(d) none of them 42
(a) 4 (b) –2
(c) 7 (d) –9.75

310 BUSINESS MATHEMATICS, LOGICAL REASONING & STATISTICS (Paper 3) [CA Foundation]
16. The first quartile of the following observations (a) 4 (b) 6
is (c) 3 (d) none
10, 8, –9, –12, 15, 0, 23, –3, –2, –13, –24, 28, 35, 25. The A.M. of two numbers is 6.5 and their G.M.
42 is 6. The two numbers are :
(a) 4 (b) –2.25 (a) 8,6 (b) 8, 5
(c) 24.25 (d) –9.75 (c) 7, 16 (d) 4, 9
17. The third quartile of the following 26. The following distribution of ages (in
observations is complete years) is obtained for the students
10, 8, –9, –12, 15, 0, 23, –3, –2, –13, –24, 28, 35, of higher secondary. The mean of the
42 distribution is
(a) 14.25 (b) –2.25 Age (in years) 15 16 17 18 19 20
21
(c) 24.25 (d) –9.75
Number of students 12 18 20 10 7 6
18. The following distribution of ages (in complete 2
years) is obtained for the students of higher
(a) 17.11 (b) 17
secondary. The median of the distribution is
(c) 18 (d) none
Age (in years) 15 16 17 18 19 20 21
27. The following distribution of ages (in
Number of
complete years) is obtained for the students
students 12 18 20 10 7 6 2
of higher secondary. The mode of the
(a) 17.11 (b) 17 distribution is
(c) 18 (d) none Age (in years) 15 16 17 18 19 20 21
19. The average of five numbers is 40 and the Number of students1218 20 10 7 6 2
average of another six numbers is 50. The (a) 16 (b) 17
average of all numbers taken together is :
(c) 18 (d) none
(a) 44.44 (b) 45.00
28. The following distribution of ages (in
(c) 45.45 (d) 90.00 complete years) is obtained for the students
20. If the two observations are 10 and 0, their of higher secondary. The first quartile of the
arithmetic mean is distribution is.
(a) 10 (b) 0 Age (in years) 15 16 17 18 19 20 21
(c) 5 (d) none Number of students 12 18 20 10 7 6 2
21. What percentage of value is greater than 3rd (a) 19 (b) 17
quartile? (c) 18 (d) none
(a) 75 per cent (b) 50 per cent 29. The first quartile of the following frequency
(c) 25 per cent (d) 0 per cent distribution is
22. What percentage of values is less than 4th Observation 8 7 6 5 4 3 2 1
decile? Frequency 10 8 12 23 30 18 19 15
(a) 50% (b) 70% (a) 4 (b) 7
(c) 40% (d) none (c) 5 (d) 2
23. What percentage of values lies between 5th 30. The second quartile of the following
and 25th percentiles frequency distribution is
(a) 20% (b) 30% Observation 8 7 6 5 4 3 2 1
(c) 75% (d) none Frequency 10 8 12 23 30 18 19 15
24. If the A.M. of a set of observations is 9 and its (a) 4 (b) 7
G.M. is 6. Then the H.M. of the set of (c) 5 (d) 2
observations is :

MEASURES OF CENTRAL TENDENCY AND DISPERSION 311


31. The third quartile of the following frequency 40. The average of n natural numbers is
distribution is (a) (n + 1)/2 (b) (2n + 1)/2
Observation 8 7 6 5 4 3 2 1 (c) n(n + 1)/2 (d) n (2n + 1)/2
Frequency 10 8 12 23 30 18 19 15 41. Mode is that value in a frequency distribution
(a) 4 (b) 7 which possesses:
(c) 5 (d) 2 (a) minimum frequency
32. Which of the following relation is true (b) maximum frequency
between 3rd decile and 30th percentile ? (c) frequency one (d) none
(a) D7 = P70 (b) D7 = P30 42. The mean of the following frequency
distribution is
(c) D3 = P30 (d) D3 = P70 Class 0–6 6–12 12–18 18–24 24–30 30–36 36–42 42–48 48–54
33. Which of the deciles are less than first quartile? Frequency 13 25 57 79 105 79 57 25 13

(a) D1 and D3 (b) D1 and D3 (a) 27 (b) 28


(c) D1 and D2 (d) None (c) 25 (d) none
34. The mean of seven observations is 18. A new 43. The median of the following frequency
observation 16 is added. The mean of eight distribution is
observations is : Class 0–6 6–12 12–18 18–24 24–30 30–36 36–42 42–48 48–54

Frequency 13 25 57 79 105 79 57 25 13
(a) 22 (b) 17
(a) 27 (b) 28
(c) 18 (d) none
(c) 25 (d) none
35. If the sum of n observations is 630 and their
mean is 21, then the value of n is : 44. The mode of the following frequency
distribution is
(a) 21 (b) 30 Class 0–6 6–12 12–18 18–24 24–30 30–36 36–42 42–48 48–54
(c) 15 (d) 20 Frequency 13 25 57 79 105 79 57 25 13
36. If the two observations are 10 and 0 then their (a) 27 (b) 28
geometric mean is : (c) 25 (d) none
(a) 10 (b) 0 45. The combined mean of the following data is
(c) 5 (d) 8 Group A B
37. If the two observations are 7 and –7, their C
geometric mean is : Number of observations 400 250
(a) 7 (b) –7 350
(c) 0 (d) None Mean 52 44
38. Can a quartile, a decile and a percentile be the 60
median ? (a) 52.8 (b) 44
(a) Only quartile but not decile (c) 52 (d) none
(b) Quartile and decile but not percentile and 46. The average marks of 120 students of a class is
percentile 48.2. If the average marks of 50 girls of that
(c) Decile and percentile but class is 54, the approximate average marks of
(d) Quartile, decile and percentile, all the the boys is
not quartile three (a) 48 (b) 58
39. When all the observations are same, then the (c) 44 (d) none
relation between A.M., G.M. and H.M. is : 47. The approximate weighted mean of the
(a) A.M. = G.M. = H.M. following data is
(b) A.M. = G.M. > H.M. xi 56 40 68 55 37
(c) A.M. > G.M. > H.M. wi 3 2 2 4 1
(d) A.M. < G.M. < H.M. (a) 55.5 (b) 58.3
(c) 56.4 (d) 53.4

312 BUSINESS MATHEMATICS, LOGICAL REASONING & STATISTICS (Paper 3) [CA Foundation]
48. The 8th decile of the following observations is (b) one of the two observations is zero
29, 18, 15, 30, 42, 35, 34, 28, 45, 34 (c) one of them is negative
(a) 44.6 (b) 40.6 (d) both of them are zero
(c) 34.15 (d) 38.15 58. Geometric mean is a good measure of central
49. The 65th percentile of the following value if the data are :
observations is (a) categorical
29, 18, 15, 30, 42, 35, 34, 28, 45, 34 (b) on ordinal scale
(c) in ratios or proportions
(a) 44.6 (b) 40.6
(d) none of the above
(c) 34.15 (d) 38.15
59. Harmonic mean is better than other means if
50. The median for the following distribution is
the data are for :
x: 2 3 4 5 6 7 8 9 10 11
(a) speed or rates
f: 3 6 9 18 20 14 10 10 7 2
(b) heights or lengths
(a) 6 (b) 5
(c) binary values like O and l
(c) 8 (d) 9
(d) ratios or proportions
51. The Q1 for the following distribution is
60. The geometric mean of 3, 7, 11, 15, 24, 28, 30, 0
x: 2 3 4 5 6 7 8 9 10 11
is
f: 3 6 9 18 20 14 10 10 7 2
(a) 6 (b) 0
(a) 6 (b) 5
(c) 8 (d) 9
(c) 8 (d) 9
61. The Harmonic mean of 5, 6, 10, 15, 20, 24 is
52. The Q3 for the following distribution is
x: 2 3 4 5 6 7 8 9 10 11
(a) 6.9 (b) 9.6
f: 3 6 9 18 20 14 10 10 7 2 (c) 8.6 (d) 9.8
(a) 6 (b) 5 62. A cyclist travels 20 kms at a speed of 16 kms.
per hour, 30 kms at a speed of 12 kms per hour
(c) 8 (d) 9
and 40 kms at a speed at 10 kms per hour. His
53. The D9 for the following distribution is average speed is
x: 2 3 4 5 6 7 8 9 10 11
(a) 16 (b) 15.5
f: 3 6 9 18 20 14 10 10 7 2
(c) 11.6 (d) 19.5
(a) 6 (b) 5
63. The mean of a distribution is 22.2 and its mode
(c) 8 (d) 9
is 23.3. The median is
54. The P82 for the following distribution is
x: 2 3 4 5 6 7 8 9 10 11 (a) 26.5 (b) 22.6
f: 3 6 9 18 20 14 10 10 7 2 (c) 28.5 (d) 26.6
(a) 6 (b) 5 64. If Z – M = 4 and Z + M = 60, then M is
(c) 8 (d) 9 (a) 26 (b) 25
55. Which of the following relations among the (c) 28 (d) 32
location parameters does not hold ? 65. If Z – M = 4 and Z + M = 60, then Z is
(a) Q2 = median (b) P50 = median (a) 32 (b) 25
(c) D5 = median (d) D4 = median (c) 28 (d) 26
56. If the group data has open end classes, one
66. If Z – M = 4 and Z + M = 60, then mean is
cannot calculate:
(a) 32 (b) 25
(a) median (b) mode
(c) mean (d) quartiles (c) 28 (d) 26
57. Geometric mean of two observations can be 67. The mean of 6 observations is 15. Four of the
calculated only if : observations are 10, 18, 23 and 9. of the
(a) both the observations are positive remaining two observations one is four times

MEASURES OF CENTRAL TENDENCY AND DISPERSION 313


the other, They are (a) mode (b) median
(a) 32 ,8 (b) 24,6 (c) sixth decile (d) all the above
(c) 28,7 (d) 26,6.5 77. The mean of 25 observations is 73.408. If one
68. The mean of 15 observations is 22. One observation 64 is removed, the revised mean
observation was taken as –12 instead of 12, is
the correct mean is (a) 72.8 (b) 73.8
(a) 32.5 (b) 25.6 (c) 80.8 (d) 76.8
(c) 28.6 (d) 23.6 78. In a group, 2 students spend ` 8 daily, 3
69. The value of the variable corresponding to the students spend ` 10 daily and 5 students spend
highest point of a frequency distribution curve ` 6 daily. The average spending of all 10
represents : students is
(a) mean (b) median (a) 7.6 (b) 5.8
(c) mode (d) none (c) 8.5 (d) 6.7
70. A frequency distribution having two modes is 79. The mean of 20 observations of a series is 18.7.
said to be : If from each of the observations 3 is subtracted
(a) unimodal (b) bimodal and then multiplied by 2, the mean of the new
(c) trimodal (d) without mode observations is
71. If modal value is not clear in a distribution, it (a) 32.4 (b) 30.4
can be ascertained by the method of: (c) 31.4 (d) none
(a) grouping (b) guessing 80. The correct relationship between A.M. G.M.
(c) summarizing (d) trial and error and H.M. is :
72. Shoe size of most of the people in a city is (a) A.M. = G.M. = H.M
No.7. Which measure of central value does it (b) G.M > A.M. > H.M
represent ? (c) H.M. > G.M. > A.M.
(a) mean (b) media (d) A.M. > G.M. > H.M.
(c) eighth decile (d) mode 81. Extreme value have no effect on :
73. In a discrete series having (2n + 1) observations, (a) average (b) median
median is : (c) geometric mean
(a) nth observation (d) harmonic mean
(b) (n + 1)th observation 82. Average of 12 members = 11.0. Average of the
(c) [(n + 2)/2]th observations first six members = 10.5 average of the last six
(d) [(2n +1)/2]th observation members =
74. To find the median , it is necessary to arrange (a) 10.5 (b) 11.5
the data in : (c) 11.0 (d) 10.0
(a) ascending order 83. For a group of 100 candidates, the mean was
(b) descending order found to be 40. Later on it was discovered that
(c) ascending or descending order a value 45 was misread as 54. The correct mean
(d) any of them is :
75. For a grouped data, the formula for median is (a) 40 (b) 39.8
based on : (c) 39.9 (d) 39.91
(a) interpolation method 84. The average runs of a cricketer in four innings
(b) extrapolation method is 32. How many runs he should score in the
(c) trial and error method fifth inning so that his average score becomes
(d) iterative method 50 runs?
76. Which of the measure of central tendency is (a) 132 (b) 122
not affected by extreme values? (c) 128 (d) 126

314 BUSINESS MATHEMATICS, LOGICAL REASONING & STATISTICS (Paper 3) [CA Foundation]
85. The combined mean of two groups is 57. The mean of the series is :
mean of the first group is 52 and the mean of (a) 20 (b) 21
the second group is 60. The proportion of (c) 22 (d) 6.57
observations in the two groups is 96. If we plot the more than type and less than
(a) 3:5 (b) 2:5 type frequency distributions of the same set
(c) 2:8 (d) 5:2 of data, their graphs intersect at the point
86. The mean of first 10 natural numbers. which is known as:
(a) 6.5 (b) 2.5 (a) median (b) mode
(c) 4.5 (d) 5.5 (c) mean (d) none
97. Mean of a set of values is based on :
87. The middle value of an ordered series is called:
(a) all values
(a) 2nd quartile (b) 5th decile
(b) 50 percent values
(c) 50th percentile (d) all the above
(c) first and last value
88. The variate values which divide a series (d) maximum & minimum value
(frequency distribution) into four equal parts
98. Which average is most affected by extreme
are called :
values?
(a) quintiles (b) quartiles (a) Geometric mean
(c) Octiles (d) percentiles (b) Harmonic mean
89. The variate values which divide a series (c) Arithmetic mean
(frequency distribution) into 100 equal parts (d) Trimmed mean
are called : 99. Harmonic mean gives more weightage to
(a) quintiles (b) quartiles (a) small values (b) large values
(c) deciles (d) (c) positive values
percentiles
(d) negative values
90. The variate values which divide a series 100. For further algebraic treatment geometric
(frequency distribution) into ten equal parts mean is
are called:
(a) suitable (b) not suitable
(a) quartiles (b) deciles (c) sometime suitable(d) none
(c) Octiles (d) percentiles 101. For further algebraic treatment harmonic
91. The number of partition values in case of mean is :
quartiles is : (a) suitable (b) not suitable
(a) 4 (b) 3 (c) sometime suitable(d) none
(c) 2 (d) 1 102. In a class test, 40 students out of 50 passed
92. The first quartile divides a frequency with mean marks 6.0 and the overall average
distribution in the ratio : of class marks was 5.5. The average marks of
(a) 4 : 1 (b) 1:4 students who failed were:
(c) 3 : 1 (d) 1:3 (a) 2.5 (b) 3.0
93. The first quartile is also known as : (c) 4.8 (d) 3.5
(a) median (b) lower quartile 103. Sum of the deviations about mean is :
(c) mode (d) third decile (a) Zero (b) minimum
94. The third quartile is also called : (c) maximum (d) One
(a) lower quartile (b) median
104. Sum of the absolute deviations about median
(c) mode (d) upper quartile is
95. If for an individual series, assumed mean, A =
(a) Zero (b) minimum
25  dx = –21 for dx = X – A and N = 7, then the (c) maximum (d) One

MEASURES OF CENTRAL TENDENCY AND DISPERSION 315


105. Sum of square of the deviations about mean (a) Range
is: (b) Mean deviation
(a) Zero (b) minimum (c) Standard deviation
(c) maximum (d) (d) Coefficient of variation
none
114. Correct formula for mean deviation from a
106. The suitable measure of central tendency for constant A of a series in which the variate
qualitative data is: values x1, x2, …..., xn have frequencies f1, f2,
(a) mode ..…, fn respectively is:
(b) arithmetic mean
1
(c) geometric mean (a)   fi xi  A
N i
(d) median
107. In a frequency distribution with open ends, 1
(b)  fi  x1  A
N i
one can not find out :
(a) mean (b) median 1
(c) mode (d) all the above (c)  | fi  x1  A |
N i
108. The geometric mean of 3, 6, 24 and 48 is
(a) 32 (b) 12 1
(d)  | fi x1  A |
N i
(c) 14 (d) 24
109. The mean annual salary of all employees in a where i = 1,2,…, k and  fi  N
company is ` 25,000. The mean salaries of male i

and female employees are ` 27,000 and ` 115. The correct relation between variance and
17,000 respectively, the percentage of males standard deviation (S.D) of a variable X is.
and females employed by the company is (a) S.D. = Var
(a) 80,20 (b) 20,80
(b) S.D. = [Var (X)]1/2
(c) 30,70 (d) 70,30
(c) S.D. = [Var (X)]2
110. If for a series the arithmetic mean is 25 and
the harmonic mean is 9, what is the geometric (d) None
mean? 116. Formula for coefficient of variation is :
(a) 25 (b) 15 100
(c) 28 (d) 16 (a) C.V. =
mean  S .D.
111. Which of the following is a unit less measure
of dispersion? mean
(b) C.V. = x 100
(a) Quartile deviation S .D.
(b) Mean deviation mean  S .D.
(c) C.V. =
(c) Coefficient of variation 100
(d) Range S .D.
112. Which one of the given measures of dispersion (d) C.V. = x 100
mean
is considered best? 117. Formula for coefficient of range of the set of
(a) Standard deviation observations X1, X2, …, Xn is :
(b) Range (a) Xmin – Xmax (b) Xmax – Xmin
(c) Mean deviation
X max  X min X max  X min
(d) Coefficient of variation (c) (d)
X max  X min X max  X min
113. For comparison of two different series, the
best measure of dispersion is

316 BUSINESS MATHEMATICS, LOGICAL REASONING & STATISTICS (Paper 3) [CA Foundation]
118. Coefficient of quartile deviation is given by (a) Mean (b) median
the formula: (c) mode (d) zero
Q3  Q1 Q3  Q1 129. If a constant value 15 is subtracted from each
(a) (b) observation of a set, the variance is:
Q1  Q3 Q1  Q3
(a) reduced by 15
Q3  Q1 Q3  Q1 (b) reduced by 225
(c) Q3  Q1 (d) Q3  Q1 (c) unaltered
119. The range from the following observations is (d) increased by 225
10, 18, 20, 28, 15, 17, 22, 25, 29, 32, 34 130. If each observation of a set is divided by 10,
(a) 24 (b) 20 the S.D. of the new observations is:
(c) 30 (d) 42 (a) 10 times of S.D. of original obs.
120. The co–efficient of range from the following
1
observations is (b) th of S.D. of original obs.
100
10, 18, 20, 28, 15, 17, 22, 25, 29, 32, 34
(a) 0.6 (b) 0.5 (c) not changed
(c) 0.55 (d) none 1
(d) th of S.D. of original obs.
121. The quartile deviation from the following 10
observations is 131. Which of the following formula for standard
10, 18, 20, 28, 15, 17, 22, 25, 29, 32, 34 deviation of a frequency distribution is not
(a) 8 (b) 6 correct?
(c) 10 (d) 5
1
122. The co–efficient of quartile deviation from the (a) s=  fi ( xi  x )2
following observations is N
10, 18, 20, 28, 15, 17, 22, 25, 29, 32, 34 1
(a) 0.26 (b) 0.16 (b) s =  fi xi2  x 2
N
(c) 0.06 (d) 0.36
123. The co– efficient of Q.D of 63, 65, 68, 75, 76, 77, 2
78, 81, 83, 70 is 1   fi x i 
(c)   fi x i2   
(a) 0.018 (b) 0.028 N i  N 
(c) 0.08 (d) 0.008
124. The co–efficient of mean deviation of 15, 17,
22, 18, 19, 11, 13, 18, 20, 17 is 2

f i x i2    i i
(a) 0.44 (b) 0.34 1  fx
(c) 0.24 (d) 0.14 (d)    
N i  N
125. The variance of 1, 2, 4, 5 and x is 2, then x is
(a) 3 (b) 6 132. The measure of dispersion which ignores signs
(c) 1 (d) 2 of the deviations from a central value is:
126. Semi inter-quartile deviation is given by the (a) Range
formula: (b) quartile deviation
Q3  Q1 (c) standard deviation
(a) Q3 – Q 1 (b) (d) mean deviation
2
(c) (Q3 + Q1)/2 (d) (Q3–Q1)/4 133. Which measure of dispersion is least affected
127. Mean deviation is minimum when deviations by extreme values ?
are taken from: (a) Range
(a) mean (b) median (b) Mean deviation
(c) mode (d) zero (c) Standard deviation
128. Sum of squares of the deviations is minimum (d) Quartile deviation
when deviations are taken from:
MEASURES OF CENTRAL TENDENCY AND DISPERSION 317
134. Which measure of dispersion is most affected 143. If Q3 + Q1 = 108.5, Q3 – Q1 = 74; then co–efficient
by extreme values ? of Q.D is
(a) Standard deviation (a) 33 (b) 3.7
(b) Mean deviation (c) 37 (d) 0.68
(c) Range 144. In a distribution 25% of the observations are
(d) Quartile deviation less than 46 and 25% of the observations are
135. Which measure of desperision ensures more than 54. The quartile deviation of the
highest degree of reliability? distribution is
(a) Range (a) 3 (b) 7
(b) Mean deviation (c) 4 (d) 6
(c) Standard deviation 145. The co-efficient of mean deviation of first five
(d) Quartile deviation natural numbers is
136. Which measure of disperision ensures lowest (a) 0.3 (b) 0.4
degree of reliability? (c) 0.5 (d) 0.7
(a) Range 146. The mean deviation of 3, 4, 5, 3 is
(b) Mean deviation (a) 0.75 (b) 0.85
(c) Quartile deviation (c) 0.65 (d) 0.55
(d) standard deviation 147. The mean and S.D. of 13, 13, 13, 13, 13 are
137. A set of values is said to be relatively uniform respectively
if it has: (a) 0,13 (b) 13, 13
(a) high dispersion (c) 13,0 (d) 0,0
(b) zero dispersion 148. The average of the sum of squares of the
(c) little dispersion deviations about mean is called:
(d) negative dispersion
(a) Variance
138. Range of a set of values is 60 and maximum
(b) absolute deviation
value in the series is 80. The minimum value
of the series is : (c) standard deviation
(a) 140 (b) 20 (d) mean deviation
(c) 70 (d) none 149. Quartile deviation is equal to:
139. If the minimum value in a set is 15 and its (a) Semi–interquartile range
range is 55, the maximum value of the set is (b) double the interquartile range
(a) 40 (b) 60 (c) Interquartile range
(c) 80 (d) 70 (d) none
140. If the values of a set are measured in kgs., the 150. Which measure of dispersion can be calculated
unit of variance will be: in case of open end intervals ?
(a) kg 2 (b) kg (a) Range
(c) kg 3 (d) no unit (b) Standard deviation
141. Which measure of dispersion has a different (c) Quartile deviation
unit other than the unit of measurement of
(d) Coefficient of variation
values:
151. If each value of a series is multiplied by 15, the
(a) Range
coefficient of variation will be increased by
(b) Mean deviation
(a) 5% (b) 0%
(c) Standard deviation
(d) Variance (c) 15% (d) 10%
142. If Q3 + Q1 = 108.5, Q3 – Q1 = 74; then Q.D. is 152. The C.V. of 4, 6, 8, 10, 12 is
(a) 33 (b) 3.7 (a) 33 (b) 35.35%
(c) 37 (d) 0.07 (c) 42 (d) 46

318 BUSINESS MATHEMATICS, LOGICAL REASONING & STATISTICS (Paper 3) [CA Foundation]
153. The sum of 10 observations is 110 and the sum 162. The relation between variance and standard
of squares of observations is 2900, their deviation is:
standard deviation is (a) variance is the square root of standard
(a) 13 (b) 8 deviation
(c) 10 (d) 9 (b) square of the standard deviation is equal
154. The sum of 25 observations is 400 and the sum to variance
of squares of observations is 8900 find C.V. (c) variance is equal to standard deviation
(a) 33.5 (b) 60.5 (d) standard deviation is the square of the
variance
(c) 62.5 (d) 70.5
163. Which of the following statements is true of a
155. The C.V of a distribution is 80% and the mean measure of dispersion ?
of the distribution is 40, the S.D. of the (a) Mean deviation does not follow algebraic
distribution is rule
(a) 33 (b) 32 (b) Range is a crudest measure
(c) 35 (d) 0.30 (c) Coefficient of variation is a relative
156. The mean of 100 observations is 18.4 and sum measure
of squares of deviations from mean is 1444, (d) All the above statements
the co-efficient of variation is
(a) 30.6 (b) 35.6
(c) 20.6 (d) 10.6
157. The standard deviation of 5 items is found to
be 15. What will be the standard deviation if
the values of all the items are increased by 5?
(a) 15 (b) 20
(c) 10 (d) none
158. What will be the relative range, if the spread
of items in a given distribution lies from 100
and 180 kg ?
(a) 0.29 (b) 0.4
(c) 0.3 (d) 0.55
159. The standard deviation of a set of 50 items is 8,
what is the standard deviation, if each item is
multiplied by 2?
(a) 32 (b) 8
(c) 4 (d) 16
160. If each value of a series is multiplied by a
constant , the coefficient of variation as
compared to original value is:
(a) increased (b) unaltered
(c) decreased (d) zero
161. If each value of a set is divided by a constant
‘d’, the coefficient of variation will be:
(a) more than original value
(b) less than original value
(c) same as original value
(d) none
MEASURES OF CENTRAL TENDENCY AND DISPERSION 319
ANSWER KEYS

1 (a) 21 (c) 41 (b) 61 (b) 81 (b) 101 (b) 121 (b) 141 (d) 161 (c)
2 (a) 22 (c) 42 (a) 62 (c) 82 (b) 102 (d) 122 (a) 142 (c) 162 (b)
3 (b) 23 (a) 43 (a) 63 (b) 83 (d) 103 (a) 123 (c) 143 (c) 163 (d)
4 (b) 24 (a) 44 (a) 64 (c) 84 (b) 104 (c) 124 (d) 144 (c)
5 (b) 25 (d) 45 (a) 65 (a) 85 (a) 105 (b) 125 (a) 145 (b)
6 (d) 26 (a) 46 (c) 66 (d) 86 (d) 106 (d) 126 (b) 146 (a)
7 (c) 27 (b) 47 (d) 67 (b) 87 (d) 107 (a) 127 (b) 147 (c)
8 (b) 28 (d) 48 (b) 68 (d) 88 (b) 108 (b) 128 (a) 148 (a)
9 (b) 29 (d) 49 (c) 69 (c) 89 (d) 109 (a) 129 (c) 149 (a)
10 (c) 30 (a) 50 (a) 70 (b) 90 (b) 110 (b) 130 (d) 150 (c)
11 (d) 31 (c) 51 (b) 71 (a) 91 (a) 111 (c) 131 (d) 151 (b)
12 (b) 32 (c) 52 (c) 72 (d) 92 (d) 112 (a) 132 (d) 152 (b)
13 (c) 33 (c) 53 (d) 73 (b) 93 (b) 113 (d) 133 (d) 153 (a)
14 (a) 34 (d) 54 (d) 74 (c) 94 (d) 114 (c) 134 (c) 154 (c)
15 (b) 35 (b) 55 (d) 75 (a) 95 (c) 115 (b) 135 (c) 155 (b)
16 (d) 36 (b) 56 (c) 76 (d) 96 (a) 116 (d) 136 (a) 156 (c)
17 (c) 37 (d) 57 (a) 77 (b) 97 (a) 117 (d) 137 (c) 157 (a)
18 (b) 38 (d) 58 (c) 78 (a) 98 (c) 118 (c) 138 (b) 158 (a)
19 (c) 39 (a) 59 (a) 79 (c) 99 (a) 119 (a) 139 (d) 159 (d)
20 (c) 40 (a) 60 (b) 80 (d) 100 (b) 120 (c) 140 (a) 160 (b)

320 BUSINESS MATHEMATICS, LOGICAL REASONING & STATISTICS (Paper 3) [CA Foundation]
8. For two variables x and y with the same mean
HOME WORK-2 the regression equation are y=2x-a and x = 2y-
 ; what is the value of common mean
(a)  (b) 0

1. If the mean of the set of observations X1, X2, (c)  (d) -


X 3 , ...... X n , is X , then the mean of the 9. The mean of 1,2,3, ………. n for first 11 natural
observation xi + Ki,
number is then the value of x is
where = I = 1, 2, 3, ......... n and
(a) X + K (n+1) (b) X + kn (a) 14 (b) 13
(c) 126 (d) 11
K n+1
(c) X+n (d) X+K 2 10. Two variables x and y satisfy the relation 3x -
2y - 25 = 0 the mode of x is 25. Then the mode
2. The mean age of a group of 100 men and of y is:
women is 25 years. If the mean age of the group (a) 25 (b) 30
of men is 26, then that of the group of women
(c) 37.5 (d) 52/3
is 21 then the ratio of women and men in the
11. for two numbers “a” and “b”, Standard
group:
Deviation given by
(a) 1:1 (b) 1:2
(c) 1:3 (d) 1:4
3. The Geometric mean of the series 1, k, k2, ....,
kn, where k is constant is
(a) K(n+1)/2 (b) Kn+1
(c) Kn+0.5 (d) Kn/2
4. If the SD of a variance X is  then Quartile
12. Which measure of dispersion is not affected
Deviation (QD) is
in the presence of extreme observations?
(a) 4/5  (b) 2/3 
(a) Range
(c) 3/2  (d) 5/4 
(b) Mean deviation
5. The mean salary for a group of for a group of 50
(c) Standard deviation
male workers is Rs.4800 per month and that
for a group of 50 female workers is Rs. 5600. (d) Quartile deviation
the combined mean salary is 13. If x and y are related as 3x + 4y = 20 and the
(a) 5100 (b) 5200 quartile deviation of x in 12. Then the Quartile
deviation of y is:
(c) 5300 (d) 5400
(a) 16 (b) 14
6. The MD about the Mean for the data
(c) 10 (d) 9
6,9,11,10,12,12
14. The sum of the squares of deviations of a Set
(a) 1.47 (b) 1.57
of observations has the smallest value. when
(c) 1.67 (d) 1.87 the deviations are taken from their:
7. The SD for the data 6, 9, 10, 3, 7 is (a) A.M (b) H.M
(a) 2.35 (b) 2.45 (c) G.M (d) None
(c) 2.55 (d) 2.65

MEASURES OF CENTRAL TENDENCY AND DISPERSION 321


15. If the mean of the following distribution is 6 24. The Algebraic sum of the deviation of a set of
then the value of P is values from their arithmetic mean is
X: 2 4 6 10 P + 5 (a) > 0 (b) = 0
F: 3 2 3 1 2 (c) < 0 (d) None
(a) 7 (b) 5 25. Which one of the following is not a central
(c) 11 (d) 8 tendency?
(a) Mean Deviation
16. If total frequencies of three series are 50,60
and 90 and their means are 12, 15 and 20 (b) Arithmetic mean
respectively, then the mean of their composite (c) Median (d) Mode
series is 26. If the range of a set of values is 65 and maximum
(a) 15.5 (b) 16 value in the set is 83, then the minimum value
in the set is
(c) 14.5 (d) 16.5
(a) 74 (b) 9
17. If the variance of 5, 7, 9 and 11 is 4, then the
(c) 18 (d) None
coefficient of variation is
(a) 25 (b) 15
(c) 17 (d) 19
18. Standard Deviation for the marks obtained by ANSWER KEYS
a student in monthly test in mathematic (out
of 50) as 30, 35,25, 20, 15 is
(a) 25 (b) 50 1 (d) 2 (d) 3 (d) 4 (c)
(c) 50 (d) 30 5 (b) 6 (c) 7 (b) 8 (b)
19. If in a moderately skewed distribution the 9 (d) 10. (a) 11. (a) 12. (d)
values of mode and mean are 32.1 and 35.4
13. (d) 14. (a) 15. (a) 16. (d)
respectively, then the value of the median is
(a) 33.3 (b) 34 17. (a) 18. (c) 19. (c) 20. (c)
(c) 34.3 (d) 33 21. (b) 22. (c) 23. (c) 24. (b)
20. If the standard deviation for the marks
25. (a) 26. (c)
obtained by a student in monthly test is 36,
then the variance is
(a) 36 (b) 6
(c) 1296 (d) None
21. The median of the data 5, 6, 7, 7, 8, 9, 10, 11, 11,
12, 15, 18, 18 and 19 is
(a) 10 (b) 10.5
(c) 11.5 (d) 11
22. The means of 20 items of a data is 5 and if each
item is multiplied by 3, then the new mean
will be
(a) 20 (b) 5
(c) 15 (d) 10
23. The Geometric mean of 3, 6, 24 and 48 is
(a) 6 (b) 8
(c) 12 (d) 24

322 BUSINESS MATHEMATICS, LOGICAL REASONING & STATISTICS (Paper 3) [CA Foundation]
CHAPTER-16
CORRELATION AND REGRESSION

CORRELATION

INTRODUCTION
1. Comparative Study of Measures of Correlation
Co-Variance Karl Pearson’s Spearman’s Concurrent
Coefficient of Rank Correlation Deviation
Correlation Coefficient Method
Who Originated Karl Pearson Karl Pearson Spearman
Based on Actual data Actual data Ranks Direction of Change
Merit It is direct measure It gives direction It is suitable for It is suitable for large
of correlation. as well as degree qualitative data no. of items
of relationship which can only be
between the rank in some
variables. order for abnormal
data.
Is Independent Yes Yes Yes Yes
of Choice of
Origin ?
Is Independent Yes Yes Yes Yes
of Choice of
Scale ?
Is Independent Yes Yes Yes Yes
of Unit of
Measurement
Limits within
which lies –  to +  –1 to +1 –1 to + 1 –1 to +1

xy
Formula COV =
N

CORRELATION AND REGRESSION 323


1. Co-variance can vary from –  to + 
2. r can vary from –1 to +1.
3. r is independent both of choice of origin and scale.
4. The value of r between –X and –Y will be same as between X and Y.
5. Karl Pearson and Rank Method would give the same answer (+1) when both the variables are
either increasing or decreasing.
6. Karl Pearson and Rank Method would give the same answer (-1) when one variable is increas-
ing and the other is decreasing.

1 r2
7. S.E. =
N
8. P.E. = .6745 S.E.
9. If |r| > 6 P.E., r is considered significant.
10. Limits of Population = r  P.E.
11. Coefficient of Determination (r2) is the ratio of explained variance to total variance.
12. Coefficient of Non-Determination (1 – r2) is the ratio of unexplained variance to total variance.
13. Both Coefficient of Determination (r2 ) and Coefficient of Non-Determination (1-r2) can never
be negative and can never exceed 1.

14. Coefficient of Alienation ( 1  r 2 ) is the square root of Coefficient of non-determination


2
(1 –r ).
15. If sum of the Products of Deviations of X and Y series from their means (i.e. Sxy) is zero. r will
be 0 since numerator is 0.
LIST OF FORMULAE
Cov.( X , Y ) xy xy
1. Karl Pearson’s r= = N x  y or
 x  y x 2  y 2

Correlation Coefficient

(a) When Deviations are where, x = (X – X ) and y = (Y – Y )


taken from Actual Mean

(d x )(d y )
d x d y 
N
(b) When Deviations are taken r= 2 (d x ) 2
2
(d y ) 2
d x – d y –
N N

from Assumed Mean Where, dx = (X – A) and dy = (Y – A)

324 BUSINESS MATHEMATICS, LOGICAL REASONING & STATISTICS (Paper 3) [CA Foundation]
fd x fd y
fd x d y 
N
(c) In a Bivariate Frequency r= (fd x ) 2
(fd y )2
fd x 2 – fd y 2 –
N N
Distribution

X Y
XY 
N
(d) When Actual Values of X and Y r= (X )2 (Y ) 2
X 2 – Y 2 –
N N
are considered
2. Spearman’s Rank Correlation
Coefficient
6D 2
(a) If Ranks are not repeated R=1– 3
N N

 1 1 
6  D 2  (m 3 – m)  (m 3 – m)     
(b) If Ranks are repeated R=1–  12 12 
3
N –N

 2C  n 
3. Concurrent Deviation Method r =± 
c  n 

1 r2
4. Standard Error (S.E.) S.E.r =
N

1 r2
5. Probable Error (P.E.) P. E. = 0.6745
N

Explained Variance
6. Coefficient of Determination = (r2) r2 =
Total Variance
7. Coefficient of Non-Determination

Un exp lained Variance


(1 – r2) = 1 – r2 =
Total Variance

8. Coefficient of Alientation = 1 – r2

9. Limits of Population Correlation = r ± Probable Error

CORRELATION AND REGRESSION 325


REGRESSION ANALYSIS

INTRODUCTION
1. Regression Coefficients are Independent of Change of Origin and not of Scale.
2. Coefficient of Correlation and both the Regression Coefficients have same signs (i.e.
either positive or negative.)
3. Both the Regression Coefficients can never exceed l. (i.e. if one is > 1, the other has to be < 1 to
the extent their product is less than or equal to 1).
4. Square Root of Product, of Two Regression Coefficients is r i.e, r = bxy  byx
5. A.M. of two Regression Coefficients is > r.
6. The Covariance, Correlation Coefficient and the two Regression Coefficients have the same
origin.
7. The Regression Lines cut each other at the point of the Mean Values of X and Y.
8. If the Regression Lines coincide, r will be +1 or -1.
9. lf r is + 1 or –1, the two lines of regression are reversible.
10. If the two lines of regression cut each other making an angle of 90°, r will be 0.
11. The farther the two regression lines cut each other, the lesser will be the degree of correlation.
12. In a bivariate. frequency distribution where width of class interval of X variable differs from
that of Y variable, the formulae for calculating b and b are adjusted by multiplying the
xy yx
ix iy
normal formulae by i and respectively.
y ix

List of Formulae
1. Regressi on Equati on of X on Y X – X = b (Y – Y )
xy
Regression Coefficient of X on Y (b )
xy
x xy
(i) If Deviations are taken from Actual Means b =r  =
xy y y 2

(d x )  (d y )
d x d y 
x N
(ii) If Deviations are taken from Assumed Means b =r  =
xy (d y )2
y d y 2 
N

(fd x )  (d x d y )
fd x d y 
x N ix

(iii) In a Bivariate Frequency Distribution b =r  =
xy (fd y )2 iy
y fd y 2 
N

326 BUSINESS MATHEMATICS, LOGICAL REASONING & STATISTICS (Paper 3) [CA Foundation]
2. Regression Equation of Y on X Y– Y =b (X – X )
yx
Regression Coefficient of Y on X (b )
yx
y xy
(i) If Deviations are taken from Actual Means b = r =
yx x x 2

( d x )  ( d y )
d x d y 
y N
2
(ii) If Deviations are taken from Assumed Means b = r = ( d x)
yx x d x 2 –
N

(fd x )  (fd y )
fd x d y  iy
y N 
2
(iii) In a Bivariate Frequency Distribution b = r = ( fd x) ix
yx x d x 2 –
N

(Re gression coefficient of X onY )


3. Coefficient of Correlation (r) r=
 (Re gressionCoefficient of Y on X )

r= bxy  b yx

( X  X c ) 2
4. Standard Error of Estimate of X (S ) S = or s.d. 1 r2
xy xy N x

(Y  Y c ) 2
5. Standard Error of Estimate of Y (S ) S = or s.d. 1  r 2
yx yx N y

6. Total Variation in Y s.d. (Y – Y )2 or s.d. y2

7. Unexplained Variation s.d. (Y – Y c )2


8. Explained Variation = Total Variation – Unexplained Variation
2 2
= s.d. (Y – Y ) – s.d. (Y – Y )
c
Explained var iation
9. Coefficient of Determination (2) = r2 =
Total var iation

 (Y – Y ) 2   (Y – Yc ) 2
=
 (Y – Y ) 2

CORRELATION AND REGRESSION 327


10. Coefficient of

Un exp lained Variation


Non-determination (1 – r2) 1 – r2 =
Total Variation

 (Y – Yc )2
=
 (Y – Y )2

Un exp lained var iation


11. Standard Deviation (Error) of Y Syx =
N

(Y  Yc ) 2
=
N

328 BUSINESS MATHEMATICS, LOGICAL REASONING & STATISTICS (Paper 3) [CA Foundation]
Correlation and Regression Analysis
Correlation analysis helps us to find an association or lack of it between the two variables X and Y. If change
in one variable is reciprocated by change in other, directly or inversely, then the variables are said to be
correlated, otherwise they are said to be dissociated or uncorrelated or independent. There are two types
of correlation positive and negative.
Positive correlation: both move in same direction. Increases (decrease) in one variable brings about increase
(decrease) in other variable. E.g. Height and weight, Yield and rainfall etc.
Negative correlation: both move in opposite directions. Increase (decrease) in one variable results in decrease
(increase) in other variable. E.g. price and demand, profits of insurance company and the number of claims.
If movement in one does not result in movement in other we say the variables are uncorrelated.
E.g. Shoe-size and intelligence
Spurious correlation or non-sense correlation is a relation without causal relation.
There are four methods to study correlation :
(1) Scatter diagram
(2) Karl Pearson’s Product movement correlation coefficient
(3) Spearman’s Rank correlation coefficient
(4) Coefficient of concurrent deviations.

Method of scatter diagram: Points (xi , yi) are plotted. The totality of the points represents scatter diagram.
If the points plotted are concentrated from lower left corner to upper right corner, it is positive correlation.
If the points plotted are concentrated from upper left corner to lower right corner, it is negative correlation.
If the points are scattered without any pattern the variables are uncorrelated.
Merits :
(1) Simple and easy to understand
(2) Can be used for all types of relations whether linear or curvilinear.

Demerits:
(1) It fails to measure the extent of correlation between the variables.

Karl Pearson’s Product movement correlation coefficient: This method is the best method for studying
correlation between two variables. However the relation should be linear. The measure is denoted by r or
.
Properties of r:
(1) It is a unit-free measure.
(2) It is not affected by shift of origin or change of scale.
(3) It lies between -1 and 1 including the limits -1 < r < 1.
r = 1 : indicates perfect positive relation between the two variables. All the points of scatter diagram will lie
on a straight line from lower left to upper right corner.

CORRELATION AND REGRESSION 329


r = -1 indicates perfect negative relation. All the points of scatter diagram will lie on a straight line from
upper left to lower right corner.
r = 0 indicates lack of linear correlation. The variables may be non-linearly related or may be independent.
Spearman’s rank correlation coefficient: This method is used for qualitative data with assigned ranks e.g.
beauty and intelligence, agreement or not between two judges. It can also be used to get a firsthand
impression about correlation between two variables.
Coefficient of concurrent deviations: a very simple and casual method, used when we are not serious about
the magnitude of the two variables.
Probable error: Method of obtaining correlation coefficient of population.

Probable error :

If r < P.E. then correlation in the population is not significant (there is no evidence).
If r > 6(P.E.) then correlation in the population is significant(presence of correlation is certain).
Probable limits of population correlation coefficient are r + P.E.
P.E. is never negative.
Regression : Establishing functional relationship between two variables. Sir Francis Galton initiated the
study of regression.
Uses: 1) It helps us to predict the value of dependent variable from the given value of independent variable.
2) It helps us to know the rate of change
Method of least square : The method which minimizes  e2 .
e is the error or residue. It is the difference between observed value and estimated value.

The line obtained by this method is known as line of best fit.


In case of y on x vertical distances are minimized.
In case of x on y horizontal distances are minimized.
Normal equation

solving the above two gives formula of byx and a

explained variance
Coefficient of determination: R2 or r2 =
Total variance

330 BUSINESS MATHEMATICS, LOGICAL REASONING & STATISTICS (Paper 3) [CA Foundation]
Range of R2 = 0 < R2 < 1

Coefficient of non-determination = 1 – r2

Coefficient of Alienation = 1 -r 2

Standard error of Y on X = y 1 -r2

Standard error of X on Y =  x 1 - r 2

Properties of regression lines


(1) coefficient byx and bxy are independent of shift of origin but not of change of scale.

(2) Two lines intersect at ( x , y )


(3) R is the geometric mean of two regression coefficient

(4) r=+ b yx  bxy

(5) If r2 is = 1 i.e. r = +1 or -1 two lines will be coincident.

(6) If r2 is = 0 i.e. r = 0 two lines will be perpendicular.

Bivariate data : data collected on two variable


Distribution with two variables is known as bivariate frequency distribution.
Bivariate table
y 0-20 20-40 40-60 60-80 total
x
0-10 3 5 1 1 10
10-20 8 20 10 7 45
20-30 4 9 12 25
total 15 25 20 20 80
There are two marginal distribution and 7 (4+3) conditional distributions.

CORRELATION AND REGRESSION 331


8. Correlation coefficient is —————— of the
units of measurement.
CLASS WORK (a) dependent (b) independent
(c) both (d) none
9. The value of correlation coefficient lies
1. –—————— is concerned with the between
measurement of the “strength of association” (a) –1 and +1
between variables. (b) –1 and 0
(a) correlation (b) regression (c) 0 and 1 Inclusive of these two values
(c) both (d) none (d) none.
10. Correlation coefficient can be found out by
2. —————— gives the mathematical
relationship of the variables. (a) Scatter Diagram (b) Rank Method
(c) both (d) none.
(a) correlation (b) regression
11. Covariance measures _________ variations of
(c) both (d) none two variables.
3. When high values of one variable are (a) joint (b) single
associated with high values of the other & low (c) both (d) none
values of one variable are associated with low
12. In calculating the Karl Pearson’s coefficient of
values of another, then they are said to be
correlation it is necessary that the data should
(a) positively correlated be of numerical measurements. The
(b) directly correlated statement is
(c) both (a) valid (b) not valid
(c) both (d) none
(d) none
13. Rank correlation coefficient lies between
4. If high values of one tend to low values of the
(a) 0 to 1
other, they are said to be
(b) –1 to +1 inclusive of these value
(a) negatively correlated
(c) –1 to 0
(b) inversely correlated
(d) both
(c) both
14. A coefficient near +1 indicates tendency for
(d) none
the larger values of one variable to be
5. Correlation coefficient between two variables associated with the larger values of the other.
is a measure of their linear relationship .
(a) true (b) false
(a) true (b) false
(c) both (d) none
(c) both (d) none
15. In rank correlation coefficient the association
6. Correlation coefficient is dependent of the need not be linear.
choice of both origin & the scale of
(a) true (b) false
observations.
(c) both (d) none
(a) True (b) false
16. In rank correlation coefficient only an
(c) both (d) none
increasing/decreasing relationship is required.
7. Correlation coefficient is a pure number.
(a) false (b) true
(a) true (b) false
(c) both (d) none
(c) both (d) none

332 BUSINESS MATHEMATICS, LOGICAL REASONING & STATISTICS (Paper 3) [CA Foundation]
17. Great advantage of ____________ is that it can (c) both (d) none
be used to rank attributes which can not be 26. The lines of regression passes through the
expressed by way of numerical value . points, bearing _________ no. of points on both
(a) concurrent correlation sides
(b) regression (a) equal (b) unequal
(c) rank correlation (c) zero (d) none
(d) none 27. Under Algebraic Method we get —————
18. The sum of the difference of rank is linear equations .

(a) 1 (b) –1 (a) one (b) two

(c) 0 (d) none. (c) three (d) none

19. Karl Pearson’s coefficient is defined from 28. In linear equations Y = a + bX and X= a + bY ‘a‘ is
the
(a) ungrouped data (b) grouped data
(a) intercept of the line
(c) both (d) none.
(b) slope (c)both
20. Correlation methods are used to study the (d) none
relationship between two time series of data
which are recorded annually, monthly, weekly, 29. In linear equations Y = a + bX and X = a + bY ‘ b ‘
daily and so on. is the

(a) True (b) false (a) intercept of the line

(c) both (d) none (b) slope of the line

21. Age of Applicants for life insurance and the (c) both
premium of insurance – correlation is (d) none
(a) positive (b) negative 30. The regression equations Y = a + bX and X = a +
(c) zero (d) none bY are based on the method of

22. “Unemployment index and the purchasing (a) greatest squares (b) least squares
power of the common man“ ——Correlation (c) both (d) none
is 31. The line Y = a + bX represents the regression
(a) positive (b) negative equation of
(c) zero (d) none (a) Y on X (b) X on Y
23. Production of pig iron and soot content in (c) both (d) none
Durgapur – Correlations are 32. The line X = a + bY represents the regression
(a) positive (b) negative equation of
(c) zero (d) none (a) Y on X (b) X onY
24. “Demand for goods and their prices under (c) both (d) none
normal times” —— Correlation is 33. Two regression lines always intersect at the
(a) positive (b) negative means.
(c) zero (d) none (a) true (b) false
25. ___________ is a relative measure of (c) both (d) none
association between two or more variables. 34. r, bxy, byx all have ______ sign.
(a) Coefficient of correlation (a) different (b) same
(b) Coefficient of regression (c) both (d) none

CORRELATION AND REGRESSION 333


35. The regression coefficients are zero if r is equal 43. In the regression equation x on y, X = 35/8 – 2Y
to /5, bxy is equal to
(a) 2 (b) –1 (a) –2/5 (b) 35/8
(c) 1 (d) 0 (c) 2/5 (d) 5/2
36. The regression lines are identical if r is equal 44. The square of coefficient of correlation ‘r’ is
to called the coefficient of
(a) +1 (b) –1 (a) determination (b) regression
(c) 1 (d) 0 (c) both (d) none
37. The regression lines are perpendicular to each 500
other if r is equal to 45. A relationship r2 = 1 – is not possible
300
(a) 0 (b) +1
(a) true (b) false
(c) –1 (d) 1
(c) both (d) none
38. Feature of Least Square regression lines are—
—— The sum of the deviations at the Y’s or 46. Whatever may be the value of r, positive or
the X’s from their regression lines are zero. negative, its square will be
(a) true (b) false (a) negative only (b) positive only
(c) both (d) none (c) zero only (d) none only
39. The coefficient of determination is defined by 47. Simple correlation is called
the formula (a) linear correlation
unexplained variance (b) nonlinear correlation
(a) r2 = 1 -
total variance (c) both (d) none
48. A scatter diagram indicates the type of
explained variance
(b) r2 = correlation between two variables.
total variance
(a) true (b) false
(c) both
(c) both (d) none
(d) none
49. If the pattern of points (or dots) on the scatter
40. If the line Y = 13 –3X /2 is the regression diagram shows a linear path diagonally across
equation of y on x then byx is the graph paper from the bottom left- hand
2 -2 corner to the top right, correlation will be
(a) (b) (a) negative (b) zero
3 3
(c) positive (d) none
3 -3
(c) (d) 50. The correlation coefficient being +1 if the
2 2 slope of the straight line in a scatter diagram is
41. In the line Y = 19 – 5X/2 is the regresson (a) positive (b) negative
equation x on y then bxy is,
(c) zero (d) none
(a) 19/2 (b) 5/2
51. The correlation coefficient being –1 if the
(c) –5/2 (d) –2/5 slope of the straight line in a scatter diagram is
42. The line X = 31/6 — Y/6 is the regression
(a) positive (b) negative
equation of
(c) zero (d) none
(a) Y on X (b) X on Y
(c) both (d) we can not say

334 BUSINESS MATHEMATICS, LOGICAL REASONING & STATISTICS (Paper 3) [CA Foundation]
52. The more scattered the points are around a 61. In case ‘Years of education and income’———
straight line in a scattered diagram the _______ (a) positive correlation
is the correlation coefficient.
(b) negative correlation
(a) zero (b) more
(c) no correlation
(c) less (d) none
(d) none
53. If the values of y are not affected by changes
in the values of x, the variables are said to be 62. In case ‘Amount of rainfall and yield of crop’—

(a) correlated (b) uncorrelated
(a) positive correlation
(c) both (d) zero
(b) negative correlation
54. If the amount of change in one variable tends
to bear a constant ratio to the amount of (c) no correlation
change in the other variable, then correlation (d) none
is said to be 63. For calculation of correlation coefficient, a
(a) non linear (b) linear change of origin is
(c) both (d) none (a) not possible (b) possible
55. Variance may be positive, negative or zero. (c) both (d) none
(a) true (b) false 64. The relation rxy = cov (x,y)/  x ,  y is
(c) both (d) none
(a) true (b) false
56. Covariance may be positive, negative or zero.
(c) both (d) none
(a) true (b) false
65. A small value of r indicates only a _________
(c) both (d) none linear type of relationship between the
57. Correlation coefficient between x and y = variables.
correlation coefficient between u and v (a) good (b) poor
(a) true (b) false (c) maximum (d) highest
(c) both (d) none 66. Two regression lines coincide when
58. In case ‘ The ages of husbands and wives’ —— (a) r= 0 (b) r= 2
—— correlation is
(c) r=  1 (d) none
(a) positive (b) negative
67. Neither y nor x can be estimated by a linear
(c) zero (d) none function of the other variable when r is equal
59. In case ‘Shoe size and intelligence’ to
(a) positive correlation (a) +1 (b) –1
(b) negative correlation (c) 0 (d) none
(c) no correlation 68. When r = 0 then cov (x,y) is equal to
(d) none (a) +1 (b) –1
60. In case ‘Insurance companies’ profits and the (c) 0 (d) none
no of claims they have to pay “—— 69. When the variables are not independent, the
(a) positive correlation correlation coefficient may be zero
(b) negative correlation (a) true (b) false
(c) no correlation (c) both (d) none
(d) none

CORRELATION AND REGRESSION 335


70. bxy is called regression coefficient of 80. Which is true?
(a) x on y (b) y on x
σ σy
(c) both (d) none (a) byx = r x (b) byx = r
σy σx
71. byx is called regression coefficient of
(a) x on y (b) y on x σ xy σ yy
(c) byx = r (d) byx = r
(c) both (d) none σx σx
72. The slopes of the regression line of y on x is
81. Maximum value of Rank Correlation
(a) byx (b) bxy coefficient is
(c) bxx (d) byy (a) –1 (b) +1
73. The slopes of the regression line of x on y is (c) 0 (d) none
(a) byx (b) bxy 82. The partial correlation coefficient lies
(c) 1/bxy (d) 1/byx between
74. The angle between the regression lines (a) –1 and +1 inclusive of these two value
depends on (b) 0 and + 1
(a) correlation coefficient (c) –1 and 1
(b) regression coefficient (d) none
(c) both 83. r12 is the correlation coefficient between
(d) none (a) x1 and x2 (b) x2 and x1
75. If x and y satisfy the relationship y = –5 + 7x, (c) x1 and x3 (d) x2 and x3
the value of r is
84. r12 is the same as r21
(a) 0 (b) – 1
(a) true (b) false
(c) +1 (d) none
(c) both (d) none
76. If byx and bxy are negative, r is
85. In case of employed persons ‘Age and income’
(a) positive (b) negative correlation is
(c) zero (d) none (a) positive (b) negative
77. Correlation coefficient r lie between the (c) zero (d) none
regression coefficients byx and bxy
86. In case ‘Speed of an automobile and the
(a) true (b) false distance required to stop the car after applying
(c) both (d) none brakes’ – correlation is
78. Since the correlation coefficient r cannot be (a) positive (b) negative
greater than 1 numerically, the product of the (c) zero (d) none
regression must
87. In case ‘Sale of woolen garments and day
(a) not exceed 1 (b) exceed 1 temperature’–––– correlation is
(c) be zero (d) none (a) positive (b) negative
79. The correlation coefficient r is the __________ (c) zero (d) none
of the two regression coefficients byx and bxy
88. In case ‘Sale of cold drinks and day
(a) A.M (b) G.M temperature’ –––––– correlation is
(c) H.M (d) none (a) positive (b) negative
(c) zero (d) none

336 BUSINESS MATHEMATICS, LOGICAL REASONING & STATISTICS (Paper 3) [CA Foundation]
89. In case of ‘Production and price per unit’ – 98. The idea of product moment correlation was
correlation is given by
(a) positive (b) negative (a) R.A. Fisher
(c) zero (d) none (b) Sir Francis Galton
(c) Karl Pearson
90. If slopes at two regression lines are equal then
r is equal to (d) Spearman
99. If the correlation between two variables X and
(a) 1 (b) 1 Y is negative, the regression coefficient of Y
(c) 0 (d) none on X is
91. Co–variance measures the joint variations of (a) positive (b) negative
two variables. (c) not certain (d) none
(a) true (b) false 100. The unit of correlation coefficients is
(c) both (d) none (a) kg (b) per cent
92. The minimum value of correlation coefficient (c) not existing (d) none
is 101. The value of correlation co-efficient r is from
(a) 0 (b) –2 (a) 0 to 1 (b) –1 to 0
(c) 1 (d) –1 (c) –1 to 1 (d) – 3 to 3
102. If r = 1, the relation between the two variables
93. The maximum value of correlation coefficient
X and Y is :
is
(a) Y is proportional to X
(a) 0 (b) 2
(b) Y is inversely proportional to X
(c) 1 (d) –1 (c) Y is equal to X
94. When r = 0 , the regression coefficients are (d) none
(a) 0 (b) 1 103. If r = 0, the variables X and Y are
(c) –1 (d) none (a) linearly related (b) independent
95. The regression equation of Y on X is, 2x + 3Y + (c) not linearly related (d) none
50 = 0. The value of bYX is 104. If r = –1, the relation between X and Y is of the
type:
(a) 2/3 (b) – 2/3
(a) When Y increases, X also increases
(c) –3/2 (d) none
(b) When Y decreases, X also decreases
96. In Method of Concurrent Deviations, only the (c) X is equal to – Y
directions of change ( Positive direction /
(d) when Y increases, X decreases
Negative direction ) in the variables are taken
proportionally
into account for calculation of
105. The correlation co-efficient between x and y
(a) coefficient of S.D is 0.78, hence the correlation co-efficient
(b) coefficient of regression. between – x and – y is
(c) coefficient of correlation (a) – 0.78 (b) 0.78
(d) (c) – 0.78 or 0.78 (d) none
none 106. If the points of the scatter diagram are not in a
97. The formula of correlation coefficient was straight line, they indicate ......... of the linear
developed in the year relationship.
(a) 1910 (b) 1890 (a) positive (b) negative
(c) 1908 (d) none (c) absence (d) can't say

CORRELATION AND REGRESSION 337


2 (a) correlation coefficient will be positive
107. If d = 0 then there is ................ correlation
(b) correlation coefficient will be negative
between the two variables.
(c) expected values are incompatible
(a) no
(d) none
(b) perfect negative
115. If r is the correlation coefficient between X
(c) perfect positive and Y, the correlation coefficient between aX
(d) partial correlation + b and Y is (a > 0).
108. The correlation co-efficient 0.6 indicates twice (a) ar (b) ar + b
the relationship than the correlation co- (c) a r 2
(d) r
efficient 0.3.
116. If two variables are perfectly positively
(a) yes (b) no correlated, r =
(c) may be (d) can't say (a) –1 (b) 1
109. If two variables are independent. (c) 0 (d) between 0 to 1
(a) r =1 (b) r=0 117. If two variables are negatively correlated, r =
(c) r = –1 (a) –1
(d) r = (a) and (c)
(b) +1
110. The correlation co-efficient between x and y
is 0.6 hence the correlation co-efficient (c) between 0 and 1
between x + 0.2 and y + 0.2 is (d) between –1 and 0
(a) 0.8 (b) –0.6 118. If the simultaneous changes in two variables
(c) 0.6 (d) 0.4 are in the same direction, the correlation
111. The correlation co-efficient between heights coefficient between them is
and weights of 1000 persons is 0.6 when (a) –1 (b) 1
heights and weights are expressed in inches (c) between 0 and1
and in lbs respectively. If heights and weights (d) between –1 and 0
are expressed in cms. and kg. respectively, the 119. If the simultaneous changes in two variables
correlation co - efficient is are in opposite direction, the correlation
(a) 0.6 (b) –0.6 coefficient between them is
(c) can't say (d) none (a) –1 (b) 1
112. If r = 0, the two variables are (c) between 0 and 1
(a) uncorrelated (d) between –1 and 0
(b) perfectly related 120. The correlation co-efficient r is independent
(c) linearly independent of change of
(d) none (a) origin (b) scale
113. A positive significant correlation between the (c) (a) and (b) both (d) none
number of shoes produced and the steel 121. If correlation co-efficient r between x and y is
produced per year is : 0.5 then r between 4x + 2 and y is
(a) a nonsense correlation (a) 2.2 (b) 0.5
(b) a spurious correlation (c) –0.5 (d) 0
(c) a meaningless correlation 122. If correlation co-efficient r between x and y is
0.5 then r between – x and – y is
(d) all the above.
(a) –1 (b) 0.5
114. Given the expected values for two variable X (c) –0.5 (d) 0
and Y as : E(X) = 8, E(X2) = 10, E(Y) = 3, E(Y2) = 20
and E(XY) = 16 . We conclude that

338 BUSINESS MATHEMATICS, LOGICAL REASONING & STATISTICS (Paper 3) [CA Foundation]
123. If correlation co-efficient r between x and y is (a) 0.6 (b) 0.66
0.5 then r between x and – y is (c) –0.66 (d) 0
(a) –1 (b) 0.5 132. The co-efficient of correlation from following
(c) –0.5 (d) 0 results is
124. If all the points, in a scatter diagram are in one
2
line then r =  x = 140,  y = 150,  (x  10) =
n = 10,
(a) –1 (b) 1 2
(c) (a) and (b) (d) (a) or (b) 180,  (y  15) = 215,  (x  10)(y  15) =
125. The value of r2 lies between 60
(a) 0 and 1 (b) –1 and 1 (a) –0.5 (b) 0.91
(c) –1 and 0 (d) none
(c) –0.9 (d) none
126. If r is the simple correlation, the quantity r2 is
133. If x and y are measured from their respective
known as
means, the correlation co-efficient from the
(a) coefficient of determination
following data
(b) coefficient of non determination
(c) coefficient of alienation n = 20,  xy = 400, Sx = 16, Sy = 2.5. is
(d) none
(a) –0.5 (b) 0.5
127. If r is the simple correlation, the quantity (1 –
r2) is called : (c) –1 (d) 1
(a) coefficient of determination 134. The correlation co-efficient between x and y
(b) coefficient of nondetermination from the following data is
(c) coefficient of alienation 2

(d) none
n = 8, x = 408,  y = 272,  (x  51) =
2
128. The ranks of ten students in two subjects A 42,  (y  34) = 60,  (x  51)(y  34) = –
and B are as follows.
16 is
A 3 5 8 4 7 10 2 1 6 9
B 7 6 2 8 4 3 10 9 5 1 (a) –0.32 (b) 0.32
Spearman's rank correlation co-efficient is (c) –1 (d) 1
(a) –0.94 (b) 0.94 135. For a bivariate sample n = 16, r = 0.8, the
(c) 0.5 (d) 0 probable error of r is
129. The rank correlation co-efficient between (a) –0.06 (b) 0.06
marks in statistics and marks in accountancy of (c) –1 (d) 1
10 students is 0.4. Later on its was noticed that 136. For a bivariate sample r = 0.5 and the probable
the difference in ranks of a student was taken error of r = 0.125, the value of number of pairs
as 3 instead of 7. The correct value of the is
correlation co-efficient is (a) 16 (b) 6
(a) –0.4 (b) 0.4 (c) 10 (d) 12
(c) 0.16 (d) 0.5 137. Find co-efficient of correlation by method of
130. If the sum of squares of differences in ranks of concurrent deviations.
two variables x and y is 126 and the correlation x : 78 80 84 89 81 83 87 94
co-efficient is – 0.5. The number of pairs are 91
(a) 8 (b) 7 y : 190 178 158 132 164 152 128 88
(c) 5 (d) 10 98
131. The co-efficient of correlation from following (a) –0.32 (b) 0.32
results is (c) –1 (d) 1
Average of x = 10.5, Average of y = 13.9, S. d. of 138. Co–efficient of correlation between two
variables x and y is 0.8 and their covariance is
x = 3.5, S. d of y = 4.1 n = 10,  xy = 1364 20. If the variance of x series is 16, the standard

CORRELATION AND REGRESSION 339


deviation of y series is 145. The term regression was introduce by :
(a) 32 (b) 12 (a) R.A. Fisher
(c) –6.25 (d) 6.25 (b) Sir Frances Galton
139. The correlation co-efficient between two (c) Karl Pearson (d) None
variables is 0.8. What is the co-efficient of 146. If X and Y are two variables, there can be at
determination? most :
(a) 0.32 (b) 0.64 (a) One regression line
(c) 6.4 (d) 0.8 (b) two regression lines
140. Find the co-efficient of correlation, using the (c) three regression lines
method of concurrent deviations, between (d) an infinite number of regression lines
supply and demand of an item for a ten year 147. In a regression line of Y on X, the variable X is
period given below. known as :
Year : 1981 1982 1983 1984 1985 1986 1987 1988 1989 (a) independent variable
1990
Supply : 125 160 164 174 155 170 165 162 172
(b) regressor
175 (c) explanatory variable
Demand : 115 125 192 190 165 174 124 127 152
(d) all the above
169
148. Regression equation is also named as :
(a) 0.75 (b) –0.75
(a) prediction equation
(c) –1 (d) 1
(b) estimating equation
141. Given that the correlation coefficent between
(c) line of average relationship
x and y is 0.5, what is the correlation coefficient
(d) all the above
between 2x – 4 and 3 – 2y ?
149. In the regression line Y = a + bX, b is called
(a) –0.5 (b) 0.5
(a) slope of the line
(c) –1 (d) 1
(b) intercept of the line
142. If the covariance between two variables is
(c) nether (a) nor (b)
positive, it means that
(d) both (a) and (b)
(a) the variables would change in the same
direction 150. If b YX and b XY are two regression coefficients,
they have
(b) the variables would change in the
opposite direction (a) same sign
(c) the variables would not change. (b) opposite sign
(d) none (c) either same or opposite signs
143. If the correlation between two variables is (d) nothing can be said
zero, it implies that : 151. The property that bYX and bXY and r have same
(a) two variable are independent signs, is called
(b) two variables do not have negative (a) fundamental property
correlation (b) signature property
(c) the two variables are not linearity related (c) magnitude property (d) none
(d) all the above. 152. The average of the two regression coefficients
144. Correlation between the direction of is always greater than or equal to the
deviations is calculated by the method of : correlation coefficient is called
(a) product moments (a) fundamental property
(b) rank correlation (b) signature property
(c) coefficient of concurrent deviation (c) magnitude property
(d) Kedndall's (d) mean property

340 BUSINESS MATHEMATICS, LOGICAL REASONING & STATISTICS (Paper 3) [CA Foundation]
153. If byx = – 0.8, bxy = – 0.45, then r = 162. The probable change in the value of y with
(a) 0.5 (b) –0.6 unit change in the value of x can be given by
(c) 0.6 (d) –0.36 (a) bxy (b) byx
154. The two regression co-efficients are 0.8 and (c) r (d) none
0.2, hence correlation co-efficient is 163. Two regression lines intersect each other at
(a) 0.4 (b) – 0.16 (a) x, y (b) Sx, Sy
(c) 0.16 (d) –0.4
(c) r (d) x, y
155. One of the regression co-efficients of two
164. The property if X and Y are independent, then
perfectly correlated variables is 0.5, hence the
bYX = bXY = 0 is called :
other regression co-efficient is
(a) fundamental property
(a) 0.5 (b) –0.5
(b) mean property
(c) 2 (d) –2
(c) independence property
156. The two regression lines are perpendicular to (d) magnitude property
each other hence the correlation co-efficient
165. The lines of regression intersect at the point :
between them is
(a) (X, Y) (b) (x, y )
(a) 0.5 (b) –1
(c) (0, 0) (d) (1, 1)
(c) 1 (d) 0
166. The coordinates ( x , y ) satisfy the lines of
157. The greater the angle between the regression
lines, .......... the correlation between the regression of :
variables. (a) Y on X
(a) lesser (b) higher (b) X on Y
(c) both the regression lines
(c) medium (d) none
(d) none of the two regression are:
158. If bYX > 1, then bXY is
167. If r = ± 1, the two lines of regression are :
(a) less than 1
(a) coincident
(b) greater than 1 (b) parallel
(c) equal to 1 (d) equal to 0 (c) perpendicular to each other
159. The property bYX > 1 implies that bXY < 1 is known (d) none
as : 168. If r = 1, the angle between the two lines of
(a) fundamental property regression is :
(b) signature property (a) zero degree (b) ninety degree
(c) magnitude property (d) none (c) sixty degree (d) thirty degree
160. If X and Y are independent, the value of 169. If r = 0, the lines of regression are :
regression coefficient bYX is equal to : (a) coincident
(a) 0 (b) parallel
(b) 1 (c) perpendicular to each other
(c) (d) none of the above
(d) any positive value 170. If r = 0, the angle between the two lines of
regression is:
161. The regression equation of y on x is y = – 3 +
0.5x and that of x on y is x = – 7 + By. (a) zero degree (b) ninety degree
(c) sixty degree (d) thirty degree
If the correlation co-efficient between x and y
is 0.1, then B = 171. If a constant 50 is subtracted from each of the
value of X and Y, the regression coefficient is:
(a) 0.5 (b) –0.5
(a) reduced by 50
(c) 0.02 (d) –0.02
(b) (1/50)th of the original regression
coefficient

CORRELATION AND REGRESSION 341


(c) increased by 50 Correlation co-efficient = 0.8
(d) not changed (a) 2.07 (b) 207
172. If each observation in the set of values ( X, Y) is (c) –20.7 (d) 20.7
divided by 100, the regression coefficient of 180. From the following information estimated
Yon X is price in Bombay when the price in Calcutta is
(a) increased by 100 ` 70 is
(b) decreased by 100 Average price in Calcutta = ` 65
(c) (1/100)th of bYX Average price in Bombay = ` 67
(d) none S. D. of price in Calcutta = ` 2.5 S. D. of
173. If each of X variate is divided by 5 and of Y by price of Bombay = ` 3.5
10, then by coded values bYX is Correlation co-efficient between prices = 0.8
(a) same as bYX (b) half of bYX (a) 72.6 (b) 73.6
(c) twice by (d) none (c) 71.6 (d) 70.6
174. If each value of X is divided by 2 and of Y is 181. If each value of X is multiplied by 10 and of Y
multiplied by 2, then bYX by coded values is by 20, bXY , the regression coefficient by coded
(a) same as bYX values is
(b) half of bYX (a) same as bXY
(c) four time of bYX
(d) eight times of bYX (b) half of bXY
175. If from each value of X and Y, constant 25 is (c) four time of bXY
subtracted and then each value is divided by (d) one fourth of bXY
10, the coded bYX is
182 If the two lines of regression are perpendicular
(a) same as bYX
to each other, the relation between the two
(b) 2½ times of bYX
regression coefficients is:
(c) 25 time of bYX
(a) bYX = bXY (b) bYX . bXY = 1
(d) 10 times of bYX
176. Two regression lines are 3x = y and 4y = 3x and (c) bYX < bXY (d) bYX = bXY = 0
Sx = 2, then r = 183. If the two lines of regression are coincident,
(a) 0.05 (b) –0.05 the relation between the two regression
(c) 0.5 (d) 0 coefficients is :
177. Two regression lines are 3x = y and 4y = 3x and (a) bYX = bXY (b) bYX . bXY = 1
Sx = 2, then Sy = (c) bYX < bXY (d) bYX = – bXY
(a) 2 (b) –2 184. Regression coefficient is independent of
(c) 3 (d) 5 change of
178. The regression equation of y on x from the (a) Origin (b) scale
following data is (c) both origin and scale
x y (d) neither origin nor scale
Average 25.5 40 185. The geometric mean of the two regression
S.d. 2.4 6 coefficients bxy and bxy is equal to
Correlation co-efficient = 0.8 (a) r (b) r2
(a) y = – 11 + 2x (b) y = – 11 – 2x (c) 1 (d) none
(c) y = 11 + 2x (d) y = – 11 – 2x
186. If the two lines of regression are x + 2y – 5 = 0
179. From the following data the estimated value
and 2x + 3y – 8 = 0, the regression line of y on x
of x for y = 25 is
is:
x y
(a) x + 2y – 5 = 0 (b) 2x + 3y – 8 = 0
Average 25.5 40
(c) any of the two lines (d) none
S.d. 2.4 6
342 BUSINESS MATHEMATICS, LOGICAL REASONING & STATISTICS (Paper 3) [CA Foundation]
187. If the two lines of regression are x + 2y – 5 = 0 (a) 15 (b) 20
and 2x + 3y – 8 = 0, the means of X and Y are (c) 25 (d) 35
(a) –3, 4 (b) 2, 4 194. The two regression equations are x = 5y – 7 and
(c) 1, 2 (d) none y = 0.1x + 1.7, the values of x , y are
188. The means of x and y are 16 and 20 respectively. (a) 3, 3 (b) 3, 2
Their standard deviations are 6 and 8 (c) 2, 5 (d) 3, 5
respectively. The correlation co-efficient
195. The two regression equations are x = 5y – 7 and
between them is 0.6. The equation of
y = 0.1x + 1.7, the value of r is
regression line of y on x is
(a) 0.71 (b) 0.75
(a) y = 7.2 + 0.8x (b) y = –7.2 + 0.8x
(c) y = 7.2 – 0.8x (d) y = –7.2 – 0.8x (c) 0.25 (d) 0.35
189. The means of x and y are 16 and 20 respectively. 196. The regression lines are X + 2Y – 5 = 0, 2X + 3Y –
Their standard deviations are 6 and 8 8 = 0 and V(X) = 12, the value of V(Y) is
respectively. (a) 16 (b) 4
The correlation co-efficient between them is (c) 3/4 (d) 4/3
0.6. The equations of regression line of x on y 197. If the two lines of regression are x = (–1/18)y +
is l; y = – 2x + m
(a) x = –7 + 0.45y (b) x = 7 – 0.45y and the mean of the distribution is at (–1, 2)
the values of l and m are :
(c) x = –7 – 0.45y (d) x = 7 + 0.45y
(a) l = 8/9, m = –5
190. The following information is obtained
regarding age of husbands and wives. (b) l = 9/8, m = –3
Average age of husbands = 30.3 years (c) l = –10/9, m = –4
Average age of wives = 24.8 years (d) l = –8/9, m = 0
S.D. of age of husbands = 5.4 years S.D. of 198. If y = mX + 4 and X = 4Y + 5 are the regression
age of wives = 4.5 years lines of Y on X and X on Y, then m lies between
The correlation co-efficient between age of the values
husbands and wives = 0.8. If the age of a (a) 0 and 1 (b) 0 and 0.5
husband is 50 years then estimate the age of (c) 0 and 0.25 (d) none
his wife is 199. Compute the value of y for x = 48 on the basis
(a) 28 (b) 30 of the following information.
(c) 35 (d) 38 x y
191. The equations of lines of regression for two Mean 40 45
variables x and y are 3x + 2y – 26 = 0 and S.D. 10 9
6x + y – 31 = 0, and the variance of x is 25. Then Karl Pearson's correlation co-efficient
x , y are between x and y = 0.50
(a) 4,7 (b) 5,6 (a) 48 (b) 48.6
(c) 4,6 (d) 6,7 (c) 25 (d) 35
192. The equations of lines of regression for two 200. If the two lines of regression in a bivariate
variables x and y are 3x + 2y – 26 = 0 and 6x + y – distribution are X + 9Y = 7 and Y + 4X = 16, then
31 = 0, and the variance of x is 25. Then r is Sx : Sy is
(a) 0.5 (b) –0.5 (a) 3 : 2 (b) 2:3
(c) 0 (d) 1 (c) 9 : 4 (d) 4:9
193. The equations of lines of regression for two
variables x and y are 3x + 2y – 26 = 0 and
6x + y–31 = 0, and the variance of x is 25. Then ---0---0---
sy is

CORRELATION AND REGRESSION 343


8. What is spurious correlation?
HOME WORK-1 (a) It is a bad relation between two variables.
(b) It is very low correlation between two
variables.
(c) It is the correlation between two
1. Bivariate Data are the data collected for
variables having no causal relation.
(a) Two variables (d) It is a negative correlation.
(b) More than two variables 9. Scatter diagram is considered for measuring
(c) Two variables at the same point of time (a) Linear relationship between two
(d) Two variables at different points of time. variables
2. For a bivariate frequency table having (p + q) (b) Curvilinear relationship between two
classification the total number of cells is variables
(a) p (b) p+q (c) Neither (a) nor (b)
(c) q (d) pq (d) Both (a) and (b).
3. Some of the cell frequencies in a bivariate 10. If the plotted points in a scatter diagram lie
frequency table may be from upper left to lower right, then the
(a) Negative (b) Zero correlation is
(a) Positive (b) Zero
(c) a or b (d) Non
(c) Negative (d) None
4. For a p x q bivariate frequency table, the
11. If the plotted points in a scatter diagram are
maximum number of marginal distributions is
evenly distributed, then the correlation is
(a) p (b) p+q
(a) Zero (b) Negative
(c) 1 (d) 2 (c) Positive (d) (a) or (b).
5. For a p x q classification of bivariate data, the 12. If all the plotted points in a scatter diagram lie
maximum number of conditional distributions on a single line, then the correlation is
is (a) Perfect positive
(a) p (b) p+q (b) Perfect negative
(c) pq (d) p or q (c) Both (a) and (b)
6. Correlation analysis aims at (d) Either (a) or (b).
(a) Predicting one variable for a given value 13. The correlation between shoe-size and
of the other variable intelligence is
(b) Establishing relation between two (a) Zero (b) Positive
variables (c) Negative (d) None
(c) Measuring the extent of relation 14. The correlation between the speed of an
between two variables automobile and the distance travelled by it
(d) Both (b) and (c). after applying the brakes is
7. Regression analysis is concerned with (a) Negative (b) Zero
(a) Establishing a mathematical relationship (c) Positive (d) None
between two variables 15. Scatter diagram helps us to
(b) Measuring the extent of association (a) Find the nature correlation between two
between two variables variables
(c) Predicting the value of the dependent (b) Compute the extent of correlation
variable for a given value of the between two variables
independent variable (c) Obtain the mathematical relationship
between two variables
(d) Both (a) and (c).
(d) Both (a) and (c).

344 BUSINESS MATHEMATICS, LOGICAL REASONING & STATISTICS (Paper 3) [CA Foundation]
16. Pearson’s correlation coefficient is used for 22. The coefficient of correlation between two
finding variables
(a) Correlation for any type of relation (a) Can have any unit.
(b) Correlation for linear relation only (b) Is expressed as the product of units of
(c) Correlation for curvilinear relation only the two variables
(d) Both (b) and (c). (c) Is a unit free measure
17. Product moment correlation coefficient is (d) None of these.
considered for 23. What are the limits of the correlation
(a) Finding the nature of correlation coefficient?
(b) Finding the amount of correlation (a) No limit
(c) Both (a) and (b) (b) —1 and 1
(d) Either (a) and (b). (c) 0 and 1, including the limits
18. If the value of correlation coefficient is (d) —1 and 1, including the limits
positive, then the points in a. scatter diagram 24. In case the correlation coefficient between
tend to cluster two variables is 1, the relationship between
(a) From lower left corner to upper right the
corner
two variables would be
(b) From lower left corner to lower right
(a) y = a + bx
corner
(c) From lower right corner to upper left (b) y=a + bx, b >0
corner (c) y = a + bx, b <0
(d) From lower right corner to upper right (d) y = a + bx, both a and b being positive.
corner. 25. If the relationship between two variables x
19. When r = 1, all the points in a scatter diagram and y is given by 2x + 3y + 4 = 0, then the value
would lie of the correlation coefficient between x and y
(a) On a straight line directed from upper left is
to lower right (a) 0 (b) 1
(b) On a straight line directed from lower left (c) —1 (d) negative.
to upper right. 26. For finding correlation between two
(c) On a straight line left to lower right attributes, we consider
(d) Both (a) and (b). (a) Pearson’s correlation coefficient -
20. Product moment correlation coefficient may (b) Scatter diagram
be defined as the ratio of
(c) Spearman’s rank correlation coefficient
(a) The product of standard deviations to the
(d) Coefficient of concurrent deviations.
product of the variances of them
27. For finding the degree of agreement about
(b) The covariance between the variables of
beauty between two Judges in a Beauty
the two variables to the covariance
Contest, we use
between them
(c) The covariance between the variables to (a) Scatter diagram
the product of their standard deviations (b) Coefficient of rank correlation
(d) Either (b) or (c). (c) Coefficient of correlation
21. The covariance between two variables is (d) Coefficient of concurrent deviation.
(a) Strictly positive 28. If there is a perfect disagreement between the
(b) Strictly negative marks in Geography and Statistics, then what
(c) Always 0 would be the value of rank correlation
(d) Either positive or negative or zero. coefficient?

CORRELATION AND REGRESSION 345


(a) Any value (b) Only 1 is known as
(c) Only -1 (d) (b) or (c) (a) Error (b) Residue
29. When we are not concerned with the (c) Deviation (d) (a) or (b).
magnitude of the two variables under 36. The errors in case of regression equations are
discussion, we consider (a) Positive (b) Negative
(a) Rank correlation coefficient (c) Zero (d) All these.
(b) Product moment correlation coefficient 37. The regression line of y on x is derived by
(c) Coefficient of concurrent deviation (a) The minimisation of vertical distances in
(d) (a) or (b) but not (c). the scatter diagram
30. What is the quickest method to find correlation (b) The minimisation of horizontal distances
between two variables? in the scatter diagram
(a) Scatter diagram (c) Both (a) and (b)
(b) Method of concurrent deviation (d) (a) or (b).
(c) Method of rank correlation 38. The two lines of regression become identical
(d) Method of product moment correlation when
31. What are the limits of the coefficient of (a) r = 1 (b) r = -1
concurrent deviations? (c) r = 0 (d) (a) or (b).
(a) No limit 39. What are the limits of the two regression
(b) Between —1 and 0, including the limiting coefficients?
values (a) No limit
(c) Between 0 and 1, including the limiting (b) Must be positive
values (c) One positive and the other negative
(d) Between —1 and 1, the limiting values (d) Product of the regression coefficient must
inclusive be numericall less than unity.
32. If there are two variables x and y, then the 40. The regression coefficients remain unchanged
number of regression equations could he due to a
(a) 1 (b) 2 (a) Shift of origin (b) Shift of scale
(c) Any number (d) 3. (c) Both (a) and (b) (d) (a) or (b).
33. Since Blood Pressure of a person depends on 41. It the coefficient of correlation between two
age, we need consider variables is —0 9, then the coefficient of
(a) The regression equation of Blood determination is
pressure on age (a) 0.9 (b) 0.81
(b) The regression equation of age on Blood (c) 0.1 (d) 0.19.
pressure
42. If the coefficient of correlation between two
(c) Both (a) and (b) variables is 0.7 then the percentage of
(d) Either (a) or (b). variation unaccounted for is
34. The method applied for deriving the (a) 70% (b) 30%
regression equations is known as (c) 51% (d) 49%
(a) Least squares 43. If for two variable x and y, the covariance,
(b) Concurrent deviation variance of x and variance of y are 40, 16 and
(c) Product moment 256 respectively, what is the value of the
(d) Normal equation. correlation coefficient?
35. The difference between the observed value (a) 0.01 (b) 0.625
and the estimated value in regression analysis (c) 0.4 (d) 0.5

346 BUSINESS MATHEMATICS, LOGICAL REASONING & STATISTICS (Paper 3) [CA Foundation]
44. If cov(x, y) = 15, what restrictions should be 50. Following are the two normal equations
put for the standard deviations of x and y? obtained for deriving the regression line of y
(a) No restriction. and x:
(b) The product of the standard deviations 5a + 10b = 40
should be more than 15. 10a + 25b = 95
(c) The product of the standard deviations The regression line of y on x is given by
should be less than 15. (a) 2x+3y=5 (b) 2y+3x=5
(d) The sum of the standard deviations (c) y = 2 + 3x (d) y=3+5x
should be less than 15. 51. If the regression line of y on x and of x on y are
45. If the rank correlation coefficient between given by 2x + 3y = —1 and 5x + 6y = -1 then the
marks in management and mathematics for a arithmetic means of x and y are given by
group of student in 0.6 and the sum of squares (a) (1, —1) (b) (-1, 1)
of the differences in ranks in 66, what is the (c) (-1 —1) (d) (2, 3)
number of students in the group? 52. Given the regression equations as 3x + y = 13
(a) 10 (b) 9 and 2x + 5y = 20, which one is the regression
(c) 8 (d) 11 equation of y on x?
46. While computing rank correlation coefficient (a) 1st equation (b) 2nd equation
between profit and investment for the last 6 (c) both (a) and (b) (d) none
years of a company the difference in rank for a 53. Given the following equations: 2x — 3y = 10
year was taken 3 instead of 4. What is the and 3x + 4y = 15, which one is the regression
rectified rank correlation coefficient if it is equation of x on y?
known that the original value of rank (a) 1st equation (b) 2nd equation
correlation coefficient was 0.4? (c) both the equations (d) none
(a) 0.3 (b) 0.2 54. If u = 2x + 5 and v = - 3y - 6 and regression-
(c) 0.25 (d) 0.28 coefficient of y on is 2.4, what is the regression
47. For 10 pairs of observations. No of concurrent coefficient of v on u?
deviations was found to be 4. What is the value (a) 3.6 (b) -3.6
of the coefficient of concurrent deviation? (c) 2.4 (d) -2.4
55. If 4y — 5x = 15 is the regression line of y on x
(a) 0.2 (b)  0.2 and the coefficient of correlation between x
(c) 1/3 (d) - 1/3 and y is 0.75, what is the value of the regression
48. The coefficient of concurrent deviation for p coefficient of x on y?
pairs of observations was found to be 1/ 3 . (a) 0.45 (b) 0.9375
(c) 0.6 (d) none
If the number of concurrent deviations was
found to-be 6, then the value of p is. 56. If the regression line of y on x and that of x on
y are given by y = -2x + 3 and 8x = -y + 3
(a) 10 (b) 9
respectively, what is the coefficient of
(c) 8 (d) none
correlation between x and y ?
49. What is the value of correlation coefficient due
to Pearson on the basis of the following data: (a) 0.5 (b) -1/ 2
x: -5 -4 -3 -2 -1 (c) - 0.5 (d) none
0 1 2 3 4 5 57. If the regression coefficient of y on x, the
y: 27 18 11 6 3 coefficient of correlation between x and y and
2 3 6 11 18 27 variance of y are - 3/4, - 3/2 and 4
(a) 1 (b) -1
respectively, what is the variance of x ?
(c) 0 (d) -0.5

CORRELATION AND REGRESSION 347


(a) 2/ 3/2 (b) 16/3 67. The two variables are known to be _________
if the movement on the part of one variable
(c) 4/3 (d) 4 does not produce any movement of other
58. If y = 3x + 4 is the regression line of y on x and variable in a particular direction.
the arithmetic mean of x is —1, what is the (a) Correlated
arithmetic mean of y ? (b) Uncorrelated
(a) 1 (b) -1 (c) Positive correlated
(c) 7 (d) none (d) Negative correlated
59. The two variables are known to be_______ if
68. The correlation between demand and price
the movement on the part of one variable does
(for normal goods) is ________.
not produce any movement of the other
(a) Zero (b) Positive
variable in a particular direction.
(c) Negative (d) None
(a) Correlated
(b) Positive correlated 69. If the coefficient of correlation between two
variables is –0.3, then the coefficient of
(c) Negative correlated (d) Uncorrelated
determination is
60. A small value of r indicates only a _________
(a) 0.3 (b) 0.09
linear type of relationship between the
(c) 0.7 (d) 0.9
variables .
(a) Good (b) Poor 70. If the coefficient of correlation between two
variables is 0.6, then the percentage of
(c) Maximum (d) Highest
variation accounted for is
61. Karl Pearson’s coefficient is defined from
(a) 60% (b) 40%
(a) Ungrouped data. (b) Grouped data
(c) 64% (d) 36%
(c) Both. (d) None
71. The coefficient of correlation
62. The line x = a + by represents the regression
(a) Has no limits.
equation of
(b) Can be less than one.
(a) y on x (b) x on y
(c) Both of above. (d) None (c) Can be more than one.
63. In case ‘The ages of husbands and wives’ (d) Varies between ± 1.
correlation is ______ . 72. Which of the following statements is not false?
(a) Positive (b) Negative (a) Scatter diagram fails to measure the
(c) Zero (d) One extent of relationship between the
64. Maximum value of Rank Correlation variables.
coefficient is (b) Scatter diagram can measure correlation
(a) –1 (b) +1 only when the variables are having a
(c) 0 (d) None linear relationship.
65. The method applied for deriving the (c) Scatter diagram can measure correlation
regression equations is known as only when the variables are having a non–
(a) Least squares. linear relationship.
(b) Concurrent deviation. (d) None of these.
(c) Product moment. 73. If two variables x and y are independent then
(d) Normal equation. the correlation coefficient between x and y is
66. The minimum value of correlation coefficient ______.
is (a) Positive (b) Negative
(a) 0 (b) –2 (c) Zero (d) One
(c) 1 (d) –1

348 BUSINESS MATHEMATICS, LOGICAL REASONING & STATISTICS (Paper 3) [CA Foundation]
74. The correlation between height and 82. In case of _______, plotted points on a scatter
intelligence is ________. diagram would be equally distributed
(a) Zero (b) Positive without depicting any particular pattern.
(c) Negative (d) None (a) Zero correlation
75. If x denotes height of a group of students (b) Positive correlation
expressed in cm. and y denotes their weight (c) Negative correlation
expressed in kg. , then the correlation (d) Simple correlation
coefficient between height and weight
83. If the coefficient of correlation between two
(a) Would be shown in kg.
variables is 0.6, then the percentage of
(b) Would be shown in cm. variation unaccounted for is
(c) Would be shown in kg. and cm.
(a) 60% (b) 40%
(d) Would be free from any unit.
(c) 64% (d) 36%
76. The correlation between sale of cold drinks
84. The errors in case of regression equation are
and day temperature is ________.
(a) Positive (b) Negative
(a) Zero (b) Positive
(c) Zero (d) All these
(c) Negative (d) None
85. The regression equation are 8x – 10y + 66 = 0
77. The coefficient of correlation between two
and 40 x – 18y = 214 find the coefficient of
variables is 0.5, then the coefficient of
correlation
determination is
(a) 4/5 (b) –4/5
(a) 0.5 (b) 0.25
(c) 3/5 (d) –1
(c) –0.5 (d) 0.5
86. r, bxy, byx all have _____________ sign.
78. In case of a _______, plotted points on a scatter
diagram concentrate from upper left to lower (a) Different (b) Same
right. (c) Both (d) None
(a) Zero correlation 87. The two regression lines obtained from certain
(b) Negative correlation data were y = x + 5 and 16 x = 9y – 94. Find the
variance of x if variance of y is 16.
(c) Positive correlation
(a) 4/16 (b) 9
(d) Multiple correlation
(c) 1 (d) 5/16
79. The correlation between Employment and
Purchasing power is ______ . 88. For a group of 8 students the sum of squares of
differences in ranks for Accounts and
(a) Positive (b) Negative
Economics marks was found to be 50. What is
(c) Zero (d) None the rank correlation coefficient.
80. If the coefficient of correlation between two (a) 0.50 (b) 0.40
variables is –0.4, then the coefficient of
(c) 0.30 (d) 0.20
determination is
89. Correlation coefficient is not a pure number
(a) 0.6 (b) 0.16
(a) True (b) False
(c) 0.4 (d) 0.2
(c) Both (d) None
81. The correlation is said to be positive 90. The quickest method to find correlation
(a) When the values of two variables move between two variables is:
in the same direction. (a) Scatter diagram
(b) When the values of two variables move (b) Method of concurrent deviation
in the opposite direction. (c) Method of Rank Correlation
(c) When the values of two variable would (d) Method of Product moment Correlation
not change.
(d) None of these.

CORRELATION AND REGRESSION 349


91. In rank correlation coefficient the association 100. For a group of 8 students, the sum of squares
need not be linear of differences in ranks for Economics and
(a) False (b) True English marks was 50. The value of rank
(c) Both (d) None correlation coefficient is ________.
92. Find the coefficient of correlation when its (a) 0.40 (b) 0.50
probable error is 0.2 and the number of pairs (c) 0.30 (d) None
of item is 9. 101. For a group of 8 students, the sum of squares
(a) 0.505 (b) 0.332 of differences in ranks for Economics and
(c) 0.414 (d) None Commerce marks was 50, the value of rank
93. If one of the regression coefficient is greater correlation coefficient is equal to
than unity, then other is less than unity. (a) 0.50 (b) 0.40
(a) True (b) False (c) 0.60 (d) None
(c) Both (d) None 102. In rank co-relation method the sum of
94. To find coefficient of correlation by scatter difference of rank is
diagram method is not suitable, if the number (a) 1 (b) –1
of observations is very large (c) 0 (d) Cannot say
(a) True (b) False 103. The relation between the production of Pig
(c) Both (d) None iron and Soot content in a factory is
95. For a set of 100 observation taking assumed (a) Positive (b) Negative
mean as 4, the sum of the deviations is –11 cm (c) 0 (d) None
and the sum of squares of these deviations is 104. If the relation between two random variables
257 cm2. Find the coefficient of variation. x and y is 2x + 3y = 4, then the correlation
(a) 41.13% (b) 14.13% coefficient between them is
(c) 25.13% (d) 52.13% (a) –2/3 (b) 1
96. The coefficient of rank correlation of marks (c) –1 (d) None
obtained by 10 students in English and 105. The correlation coefficient r is the ………….. of
Economics was found to be 0.5 it was later the two regression coefficients.
discovered that the difference in ranks in the (a) G.M. (b) H.M.
two subjects obtained by one student was (c) Arithmetic Mean (d) None
wrongly taken as 3 instead of 7. Find correct 106. If r = 0, then
coefficient of rank correlation. (a) There is a perfect correlation between x
(a) 0.514 (b) 0.26 & y.
(c) 0.15 (d) None (b) x and y are not correlated.
97. If 2x + 5y – 9=0 and 3x-y-5=0 are two regression (c) There is a positive correlation between x
equation, then find the value of mean of x and & y.
mean of y. (d) Do not exist.
(a) 1,2 (b) 2,2 107. If Covariance (x, y) < 0; then the relation
(c) 2,1 (d) 1,1 between two variable is
98. After settlement the average weekly wage in (a) Positive (b) Negative
a factory has increased from Rs. 8 to Rs. 12 and (c) (a) or (b) (d) None
standard deviation has increased from 2 to 2.5. 108. Consider the two regression lines 3x+2y = 26 &
Find the coefficient of variation after the 6x + y = 31. Find the mean values of x and y.
settlement. (a) x = 4 & y = 7 (b) x=7&y=4
(a) 25% (b) 20.83% (c) x = 5 & y = 6 (d) None
(c) 24.04% (d) 26.30% 109. Consider the two regression lines 3x+2y = 26 &
99. If r = 0.8 then coefficient of determination shall 6x + y = 31. Find the correlation coefficient
be between x & y.
(a) 0.64 (b) 0.40 (a) 0.5 (b) –0.5
(c) 0.60 (d) 0.80 (c) 0.6 (d) None

350 BUSINESS MATHEMATICS, LOGICAL REASONING & STATISTICS (Paper 3) [CA Foundation]
110. Two regression lines are (c) Cannot say (d) None
(a) Reversible (b) not reversible 119. Find the correlation coefficient between the
(c) cannot say (d) None following set of observation.
111. The two regression lines are 5x = 22 + y & 64x =
24 + 45y. Find the Standard Deviation of y from
the given information.
(a) 4 (b) 5
(c) Cannot determined (d) None (a) 1 (b) –1
112. Which one of the following is a true (c) 0 (d) None
statement? 120. The value of Spearman’s rank correlation
(a) 1/2 (bxy + byx) = r (b) 1/2 (bxy+ byx) < r coefficient of a certain number of observations
(c) 1/2 (byx + byx) > r (d) None was to be 2/3. The sum of the squares of
113. The correlation between two variables x and y differences between the corresponding ranks
is found to be 0.4. What is the correlation was 55.
between 2x and (y) ? Find the number of Pairs.
(a) 0.4 (b) -0.4 (a) 10 (b) 12
(c) 0.6 (d) None (c) 11 (d) None
114. Find the coefficient of correlation between 121. The coefficient of regression of Y on X is byx =
the following set of observation:
1.2. If U = and V= find bvu

(a) 0.9 (b) 0.8


(a) 1 (b) -1 (c) 0.7 (d) None
(c) 0 (d) None 122. Two regression coefficient bxy and byx are 1.2
115. Find the correlation coefficient between the and –0.5. This is
following set of observation. (a) True (b) False
(c) Either (a) or (b) (d) None
123. Two lines of regression are given by 5x+7y–
22=0 and 6x+2y–22=0. If the variance of y is 15
find the standard deviation of x.
(a) 1 (b) -1
(a) 5 (b) 6
(c) 0 (d) None
(c) 7 (d) 8
116. For the bivariate data [(x, y)] = [(20,5), (21,4),
(22,3)], the correlation coefficient between x
and y is
(a) 0 (b) 1
(c) - 1 (d) 0.5
117. The regression of y on x is 2y + 3x = 4 and the
correlation coefficient between x and y is 0.8.
This statement is
(a) True (b) False
(c) Cannot say (d) None
118. The correlation coefficient of 3x and -2y is the
same as the correlation coefficient Of x and y.
This statement is
(a) True (b) False

CORRELATION AND REGRESSION 351


ANSWER KEYS

1 (c) 21 (d) 41 (b) 61 (c) 81 (a) 101 (b)


2 (d) 22 (c) 42 (c) 62 (b) 82 (a) 102 (c)
3 (b) 23 (d) 43 (b) 63 (a) 83 (c) 103 (a)
4 (d) 24 (b) 44 (b) 64 (b) 84 (d) 104 (c)
5 (b) 25 (c) 45 (a) 65 (a) 85 (c) 105 (a)
6 (d) 26 (c) 46 (b) 66 (d) 86 (b) 106 (b)
7 (d) 27 (b) 47 (d) 67 (b) 87 (b) 107 (b)
8 (c) 28 (c) 48 (a) 68 (c) 88 (b) 108 (a)
9 (d) 29 (c) 49 (c) 69 (b) 89 (b) 109 (b)
10 (c) 30 (b) 50 (c) 70 (d) 90 (b) 110 (a)
11 (a) 31 (d) 51 (a) 71 (d) 91 (b) 111 (c)
12 (d) 32 (b) 52 (b) 72 (a) 92 (b) 112 (c)
13 (a) 33 (a) 53 (d) 73 (c) 93 (a) 113 (a)
14 (a) 34 (a) 54 (b) 74 (a) 94 (a) 114 (a)
15 (a) 35 (d) 55 (a) 75 (d) 95 (a) 115 (b)
16 (b) 36 (d) 56 (c) 76 (b) 96 (b) 116 (c)
17 (c) 37 (a) 57 (b) 77 (b) 97 (c) 117 (b)
18 (a) 38 (d) 58 (a) 78 (b) 98 (b) 118 (b) 121 (b)
19 (a) 39 (d) 59 (d) 79 (a) 99 (a) 119 (c) 122 (b)
20 (c) 40 (a) 60 (b) 80 (b) 100 (a) 120 (a) 123 (c)

352 BUSINESS MATHEMATICS, LOGICAL REASONING & STATISTICS (Paper 3) [CA Foundation]
the number of towns:
(a) 10 (b) 11
HOME WORK-2
(c) 12 (d) 9
9. For a bivariate frequency table having (p + q)
classifica tion the total number of cells is
1. The correlation coefficient between x and y is
(a) P (b) P+q
0.8, the correlation coefficient between u and
v are 2u + x + 4 = 0 and 4v + 16x + 11 = 0 (c) q (d) pq
10. The two lines of regression becomes identical
(a) r = 0.8 (b) r = -0.8
when
(c) r = 0 (d) r = +1 (a) r = 1 (b) r = -1
2. If three Judges appointed for a beauty (c) r = 0 (d) (a) or (b)
competition, then how many different rank 11. If x and y are two correlated variables with
correlation coefficients are required to analyse correlation coefficient 0.60. If u =3x +5 and V =
the judge competition. 5y-7. The correlation coefficient of U and V is:
(a) 3 (b) 1 (a) - 0.60 (b) 0.60
(c) 2 (d) 6 (c) 1 (d) 0.36
3. In a bivariate population, the linear regression 12. If the two regression co -efficient are 4 and 16
lines 3x+y-2=0 and y+x = 0 then the coefficient the percentage of unexplained variation is:
of correlation is (a) 64 (b) 36
(a) 0 (b) -1/3 (c) 54 (d) 46
(c) 1/3 (d) -1/ 3 13. r, bxy, byx all have___________sign.
(a) Different (b) Same
4. If the two regression co-efficients are 4 and 16
(c) Both (d) None of them
the percentage of unexplained variation is:
14. If cov (x, y) = 25, what restrictions should put
(a) 64 (b) 36
for the standard deviations of x and y?
(c) 54 (d) 46
5. Correlation Co-efficient is _______ of the units (a) No restriction
of measurements (b) The product of Standard deviations
(a) Independent (b) Dependent should be more than 25
(c) Both (d) none of these (c) The product of Standard deviations
6. If for two variable x and y, the covariance, should be less than 25
variance of x and variance of y are 40, 16 (d) The sum of Standard deviations should
and 256 respectively, what is the value of the be less than 25
correlation coefficient? 15. The two lines of regression intersect at the
(a) 0.01 (b) 0.625 point:
(c) 0.4 (d) 0.5 (a) Mean (b) Median
7. Two lines of regression are given by 5x+7y – (c) Mode (d) None
22=0 and 6x+2y–22=0. If the variance of y is 15, 16. If the two lines of regression are x +2y – 5 = 0
find the standard deviation of x? and 2x+3y –8=0, then the regression line of y
(a) 5 (b) 7 on x is
(a) x +2y – 5 = 0 (b) x +2y = 0
(c) 6 (d) 8 (c) 2x + 3y – 8 = 0 (d) 2x + 3y = 0
8. For the two of towns, the co-efficient of rank 17. If the two regression lines are 3X = Y and 8Y =
correlation between the people living below 6X, then the value of correlation coefficient is
the poverty line and increase population in
(a) –0.5 (b) 0.5
0.50. The sum of the squares difference in
(c) 0.75 (d) –0.80
ranks awarded to their factors is 82.50, find
CORRELATION AND REGRESSION 353
18. The regression coefficient is independent of
the change of
(a) Origin (b) Scale
(c) Scale and origin both (d) None of these
19. If the correlation coefficient between the
variables X and Y is 0.5, then the correlation
coefficient between the variables 2x – 4 and
3– 2y is
(a) 0.5 (b) 1
(c) –0.5 (d) 0

ANSWER KEYS

1 (a) 2 (a)
3 (d) 4 (b)
5 (a) 6 (b)
7 (b) 8 (a)
9 (d) 10. (d)
11. (b) 12. (b)
13. (b) 14. (b)
15. (a) 16. (a)
17. (b) 18. (a)
19. (c)

354 BUSINESS MATHEMATICS, LOGICAL REASONING & STATISTICS (Paper 3) [CA Foundation]
CHAPTER-17
PROBABILITY

INTRODUCTION
1. Probability rangs from 0 to 1.
2. The Classical definition of probability assumes that all possible outcomes of an experiment
are equally likely.
3. If an event cannot take place, probability will be 0.
4. p + q would always be 1.
5. If two events A and B are independent, then their complements are also independent.
6. If two events A and B are mutually exclusive the probability of occurrence of either A or B is
given by P(A) + P(B)
7. If two events A and B are overlapping events, the probability of occurrence of either A or B is
given by P(A) + P(B) – P(A and B).
8. If two events A and B are independent, the probability that they will both occur is given by
P(A) × P(B).
9. If two events A and B are dependent, the conditional probability of B given A is given by
P(AB) / P(A).
10. Dependent events are those in which the outcome of one affects and is affected by the other.
11. Joint Probability is the probability of the joint or simultaneous occurrence of two or more
events.
12. The probability obtained by following relative frequency is called posterior probability.
13. The order of arrangement is important in permutations.
14. Permutation exceeds combinations.
n
15. Cr is equal to 1 when n = r
n
16. Cr is equal to n where r = 1
n
17. Cr is equal to n where r = n–1
n
18. Cr is equal to 1 when n = r.
7 7
19. C4 = C3

PROBABILITY 355
LIST OF FORMULA
1. Range of probability of an event 0 to 1
2. Probability of an event which cannot
take place 0
3. Probability of an event which is certain
i.e., bound to occure 1
4. Probability of the entire sample space P(S) = 1
5. Addition Rule of Probability
(a) When events are Probability of occurrence of either A or B
Mutually Exclusive P(A  B) = P(A) + P(B)
(b) When events are Not Probability of occurrence of atleast A or B
Mutually Exclusive P(A  B) = P(A) + P(B) – P(A  B)
6. Multiplication Rule of (i) Probability of occurrence of both A & B
Probability When events P(A  B) = P(A) × P(B)
are Independent
(ii) Probability of non-occurrence of both A & B

P ( A  B) = P ( A ) × P ( B )

(iii) Probability of occurrence of A and not B

P ( A  B) = P(A) × P ( B )

(iv) Probability of occurrence of B and not A

P ( A  B) = P ( A ) × P (B)

(v) Probability of occurrence of only one event

P ( A  B) + P ( A  B)

(vi) Probability of occurrence of atleast one event

= 1 – P ( A  B)

(vii) Probability of occurrence of an event

= P(A  B) + P ( A  B) + P ( A  B)

7. Conditional Probability P(A  B) = P(A) × P(B/A)

356 BUSINESS MATHEMATICS, LOGICAL REASONING & STATISTICS (Paper 3) [CA Foundation]
Jo int Pr obability
8. Posterior Probability =
Sum of Jo int Pr obabilities
here, Joint Probability
= Prior Probability × Conditional Probability

9. Expected Value (X) E(X) = P1X1 + P2X2 + ..... + PkXk


2
10. Variance of Expectation E (X2) – [E(X)]

Where, E(X2) =  px2

PROBABILITY 357
10. If A and B are two events which have no point
CLASS WORK in common, the events A and B are:
(a) complementary to each other
(b) independent
(c) mutually exclusive
1. Classical probability is measured in terms of : (d) dependent
(a) absolute value and ratio both 11. Each outcome of a random experiment is
(b) a ratio called:
(c) an absolute value (a) Primary event
(d) none (b) probable event
2. Probability can take values from (c) complementary event
(a) – 3 to 3 (b) – 3 to 1 (d) none of them
(c) 0 to 1 (d) – 1 to 1 12. If A and B are two events, the probability of
3. Probability is expressed as: occurrence of either A or B is given as:
(a) percentage (b) ratio (a) P (A) + P (B) (b) P (A U B)
(c) proportion (d) all (a), (b), (c) (c) P (A) P (B) (d) P (A B)
4. Classical probability is also known as : 13. If A and B are two events, the probability of
(a) a priori probability occurrence of A and B simultaneously is given
(b) mathematical probability as
(c) laplace's probability (a) P (A) + P (B) (b) P (A U B)
(d) all the above (c) P (A B) (d) P (A) P (B)
5. The limiting relative frequency approach of 14. If it is known that an event A has occurred, the
probability is known as: probability of an event B given A is called
(a) statistical probability (a) empirical probability
(b) classical probability (b) subjective probability
(c) mathematical probability (c) conditional probability
(d) all the above (d) posteriori probability
6. The definition of statistical probability was 15. Classical probability is possible in case of:
originally given by : (a) equally likely outcomes
(a) Feller (b) Laplace
(b) unequally likely outcomes
(c) Von-Mises (d) Pascal
(c) either unequally likely or equally likely
7. The definition of apriori probability was
out comes
originally given by :
(a) De Moivre (b) Laplace (d) all the above
(c) Von-Mises (d) Pascal 16. An event consisting of those elements which
8. The idea of posteriori probabilities was are not in A is called
introduced by (a) union event
(a) Pascal (b) Peter and Paul (b) intersection event
(c) Thomas Bayes (d) Karl Pearson (c) event
9. Two events are said to be independent if:
(d) complementary event of A
(a) both the events have only one point
(b) there is no common point in between 17. The probability of all possible outcomes of a
them random experiment is always equal to:
(c) one does not affect the occurrence of the (a) 100 (b) one
other (c) zero (d) infinity
(d) each outcome has equal chance of
occurrence

358 BUSINESS MATHEMATICS, LOGICAL REASONING & STATISTICS (Paper 3) [CA Foundation]
18. The probability of the intersection of two selecting a fair complexioned rich girl ?
mutually exclusive events is always: (a) 0.85 (b) 0.02
(a) –1 (b) 0 (c) 0.24 (d) 0.64
(c) 1 (d) none 26. A card is drawn at random from a pack of 52
19. The individual probabilities of occurrence of cards, the probability of getting a club is
two events A and B are known, the probability (a) 1/4 (b) 1/52
of occurrence of both the events together will (c) 1/13 (d) none
be :
27. A card is drawn at random from a pack of 52
(a) 1 (b) decreased cards, the probability of getting a queen is
(c) 0 (d) increased (a) 1/4 (b) 1/52
20. If A, B and C are three mutually exclusive (c) 1/13 (d) 4/13
events, the probability of their union is equal
28. A card is drawn at random from a pack of 52
to:
cards, the probability of getting a club queen
(a) P (A) P (B) P (C) is
(b) P (A) + P (B) + P (C) – P (ABC) (a) 1/14 (b) 1/52
(c) P (A) + P (B) + P (C) (c) 1/13 (d) 4/13
(d) P (A) P (B) + P (C) P (A) + P (B) P (C) 29. A card is drawn at random from a pack of 52
21. If A, B and C are three independent events, cards, find the probability of getting a club or a
the probability of their joint occurrence is equal queen is
to (a) 1/14 (b) 1/52
(a) P (A) P (B) P(C) (c) 1/13 (d) 4/13
(b) 1/[P (A) P (B) P (C)] 30. A box contains 6 black and 4 white balls. Two
(c) P (A) + P (B) + P(C) balls are drawn at random from it, the
(d) P (AB) + P (AC) + P (BC) probability that both the balls are black is
22. If A is an impossible event, the conditional (a) 1/2 (b) 1/3
probability of B given A is equal to: (c) 2/3 (d) 1/4
(a) zero (b) one 31. A box contains 6 black and 4 white balls. Two
(c) infinite balls are drawn at random from it,the
(d) indeterminate quantity probability that both are white is
23. If an event B has occurred and it is known that (a) 1/15 (b) 1/5
P (B) = 1, the conditional probability P (A/B) is (c) 2/15 (d) 4/15
equal to : 32. A box contains 6 black and 4 white balls. Two
(a) P (A) (b) P (B) balls are drawn at random from it,the
(c) one (d) zero probability that both are of different colour is
24. If A and B are two independent events, then P (a) 4/15 (b) 5/15
(A  B) is equal to: (c) 7/15 (d) 8/15
(a) P (A) P (B) 33. Two cubical dice are thrown simultaneously,
(b) 1 – P ( A'  B' ) the probability of getting total '9' is
(c) [ 1 - P(A')] [1- P (B')] (a) 1/4 (b) 1/6
(d) all the above (c) 1/9 (d) 1/36
25. A class consists of 100 students, 25 of them are 34. Two cubical dice are thrown simultaneously,
girls and 75 boys. 20 of them are rich and the probability of getting total at least '9' is
remaining poor. 40 of them are fair (a) 5/36 (b) 5/18
complexioned. What is the probability of (c) 10/18 (d) 1/9

PROBABILITY 359
35. One bag contains 5 black and 3 white balls. 43. The present age of a person A is 35. The odds
Another bag contains 4 black and 5 white balls. in favour of his living upto the age of 65 is 3 : 2.
One ball is drawn from each bag, the The age of another person B is 40 at present.
probability that they are of different colour is The odds against his living upto the age of 70 is
(a) 37/72 (b) 27/72 4 : 1. The probability that atleast one of them
(c) 8/72 (d) 17/72 will be alive after 30 years is
36. Two dice are thrown simultaneously, the (a) 17/30 (b) 17/25
probability that the sum of the numbers is (c) 18/72 (d) 7/25
divisible by 3 or 4 is 44. In three different families, there are
(a) 5/36 (b) 5/18 respectively 3 boys and 2 girls, 2 boys and 3
(c) 5/9 (d) 7/36 girls, 4 boys and 1 girl. A family is selected at
random and from it 2 children are taken at
37. The probability of getting total at the most '6'
random, the probability that both are boys is
when three cubical dice are thrown is
(a) 5/54 (b) 5/ 72 (a) 1/3 (b) 2/3
(c) 5/36 (d) 5/216 (c) 3/7` (d) 7/72
38. An urn contains 5 white, 4 red and 3 black balls. 45. A group consists of 7 men and some women.
Three balls are drawn at random from it, the The probability of selecting 2 women from
probabilities that all are black is them is 1/15. The number of women in the
(a) 1/216 (b) 1/220 group is
(c) 1/72 (d) 1/60 (a) 5 (b) 3
39. An urn contains 5 white, 4 red and 3 black balls. (c) 8 (d) 7
Three balls are drawn at random from it, the 46. A number is taken at random from the numbers
probabilities that two are of the same colour 1 to 100, the probability that the number is
is divisible by '3' is
(a) 7/77 (b) 7/72 (a) 33/100 (b) 25/100
(c) 8/44 (d) 29/44 (c) 30/100 (d) none
40. A card as drawn from a pack of 52 cards and it is 47. A number is taken at random from the numbers
thrown away. Then another card is drawn, the 1 to 100, the probability that the number is
probability that it is a queen. divisible by '7' is
(a) 1/4 (b) 1/52 (a) 33/100 (b) 28/100
(c) 4/13 (d) 1/13 (c) 14/100 (d) 1/5
41. Three persons A, B and C aim a target. The 48. A number is taken at random from the numbers
probabilities of their hitting the target are 1 to 100, the probability that the number is
respectively 2/3, 1/4, 1/2 . What is the divisible by '3' or '7' is
probability that the target will be hit? (a) 45/100 (b) 43/100
(a) 1/8 (b) 3/8 (c) 47/100 (d) 51/100
(c) 5/8 (d) 7/8 49. A and B choose any one digit at random from
42. An example of statistics is given to three the digits 0, 1, 2, ... 9. independently. The
students A, B and C. Their probabilities of probability that the product of the two digits
solving the example correctly are respectively is zero is
1/2, 3/4, 1/4, the probability that the example (a) 0.20 (b) 0.22
will be solved is (c) 0.18 (d) 0.19
(a) 20/32 (b) 27/32 50. There are 5 red and 4 black balls in a bag. Two
(c) 28/32 (d) 29/32 draws of two balls are made from it. The
probabilities that the first drawing gives 2 red

360 BUSINESS MATHEMATICS, LOGICAL REASONING & STATISTICS (Paper 3) [CA Foundation]
balls and the second drawing gives 2 black balls draw is equal to
if the balls are not replaced is (a) 4/144 (b) 5/144
(a) 5/62 (b) 5/108 (c) 6/144 (d) 1
(c) 5/63 (d) 5/72 58. In a locality 65% can read Gujarati, 36% can read
51. There are 5 red and 4 black balls in a bag. Two Hindi and 30% can read English. 18% can read
draws of two balls are made from it. The Gujarati and Hindi. 17% can read Gujarati and
probability that the first drawing gives 2 red English and 13% can read Hindi and English.
balls and the second drawing gives 2 black balls 5% can read all the three languages. The
if the balls are replaced is probability that a person selected at random
(a) 5/62 (b) 5/108 can read, at least one of the three languages is
(c) 5/63 (d) 5/72 (a) 0.78 (b) 0.80
52. In a city 60% read newspaper A, 40% read (c) 0.88 (d) 0.55
newspaper B and 30% read newspaper C, 20% 59. An urn contains 9 red, 7 black and 5 white balls.
read A and B, 30% read A and C, 10% read B and If three balls are drawn one after the other
C. Also 5% read papers A, B and C. The without replacement, the probability that they
percentage of people who do not read any of are red, black and white respectively is
these newspapers is : (a) 5/76 (b) 3/76
(a) 15% (b) 55% (c) 5/63 (d) 3/63
(c) 35% (d) none 60. In a group of 20 persons, there are 5 graduates.
53. A bag contains 4 white and 3 black balls. Two If 3 persons are selected at random from the
draws of 2 balls are successively made, the group, the probability that all are graduates is
probability of getting 2 white balls at first draw (a) 1/120 (b) 15/108
and 2 black balls at second draw when the balls (c) 1/114 (d) 5/114
drawn at first draw were replaced is:
61. In a group of 20 persons, there are 5 graduates.
(a) 3/49 (b) 1/49 If 3 persons are selected at random from the
(c) 9/49 (d) 2/49 group, the probability that at least one is
54. In tossing three coins at a time, the probability graduate is
of getting at most one head is : (a) 115/228 (b) 135/228
(a) 3/8 (b) 7/8 (c) 225/228 (d) 137/228
(c) 3/6 (d) 1/6 62. One urn contains 4 red and 5 white balls and
55. There is 80% chance that a problem will be the second urn contains 6 red and 3 white balls.
solved by a statistics student and 60% chance One of the urns is selected at random and two
that the same problem will be solved by a balls are drawn from it. The probability that
mathematics student. The probability that the both the balls are red is
problem will be solved is (a) 5/24 (b) 5/48
(a) 0.82 (b) 0.92 (c) 7/48 (d) 7/24
(c) 0.20 (d) 0.48 63. An urn A contains 5 white and 3 black balls and
56. The probability of two persons being borne B contains 4 white and 4 black balls. An urn is
on the same day (ignoring date) is: selected and a ball is drawn from it, the
(a) 1/49 (b) 1/365 probability that the ball is white is :
(c) 1/7 (d) none (a) 9/16 (b) 9/18
57. An urn contains 5 red, 4 white and 3 black balls. (c) 10/32 (d) 10/16
Three balls are successively drawn from it. The 64. From a pack of 52 cards, two cards are drawn at
probability of three balls being of different random. The probability that one is an ace and
colours when the ball is replaced after each the other is a king is :

PROBABILITY 361
(a) 2/13 (b) 1/169 (a) 616/4845 (b) 308/4845
(c) 8/663 (d) 28/663 (c) 154/1845 (d) 72/625
65. Two dice are rolled by two players A and B. A 72. A bag contains 12 white and 8 red balls. If four
throws total 10, the probability that B throws balls are taken one after the other from the
more than A is bag with replacement, the probability that
(a) 1/12 (b) 1/36 they are alternatively of different colour is
(c) 1/18 (d) none (a) 616/4845 (b) 308/4845
66. The data reveals that 10 per cent patients die (c) 154/1845 (d) 72/625
in a particular type of operation. A doctor 73. There are two bags. One bag contains 4 red
performed 9 operations and all of them and 5 black balls and the other 5 red and 4 black
survived. Whether the 10th patient on being balls. One ball is to be drawn from either of
operated: the two bags. The probability of drawing a
(a) will survive black ball is :
(b) will die (a) 1 (b) 16/80
(c) may survive or may die (c) 40/80 (d) 10/80
(d) none 74. Three dice are rolled simultaneously. The
67. There are two groups of students consisting of probability of getting 12 spots is :
4 boys and 2 girls, 3 boys and 1 girl. One student (a) 1/64 (b) 25/216
is selected from each groups. The probability (c) 1/36 (d) none
of one boy and one girl being selected is : 75. Given that P (A) = 1/3, P (B) = 1/4, P ( A | B) = 1/
(a) 1/12 (b) 5/12 6, the probability P (B | A) is equal to :
(c) 1 (d) 1/2 (a) 4/8 (b) 3/8
68. In a shooting competition, Mr. X can shoot at (c) 2/8 (d) 1/8
the bulls eye 4 times out of 5 shots and Mr. Y, 5 76. A number is selected randomly from each of
times out of six shots and Mr. Z, 3 times out of the two sets
4 shots. The probability that the target will be 1, 2, 3, 4, 5, 6, 7, 8
hit at least twice is :
2, 3, 4, 5, 6, 7, 8, 9
(a) 100/120 (b) 50/120
The probability that the sum of the numbers is
(c) 110/120 (d) 107/120 equal to 9 is :
69. A and B toss a coin alternately. One who gets (a) 8/64 (b) 8/72
head first wins. The probabilities of their
(c) 14/81 (d) 7/64
winning are respectively
77. A bag contains 3 white, 1 black and 3 red balls.
(a) 2/3,1/3 (b) 1/2,1/2
Two balls are drawn from the well shaked bag.
(c) 3/4,1/4 (d) 5/6 , 1/6 The probability of both the balls being black is
70. A can hit a target 3 times out of 5 trials, B can :
hit the target 2 times out of 5 trials; C can hit (a) 0 (b) 1
the target 3 times out of 4 trials. If all the three
(c) 1/7 (d) 1/9
try simultaneously find the probability that at
least 2 will hit the target. 78. The chance of winning the race of the horse A
is 1/5 and that of horse B is 1/6. The probability
(a) 0.63 (b) 0.5
that the race will be won by A or B is :
(c) 0.69 (d) 0.65
(a) 11/30 (b) 1/3
71. A bag contains 12 white and 8 red balls. If four
(c) 1/30 (d) none
balls are taken one after the other from the
bag without replacement , the probability that 79. Four cards are drawn from a pack of 52 cards.
they are alternatively of different colour is The probability that out of 4 cards 2 are red
and 2 are black is

362 BUSINESS MATHEMATICS, LOGICAL REASONING & STATISTICS (Paper 3) [CA Foundation]
(a) 325/833 (b) 235/833 88. If two events A and B are such that A  B, the
(c) 352/833 (d) none relation between the conditional probabilities
80. The probability of Mr. A living 20 years more is P (A/C) and P (B/C) is :
1/5 and that of Mr. B is 1/7. The probability that (a) P (A/C) = P (B/C)
at least one of them will survive 20 years hence (b) P (A/C) < P (B/C)
is : (c) P (A/C) > P (B/C)
(a) 12/35 (b) 1/35 (d) Can’t say
(c) 11/35 (d) 14/35 89. For any two events A and B, P (A-B) is equal to:
81. A candidate is selected for interviews for three (a) P (A) – P (B) (b) P (B) – P (A)
posts. For the first post there were 3 (c) P (A) – P (A  B) (d) P (B) – (AB)
candidates, for the second 4 and for the third
90. For any two events A and B, P (B – A) is equal
2. What is the probability that the candidate is
to:
selected for at least one post ?
(a) P (A) – P (B) (b) P (B) – P (A)
(a) 1/4 (b) 2/4
(c) P (A) – P (A  B) (d)P (B) – P(A  B)
(c) 3/4 (d) 5/6
91. The probability distribution of a random
82. A person applies for a job in two firms X and Y.
variable x is as follows
The probability of his being selected in firm X
2 3 4 5 6 7 8 9 10
is 0.7 and being rejected in firm Y is 0.5. The
probability of at least one of his applications 0.05 .10 .30 .20 .05 .10 .05 .10 .05
being rejected is 0.6. What is the probability The mean of x is
that he will be selected in one of the two firms? (a) 1.9 (b) 5.4
(a) 0.9 (b) 0.8 (c) 3.6 (d) 6.5
(c) 0.6 (d) 0.7 92. The probability distribution of a random
variable is as follows :
83. The probability of drawing a white ball in the
15 16 17 18 19 20
first draw and again a white ball in the second
draw with replacement from a bag containing .04 .19 3p .26 p .07
6 white and 4 blue balls is : The value of p is
(a) 2/100 (b) 6/100 (a) 0.11 (b) 0.15
(c) 36/100 (d) 1/100 (c) 0.10 (d) none
84. Out of 20 employees in a company, five are 93. 4 coins are tossed simultaneously, the
graduates. Three employees are selected at expected number of heads is
random. The probability of all the three being (a) 1 (b) 2.5
graduates is (c) 3 (d) 2
(a) 1/32 (b) 2/125 94. Two coins are tossed simultaneously. A person
(c) 2/114 (d) 1/114 receives ` 8 for each head and loses ` 10 for
85. If two events A and B are such that A  B and each tail. The expected value of the amount
B  A, the relation between P (A) and P (B) is: gained by him is
(a) P (A) = P (B) (b) P (A) > P (B) (a) 1 (b) –2
(c) P (A) < P (B) (d) none (c) 3 (d) –3
86. If A  B, the probability, P(A/B) is equal to o 95. There are 5 white and 3 black balls in a box. 3
balls are taken at random from the box. The
(a) zero (b) one
expected number of black balls is
(c) P (B)/P (A) (d) P (A)/P (B) (a) 2 (b) 6/9
87. If B  A, the probability P(A/B) is equal to (c) 3 (d) 9/8
(a) zero (b) one 96. There are 4 black and 2 white balls in a box and
(c) P (A)/ P (B) (d) P (B)/P (A) 2 balls are taken at random from it. If a person

PROBABILITY 363
receives ` 4 for each white ball and loses ` 2 –2 –1 0 1 2
for each black ball, the mathematical 0.15 0.30 0.30 0.15 0.10
expectation of the amount received by him is The value of the V(x) is
(a) 1 (b) 0 (a) 1.25 (b) 1.20
(c) –3 (d) 5 (c) 1.3875 (d) 1.6250
97. There are 3 black and 2 white balls in a box. 2 105. Daily demand for transistors is has the
balls are taken from it. ` 24 is given for each following probability distribution.
black ball. What amount should be charged for Demand : 1 2 3 4 5 6
each white ball so that the game is fair ? Probability : 0.10 0.15 0.20 0.25 0.18 0.12
(a) 72 (b) 54 The expected daily demand for transistors is
(c) 36 (d) 33 (a) 2.25 (b) 5.4
98. There are 8 screws in a packet of which 2 are (c) 3.62 (d) 3.3
defective. If 2 screws are taken at random, the 106. All possible outcomes of a random experiment
expected number of defective screws is forms the
(a) 2 (b) 4 (a) events (b) sample space
(c) 0.5 (d) 3 (c) both (d) none
99. There are 5 tickets in a box numbered 1, 1, 2, 2, 107. If one of outcomes cannot be expected to occur
2 respectively. Two tickets are taken at random in preference to the other in an experiment
from it, the expectation of the total of the the events are
numbers on the tickets is (a) simple events (b) compound events
(a) 7.2 (b) 5.4 (c) favourable events
(c) 3.2 (d) 3.3 (d) equally likely events
100. There are 100 tickets in a lottery of Re. 1 each. 108. If two events cannot occur simultaneously in
There is only one ticket in the lottery bearing the same trial then they are
a prize of ` 80. A person purchases 1 ticket. His (a) mutually exclusive events
expectation is (b) simple events
(a) 2 (b) 4 (c) favourable events (d) none
(c) –0.5 (d) –0.2 109. When the number of cases favourable to the
101. A person takes an insurance of ` 1000 and pays event A is none then P(A) is equal to
premium of ` 20. The probability that any (a) 1 (b) 0
person of his age group dies within a year is
1
0.01, the expected gain of the insurance (c) (d) none
company is 2
110. A card is drawn from a well-shuffled pack of
(a) 12 (b) 14
playing cards. The probability that it is a spade
(c) 10 (d) 20
is
102. Two tickets are taken at random from 5 tickets
numbered from 1 to 5, the expected value of 1 1
(a) (b)
the sum obtained on the two tickets is 13 4
(a) 7 (b) 5 3
(c) 36 (d) 6 (c) (d) none
13
103. For a random variable x, E(x) = 2, the value of
111. A card is drawn from a well-shuffled pack of
the E(2x + 3) is
playing cards. The probability that it is a king is
(a) 7 (b) 5
(c) 4 (d) 3 1 1
(a) (b)
104. The probability distribution of a random 13 4
variable x is as follows : 4
(c) (d) none
13
364 BUSINESS MATHEMATICS, LOGICAL REASONING & STATISTICS (Paper 3) [CA Foundation]
112. A card is drawn from a well-shuffled pack of 121. For the above table the probability that his
playing cards. The probability that it is the ace wages between ‘ 160 and 220 is
of clubs is (a) 30/100 (b) 10/100
1 1 (c) 38/100 (d) 18/100
(a) (b)
13 4 122. The table below shows the history of 1000 men
:
1
(c) (d) none Life (in years) : 60 70 80 90
52
No. survived : 1000 500 100 60
113. In a single throw with two dice the probability
of getting a sum of five on the two dice is The probability that a man will survived to age 90 is
(a) 1/9 (b) 5/36 (a) 60/1000 (b) 160/1000
(c) 5/9 (d) none (c) 660/1000 (d) none
114. In a single throw with two dice, the probability 123. The terms “chance” and probability are
of getting a sum of six on the two dice is synonymous
(a) 1/9 (b) 5/36 (a) True (b) false
(c) 5/9 (d) none (c) both (d) none
115. The probability that exactly one head appears 124. If probability of drawing a spade from a well-
in a single throw of two fair coins is shuffled pack of playing cards is 1/4 then the
(a) 3/4 (b) 1/2 probability that of the card drawn from a well-
(c) 1/4 (d) none shuffled pack of playing cards is ‘not a spade’
116. The probability that at least one head appears is
in a single throw of three fair coins is (a) 1 (b) 1/2
(a) 1/8 (b) 7/8 (c) 1/4 (d) 3/4
(c) 1/3 (d) none 125. Probability of the sample space is
117. The definition of probability fails when the no (a) 0 (b) 1/2
of possible outcomes of the experiment is (c) 1 (d) none
infinite 126. Sum of all probabilities of mutually exclusive
(a) True (b) false and exhaustive events is equal to
(c) both (d) none (a) 0 (b) 1/2
118. The following table gives distribution of wages (c) 3/4 (d) 1
of 100 workers – 127. Let a sample space be S = {X1, X2, X3} which of
Wages (in ‘) 120–140 140–160 160–180 180–200 200–220 220–240 240–260
the fallowing defines probability space on S ?
No. of

workers 9 20 0 10 8 35 18
(a) P(X1)= 1/4 , P(X2)= 1/3 , P(X3)= 1/3
The probability that his wages are under ‘ 140 is (b) P(X1)= 0, P(X2)= 1/3 , P(X3)= 2/3
(a) 20/100 (b) 9/100 (c) P(X1)= 2/3 , P(X2)= 1/3 , P(X3)= 2/3
(c) 29/100 (d) none (d) none
119. An individual is selected at random from the 128. Let P be a probability function on S = {X1 , X2 ,
above group. The probability that his wages X3} if P(X1)= 1/4 and P(X 3) = 1/3 then P(X2) is
are under ‘160 is equal to
(a) 9/100 (b) 20/100 (a) 5/12 (b) 7/12
(c) 29/100 (d) none (c) 3/4 (d) none
120. For the above table the probability that his 129. The chance of getting a sum of 10 in a single
wages are above ‘ 200 is throw with two dice is
(a) 43/100 (b) 35/100 (a) 10/36 (b) 1/12
(c) 53/100 (d) 61/100 (c) 5/36 (d) none

PROBABILITY 365
130. The chance of getting a sum of 6 in a single (a) 2/4 (b) 1/4
throw with two dice is (c) 3/4 (d) 0
(a) 3/36 (b) 4/36 140. When two unbiased coins are tossed, the
(c) 6/36 (d) 5/36 probability of obtaining not more than 3 heads
131. P (B/A) defines the probability that event B is
occurs on the assumption that A has happened (a) 3/4 (b) 1/2
(a) Yes (b) no (c) 1 (d) 0
(c) both (d) none 141. When two unbiased coins are tossed, the
132. The complete group of all possible outcomes probability of getting both heads or both tails
of a random experiment given an ________ set is
of events. (a) 1/2 (b) 3/4
(a) mutually exclusive (c) 1/4 (d) none
(b) exhaustive 142. Two dice with face marked 1, 2, 3, 4, 5, 6 are
(c) both (d) none thrown simultaneously and the points on the
133. When the event is ‘certain’ the probability of dice are multiplied together. The probability
it is that product is 12 is
(a) 0 (b) 1/2 (a) 4/36 (b) 5/36
(c) 1 (d) none (c) 12/36 (d) none
134. The classical definition of probability is based 143. A bag contain 6 white and 5 black balls. One
on the feasibility at subdividing the possible ball is drawn. The probability that it is white is
outcomes of the experiments into (a) 5/11 (b) 1
(a) mutually exclusive and exhaustive (c) 6/11 (d) 1/11
(b) mutually exclusive and equally likely 144. Probability of occurrence of at least one of the
(c) exhaustive and equally likely events A and B is denoted by
(d) mutually exclusive,exhaustive and (a) P(AB) (b) P(A+B)
equally likely cases. (c) P(A/B) (d) none
135. Two unbiased coins are tossed. The probability 145. Probability of occurrence of A as well as B is
of obtaining ‘both heads’ is denoted by
(a) 1/4 (b) 2/4 (a) P(AB) (b) P(A+B)
(c) 3/4 (d) none (c) P(A/B) (d) none
136. Two unbiased coins are tossed. The probability 146. Which of the following relation is true ?
of obtaining one head and one tail is (a) P(A)– P(AC)= 1 (b) P(A)+ P(AC)= 1
C
(a) 1/4 (b) 2/4 (c) P(A) P(A )= 1 (d) none
(c) 3/4 (d) none 147. If events A and B are mutually exclusive, the
137. Two unbiased coins are tossed. The probability probability that either A or B occurs is given by
of obtaining both tail is (a) P (A+B)= P(A)– P(B)
(a) 2/4 (b) 3/4 (b) P (A+B)=P(A)+ P(B)– P(AB)
(c) 1/4 (d) none (c) P (A+B)= P(A)– P(B)+ P(AB)
138. Two unbiased coins are tossed. The probability (d) P (A+B)= P(A)+ P(B)
of obtaining at least one head is 148. The probability of occurrence of at least one
(a) 1/4 (b) 2/4 of the 2 events A and B (which may not be
(c) 3/4 (d) none mutually exclusive) is given by
139. When two unbiased coins are tossed, the (a) P(A+B)= P(A)– P(B)
probability of obtaining 3 heads is (b) P(A+B)= P(A)+ P(B)– P(AB)

366 BUSINESS MATHEMATICS, LOGICAL REASONING & STATISTICS (Paper 3) [CA Foundation]
(c) P(A+B)= P(A)– P(B)+ P(AB) 158. If P (A)= 1/4, P(B) = 2/5, P(A+B) = 1/2 then P(AB)is
(d) P(A+B)= P(A)+P (B) equal to
149. If events A and B are independent, the (a) 3/4 (b) 2/20
probability of occurrence of A as well as B is (c) 13/20 (d) 3/20
given by 159. If events A and B are independent and P(A)=
(a) P(AB)= P(A/B) 2/3 , P(B)= 3/5 then P(A+B)is equal to
(b) P(AB)= P(A)/ P(B) (a) 13/15 (b) 6/15
(c) P(AB)= P(A)P(B) (d) None (c) 1/15 (d) none
150. For the condition P(AB)= P(A)P(B)two events 160. The expected number of head in 100 tosses of
A and B are said to be an unbiased coin is
(a) dependent (b) independent (a) 100 (b) 50
(c) equally like (d) none (c) 25 (d) none
151. The conditional probability of an event B on 161. A and B are two events such that P(A)= 1/3,
the assumption that another event A has P(B) = 1/4, P(A+B)= 1/2, than P(B/A) is equal to
actually occurred is given by (a) 1/4 (b) 1/3
(a) P(B/A)= P(AB)/P(A) (c) 1/2 (d) none
(b) P(A/B)= P(AB)/ P(B) 162. Probability mass function is always
(c) P(B/A)= P(AB) (a) 0 (b) greater than 0
(d) P(A/B)= P(AB)/ P(A)P(B) (c) greater than equal to 0
152. Given P(A)= 1/2, P(B)= 1/3, P(AB)= 1/4, the value (d) less than 0
of P(A+B) is 163. The sum of probability mass function is equal
(a) 3/4 (b) 7/12 to
(c) 5/6 (d) 1/6 (a) –1 (b) 0
153. Given P(A)= 1/2, , P(B)= 1/3, P (AB)= 1/4, the (c) 1 (d) none
value of P (A/B) is 164. When X is a continues function f(x)is called
(a) 1/2 (b) 1/6 (a) probability mass function
(c) 2/3 (d) 3/4 (b) probability density function
154. If P (A)= 1/3, P(B)= 1/4, the events A & B are (c) both (d) none
(a) not equally likely 165. Which of the following set of function define
(b) mutually exclusive a probability space on S = {a1, a2, a3}
(c) equally likely (d) none (a) P(a1)=1/3, P(a2) = 1/2, , P(a3)= 1/4
155. If events A and B are independent then (b) P(a1)= 1/3, P(a2)= 1/6,P(a3) = 1/2
a) AC and BC are dependent
(c) P(a1)= P(a2)= 2/3, P(a3)= 1/4
b) AC and B are dependent
(d) None
c) A and BC are dependent
166. If P (a1)= 0, P(a2)= 1/3, P (a3) = 2/3 then S = {a1,
d) AC and BC are also independent
a2, a3} is a probability space
156. A card is drown from each of two well-shuffled
packs of cards.The probability that at least one (a) true (b) false
of them is an ace is (c) both (d) none
(a) 1/69 (b) 25/169 167. If two events are independent then
(c) 2/13 (d) none (a) P(B/A)= P(AB) P(A)
157. When a die is tossed, the sample space is (b) P(B/A)= P(AB) P(B)
(a) S =(1,2,3,4,5) (b) S =(1,2,3,4) (c) P(B/A)= P(B)
(c) S =(1,2,3,4,5,6) (d) none (d) P(B/A)P(A)

PROBABILITY 367
168. When expected value is negative the result is 177. Two dice are thrown at a time. The probability
(a) favourable (b) unfavourable that the numbers shown are equal is
(c) both (d) none to the above (a) 2/6 (b) 5/6
169. The expected value of X, the sum of the scores, (c) 1/6 (d) none
when two dice are rolled is 178. Two dice are thrown at a time. The probability
(a) 9 (b) 8 that ‘the difference of numbers shown is 1’ is
(c) 6 (d) 7 (a) 11/18 (b) 5/18
170. Let A and B be the events with P(A)= 1/3, P(B) (c) 7/18 (d) none
= 1/4 and P(AB)= 1/12 then P(A/B) is equal to 179. Two dice are thrown together. The probability
(a) 1/3 (b) 1/4 that ‘the event the difference of numbers
(c) 3/4 (d) 2/3 shown is 2’ is
171. Let A and B be the events with P(A)= 2/3, P(B)= (a) 2/9 (b) 5/9
1/4 and P(AB)= 1/12 then P(B/A) is equal to (c) 4/9 (d) 7/9
(a) 7/8 (b) 1/3 180. The probability space in tossing two coins is
(c) 1/8 (d) none (a) {(H,H),(H,T),(T,H)} (b) {(H,T),(T,H),(T,T)}
172. The odds in favour of one student passing a (c) {(H,H),(H,T),(T.H), (T,T)} (d) none
test are 3:7.The odds against another student 181. The probability of drawing a white ball from a
passing at are 3:5.The probability that both pass bag containing 3 white and 8 balls is
is (a) 3/5 (b) 3/11
(a) 7/16 (b) 21/80 (c) 8/11 (d) none
(c) 9/80 (d) 3/16 182. Two dice are thrown together. The probability
173. The odds in favour of one student passing a of the event that the sum of numbers shown
test are 3:7.The odds against another student is greater than 5 is
passing at are 3:5. The probability that both (a) 13/18 (b) 15/18
fail is (c) 1 (d) none
7 21 183. A traffic census show that out of 1000 vehicles
(a) (b) passing a junction point on a highway 600
16 80
turned to the right. The probability of an
9 3 automobile turning the right is
(c) (d)
80 16 (a) 2/5 (b) 3/5
174. In formula P(B/A), P(A) is (c) 4/5 (d) none
(a) greater than zero (b) less than zero 184. Three coins are tossed together. The
(c) equal to zero probability of getting three tails is
(d) greater than equal to zero (a) 5/8 (b) 3/8
175. Two events A and B are mutually exclusive (c) 1/8 (d) none
means they are 185. Three coins are tossed together.The probability
(a) not disjoint (b) disjoint of getting exactly two heads is
(c) equally likely (d) none (a) 5/8 (b) 3/8
176. A bag contains 10 white and 10 black balls A (c) 1/8 (d) none
ball is drawn from it. The probability that it 186. Three coins are tossed together. The
will be white is probability of getting at least two heads is
(a) 1/10 (b) 1 (a) 1/2 (b) 3/8
(c) 1/2 (d) none (c) 1/8 (d) none

368 BUSINESS MATHEMATICS, LOGICAL REASONING & STATISTICS (Paper 3) [CA Foundation]
187. 4 coins are tossed. The probability that there that one of the horses will win
are 2 heads is (a) 5/12 (b) 7/12
(a) 1/2 (b) 3/8 (c) 1/12 (d) none
(c) 1/8 (d) none 198. If the probability of a horse A winning a race is
188. If 4 coins are tossed. The chance that there 1/6 and the probability of a horse B winning
should be two tails is the same race is 1/4 , What is the probability
(a) 1/2 (b) 3/8 that none of them will win
(c) 1/8 (d) none (a) 5/12 (b) 7/12
189. If A is an event and AC its complementary (c) 1/12 (d) none
event then 199. If P (A)= 7/8 then(P(AC) is equal to
(a) P(A)=P(A C)–1 (a) 1 (b) 0
(b) P(AC)= 1 – P(A) (c) 7/8 (d) 1/8
(c) P(A)= 1 + P(AC) (d) none 200. The value of P(S) were S is the sample space is
190. If P(A)= 3/8, P(B)= 1/3 and P(AB)= 1/4 then P(AC) (a) –1 (b) 0
is equal to (c) 1 (d) none
(a) 5/8 (b) 3/8 201. A man can kill a bird once in three shots.The
(c) 1/8 (d) none probabilities that a bird is not killed is
191. If P(A)= 3/8, P(B)= 1/3 then P( B ) is equal to
o (a) 1/3 (b) 2/3
(a) 1 (b) 1/3 (c) 1 (d) 0
(c) 2/3 (d) none 202. If on an average 9 shops out of 10 return safely
to a port, the probability of one ship returns
192. If P(A)= 3/8, P(B)= 1/3 and P(AB)= 1/4 then P(A +
safely is
B)is
(a) 1/10 (b) 8/10
(a) 13/24 (b) 11/24
(c) 9/10 (d) none
(c) 17/24 (d) none
203. If on an average 9 shops out of 10 return safely
193. If P(A)= 1/5, P(B)= 1/2 and A and B are mutually
to a port, the probability of one ship does not
exclusive then P(AB) is
reach safely is
(a) 7/10 (b) 3/10
(a) 1/10 (b) 8/10
(c) 1/5 (d) none
(c) 9/10 (d) none
194. The probability of throwing more than 4 in a
204. The probability of winning of a person is 6/11
single throw from an ordinary die is
and at a result he gets ` 77/-. The expectation
(a) 2/3 (b) 1/3 of this person is
(c) 1 (d) none (a) ` 35/- (b) ` 42/-
195. The probability that a card drawn at random (c) ` 58/- (d) none
from the pack of playing cards may be either a
205. A family has 2 children. The probability that
queen or an ace is
both of them are boys if it is known that one of
(a) 2/13 (b) 11/13 them is a boy
(c) 9/13 (d) none (a) 1 (b) 1/2
196. The chance of getting 7 or 11 in a throw of 2 (c) 3/4 (d) none
dice is
206. The Probability of the occurrence of a number
(a) 7/9 (b) 5/9 greater then 2 in a throw of a die if it is known
(c) 2/9 (d) none that only even numbers can occur is
197. If the probability of a horse A winning a race is (a) 1/3 (b) 1/2
1/6 and the probability of a horse B winning (c) 2/3 (d) none
the same race is 1/4 , what is the probability

PROBABILITY 369
207. A player has 7 cards in hand of which 5 are red
and of these five 2 are kings. A card is drawn at HOME WORK-1
random. The probability that it is a king, it
being known that it is red is
(a) 2/5 (b) 3/5
(c) 4/5 (d) none
1. Initially, probability was a branch of
208. In a class 40 % students read Mathematics, 25
(a) Physics (b) Statistics
% Biology and 15 % both Mathematics and
Biology. One student is select at random. The (c) Mathematics (d) Economics.
probability that he reads Mathematics if it is 2. Two broad divisions of probability are
known that he reads Biology is (a) Subjective probability and objective
(a) 2/5 (b) 3/5 probability
(c) 4/5 (d) none (b) Deductive probability and non-deductive
209. In a class 40 % students read Mathematics, 25 probability
% Biology and 15 % both Mathematics and (c) Statistical probability and Mathematical
Biology. One student is select at random.The probability
probability that he reads Biology if he reads (d) None of these.
Mathematics 3. If P(A  B)  0 then the two events A and
(a) 7/8 (b) 1/8 Bare
(c) 3/8 (d) none (a) Mutually exclusive(b) Exhaustive
210. Probability of throwing an odd no with an (c) Equally likely (d) Independent.
ordinary six faced die is 4. If for two events A and B, P(AUB) = 1, then A
(a) 1/2 (b) 1 and B are
(c) –1/2 (d) 0 (a) Mutually exclusive events
211. For a event A which is certain, P (A) is equal to (b) Equally likely events
(a) 1 (b) 0 (c) Exhaustive events
(c) –1 (d) none (d) Dependent events .
212. When none of the outcomes is favourable to 5. If an unbiased coin is tossed once, then the
the event then the event is said to be two events Head and Tail are
(a) Mutually exclusive
(a) certain (b) sample
(b) Exhaustive
(c) impossible (d) none
(c) Equally likely
(d) All these (a), (b) and (c).
6. If for two events A and B,
P(A  B)  P(A)XP(B) then the two events
---0---0---
A and B are
(a) Independent
(b) Dependent
(c) Not equally likely
(d) Not exhaustive.
7. If P(A/B) = P(A), then
(a) A is independent of B
(b) B is independent of A
(c) B is dependent of A
(d) Both (a) and (b).

370 BUSINESS MATHEMATICS, LOGICAL REASONING & STATISTICS (Paper 3) [CA Foundation]
8. If two events A and B are independent, then
q
(a) A and the complement of B are (c) (d) none
independent
pq
(b) B and the complement of A are
independent 16. If P(A) = 5/9, then the odds against the event A
(c) Complements of A and B are independent is
(d) All of these (a), (b) and (c). (a) 5: 9 (b) 5: 4
9. If two events A and Bare independent, then (c) 4: 5 (d) 5: 14
(a) They can be mutually exclusive 17. If A, Band C are mutually exclusive and
exhaustive events, then P(A) + P(B) + P(C)
(b) They can not be mutually exclusive
equals to
(c) They can not be exhaustive
(a) 1/3
(d) Both (b) and (c)
(b) 1
10. If two events A and B are mutually exclusive,
(c) a
then
(d) any value between a and 1.
(a) They are always independent
18. If A denotes that a student reads in a school
(b) They may be independent
and B denotes that he plays cricket, then
(c) They can not be independent
(a) P(A  B)  1 (b) P(A  B)  1
(d) They can not be equally likely.
11. If a coin is tossed twice, then. the events (c) P(A  B)  0 (d) P(A)= P(B)
‘occurrence of one head’, ‘occurrence of 2 19. P(B/ A) is defined only when
heads’ and ‘occurrence of no head’ are
(a) A is a sure event
(a) Independent
(b) B is a sure event
(b) Equally likely
(c) A is not an impossible event
(c) Not equally likely
(d) B is an impossible event.
(d) Both (a) and (b).
20. P(A/B’) is defined only when
12. The probability of an event can assume any
(a) B is not a sure event
value between
(b) B is a sure event
(a) - 1 and 1 (b) 0 and 1
(c) B is an impossible event
(c) - 1 and 0 (d) none
(d) B is not an impossible event.
13. If P(A) = 0, then the event A
21. For two events A and B, P(A  B) = P(A) + P(B)
(a) will never happen
only when
(b) will always happen
(a) A and B are equally likely events
(c) may happen
(b) A and B are exhaustive events
(d) may not happen.
(c) A and B are mutually independent
14. If P(A) = 1, then the event A is known as
(d) A and B are mutually exclusive.
(a) symmetric event
22. Addition Theorem of ProbabiIity states that
(b) dependent event for any two events A and B,
(c) improbable event (a) P(A  B) = P(A) + P(B)
(d) sure event. (b) P(A  B) = P(A) + P(B) + P(A  B)
15. If P : q are the odds in favour of an event, then
(c) P(A  B) = P(A) + P(B) - P(A  B)
the probability of that event is
(d) P(A  B) = P(A) x P(B)
P
(a) plq (b)
pq

PROBABILITY 371
23. For any two events A and B, 29. If two events A and B are independent, then
(a) P(A-B) = P(A) - P(B) p(A  B)
(b) P(A-B) = P(A) - P(A  B) (a) equals to P(A) + P(B)
(c) P(A-B) = P(B) - P(A  B) (b) equals to P(A) x P(B)
(d) P(B-A) = P(B) + P(A  B). (c) equals to P(A) x P(B/ A)
24. The limitations of the classical definition of (d) equals to P(B) x P(A/B).
probability 30. Values of a random variable are
(a) it is applicable when the total number of (a) always positive numbers.
elementary events is finite (b) always positive real numbers.
(b) it is applicable if the elementary events (c) real numbers.
are equally likely (d) natural numbers.
(c) it is applicable if the elementary events 31. Expected value of a random variable
are mutually independent (a) is always positive
(d) (a) and (b). (b) may be positive or negative
25. According to the statistical definition of (c) may be positive or negative or zero
probability, the probability of an event A is (d) can never be zero.
the 32. If all the values taken by a random variable are
(a) limiting value of the ratio of the no. of equal then
times the event A occurs to the number (a) its expected value is zero
of times the experiment is repeated (b) its standard deviation is zero
(b) the ratio of the frequency of the (c) its standard deviation is positive
occurrences (d) its standard deviation is a real number
of A to the total frequency occurrences 33. If x and yare independent, then
of A to the non-occurrence of A (a) E(xy) = E(x) x E(y)
(c) the ratio of the frequency of the (b) E(xy) = E(x) + E(y) ,
(d) the ratio of the favourable elementary (c) E(x + y) = E(x) + E(y)
events (d) E(x - y) = E(x) -x E(y)
to A to the total number of elementary 34. If a random variable x assumes the values X1 ,
events. x2 , x3 , x4 with corresponding probabilities P1,
26. The Theorem of Compound Probability states P2 , P3 , P4 then the expected value of x is
that for any two events A and B (a) P1 + P2 + P3 + P4
(a) P(A  B)=P(A) x P(B/A) (b) XI P1 + x2 P3 + x3 P2 + x1 P4
(b) P(A  B)=P(A) x P(B/A) (c) P1 x1 + P2 x2 + P3 x3 + P4 x4
(c) P(A  B) = P(A) x P(B) (d) none
(d) P(A  B) = P(B) + P(B) - P(A  B) 35. Variance of a random variable x is given by
27. If A and B are mutually exclusive events, then (a) E(x  )2 (b) E [x  E(x)]2
(a) P(A) = P(A-B).
(b) P(B) = P(A-B). (c) E(x 2  ) (d) (a) or (b)
(c) P(A) = P(A  B). 36. If two random variables x and yare related by
(d) P(B) = P(A  B). y = 2 - 3x, then the SD of y is given by
28. If P(A-B) = P(B-A), then the two events A and B (a) -3 x SD of x (b) 3 x SD of x.
satisfy the condition (c) 9 x SD of x (d) 2 x SD of x.
(a) P(A) = P(B). 37. Probability of getting a head when two
(b) P(A) + P(B) = 1 unbiased coins are tossed simultaneously is
(c) P(A  B) = 0 (a) 0.25 (b) 0.50
(d) P(A  B) = 1 (c) 0.20 (d) 0.75
372 BUSINESS MATHEMATICS, LOGICAL REASONING & STATISTICS (Paper 3) [CA Foundation]
38. If an unbiased coin is tossed twice, the 3 3
probability of obtaining at least one tail is  1 5
(c) 1-   (d) 1-  
(a) 0.25 (b) 0.50 6 6
(c) 0.75 (d) 1.00 47. Two balls are drawn from a bag containing 5
39. If an unbiased die is roIled once, the odds in white and 7 black balls at random. What is the
favour of getting a point which is a multiple of probability that they would be of different
3 is colours?
(a) 1:2 (b) 2:1 (a) 35/66 (b) 30/66
(c) 1:3 (d) 3:1 (c) 12/66 (d) None
40. A bag confains 15 one rupee coins, 25 two rupee 48. What is the chance of throwing at least 7 in a
coins and 10 five rupee coins. If a coin is single cast with 2 dice?
selected at random from the bag, then the (a) 5/12 (b) 7/12
probability of not selecting a one rupee coin is
(c) 1/4 (d) 17/36
(a) 0.30 (b) 0.70
49. What is the chance of getting at least one
(c) 0.25 (d) 0.20 defective item if 3 items are drawn randomly
41. A, B, C are three mutually independent with from a lot containing 6 items of which 2 are
probabilities 0.3, 0.2 and 0.4 respectively. What defective item?
is P (A  B  C)? (a) 0.30 (b) 0.20
(a) 0.400 (b) 0.240 (c) 0.80 (d) 0.50
(c) 0.024 (d) 0.500 50. If two unbiased dice are rolled together, what
42. If two letters are taken at random from the is the probability of getting no difference of
word HOME, what is the Probability that none points?
of the letters would be vowels? (a) 1/2 (b) 1/3
(a) 1/ 6 (b) 1/2 (c) 1/5 (d) 1/6
(c) 1/3 (d) 1/4 51. If A, Band C are mutually exclusive
43. If a card is drawn at random from a pack at 52 independent and exhaustive events then what
cards, what is the chance of getting a Spade or is the probabIlity that they occur
an ace? simultaneously?
(a) 4/13 (b) 5/13 (a) 1
(c) 0.25 (d) 0.20 (b) 0.50
44. If x and yare random variables having expected (c) 0
values as 4.5 and 2.5 respectively, then the (d) any value between 0 and 1
expected value of (x-y) is
52. There are 10 balls numbered from 1 to 10 in a
(a) 2 (b) 7 box. If one of them is selected at random, what
(c) 6 (d) a is the probability that the number printed on
45. If variance of a random variable x is 23, then the ball would be an odd number greater than
what is the variance of 2x+10 ? 4?
(a) 56 (b) 33 (a) 0.50 (b) 0.40
(c) 46 (d) 92 (c) 0.60 (d) 0.30
46. What is the probability of having at least one 53. Following are the wages of 8 workers in
‘six’ from 3 throws of a perfect die? rupees:
3 50, 62, 40, 70, 45, 56, 32, 45
(a) 5/6 (b) 5 6 If one of the workers is selected at random,
what is the probability that his wage would be
lower than the average wage?

PROBABILITY 373
(a) 0.625 (b) 0.500 also a girl ?
(c) 0.375 (d) 0.450 (a) 0.50 (b) 0.75
54. A, Band C are three mutually exclusive and (c) 1/3 (d) 2/3
exhaustive events such that P (A) = 2 P (B) = 63. Two coins are tossed simultaneously. What is
3P(C). What is P (B)? the probability that the second coin would
(a) 6/11 (b) 6/22 show a tail given that the first coin has shown
(c) 1/6 (d) 1/3 a head?
55. For two events A and B, P (B) = 0.3, P (A but not (a) 0.50 (b) 0.25
B) = 0.4 and P (not A) = 0.6. The events A and (c) 0.75 (d) 0.125
Bare 64. If a random variable x assumes the values 0, 1
(a) exhaustive and 2 with probabilities 0.30, 0.50 and 0.20,
(b) independent then its expected value is
(c) equally likely (a) 1.50 (b) 3
(d) mutually exclusive (c) 0.90 (d) 1
56. A bag contains 12 balls which are numbered 65. If two random variables x and yare reiated as y
from 1 to 12. If a ball is selected at random, = -3x + 4 and standard deviation of x is 2, then
what is the probability that the number of the the standard deviation of y is related
ball will be a multiple of 5 or 6 ? (a) - 6 (b) 6
(a) 0.30 (b) 0.25 (c) 18 (d) 3.50
(c) 0.20 (d) 1/3 66. If 2x + 3y + 4 = 0 and v(x) = 6 then v (y) is
(a) 8/3 (b) 9
57. For two independent events A and B, what is P
(c) - 9 (d) 6
(A+B), given P(A) = 3/5 and P(B) = 2/3?
67. What is the probability that a leap year
(a) 11 / 15 (b) 13 /15
selected at random would contain 53
(c) 7/15 (d) 0.65 Saturdays?
58. If for two independent events A and B, P ( A (a) 1/7 (b) 2/7
 B) = 2/3 and P (A) = 2/5, what is P (B)? (c) 1/12 (d) 1/4
(a) 4/15 (b) 4/9 68. If an unbiased coin is tossed three times, what
(c) 5/9 (d) 7/15 is the probability of getting more that one
59. If P (A) = 2/3, P (B) =3/4, P (A/B) = 2/3, then what head?
is P (B / A)? (a) 1 /8 (b) 3/8
(a) 1/3 (b) 2/3 (c) 1/2 (d) -1/3
(c) 3/4 (d) 1/2 69. If two unbiased dice are rolled, what is the
60. If P (A) = a, P (B) = b and P (P (A  B) = c then probability of getting points neither 6 nor 9?
the expression of P (A’  B’) in terms of a, b (a) 0.25 (b) 0.50
and cis (c) 0.75 (d) 0.80
(a) 1 - a - b - c (b) a+b-c 70. What is the probahility that 4 children selected
(c) 1+ a - b - c (d) 1-a-b+c at random would have different birthdays?
61. For three events A, Band C, the probability that
only A occur is 364 x 363 x 362 6x5x4
(a) (b)
(a) P (A) (365)3 73
(b) P(A  B  C)
(c) 1/365 (d) (1/7) 3
(c) P ( A   B  C)
71. A box contains 5 white and 7 black balls. Two
(d) P (A  B  C ) successive drawn of 3 balls are made (i) with
62. It is given that a family of 2 children has a girl, replacement (ii) without replacement. The
what is the probability that the other child is probability that the first draw would produce
white balls and the second draw would

374 BUSINESS MATHEMATICS, LOGICAL REASONING & STATISTICS (Paper 3) [CA Foundation]
produce black balls are respectively (c) 0.392 (d) 0.388
(a) 6/321 and 3/926 78. Tom speaks truth in 30 percent cases and Dick
(b) 1/20 and 1/30 speaks truth in 25 percent cases. What is the
(c) 35/144 and 35/108 probability that they would contradict each
(d) 7/968 and 5/264 other?
72. There are three boxes with the following (a) 0.325 (b) 0.400
composition: (c) 0.925 (d) 0.075
Box I: 5 Red + 7 White + 6 Blue balls 79. There are two urns. The first urn contains 3 red
Box II: 4 Red + 8 White + 6 Blue balls and 5 white balls whereas the second urn
Box III: 3 Red + 4 White + 2 Blue balls contains 4 red and 6 white billls. A ball is taken
If one ball is drawn (from each) at random, then at random from the first urn.and is transferred
what is the probability that they would be of same to the second urn. Now another ball is selected
colour? at random from the second arm. The
(a) 89/729 (b) 97/729 probability that the second ball would be red
(c) 82/729 (d) 23/32 is
73. A number is selected at random from the first (a) 7/20 (b) 35/88
1000 natural numbers. What is the probability
(c) 17/52 (d) 3/20
that the number so selected would be a
80. For a group of students, 30%,40% and 50%
multiple of 7 or 11 ?
failed in Physics, Chemistry and at least one of
(a) 0.25 (b) 0.32
the tvvo subjects respectively. If an examinee
(c) 0.22 (d) 0.33
is selected at random, what is the probability
74. A bag contains 8 red and 5 white balls. Two
that he passed in Physics if it is known that he
successive draws of 3 balls are made without
failed in Chemistry ?
replacement. The probability that the first
draw will produce 3 white balls and the second (a) 1 / 2 (b) 1/3
3 red balls is (c) 1/4 (d) 1/6
(a) 5/223 (b) 6/257 81. A packet of 10 electronic components is known
(c) 7/429 (d) 3/548 to include 2 defectives. If a sample of 4
75. There are two boxes containing 5 white and 6 components is selected at random from the
blue balls and 3 white and 7 blue balls packet, what is the probability that the sample
respectively. If one of the the boxes is selected does not contain more than 1 defective?
at random and a ball is drawn from it, then the ( a) 1 / 3 (b) 2/3
probability that the ball is blue is (c) 13/15 (d) 3/15
(a) 115/227 (b) 83/250 82. X and Y stand in a line with 6 other people.
(c) 137/220 (d) 127/250 What is the probability that there are 3 persons
76. A problem in probability was given to three between them?
CA students A, Band C whose chances of (a) 1 / 5 (b) 1 /6
solving it are 1/3, 1/5 and 1/2 respectively. What (c) 1/7 (d) 1/3
is the probability that the problem would be 83. Given that P (A) = 1/2, P (B) = 1/3, P (A  B) = 1/
solved? 4, what is P (A’ /B’)
(a) 4/15 (b) 7/8 (a) 1/2 (b) 7/8
(c) 8/15 (d) 11/15 (c) 5/8 (d) 2/3
77. There are three persons aged 60, 65 and 70
84. Four digits 1,2,4 and 6 are selected at random
years old. The survivals probabilities for these
to form a four digit number. What is the
three persons for another 5 years are 0.7, 0.4
probability that the number so formed, would
and 0.2 respectively. What is the probability
be divisible by 4?
that at least two of them would survive another
five vears? (a) 1/2 (b) 1/5
(a) 0.425 (b) 0.456 (c) 1/4 (d) 1/3

PROBABILITY 375
85. The probability distribution of a random (a) 86/104 (b) 13/26
variable x is given below: (c) 2/26 (d) 42/52
x: 1 2 4 5 6 92. The chance of Ram to stand first in the class is
P: 0.15 0.25 0.20 0.30 0.10 1/3 and that of Rahim is 1/5. The probability
that either of the two will stand first in the
What is the standard deviation of x?
class is :
(a) 1.49 (c)1.69 (a) 7/15 (b) 6/15
(b) 1.56 (d) 1.72 (c) 8/15 (d) 5/15
86. A packet of 10 electronic components is known 93. The probability of throwing an odd sum with
to include 3 defectives. If 4 components are two fair dice is :
selected from the packet at random, what is (a) 1/3 (b) 1/6
the expected value of the number of (c) 1 (d) 1/2
defective? 94. The probability that there is at least one spot
(a) 1.20 (c)1.69 in two rollings of a die is :
(b) 1. 21 (d) 1.72 (a) 11/36 (b) 15/18
87. The probability that there is at least one error (c) 1/36 (d) 1/18
in an account statement prepared by 3 persons 95. The probabilities of Mr. X and Mr. Y not living
A, Band Care 0.2, 0.3 and 0.1 respectively. If A, for one more year are 1/9 and 1/7 respectively.
Band C prepare 60, 70 and 90 such statements, The probability of living one more year of
then the expected number or correct either one or both is
statements (a) 2/21 (b) 6/63
(a) 170 (b) 176 (c) 15/63 (d) 62/63
(c) 178 (d) 180 96. In a group there are 4 boys and some girls. The
88. A bag contains 6 white and 4 red balls. If a probability of selecting two girls at random
person draws 2 balls and receives ` 10 and ` from the group is 13 , the number of girls in
20 for a white and red balls respectively, then the group is
his expected amount is (a) 5 (b) 4
(a) ` 25 (b) ` 26 (c) 3 (d) 6
(c) ` 29 (d) ` 28 97. Four dice are thrown, find the probability that
89. The probability distribution of a random 1 will appear on at least one of the dice.
variable is as follows: (a) 616/1296 (b) 308/1296
x: 1 2 4 6 8 (c) 671/1296 (d) 72/1296
P: k 2k 3k 3k k 98. A husband and his wife appear in an interview
The variance of x is for two vacancies. The probability of their
(a) 2.1 (b) 4.41 selection are respectively1/4 and 1/3. What is
(c) 2.32 (d) 2.47 the probability that only one of them will be
selected?
(a) 5/11 (b) 5/12
90. For a 60 year old person living upto the age of
(c) 5/13 (d) 5/14
70, it is 7 : 5 against him and for another 70
99. A group consists of 4 men, 3 women and 2 boys.
years old person surviving upto the age of 80,
Three persons are selected at random. The
it is 5 : 2 against him. The probability that only
probability that 2 men are selected is :
one of them will survive for 10 years more is :
(a) 2/28 (b) 5/28
(a) 15/42 (b) 39/84
(c) 15/28 (d) 10/28
(c) 49/84 (d) 40/84
100. The probability that a leap year will have 53
91. If 7 : 6 is in favour of P to survive 10 years more Sundays is :
and 5 : 3 in favour of Q to survive 10 years (a) 1/7 (b) 2/7
more, the probability that all least one of (c) 3/7 (d) 1/53
them will survive for 10 years more is :
376 BUSINESS MATHEMATICS, LOGICAL REASONING & STATISTICS (Paper 3) [CA Foundation]
101. With a pair of dice thrown at a time, the
probability of getting a sum more than 9 is ANSWER KEYS
(a) 15/18 (b) 17/36
(c) 5/6 (d) none
102. If the chance of A hitting a target is 3 times out 1 (c) 2 (a) 3 (a) 4 (c)
of 4 and of B 4 times out of 5 and of C 5 times 5 (d) 6 (b) 7 (d) 8 (d)
out of 6. The probability that the target will be
hit in two hits is : 9 (b) 10 (c) 11 (c) 12 (d)
(a) 190/240 (b) 20/30 13 (a) 14 (d) 15 (b) 16 (c)
(c) 60/120 (d) 47/120 17 (b) 18 (c) 19 (c) 20 (a)
103. An urn contains 5 yellow, 4 black and 3 white
balls. Three balls are drawn at random. The 21 (d) 22 (c) 23 (b) 24 (d)
probability that no black ball is selected is : 25 (a) 26 (a) 27 (a) 28 (a)
(a) 6/55 (b) 14/55 29 (a) 30 (c) 31 (c) 32 (b)
(c) 8/55 (d) 9/55
33 (a) 34 (c) 35 (d) 36 (b)
104. A bag contains 3 White and 5 red balls. A game
is played such that a ball is drawn, its colour is 37 (b) 38 (c) 39 (a) 40 (b)
noted and replaced with two additional balls 41 (c) 42 (a) 43 (a) 44 (b)
of the same colour. The selection is made three
45 (d) 46 (d) 47 (a) 48 (b)
times. The probability that a white ball is
selected at each trial is : 49 (c) 50 (d) 51 (c) 52 (d)
(a) 7/64 (b) 21/44 53 (b) 54 (b) 55 (d) 56 (d)
(c) 105/512 (d) 9/320 57 (b) 58 (b) 59 (c) 60 (d)
105. Given that P (A) = 1/3, P (B) = 3/4 and P (A U B) =
11/12, the probability, P (B/A) is 61 (d) 62 (a) 63 (a) 64 (c)
(a) 1/6 (b) 4/9 65 (b) 66 (a) 67 (b) 68 (c)
(c) 1/2 (d) 1/4 69 (c) 70 (a) 71 (d) 72 (a)
106. A can hit a target 2 times in 5 shots, B 3 times in
73 (c) 74 (c) 75 (c) 76 (d)
5 shots and C 4 times in 5 shots, They fire a
volley (each try once to hit the target). The 77 (d) 78 (b) 79 (b) 80 (a)
probability that two shots hit is 81 (c) 82 (c) 83 (c) 84 (d)
(a) 20/125 (b) 58/125
85 (c) 86 (a) 87 (c) 88 (d)
(c) 120/125 (d) 55/125
107. X can solve 80% of the problems while Y can 89 (b) 90 (b) 91 (a) 92 (c)
solve 90% of the problems given in a statistics 93 (d) 94 (a) 95 (d) 96 (d)
book. A problem is selected at random. What 97 (c) 98 (b) 99 (d) 100 (b)
is the probability that at least one of them will
solve the problem. 101 (d) 102 (d) 103 (b) 104 (a)
(a) 0.95 (b) 0.98 105 (c) 106 (b) 107 (b) 108 (a)
(c) 0.96 (d) 0.99 109 (b)
108. If a pair of dice is thrown, the probability that
the sum of the digits is neither 7 nor 11 is
(a) 7/9 (b) 9/10
(c) 7/10 (d) 6/9
109. Out of numbers 1 to 120, one is selected at
random, what is the probability that it is
divisible by 8 or 10 ?
(a) 0.9 (b) 0.2
(c) 0.6 (d) 0.7
PROBABILITY 377
8.  
If (A U B) = 0.8 and P (A  B) = 0.3 then P A +
HOME WORK-2

P B is equal to:
(a) 0.3 (b) 0.5
1. If P(A) = 1 and P(B) = 1/3 then P(A/B) =
(c) 0.9 (d) 0.7
(a) 1/3 (b) 2/3
(c) 1 (d) ½
2. The probability of A solving a problem is 7 /12 ANSWER KEYS
the odds against solving a problem
(a) 5:7 (b) 4:7 1 (c) 2 (a)
(c) 5:8 (d) 4:5
3 (a) 4 (b)
3. Two events A& B Probabilities 0.24 and 0.52
respectively. If the probability of both A and B 5 (a) 6 (b)
occurs simultaneously is 0.15. then the 7 (a) 8 (c)
probabilities that neither A nor B is.
(a) 0.39 (b) 0.375
(c) 0.61 (d) 0.86
4. From a bag is containing 10 black and 20 white
balls, a ball is drawn at random. What is the
probability that is black?
(a) ½ (b) 1/3
(c) 1 (d) 2
5. The odds in favour of an event is 2:3 and the
odds against another event is 3:7. Find the
probability that only one of the two events
occurs.
27 17
(a) (b)
50 50
37
(c) (d) None
50
1 1
6. If P(A) = , P(B) = , and P(A  B) is equal to
2 3
11 07
(a) (b)
12 12
10 1
(c) (d)
12 6
7. Two different dice are thrown simultaneously,
then the probability, that the sum of two
numbers appearing on the top of dice is 9 is
1 8
(a) (b)
9 9
7
(c) (d) None
9

378 BUSINESS MATHEMATICS, LOGICAL REASONING & STATISTICS (Paper 3) [CA Foundation]
CHAPTER-18
PROBABILITY (THEORETICAL) DISTRIBUTION

INTRODUCTION
THEORETICAL DISTRIBUTIONS
- BINOMIAL DISTRIBUTION
1. Binomial Distribution is associated with the name of the french Mathemaician James Bernoulli.
2. Binomial Distributions are discrete probability distributions.
3. In a Binomial Distribution, variance is always less than mean.
4. The Mean of Distribution is np.
5. The variance of Binomial Distribution is npq.
6. The Variance of a Binomial Distribution can not excced n/4.
7. If Value of P = 0.5, the Binomial Distribution is symmetrical.
8. If Value of P > 0.5, the Binomial Distribution is skewed to the left.
9. As p increases for fixed n, the Binomial Distribution shifts to the right.
10. As n increases for fixed p, the Binomial Distribution shifts to the right.
11. As p increases for a fixed n, both the mean and mode increase.
12. If in Binomial Distributin Mean is 10 and Standard Deviation is 3, q will be 0.9.
13. The results calculated by a student for a Binomial Distribution as mean 10 and standard
deviation 4, are not correct.
14. The Sampling Distribution of the Numbers of Successes follows a Binomial Probability
Distribution.
15. The First Moment of Binomial Distribution is = 0.
16. The second Moment of Binomial Distribution is = npq.
17. Binomial Distribution tends to be a Normal Distribution when (a) ‘N’ is large (b) Neither ‘p’ or
‘q’ is close to zero. In practice, the approximation is very good if both np and nq are greater
than s.d.

PROBABILITY DISTRIBUTION 379


LIST OF FORMULAE

1. Probability Function P(x = r) = nCrPr qn-r, where r = 0, 1, 2,...,n


P = Probability of success in a single trial
q=1–p
n = Number of trials
r = Numberof successes in n trials
Probability of 0 event P(r = 0) nC0 P0 qn-0 = qn
Probability of 1 event P(r = 1) nC1 P1 qn-1
Probability of 2 event P(r = 2) nC2 P2 qn-2
Probability of 3 event P(r = 3) = nC3 P3 qn-3
Probability of at least 1 event P(r = 1 or more) = 1 – P (r = 0)
Probability of at least 2 events P(r = 2 or more) = P (r = 0) + P(r = 1) + P(r = 2)
Probability of less than 2 events P(r = 0 or 1) = P (r = 0) + P(r = 1)
Probability of more than 2 event P(r = 3 or more) = 1 – [P (r = 0) + P(r = 1) + P(r = 2)]
2. Expected Frequency Function N.P (r) = N.nCr, Pr qn-r
Thus, expected frequency = Probability of an event ×
Total no. of observations (N)
Note : Fitting a binomial distribution means finding out
the theoretical expected frequencies.
3. Mean = np
4. Variance = npq

380 BUSINESS MATHEMATICS, LOGICAL REASONING & STATISTICS (Paper 3) [CA Foundation]
INTRODUCTION
THEORETICAL DISTRIBUTIONS
- POISSON DISTRIBUTION
1. Poisson Distribution is associated with the name of French Mathematician Simon Denis Poisson.
2. Poisson Distributions are discrete probability distributions.
3. In case of Poisson Distribution Mean & Variance are equal.
4. In case of Poisson Distribution Mean and Variance is m.
5. In case of Poisson Distribution p is less than q.
6. In case of Poisson Distribution m must be greater than 0.
7. In case of Poisson Distribution e = 2.7183.
8. In case Poisson Distribution as m or n increases, the distribution shifts to right.
9. In case of Poisson Distribution Mean, Variance, Second Moment and Third Moment are
equal.

10. In case of Poisson Distribution Skewness (  1) and Coefficient of Kurtosis (  2) are same
and are equal to reciprocal of mean/variance.
11. In case of Poisson Distribution Coefficient of skewness is square root of reciprocal of
mean/variance.
12. The First Moment of Poisson Distribution is = 0.
13. The Second and Third Moment of poisson Distribution is = m.
14. The Poisson Distribution can frequently be used to approximate Binomial Distribution
when -
(i) n, i.e. number of trials, is indefinitely large, i.e. n  
(ii) p, i.e. the probability of success for each trial is indefinitely small, i.e. p  0, and
(iii) np = m (say) is finite.
Note : In practice, the Poisson distribution may be used in place of the binomial distribution,
where n > 20 and P < 0.1.

PROBABILITY DISTRIBUTION 381


LIST OF FORMULAE
em mr
1. Probability Function P(r) =
r!
Where, r = 0, 1, 2,....e=2.7183
m = mean i.e. np or the average num-
ber of occurrences of an event.

em m0
Probability of 0 event P(r = 0) = = e-m
0!

e m m1
Probability of 1 event P(r = 1) =
1!

em m2
Probability of 2 events P(r = 2) =
2!

e  m m3
Probability of 3 events P(r = 3) =
3!
Probability of at least 1 event P(r = 1 or more) = 1 – P(r = 0)
Probability of at least 2 events P(r = 2 or more) = 1 – [P(r = 0) + P(r = 1)]
Probability of at the most 2 events P(r = 0, 1 or 2) = P(r = 0) + P(r = 1) + P(r = 2)
Probability of less than 2 events P (r = 0 or 1) = P(r = 0) + P(r = 1)
Probability of more than 2 events P (r = 3 or more) = 1 – [P (r = 0) + P(r = 1) + P
(r = 2)]

em mr
2. Expected Frequency Function N.P(r) = N.
r!
Thus expected frequency = Probability of an
event × Total No. of Observations (N).
Note : Fitting a Poisson distribution means find
ing out the theoretical expected frequencies.
3. Mean = m = np
4. Variance =m

382 BUSINESS MATHEMATICS, LOGICAL REASONING & STATISTICS (Paper 3) [CA Foundation]
THEORETICAL DISTRIBUTIONS - NORMAL DISTRIBUTION

1. Normal Distribution (t-distribution,  - distribution, F-distribution) is continuous distribution.


2. Nomral Distribution is bell shaped and unimodal.
3. In case of Normal Distribution Mean = Median = Mode.
4. In case of Normal Distribution Skewness = 0
5. In case of Normal Distribution Mean Deviation = 4/5 s.d. and Quartile Devation = 2/3s.d.
6. In case of Normal Distribution Med. – Q1 = Q3 – Med.
7. In case of Normal Distribution Highest Value = Mean + Half of Range &
Lowest Value = Mean – Half of Range
8. In case of Normal Distribution Mean ± 3 s.d. covers 99.73% of items
9. Normal Distribution with X = 0 and s.d. = 1 is known as Standard Normal Distribution.
10. The height of normal curve is maximum at the Mean value.

LIST OF FORMULA
1. Mean Mean = 

2. Variance Variance = s.d.2 =  2

2
1 1 x 
3. Probability Function P(x) = e   (–  < x <  )
 2 2  

(or Density Function) where,  = Mean of the normal random variable X.


s.d.= Standard deviation of the given
normal dis tribution [Two parameters]

2 = 2.5066
e = 2.7183 [Two constants]

PROBABILITY DISTRIBUTION 383


1
1  ( z )2
2 X 
4. Density Function in Terms of P(z) = e Where, z =
 2 
Standard Normal Variable Mean = 0
Standard deviation = 1
COMPARATIVE STUDY OF THEORETICAL DISTRIBUTIONS

Basis of Comparison Binomial Distribution Poisson Distribution Normal Distribution


1. Type of Distribution Discrete Probability Discrete Probability Continuous Distribution
Distribution Probability Distribution

2. Parameters Biparametric n,p Uniparametric m , 2


3. Restriction on 0<p<1 m>0   = 0 for std. normal
Parameters  2 = 1 distribution
4. Mean ( X ) X = np X =m X =
5. Variance (  2 )  2 = npq 2 =m 2
2
1  x  
em mr 1  
 
6. Probability Function P(r) = nCrprqn-r P(r) = P(x) = e 2
r!  2
(or Density Function) Where, (–  < x <  )
r = 0, 1, 2.......... where,   = Mean

m = Mean e = 2.7183 (the base of


natural logarithms)
s.d.= Standard Deviation

em mr
7. Expected Frequency N.P. (r) N.P. (r) = N.
r!
Distribution = N. nCr pr qn-r
1 2
1  z
2
8. Density Function in P(z) = e
 2
X 
terms of Standard where, z =

Normal Variate Mean = 0
Standard deviation = 1
9. Skewness/Shape p = 0.5 Normal Positively Skewed to Skewnes = 0
P < 0.5 Skewed to right right Bell-Shaped

384 BUSINESS MATHEMATICS, LOGICAL REASONING & STATISTICS (Paper 3) [CA Foundation]
P> 0.5 Skewed to left
10. Mode Unimodal or bimodal Unimodal
11. Relationship - - Mean = Mode = Median
between Mean,
Mode and Median
Basis of Comparison Binomial Distribution Poisson Distribution Normal Distribution
11. Conditions 1. Discrete Variable 1. Discrete Variable
Under which used 2. Independent Trials 2. Independent Trials
3. Finite and Fixed 3. Large No. of Trials
No. of Trials 4. Small p.
4. Only Two Outcomes 5. Finite Mean
(Success/Failure)
5. Fixed Prob. for each
Trial

---0---0---

PROBABILITY DISTRIBUTION 385


9. Variance of Binomial distribution is
(a) npq (b) np
CLASS WORK (c) both (d) none
10. When p = 0.1 the binomial distribution is
1. When a coin is tossed 10 times then we use skewed to the
(a) Normal Distribution (a) left (b) right
(b) Poisson Distribution (c) both (d) none
(c) Binomial Distribution 11. If in Binomial distribution np = 9 and npq = 2.
(d) None 25 then q is equal to

2. In Binomial Distribution ‘n‘ means (a) 0.25 (b) 0.75

(a) Number of trials of the experiment (c) 1 (d) none

(b) the probability of getting success 12. In Binomial Distribution

(c) Number of success (a) mean is greater than variance

(d) none (b) mean is less than variance

3. Binomial probability Distribution is a (c) mean is equal to variance

(a) Continuous (b) discrete (d) none

(c) both (d) none 13. Standard deviation of binomial distribution is


4. When there are a fixed number of repeated (a) (npq)2 (b) npq
trial of any experiments under identical
conditions for which only one of two mutually (c) (np)2 (d) np
exclusive outcomes, success or failure can
result in each trial then, we use 14. _________ distribution is a limiting case of
Binomial distribution
(a) Normal Distribution
(a) Normal (b) Poisson
(b) Binomial Distribution
(c) Both (d) none
(c) Poisson Distribution
15. When the number of trials is large and
(d) None probability of success is small then we use the
5. In Binomial Distribution ‘p’ denotes Probability distribution
of (a) Normal (b) Poisson
(a) Success (b) Failure (c) Binomial (d) none
(c) Both (d) None 16. In Poisson Distribution, probability of success
6. When p = 0.5 5, the binomial distribution is is very close to
(a) asymmetrical (b) symmetrical (a) 1 (b) – 1
(c) Both (d) None (c) 0 (d) none
7. When ‘p’ is larger than 0. 5, the binomial 17. In Poisson Distribution np is
distribution is (a) finite (b) infinite
(a) asymmetrical (b) symmetrical (c) 0 (d) none
(c) Both (d) None 18. In ________________ distribution, mean =
variance
8. Mean of Binomial distribution is
(a) Normal (b) Binomial
(a) npq (b) np
(c) Poisson (d) none
(c) both (d) none

386 BUSINESS MATHEMATICS, LOGICAL REASONING & STATISTICS (Paper 3) [CA Foundation]
19. In Poisson distribution mean is equal to (a) Poisson (b) Binomial
(c) Normal (d) none
(a)   29. The curve of _________ distribution is
(b) np unimodal and bell shaped with the highest
point over the mean
(c) square root mp
(a) Poisson (b) Normal
(d) square root mpq
(c) Binomial (d) none
20. In Binomial distribution standard deviation is 30. Because of the symmetry of Normal
equal to distribution the median and the mode have
(a) np (b) (np)2 the ______ value as that of the mean
(a) greater (b) smaller
(c) npq (d) (npq)2 (c) same (d) none
21. For continuous events _________________ 31. For a Normal distribution, the total area under
distribution is used. the normal curve is
(a) Normal (b) Poisson (a) 0 (b) 1
(c) Binomial (d) none (c) 2 (d) –1
22. Probability density function is associated with 32. In Normal distribution the probability has the
(a) discrete random variable maximum value at the
(b) continuous random variables (a) mode (b) mean
(c) both (c) median (d) none
(d) none 33. In Normal distribution the probability
23. Probability density function is always decreases gradually on either side of the mean
but never touches the axis.
(a) greater than 0
(a) True (b) false
(b) greater than equal to 0
(c) both (d) none
(c) less than 0
34. Whatever may be the parameter of
(d) less than equal to 0
__________ distribution, it has same shape.
24. For continuous random variables probability
(a) Normal (b) Binomial
of the entire space is
(c) Poisson (d) none
(a) 0 (b) –1
35. In Standard Normal distribution
(c) 1 (d) none
(a) mean=1, S.D=0
25. For discrete random variables the probability
(b) mean=1, S.D=1
of the entire space is
(a) 0 (b) 1 (c) mean = 0, S.D = 1
(d) mean=0, S. D=0
(c) –1 (d) none
36. The number of methods for fitting the normal
26. Binomial distribution is symmetrical if
curve is
(a) p > q (b) p < q
(a) 1 (b) 2
(c) p = q (d) none
(c) 3 (d) 4
27. The Poisson distribution tends to be
37. ____________ distribution is symmetrical
symmetrical if the mean value is
around t = 0
(a) high (b) low
(a) Normal (b) Poisson
(c) zero (d) none
(c) Binomial (d) t
28. The curve of ____________ distribution has
single peak

PROBABILITY DISTRIBUTION 387


38. As the degree of freedom increases, the 47. The probability distribution of x is given
________ distribution approaches the Standard below:
Normal distribution value of x : 1 0
(a) t (b) Binomial Total
(c) Poisson (d) Normal probability : p 1–p
39. _________ distribution is asymptotic to the 1
horizontal axis. Mean is equal to
(a) Binomial (b) Normal (a) p (b) 1–p
(c) Poisson (d) t (c) 0 (d) 1
40. ________ distribution has a greater spread than 48. For n independent trials in Binomial
Normal distribution curve distribution the sum of the powers of p and q
(a) t (b) Binomial is always n , whatever be the no. of success.
(c) Poisson (d) none (a) True (b) false
41. In Binomial Distribution if n is infinitely large, (c) both (d) none
the probability p of occurrence of event’ is 49. In Binomial distribution parameters are
close to _______ and q is close to _________ (a) n and q (b) n and p
(a) 0 , 1 (b) 1 , 0 (c) p and q (d) none
(c) 1 , 1 (d) none 50. Poisson distribution is a ___________
probability distribution .
42. Poisson distribution approaches a Normal
distribution as n (a) discrete (b) continuous
(a) increase infinitely (c) both (d) none
(b) decrease 51. Number of radio- active atoms decaying in a
given interval of time is an example of
(c) increases moderately
(a) Binomial distribution
(d) none
(b) Normal distribution
43. If neither p nor q is very small but n sufficiently
large, the Binomial distribution is very closely (c) Poisson distribution
approximated by _________ distribution (d) None
(a) Poisson (b) Normal 52. __________ distribution is sometimes known
(c) t (d) none as the “distribution of rare events“.
44. For discrete random variable x, Expected value (a) Poisson (b) Normal
of x (i.e E(x)) is defined as the sum of products (c) Binomial (d) none
of the different values and the corresponding 53. The probability that x assumes a specified
probabilities. value in continuous probability distribution is
(a) True (b) false (a) 1 (b) 0
(c) both (d) none (c) –1 (d) none
45. For a probability distribution, —————— is 54. In Normal distribution mean, median and
the expected value of x. mode are
(a) median (b) mode (a) equal (b) not equal
(c) mean (d) none (c) zero (d) none
46. _________ is the expected value of (x – m)2 , 55. Probability function is known as
where m is the mean. (a) frequency function
(a) median (b) variance (b) continuous function
(c) standard deviation (c) discrete function
(d) mode (d) none

388 BUSINESS MATHEMATICS, LOGICAL REASONING & STATISTICS (Paper 3) [CA Foundation]
56. The number of points obtained in a single (a) 50 (b) 100
throw of an unbiased die follows : (c) 800 (d) 25
(a) Binomial distribution 65. Assuming that boys and girls are equally
(b) Poisson distribution probable, the number of families out of 1600
(c) Uniform distribution families each having 5 children, with 2 or 3 boys
are
(d) None
(a) 800 (b) 600
57. The number of points in a single throw of an
(c) 1000 (d) 100
unbiased die has frequency function
66. Assuming that boys and girls are equally
(a) f(x)=1/4 (b) f(x)= 1/5
probable, the number of families out of 1600
(c) f(x) = 1/6 (d) none families each having 5 children, with all
58. In uniform distribution random variable x children of the same sex are
assumes n values with (a) 800 (b) 600
(a) equal probability (c) 1000 (d) 100
(b) unequal probability 67. Five dice are thrown simultaneously for 96
(c) zero times. An even number is regarded as success,
the expected frequencies of all five successes
(d) none
are
59. Seven coins are tossed simultaneously, the
(a) 8 (b) 16
probability of five heads is
(c) 3 (d) 32
(a) 12/128 (b) 17/128
(c) 21/128 (d) 11/128 68 For Bernoulli distribution with probability p
of a success and q of a failure, the relation
60. Seven coins are tossed simultaneously the
between mean and variance that holds is :
probability of at least five heads is
(a) 12/128 (b) 17/128 (a) mean > variance
(c) 21/128 (d) 29/128 (b) mean < variance
61. The probability that a person hits a target is 1/ (c) mean = variance
3, the probability that he will hit the target in 3 (d) mean < variance
times out of 5 trials is 69. The outcomes of an experiment classified as
(a) 12/243 (b) 17/243 success A or failure A will follow a Bernoulli
(c) 40/243 (d) 60/243 distribution if:
62. The probability of a smoker from a group of (a) P (A) = 1/2 (b) P (A) = 0
persons is 2/3. Five persons are selected at (c) P (A) = 1
random from the group, the probability that (d) P (A) remains constant in all trials
at least 4 of them are smokers is
70. The mean and variance of a binomial
(a) 112 / 243 (b) 117/243 distribution are 8 and 4, respectively. Then, P
(c) 221/243 (d) 32/243 (X = 1) is equal to :
63. Assuming that boys and girls are equally (a) 1/210 (b) 1/212
probable, the number of families out of 1600
(c) 1/216 (d) 1/26
families each having 5 children, with 3 boys
are 71. If for a binomial distribution, b(n, p), n = 4 and
(a) 800 (b) 500 also P (X=2) = 3P (X=3), the value of p is :
(c) 200 (d) 100 (a) 9/11 (b) 1
64. Assuming that boys and girls are equally (c) 1/4 (d) 1/3
probable, the number of families out of 1600 72. A and B play a game in which the probability of
families each having 5 children, with 5 girls winning of A is 2/3, the probability that A will
are win at least 6 times out of 8 trials is

PROBABILITY DISTRIBUTION 389


(a) 512/ 2187 (b) 64/2187 81. A six faced die is so constructed that the
(c) 32/2187 (d) 1024/2187 probability of getting an even number is twice
73. The mean of Binomial distribution is 4 and its the probability of getting an odd number, the
variance is 2.4, the value of p is probability that out of 5 trials all the five will
give even numbers is
(a) 0.4 (b) 0.5
(a) 16/243 (b) 16/81
(c) 0.3 (d) 0.2
(c) 64/243 (d) 32/243
74. The mean of Binomial distribution is 4 and its
82. If for a binomial distribution b (n, p), mean = 4,
variance is 2.4, the value of n is
variance = 4/3, the probability, P (X > 5) is equal
(a) 4 (b) 5 to
(c) 8 (d) 10 (a) (2/3)6 (b) (1/3)5 (2/3)
75. If 9 ships out of 10 ships safely reach the (c) (1/3) 6
(d) 35 (2/3)6
destination, the mean of number of ships 83. An experiment succeeds twice as often as it
reaching safely to the destination out of 400 fails. The chance that in the next six trials, there
ships is shall be at least four successes is :
(a) 400 (b) 50 (a) 243/729 (b) 498/729
(c) 360 (d) 60 (c) 496/729 (d) 329/729
76. If 9 ships out of 10 ships safely reach the 84. A six faced die is so constructed that the
destination, the standard deviation of number probability of getting an even number is twice
of ships reaching safely to the destination out the probability of getting an odd number, the
of 400 ships is probability that out of 5 trials 2 or 3 trials will
(a) 6 (b) 5 give even numbers is
(c) 3 (d) 2 (a) 40/243 (b) 40/18
77. The probability of a defective screw in a (c) 40/81 (d) 80/243
manufacturing process is 1/10. The mean of 85 For a Binomial distribution, mean = 20 and S.D.
defective screws in a sample of 400 screws is = 2 then the value of q is
(a) 30 (b) 50 (a) 0.4 (b) 0.2
(c) 40 (d) 360 (c) 0.3 (d) 0.5
86. For a Binomial distribution, mean = 20 and S.D.
78. The probability of a defective screw in a
= 2 then parameter n is
manufacturing process is 1/10. The variance of
defective screws in a sample of 400 screws. (a) 20 (b) 24
(c) 30 (d) 25
(a) 4 (b) 16
87. For a Binomial variate n = 6 and P(3) : P(4) = 8 :
(c) 18 (d) 36
3, the value of p is
79. It is given that on an average it rains for 10 (a) 0.4 (b) 0.5
days out of 30 days. The probability that it will
(c) 0.33 (d) 0.2
rain for at least 3 days in a week is
88 In a Binomial distribution with n = 5, P(x = 2) =
(a) 219/729 (b) 313/729
0.2048 and P(x = 3) = 0.0512, the value of p is
(c) 330/729 (d) 335/729 (a) 0.25 (b) 0.5
80. Assuming that half of the population is (c) 0.35 (d) 0.2
vegetarian, estimate how many investigators 89. For a Binomial distribution with n = 6, 9P(x = 4)
out of 100 will report that 3 or less are = P(x = 2), find the value of q is
vegetarians if each takes a sample of 10 (a) 0.4 (b) 0.5
individuals.
(c) 0.75 (d) 0.25
(a) 15 (b) 50
90. The probability of success p for a binomial
(c) 17 (d) 12 distribution if n = 6 and 4 P (x = 4) = P (x = 2) is

390 BUSINESS MATHEMATICS, LOGICAL REASONING & STATISTICS (Paper 3) [CA Foundation]
(a) 1/3 (b) 2/3 switches, there are at most two defective is :
(c) 0.5 (d) 0.25 (a) 2 e–1 (b) e–1
91. A and B play 12 games of chess of which 6 are (c) 2.5e –1
(d) 3e–1
won by A, 4 by B and 2 end in a tie. They agree 99. A person has some cars, and the average
to play 3 more games, the probability that A demand of cars per day is 3, the probability
wins all the three games is that on any day not more than 2 cars are in use
(a) 7/8 (b) 5/8 is (e–3 = 0.0498)
(c) 6/8 (d) 1/8 (a) 0.5533 (b) 0.4233
92. A and B play 12 games of chess of which 6 are (c) 0.7533 (d) 0.2533
won by A, 4 by B and 2 end in a tie. They agree
100. The probability that a match stick is found
to play 3 more games, the probability that two
without head is 1/100. Each match box contains
games end in a tie is
50 sticks. Using Poisson distribution we can say
(a) 5/36 (b) 5/72 that the percentage of number of boxes having
(c) 5/144 (d) 5/60 0 sticks without head is (e–0.5 = 0.61)
93. A and B play 12 games of chess of which 6 are (a) 40 (b) 51
won by A, 4 by B and 2 end in a tie. They agree
to play 3 more games, the probability that A (c) 61 (d) 25
and B win alternatively is 101. The probability that a match stick is found
(a) 5/36 (b) 5/72 without head is 1/100. Each match box contains
50 sticks. Using Poisson distribution we can say
(c) 5/144 (d) 5/60
that the percentage of number of boxes having
94. A and B play 12 games of chess of which 6 are 1 stick without head is
won by A, 4 by B and 2 end in a tie. They agree
(a) 40.5 (b) 50.5
to play 3 more games, the probability that B
wins at least one game is (c) 75.5 (d) 30.5
(a) 19/27 (b) 9/27 102. The probability that a match stick is found
without head is 1/100. Each match box cotains
(c) 9/18 (d) 5/27
50 sticks. Using Poisson distribution we can say
95. What is probability of getting exactly 2 heads that the percentage of number of boxes having
in 7 tosses of a fair coin? 2 sticks without heads is
(a) 12/128 (b) 16/128 (a) 8.5 (b) 7.63
(c) 21/128 (d) 15/128 (c) 8.75 (d) 10.25
96. X is a binomial variate with parameters n and 103. In the production of electric fuses 2% are
p. If n = 1, the distribution of X reduces to: defective, the probability of getting at the most
(a) Poisson distribution 2 defective fuses in a box containing 200 fuses
(b) Bernoulli distribution is (e–4 = 0.0183)
(c) binomial distribution itself (a) 0.238 (b) 0.763
(d) Normal distribution (c) 0.875 (d) 0.252
97. A distributions in which mean is equal to 104. In the production of electric fuses 2% are
variance is : defective, the probability of getting 3
(a) Poisson distribution defective fuses in a box containing 200 fuses.
(b) gamma distribution is (e–4 = 0.0183)
(c) normal distribution (a) 0.2952 (b) 0.1952
(d) binomial distribution (c) 0.3952 (d) 0.4952
98. A manufacturer produces switches and 105. In the manufacturing of cotter pins it is known
experiences that 2 per cent switches are that 5% of the pins are defective. The pins are
defective. The probability that in a box of 50 sold in boxes of 100 and it is guaranteed that

PROBABILITY DISTRIBUTION 391


not more than 4 pins will be defective in a box. (a) 2 (b) 3
What is the probability that a box will meet (c) 1 (d) none
this guarantee? (e–5 = 0.0067) 114. x is a Poisson variate such that P(x = 3) = P(x =
(a) 0.4480 (b) 0.5480 4), its S.D. is
(c) 0.6480 (d) 0.4380 (a) 4 (b) 3
106. If 3% of electric bulbs manufactured by a (c) 2 (d) none
company are defective, find the probability 115. If for a Poisson variate x, P (x = 0) = P(x = 1) = k,
that in a sample of 100 bulbs, exactly 5 bulbs then k =
are defective. (e–3 = 0.0498) (a) 0.3681 (b) 2.7183
(a) 0.2952 (b) 0.1008 (c) 0.5 (d) none
(c) 0.2008 (d) 0.3008 116. x is a Poisson variate and P(x = 1) = P(x = 2), find
107. In one hospital 3 percent of the patients P(x = 0) is
demand special rooms. On a particular day 3 (a) e–2 (b) e–0.5
special rooms were vacant. If 50 patients were (c) e–1 (d) none
admitted in the hospital on that day, the 117. x is a Poisson variate and P(x = 2) = 9.P(x = 4) +
probability that the demands for special room 90.P(x = 6) then its mean is
were not met is (e–1.5 = 0.2231) (a) 4 (b) 3
(a) 0.0658 (b) 0.1952 (c) 1 (d) none
(c) 0.1304 (d) 0.1316 118. The number of accidents in a year attributed
108. In a book, on an average there are 3 misprints to taxi drivers in a city follows poisson
in 5 pages. The number of pages having more distribution with mean 3. Out of 1000 taxi
than 2 misprints in that book of 100 pages are drivers, the number of drivers with no accident
(Use Poisson distribution) (e–0.6 = 0.5488) in a year is (e–3 = 0.0498)
(a) 3.31 (b) 4.31 (a) 40 (b) 30
(c) 2.31 (d) 5.31 (c) 50 (d) none
109. A factory produces 0.5% defective articles. If a 119. The number of accidents in a year attributed
sample of 100 articles is taken from the to taxi drivers in a city follows poisson
production, the probability of getting 2 or more distribution with mean 3. Out of 1000 taxi
defective articles is (e–0.5 = 0.6065) drivers, the number of drivers with at least 3
(a) 0.0902 (b) 0.1902 accidents in a year is (e–3 = 0.0498)
(c) 0.1302 (d) 0.1102 (a) 477 (b) 377
110. A random variable x follows Poisson (c) 177 (d) none
distribution with mean 2 then P( X > 0) is equal 120. The average number of customers, who
to (e–2 = 0.1353) appear at a counter of a certain bank per
(a) 0.1353 (b) 0.2706 minute is 2, the probability that during a given
(c) 0.8647 (d) none minute three or more customers appear is (e–
111. The mean of a Poisson variate is 0.81, then its 2
= 0.1353)
S.D. is (a) 0.32354 (b) 0.1353
(a) 0.81 (b) 0.9 (c) 1 (d) none
(c) 0.8647 (d) none
121. To which probability distribution mean is
112. x is a Poisson variate such that P(x = 3) = P(x = greater than variance .
4), mean =
(a) Poission (b) Binomial
(a) 2 (b) 3
(c) normal (d) none of them
(c) 4 (d) none
113. For a Poisson variate x its P(x = 1) = P(x = 2), 122. An approximate relation between Q.D. and S.
variance is D. of normal distribution is :

392 BUSINESS MATHEMATICS, LOGICAL REASONING & STATISTICS (Paper 3) [CA Foundation]
(a) 3 Q.D. = 2 S. D. (b) 4 Q.D. = 5 S. D. (a) 0.9772 (b) 0.2857
(c) 2 Q.D = 3 S. D. (d) 5 Q. D. = 4 S. D. (c) 0.0228 (d) 0.9332
123. A approximate relation between M. D. about 132. The mean of a normal variate is 20 and its
mean and S.D. of a normal distribution is : variance is 9. The area for the values of the
(a) 4 M.D. = 5 S. D. (b) 5 M.D. = 4 S. D. variate less than 24.5 is
(c) 3 M.D. = 3 S.D. (d) 3 M.D. = 2 S. D. (a) 0.6247 (b) 0.2857
(c) 0.0228 (d) 0.9332
124. The area under the standard normal curve
beyond the lines ± 1.96 is 133. In an intelligence test administered to 1000
children the average score is 42 and its s.d. is
(a) 95% (b) 90%
24. Assuming that the scores are normally
(c) 99.73% (d) 5% distributed, the number of children exceeding
125. If Z is a standard normal variate, the proportion score 50 are
of items lying between Z = – 0.5 and Z = – 3.0 is (a) 371 (b) 383
(a) 0.5 (b) 0.1915 (c) 72.7 (d) none
(c) 0.3172 (d) 0.3072 134. In an intelligence test administered to 1000
126. If X is a normal variate representing the income children the average score is 42 and its s.d. is
in ` per day with mean = 50 and S. D. = 10 of the 24. Assuming that the scores are normally
workers in a factory. The total number of distributed, the number of children getting
workers is 1000. The number of workers having score between 30 and 54 is
income more than ` 62.00 per day is (a) 371 (b) 383
(a) 246 (b) 150 (c) 72.7 (d) none
(c) 738 (d) 115 135 . In an intelligence test administered to 1000
127. Assume that distribution of diameters of shafts children the average score is 42 and its s.d. is
as normal with mean = 5 and S. D. = 0.05. The 24. Assuming that the scores are normally
tolerance limit of shafts is 4.90 to 5.10 cms. In a distributed, the minimum score of the most
consignment of 200 shafts, the number of intelligent 100 students is
shafts out of tolerance limits is : (a) 371 (b) 383
(a) 15 (b) 9 (c) 72.7 (d) none
(c) 20 (d) 25 136. The customer accounts at a certain
128. The mean of a normal variate is 20 and its departmental store have an average balance
variance is 9. The area between the values of of ` 120 and a standard deviation of ` 40.
the variate 15.5 and 21.5 is Assuming that account balances are normally
(a) 0.6247 (b) 0.2857 distributed, what percentage of the accounts
(c) 0.0228 (d) 0.9332 are over ` 150 ?
129. The mean of a normal variate is 20 and its (a) 25.5 (b) 22.66
variance is 9. The area between the values of (c) 72.7 (d) 46.49
the variate 14 and 18.5 is 137. The customer accounts at a certain
(a) 0.6247 (b) 0.2857 departmental store have an average balance
(c) 0.0228 (d) 0.9332 of ` 120 and a standard deviation of ` 40.
130. The mean of a normal variate is 20 and its Assuming that account balances are normally
variance is 9. The area for the values of variate distributed, what percentage of the accounts
more than 26 is are between ` 100 and ` 150 ?
(a) 0.6247 (b) 0.2857 (a) 25.5 (b) 22.66
(c) 0.0228 (d) 0.9332 (c) 72.7 (d) 46.49
131 . The mean of a normal variate is 20 and its 138. The average life of a battery is 400 hours and
variance is 9. The area for the values of variate its standard deviation is 50 hours. Assuming
more than 14 is normal distribution, the percentage of

PROBABILITY DISTRIBUTION 393


batteries having life more than 350 hours is number of students getting more than 25
(a) 84.13 (b) 22.66 marks is 50. The number of students of that
(c) 72.7 (d) 46.49 class is
139. The average life of a battery is 400 hours and (a) 682 (b) 653
its standard deviation is 50 hours. Assuming (c) 315 (d) none
normal distribution, approximate after how 147. The mean and S.D. of a normal distribution are
many hours 25% of the batteries will still be 20.5 and 5. The median is
working ? (a) 20.5 (b) 5
(a) 284 (b) 422 (c) 3.33 (d) 4
(c) 372 (d) 434 148. The mean and S.D. of a normal distribution are
140. The average life of a battery is 400 hours and 20.5 and 5. The mode is
its standard deviation is 50 hours. Assuming (a) 20.5 (b) 5
normal distribution, the percentage of (c) 3.33 (d) 4
batteries with life time between 300 and 500 149. The mean and S.D. of a normal distribution are
hours is 20.5 and 5. The quartile deviation is
(a) 84.13 (b) 22.66 (a) 20.5 (b) 5
(c) 95.44 (d) 19.35 (c) 3.33 (d) 4
141. In a normal distribution 50% of the 150. The mean and S.D. of a normal distribution are
observations are less than 35 and 89% of the 20.5 and 5. The mean deviation is
observations are less than 63. The mean of the (a) 20.5 (b) 5
distribution is (c) 3.33 (d) 4
(a) 63 (b) 35 151. The quartiles of a normal distribution are
(c) 89 (d) none respectively 8.64 and 14.32, the mean of the
distribution is
142. In a normal distribution 50% of the
(a) 11.48 (b) 4.26
observations are less than 35 and 89% of the
(c) 3.41 (d) none
observations are less than 63. The standard
152. The quartiles of a normal distribution are
deviation of the distribution is
respectively 8.64 and 14.32, the median of the
(a) 63 (b) 35 distribution is
(c) 89 (d) none (a) 11.48 (b) 4.26
143. The mean and S.D. of a normal variate are 30 (c) 3.41 (d) none
and 5 respectively, the probability of 26 < x < 153. The quartiles of a normal distribution are
40 is respectively 8.64 and 14.32, the standard
(a) 0.0013 (b) 0.7653 deviation of the distribution is
(c) 0.6753 (d) none (a) 11.48 (b) 4.26
(c) 3.41 (d) none
144. The mean and S.D. of a normal variate are 30
and 5 respectively, the probability of x > 45 is 154. The quartiles of a normal distribution are
respectively 8.64 and 14.32, the mean deviation
(a) 0.6826 (b) 0.7653
of the distribution is
(c) 0.0013 (d) none
(a) 11.48 (b) 4.26
145. The mean and S.D. of a normal variate are 30
and 5 respectively, the probability of (c) 3.41 (d) none
| x – 30 | < 5 is 155. What percentage of values will be within  ±
(a) 0.6826 (b) 0.0013 s when the characteristic follows a normal
(c) 0.7653 (d) none distribution?
146. The distribution of marks of the students in a (a) 95.45 (b) 68.26
class is normal with mean 20 and s.d. 5. The (c) 99.73 (d) none

394 BUSINESS MATHEMATICS, LOGICAL REASONING & STATISTICS (Paper 3) [CA Foundation]
156. What percentage of values will be within  ±
2s when the characteristic follows a normal
distribution?
HOME WORK-1
(a) 95.45 (b) 68.26
1. Probability distribution may be
(c) 99.73 (d) none
(a) discrete.
157. What percentage of values will be within m ±
3s when the characteristic follows a normal (b) continuous.
distribution? (c) infinite.
(a) 95.45 (b) 68.26 (d) both (a) and (b).
2. An important discrete probability distribution
(c) 99.73 (d) none
is
158. What percentage of values will be within m ±
(a) poisson distribution.
1.5s when the characteristic follows a normal
distribution? (b) Normal distribution.
(c) Cauchy distribution.
(a) 95.45 (b) 68.26
(d) Log normal distribution.
(c) 99.73 (d) none
3. An important continuous probability
159. In Binomial distribution if n = 4 and p = 1/3 distribution
then the value of variance is
(a) Binomial distribution.
(a) 8/3 (b) 8/9
(b) poisson distribution.
(c) 4/3 (d) none
(c) Geometric distribution.
160. In Binomial distribution if mean = 20, S.D.= 4
(d) Chi-square distribution.
then q is equal to
4. Parameter is a characteristic of
(a) 2/5 (b) 3/8
(a) population.
(c) 1/5 (d) 4/5
(b) sample.
161. If in a Binomial distribution mean = 20 , S.D.= 4
then p is equal to (c) probabiiity distribution.
(a) 2/5 (b) 3/5 (d) both (a) and (b)
(c) 1/5 (d) 4/5 5. An example of a parameter is
162. If is a Binomial distribution mean = 20 , S.D.= 4 (a) sample mean.
then n is equal to (b) population mean.
(a) 80 (b) 100 (c) binomial distribution.
(c) 90 (d) none (d) sample size.
6. The important characteristic(s) of Bernoulh
trials
(a) each trial is associated with just
(b) trials are independent.
two possible outcomes
(c) trials are infinite.
(d) both (a) and (b).
7. The probability mass function of binomial
distribution is given by
(a) f(x) = p x q n-x.
(b) f(x) = nCx px q n-x
(c) f(x) = ncx qx p n-x.
(d) f(x) = nCx pn-x q x
PROBABILITY DISTRIBUTION 395
8. If x is a binomial variable with parameters nand (a) n/2. (b) n/4.
p, then x can assume (c) np (1 - p). (d) 2n.
(a) any value between 0 and n. 17. Which one is uniparametric distribution?
(b) any value between 0 and n, both (a) Binomial.
inclusive. (b) poisson.
(c) any whole number between 0 and n, both (c) Normal.
inclusive.
(d) Hyper geometric.
(d) any number between 0 and infinity.
18. For a Poisson distribution,
9. A binomial distribution is
(a) mean and standard deviation are equal.
(a) never symmetrical.
(b) mean and variance are equal.
(b) never positively skewed. (c) standard deviation and variance are
(c) never negatively skewed. equal.
(d) symmetrical when p = 0.5. (d) both (a) and (b)
10. The mean of a binomial distribution with 19. poisson distribution may be
parameter nand p is (a) unimodal. (b) bimodal.
(a) n (1- p). (b) np (1 - p). (c) Multi-modal. (d) (a) or (b).
(c) np. (d) np (1  p). 20. poisson distribution is
(a) always symmetric.
11. The variance of a binomial distribution with
(b) always positively skewed.
parameters n and p is
(c) always negatively skewed.
(a) np2 (1 - p). (b) np (1  p). (d) symmetric only when m = 2.
(c) nq (1 - q). (d) n p (1- p)2.
2 2
21. A binomial distribution with parameters n and
12. An example of a bi-parametric discrete p can be approximated by a poisson
probability distribution is distribution with parameter m = np is
(a) binomial distribution. (a) n  
(b) poisson distribution.
(b) p  o
(c) normal distribution.
(d) both (a) and (b). (c) n   and p  o.
13. For a binomial distribution, mean and mode (d) n   and p  o x and np remains finite.
(a) are never, equal.
22. Poisson fitting to an observed frequency
(b) are always equal. distribution,
(c) are equal when q = 0.50. (a) we equate the Poisson parameter to the
(d) do not always exist. mean of the frequency distribution.
14. The mean of binomial distribution is (b) we equate the Poisson parameter to the
(a) always more than its variance. median of the distribution.
(b) always equal to its variance. (c) we equate the Poisson parameter to the
(e) always less than its variance. mode of the distribution.
(d) always equal to its standard deviation. (d) none of these.
15. For a binomial distribution, there may be 23. The most important continuous probability
(a) one mode. (b) two mode. distribution is known as
(c) (a). (d) (a) or (b). (a) Binomial distribution.
16. The maximum value of the variance of a (b) Normal distribution.
binomial distribution with parameters nand p (c) Chi-square distribution.
is
(d) sampling distribution.

396 BUSINESS MATHEMATICS, LOGICAL REASONING & STATISTICS (Paper 3) [CA Foundation]
24. The probability density function of a normal 29. The mean and mode of a normal distribution
variable x is given by (a) may be equal.
(b) may be different.
2 (c) are always equal.
1  x   
1 
2    (d) (a) or (b)
(a) f ( x )  .e 30. The mean deviation about median of a
 2
standard normal variate is
for    x   (a) 0.675  (b) 0.675.
(c) 0.80  (d) 0.80.
2
1  x   31. The quartile deviation of a normal distribution
(b) f ( x )  e with mean 10 and SD 4 is
 2 2 2 (a) 0.675 (b) 67,50
(c) 2.70 (d) 3.20
32. For a standard normal distribution, the points
for 0  x   of inflexion are given by
2 (a)    and    (b)   and 
  x  2 
  (c) -1 and 1. (d) 0 and 1.
1  2 2



(c ) f ( x )  e
 2  33. The symbol  (a) indicates the area of the
standard normal curve between
(a) 0 to a. (b) a to 00.
(c) -  to a. (d) -  to  .
for    x  
34. The interval (  3,   3) covers
(d) none of these.
25. The total area of the normal curve is (a) 95% area of a normal distribution.
(b) 96% area of a normal distribution.
(a) one.
(c) 99% area of a normal distribution.
(b) 50 per cent.
(d) all but 0.27% area of a normal distribution.
(c) 0.50.
35. Number of misprints per page of a thick book
(d) any value between 0 and 1. follows
26. The normal curve is (a) Normal distribution.
(a) Bell-shaped. (b) poisson distribution.
(b) U- shaped (c) Binomial distribution.
(c) J- shaped. (d) Standard normal distribution.
(d) Inverted J - shaped. 36. The result of ODI matches between India and
27. The normal curve is Pakistan follows
(a) positively skewed. (a) Binomial distribution.
(b) negatively skewed. (b) poisson distribution.
(c) Symmetrical. (c) Normal distribution.
(d) (b) or (c).
(d) all these.
37. The wage of workers of a factory follow
28. Area of the normal curve is
(a) Binomial distribution.
(a) between -  to  is 0.50.
(b) poisson distribution.
(b) between  to  is 0.50. (c) Normal distribution.
(c) between -  < to  is 0.50. (d) Chi-square distribution.
(d) both (a) and (b).

PROBABILITY DISTRIBUTION 397


38. Which is the standard deviation of the number 48. If X ~ P (m) and its coefficient of variation is 50,
of recoveries among 48 patients when the what is the probability that X would assume
probability of recovering is 0.75? only non-zero vall les?
(a) 36 (b) 81 (a) 0.018 (b) 0.982
(c) 9 (d) 3 (c) 0.989 (d) 0.976
39. X is a binomial variable with n = 20. What is the 49. If 1.5 per cent of items produced by a
mean of X if it is known that x is symmetric? manufacturing units are known to be
(a) 5 (b) 10 defective, what is the probability that a sample
(c) 2 (d) 8 of 200 items would contain no defective item?
40. If X B (n, p), what would be the maximum (a) 0.05 (b) 0.15
value of the variance of x when n = 16? (c) 0.20 (d) 0.22
(a) 2 (b) 4 50. For a Poisson variate X, P (X = 1) = P (X = 2).
What is the mean of X?
(c) 8 (d) 5 (a) 1.00 (b) 1.50
41. If x is a binomial variate vvith parameter 15 (c) 2.00 (d) 2.50
and 1/3, what is the value of mode of the 51. If 1 per cent of an airline’s flights suffer a minor
distribution equipment failure in an aircraft, what is the
(a) 5 and 6 (b) 5 probability that there will be exactly two such
(c) 5.50 (d) 6 failures in the next 100 such flights? e-1=0.3679
42. What is the no. of trials of a binomial (a) 0.50 (b) 0.184
distribution having mean and SD as 3 and 1.5 (c) 0.265 (d) 0.256
respectively? 52. If for a Poisson variable X, f(2) = 3 f(4), what is
(a) 2 (b) 4 the variance of X?
(c) 8 (d) 12 (a) 2 (b) 4
43. What is the probability of getting 3 heads if 6
(c) 2 (d) 3
unbiased coins are tossed simultaneously?
(a) 0.50 (b) 0.25 53. What is the coefficient of variation of x,
(c) 0.3125 (d) 0.6875 characterised by the following probability
44. If the overall percentage of success in an exam density
is 60, what is the probability that out of a group ( x 10 )2
of 4 students, at least one has passed’? 1 
32
funcation : f (x)  e for -
(a) 0.6525 (b) 0.9744 4 2
(c) 0.8704 (d) 0.0256
 <x< 
45. What is the probability of making 3 correct
(a) 50 (b) 60
guesses in 5 True - False answer type
questions? (c) 40 (d) 30
(a) 0.3125 (b) 0.5676 54. What is the first quartile of X having the
(c) 0.6875 (d) 0.4325 following probability density function?
46. If the standard deviation of a Poisson variate X ( x 10 )2
1 
is 2, what is P (1.5 < X < 2.9)? e-4 = 0.0183 f(x)  e 72
for -  < x < 
(a) 0.231 (b) 0.158 72 
(c) 0.15 (d) 0.1464 (a) 4 (b) 5
47. If the mean of Poisson variable X is 1, what is P (c) 5.95 (d) 6.75
(X = at least one)?
55. If the two quartiles of N (  , 2 ) are 14.6 and
(a) 0.456 (b) 0.821
(c) 0.632 (d) 0.254. 25.4 respectively, what is the standard
deviation of the distribution?

398 BUSINESS MATHEMATICS, LOGICAL REASONING & STATISTICS (Paper 3) [CA Foundation]
(a) 9 (b) 6 64. Assuming that one-third of the population are
(c) 10 (d) 8 tea drinkers and each of 1000 enumerators
56. If the mean deviation of a normal variable is takes a sample of 8 individuals to find out
16, what is its quartile deviation? whether they are tea drinkers or not, how
many enumerators are expected to report that
(a) 10.00 (b) 13.50
five or more people are tea drinkers?
(c) 15.00 (d) 12.05
(a) 100 (b) 95
57. If the points of inflexion of a normal curve are
(c) 88 (d) 90
40 and 60 respectively, then its mean deviation
is 65. Out of 128 families with 4 children each, how
many are expecled to have at least one boy
(a) 8 (b) 45
and one girl?
(c) 50 (d) 60
(a) 100 (b) 105
58. If the quartile deviation of a normal curve is
(c) 108 (d) 112
4.05, then its mean deviation is
66. If a binomial distribution is fitted to the
(a) 5.26 (b) 6.24
following data:
(c) 4.24 (d) 4.80
x 0 1 2 3 4
59. If the 1st quartile and mean deviation about
f 16 25 32 17 10
median of a normal distribution are 13.25 and
8 respectively, then the mode of the then the sum of the expected freque:lcies for
distribution is x :::: 2, 3 and 4 would be
(a) 20 (b) 10 (a) 58 (b) 59
(c) 15 (d) 12 (c) 60 (d) 61
60. If the area of standard normal curve between 67. X is a Poisson variate satisfying the following
condition 9 P (X = 4) + 90 P (X = 6) = P (X = 2).
z = 0 to z = 1 is 0.3413, then the value of  (1) is
What is the value of P (X < I)?
(a) 0.5000 (b) 0.8413 (a) 0.5655 (b) 0.6559
(c) -0.5000 (d) 1 (c) 0.7358 (d) 0.8201
61. If X and Yare 2 independent normal variables 68. A random-variable x follows Poisson
with mean as 10 and 12 and SD as 3 and 4, then distribution and. its coefficient of variation is
(X+ Y) is normally distributed with 50 What is the value of P (x > 1 / x > 0)?
(a) mean = 22 and SD = 7. (a) 0.1876 (b) 0.2341
(b) mean = 22 and SD = 25. (c) 0.9254 (d) 0.8756
(c) mean = 22 and SD = 5. 69. A renowned hospital usually admits 200
(d) mean = 22 and SD = 49. patients every day. One per cent patients, on
62. If it is known that the probability of a missile an average. require special room facilities. On
hItting a target is 1/8, what is the probability one particular morning, it was found that only
that out of 10 missiles fired, at least 2 will hit one special room is available. What is the
the target? probability that more than 3 patients would
(a) 0.4258 (b) 0.3968 require special room facilities?
(c) 0.5238 (d) 0.3611 (a) 0.1428 (b) 0.1732
63. X is a binomial variable such that 2 P(X = 2) = (c) 0.2235 (d) 0.3450
P(X = 3) and mean of X is known to be 70. A car hire firm has 2 cars which is hired out
10/3. What would be the probability that X everyday. The number of demands per day for
assumes at most the value 2? a car follows Poisson distribution with mean
(a) 16/81 (b) 17/81 1.20. What is the proportion of days on which
some demand is refused? (Given e 1.20 = 3.32).
(c) 47/243 (d) 46/243

PROBABILITY DISTRIBUTION 399


(a) 0.25 (b) 0.3012 belonging to this group, at least one family
(c) 0.12 (d) 0.03 has weekly food expenditure in excess of `
71. If a Poisson distribution is fitted to the 2,100? Given  (1) = 0.84.
following data: (a) 0.418 (b) 0.582
Mistake per page 0 1 2 3 4 5 (c) 0.386 (d) 0.614
No. of pages 76 74 29 17 3 1 77. If the weekly wages of 5000 workers in a factory
Then the sum of the expected frequencies for follows normal distribution with mean and SD
x = 0, 1 and 2 is as ` 700 and ` 50 respectively, what is the
(a) 150 (b) 184 expected number of workers with wages
(c) 165 (d) 148 between ` 660 and ` 720?
72. The number of accidents in a year attributed (a) 2,050 (b) 2,200
to taxi drivers in a locality follows Poisson (c) 2,218 (d) 2,300
distribution with an average 2. Out of 500 taxi 78. 50 per cent of a certain product have weight 60
drivers of that area, what is the number of Kg or more whereas 10 per cent have weight
drivers with at least 3 accidents in a year? 55 Kg or less. On the assumption of normality,
(a) 162 (b) 180 what is the variance of weight?
(c) 201 (d) 190 Given  (1.28) = 0.90.
73. In a sample of 800 students, the mean weight (a) 15.21 (b) 9.00
and standard deviation of weight are found to (c) 16.00 (d) 22.68
be 50 Kg and 20 Kg respectively. On the 79. A discrete random variable x follows uniform
assumption of normality, what is the number distribution and takes only the values 6, 8, 11,
of students weighing between 46 Kg and 62 12, 17 The probability of P( x = 8) is
Kg? Given area of the standard normal curve (a) 1/5 (b) 3/5
between Z = 0 to Z = 0.20 = 0.0793 and area (c) 2/8 (d) 3/8
between Z = 0 to Z = 0.60 = 0.2257.
80. A discrete random variable x follows uniform
(a) 250 (b) 244 distribution and takes the values 6, 9, 10, 11,
(c) 240 (d) 260 13
74. The salary of workers of a factory is known to The probability of P( x = 12) is
follow normal distribution with an average (a) 1/5 (b) 3/5
salary of ` 10,000 and standard deviation of (c) 4/5 (d) 0
salary as ` 2,000. If 50 workers receive salary 81. A discrete random variable x follows uniform
more than ` 14,000, then the total no. of distribution and takes the values 6, 8, 11, 12,
workers in the factory is 17
(a) 2,193 (b) 2,000 The probability of P(x < 12) is
(c) 2,200 (d) 2,500 (a) 3/5 (b) 4/5
75. For a normal distribution with mean as 500 and (c) 1/5 (d) none
SD as 120, what is the value of k so that the 82. A discrete random variable x follows uniform
interval [500, k] covers 40.32 per cent area of distribution and takes the values 6, 8, 10, 12,
the normal curve? Given  (1.30) = 0.9032. 18
(a) 740 (b) 750 The probability of P( x < 12) is
(a) 1/5 (b) 4/5
(c) 656 (d) 800
(c) 3/5 (d) none
76. The average weekly food expenditure of a
group of families has a normal distribution
with mean ` 1,800 and standard deviation ` 300.
What is the probability that out of 5 families

400 BUSINESS MATHEMATICS, LOGICAL REASONING & STATISTICS (Paper 3) [CA Foundation]
83. A discrete random variable x follows uniform
distribution and takes the values 5, 7, 12, 15, ANSWER KEYS
18
The probability of P( x > 10) is
(a) 3/5 (b) 2/5 1 (d) 2 (a) 3 (d) 4 (a)
(c) 4/5 (d) none 5 (b) 6 (d) 7 (b) 8 (c)
84. The probability distribution whose frequency 9 (d) 10 (c) 11 (c) 12 (a)
function f(x)= 1/n( x = x1, x2, …, xn) is known as
13 (c) 14 (a) 15 (d) 16 (b)
(a) Binomial distribution
(b) Poisson distribution 17 (b) 18 (b) 19 (d) 20 (b)
(c) Uniform distribution 21 (d) 22 (a) 23 (b) 24 (a)
(d) Normal distribution 25 (a) 26 (a) 27 (c) 28 (d)
85. In uniform distribution random variable x
29 (c) 30 (d) 31 (c) 32 (c)
assumes n values with
(a) equal probability 33 (c) 34 (d) 35 (b) 36 (a)
(b) unequal probability 37 (c) 38 (d) 39 (b) 40 (b)
(c) zero 41 (b) 42 (d) 43 (c) 44 (b)
(d) none 45 (a) 46 (d) 47 (c) 48 (b)
86. In a discrete random variable x follows uniform
distribution and assumes only the values 8 , 9, 49 (a) 50 (c) 51 (b) 52 (a)
11, 15, 18, 20. Then P(x = 12) is 53 (c) 54 (c) 55 (d) 56 (b)
(a) 1/6 (b) 0 57 (a) 58 (d) 59 (a) 60 (b)
(c) 1/7 (d) none 61 (c) 62 (d) 63 (b) 64 (c)
87. In a discrete random variable x follows uniform
distribution and assumes only the values 8, 9, 65 (d) 66 (d) 67 (c) 68 (c)
11, 15, 18, 20. Then P(x < 15) is 69 (a) 70 (c) 71 (b) 72 (a)
(a) 1/2 (b) 2/3 73 (b) 74 (a) 75 (c) 76 (b)
(c) 1 (d) none 77 (c) 78 (a) 79 (a) 80 (d)
88. In a discrete random variable x follows uniform
distribution and assumes only the values 8 , 9, 81 (b) 82 (c) 83 (a) 84 (c)
11, 15, 18, 20. Then P (x < 15) is 85 (a) 86 (b) 87 (a) 88 (a)
(a) 2/3 (b) 1/3 89 (c)
(c) 1 (d) none
89. In a discrete random variable x follows uniform
distribution and assumes only the values 8, 9,
11, 15, 18, 20. Then P(|x – 14| < 5) is
(a) 1/3 (b) 2/3
(c) 1/2 (d) 1

PROBABILITY DISTRIBUTION 401


8. If x and y are independent normal variably with
HOME WORK-2 mean 100 and 80 respectively and Standard
deviation as 4 and 3 respectively. What is the
Standard deviation of (x+y) ?
1. The mean of binomial distribution is (a) (180, 5) (b) (180, 25)
(a) Always more than its variance (c) (100, 15) (d) None of them
(b) always equal to variance 9. The value of e is
(a) 2.7183 (b) 2.1786
(c) less than its variance
(c) 2.1643 (d) 0
(d) always equal to Standard deviation
2. If X is normal variate with mean 6 and variance 10. For a Poisson variate X, P(X=2) = 3P (X = 4),
16 then the value of the probability. then the standard deviation of X is
p (2  X  10) is equal to (a) 2 (b) 3
(a) 2P(2<X<10) (c) P(0<X<6) (c) 4 (d) 2
(b) 2P(6<X<10) (d) 3P(6<X<10)
1
3. In Binomial Distribution the trails are statistics 11. The mean of the Binomial distribution B(4, )
3
(a) dependent
(b) independent is equal to
(c) either independent or dependent 3 4
(a) (b)
(d) none of these 5 3
4. If p is increased for a fixed n; the Binomial 8 3
distribution shifts to the (c) (d)
3 4
(a) Right (b) left
12. If for a normal distribution Q1 = 54.52 and Q =
(c) Above (d) Below 78.86, then the median of the distribution is
5. In a normal distribution skewness is ___ (a) 12.17 (b) 66.69
(a) 0 (b) >3 (c) 39.43 (d) None
(c) <3 (d) <1 13. What is the mean of X having the following
6. For a random variable x; the probability density function?
deviatory function is given by: F(x)= for – 8 < x < 8
(a) 4 (b) 10
Find the mean and variance of its distribution. (c) 40 (d) None of the above
( x10)2
14. The probability that a student is not a swimmer
1 is 1/5, then the probability that out of five
f (x)  .e 32
for    x  
4 2 students four are swimmer is
4 4
1 1 4 1 1 4
(a)  =2; 2 = (b)  = 4; 2 = (a)     (b) 5
C1    
4 2  5   5 5  5
1 1 4
(c)  = ; 2 = (d) None 5
4 1
4 2 (c) C4     (d) None
 5   5
7. Find the points of inflexion of the normal
curve.
 ( x 10)2
ANSWER KEYS
1 32
f (x)  .e for    x   1 (a) 2 (b) 3 (b) 4 (a)
4 2
(a) 6 and 14 (b) 6 and 12 5 (a) 6 (b) 7 (a) 8 (a)
(c) 7 and 10 (d) 10 and 12 9 (a) 10. (d) 11. (b) 12. (b)
13. (b) 14. (c)
402 BUSINESS MATHEMATICS, LOGICAL REASONING & STATISTICS (Paper 3) [CA Foundation]
CHAPTER-19
INDEX NUMBER AND TIME SERIES

UNIT 1 : INDEX NUMBER

INTRODUCTION
1. Theoretically the best average in the construction of index numbers is Geometric Mean.
2. Fisher’s Index is known as an ideal index because it satisfies Time Reversal Test and Factor
Reversal Test.
3. Fisher’s Index is the Geometric Mean of Laspeyre’s Index and Paasche’s Index.
4. Dorbish Bowley Index Number is the Arithmetic Mean of Laspeyre’s and Paasche’s Index.
5. Index number based on arithmetic mean is higher than the index number based on geomet-
ric mean.
6. Time Reversal Test is satisfied when P × P = 1
01 10
P1q1
7. Factor Reversal Test is satisfied when P ×Q =
01 01 P0 q0
8. Circular Test is satisfied when P × P × P = 1
01 12 20
9. Circular Test is not met by any of weighted aggregative with changing weights.
10. Circular Test is met by simple geometric mean of price relatives and weighted aggregative
with fixed weights.
11. Time Reversal Test is the test which requires that the product of price index for year 1 on the
base year 0 and price index for year 0 on the base year 1 should be equal to 1.
12. Factor Reversal Test is the test which requires that the product of a price index and the
quantity index should be equal to the corresponding value index.
13. Chain Base Index is the index method which permits the introduction of new item and dele-
tion of old items without necessitating the recalculation of entire series and in which the
deletion of old items without necessitating the recalculation of entire series and in which the
base changes from year to year.
14. Base Shifting is The technique of changing the old base period to new base period.
15. Splicing is the technique of linking two or more index number series.
16. Deflating is the technique to eliminate effect of changing price levels.
17. Consumer Price Index which measures how much the consumers of a particular class have
to pay more or less for a certain basket of goods and services in a given period.

INDEX NUMBER AND TIME SERIES 403


LIST OF FORMULAE
1. Unweighted Index Numbers

P1
(a) Simple Aggregative P01=
P0 × 100

(b) Simple Average of Relatives

P 
  1 100 
(i) When Arithmetic Mean is used P01 =  P0 
N

  P1 
  log  100  
  P0 
(ii) When Geometric Mean is used P01 = Antilog  N 
 
 

2. WEIGHTED INDEX NUMBERS


(a) Weighted Aggregative Indices

P1q0
(i) Laspeyres Method P01 =
P0 q0 × 100

P1q1
(ii) Paasche Method P01 =
P0 q1 × 100

p1q0 p1q1
LP 
(iii) Dorbish & Bowley’s Method P01 = = P0 q0 p0 q1 100
2 2

p1q0 p1q1
(iv) Fisher’s Ideal Method P01 = LP = 
p0 q0 p0 q1 × 100

(q0  q1 )  p1 p1q0  p1q1


(v) Marshall-Edgeworth Method P01 = 100 =
(q0  q1 )  p0 p0 q0  p0 q1 × 100

(vi) Kelly’s Method

p1q
(a) if fixed quantities are P01 =
p0 q × 100
given as weights

404 BUSINESS MATHEMATICS, LOGICAL REASONING & STATISTICS (Paper 3) [CA Foundation]
(b) If average of the quantities of
p1q q0  q1
two years is used as weights P01 = × 100 where q =
p0 q 2
(b) Weighted Average of Relatives
(a) if fixed qua
PV
(i) If Arithmetic Mean is used P01 =
V

P1
Where, P = Price Relative =
P0 × 100

V = Value Weights = P0q0

 V log p 
(ii) If Geometric Mean is used P01 = Antilog  
 V 
3. QUANTITY INDEX NUMBERS
(a) Weighted Aggregative Indices

q1 p0
(i) Laspeyres Method Q01 =
q0 p0 × 100

q1 p1
(ii) Paasche Method Q01 =
q0 p1 × 100

q1 p0 q1 p1
LP 
(iii) Dorbish & Bowley’s Method Q = = q 0 p0 q0 p1
01 2 100
2

q1 p0 q1 p1
(iv) Fisher’s Ideal Method Q01 = LP = 
q0 p0 q0 p1 × 100

( p0  p1 )  q1 q1P0  q1P1
(v) Marshall-Edgeworth Method Q01 = 100 =
( p0  p1 )  q0 q0 P0  q0 P1 × 100

(vi) Kelly’s Method

q1 p
(a) if fixed quantities are Q01 =
q0 p × 100
given as weights
(b) If average of the quantities of

INDEX NUMBER AND TIME SERIES 405


q1 p p0  p1
two years is used as weights Q01 =
q0 p × 100 where p = 2
(b) Weighted Average of Relatives

PV
(i) If Arithmetic Mean is used Q01 =
V

Q1
where, Q = Quantity Relatives
Q0 × 100

V = Value Weights = p0q0

 V log Q 
(ii) If Geometric Mean is used Q01 = Antilog 
 V 

p1q1
4. Value Index Number V = P01 × Q01 =
p0 q0

5. TESTS OF ADEQUACY
(a) Time Reversal Test P01 × P10 = 1
Note : Fisher’s Ideal Index and Marshall Edge worth
Index satisfy Time Reversal Test.

p1q1
(b) Factor Reversal Test P01 × Q01 =
p0 q0

Note : The Factor Reversal Test is satisfied only by


the Fisher’s Ideal Index.
(c) Circular Test P01 × P12 × P20 = 1
Note : The Circular Test is satisfied only by simple
geometric mean of price relatives and the weighted
aggregative fixed weighted.
6. Chain Base Index Number =

Average Link Re lative of Current Year  Chain Index of Pr eviousYear


100

Current Year ' s Pr ice


Note : Link Relative = × 100
Pr oviousYear ' s Pr ice

406 BUSINESS MATHEMATICS, LOGICAL REASONING & STATISTICS (Paper 3) [CA Foundation]
7. Conversion of Chain Base Index into Fixed Base Index

Current Year ' s C .B.I .  Pr evious year ' s F .B.I


Current year’s F.B.I. =
100
8. Conversion of Fixed Base Index into Chain Base Index

Current Year ' s F .B.I .


Current year’s C.B.I. = × 100
Pr eviousYear ' s F .B.I .
9. Conversion of Link Relative to Price Relative
Current year’s Price Relative

Current Year ' s link Re lative  Pr evious year ' s Pr ice Re lative
=
100
10. Base Shifting
New Index Number using New Base

Old Index Number u sin g old Base


= × 100
Index Number Correspondingto New Base period
11. Splicing
(a) Index No. of old series under forward splicing

100
= × Given Index No. of old series
Overlapping year ' s Index No. of old series
(b) Index No. of new series under backward splicing

Overlapping year ' s Index no. of old series


= × Given Index No. of new series
100
12. Deflating

MoneyWage
(i) Real Wage = × 100
Pr ice Index

Re alWage
(ii) Money Wage Index = × 100
MoneyWage of the BaseYear

MeneyWage Index
(iii) Real Wage Index = × 100
Pr ice Index
13. Consumer Price Index

INDEX NUMBER AND TIME SERIES 407


p1q0
(i) Consumer price Index =
p0 q0 × 100

PW
(ii) Consumer Price Index =
W

IW
(iii) Consumer Price Index =
W

408 BUSINESS MATHEMATICS, LOGICAL REASONING & STATISTICS (Paper 3) [CA Foundation]
4. The time reversal test is satisfied by

CLASS WORK (a) Fisher’s index number.


(b) Paasche’s index number.
(c) Laspeyre’s index number.
1. Each of the following statements is either True (d) None of these
or False write your choice of the answer by
5. The factor reversal test is satisfied by
writing T for True
(a) Index Numbers are the signs and (a) Simple aggregative index number.
guideposts along the business highway (b) Paasche’s index number.
that indicate to the businessman how he (c) Laspeyre’s index number.
should drive or manage.
(d) None of these.
(b) “For Construction index number, the best
6. The circular test is satisfied by
method on theoretical ground is not the
best method from practical point of (a) Fisher’s index number.
view”. (b) Paasche’s index number.
(c) Weighting index numbers makes them (c) Laspeyre’s index number.
less representative. (d) None of these.
(d) Fisher’s index number is not an ideal
7. Fisher’s index number is based on
index number.
(a) The Arithmetic mean of Laspeyre’s and
2. Each of the following statements is either True
Paasche’s index numbers.
or False. Write your choice of the answer by
writing F for false. (b) The Median of Laspeyre’s and Paasche’s
index numbers.
(a) Geometric mean is the most appropriate
average to be used for constructing an (c) the Mode of Laspeyre’s and Paasche’s
index number. index numbers.
(b) Weighted average of relatives and (d) None of these.
weighted aggregative methods render 8. Paasche index is based on
the same result. (a) Base year quantities.
(c) “Fisher’s Ideal Index Number is a (b) Current year quantities.
compromise between two well known
indices – not a right compromise, (c) Average of current and base year.
economically speaking”. (d) None of these.
(d) “Like all statistical tools, index numbers 9. Fisher’s ideal index number is
must be used with great caution”. (a) The Median of Laspeyre’s and Paasche’s
3. The best average for constructing an index index numbers
numbers is (b) The Arithmetic Mean of Laspeyre’s and
(a) Arithmetic Mean Paasche’s index numbers
(b) Harmonic Mean (c) The Geometric Mean of Laspeyre’s and
(c) Geometric Mean Paasche’s index numbers

(d) None of these. (d) None of these.

INDEX NUMBER AND TIME SERIES 409


10. Price-relative is expressed in term of 15. If the prices of all commodities in a place have
increased 1.25 times in comparison to the base
P P
(a) P= n (b) P= o period, the index number of prices of that place
Po Pn now is
(a) 125 (b) 150
P (c) 225 (d) None of these.
P = n × 100 P
(c) Po (d) P = o × 100 16. If the index number of prices at a place in 1994
Pn is 250 with 1984 as base year, then the prices
have increased on average by
11. Paasehe’s index number is expressed in terms (a) 250% (b) 150%
of : (c) 350% (d) None of these.
 Pnqn  Poqo 17. If the prices of all commodities in a place have
(a) (b) decreased 35% over the base period prices,
 Poqn  Pnqn then the index number of prices of that place
is now
 Pnqn  Pnqo (a) 35 (b) 135
(c) × 100 (d) × 100
 Poqn  Poqo (c) 65 (d) None of these.
12. Time reversal Test is satisfied by following 18. Link relative index number is expressed for
index number formula is period n is
(a) Laspeyre’s Index number. Pn P0
(b) Simple Arithmetic Mean of price relative (a) (b)
Pn + 1 Pn - 1
formula
(c) Marshall-Edge worth formula. Pn
(c) × 100 (d) None of these.
(d) None of these. Pn - 1
13. Cost of Living Index number (C. L. I.) is 19. Fisher’s Ideal Index number is expressed in
expressed in terms of : terms of :

 Pnqo  Pnqn F
(a) × 100 (b)
(a) Pon  = Laspeyre's Index × Paasche's Indexo p
 Poqo  Poqo
F
(b) Pon  = Laspeyre’s Index X Paasche’s Index
 Poqn
(c) × 100 (d) None of these (c)
 Pnqn
F
14. If the ratio between Laspeyre’s index number Pon  = Marshall Edge worth Index × Paasche's
and Paasche’s Index number is 28 : 27. Then (d) None of these
the missing figure in the following table P is : 20. Factor Reversal Test According to Fisher is P01
Commodity Base Year Current Year × Q01=
Price Quantity Price Quantity  Poqo  Pnqn
X L 10 2 5 (a) (b)
 Pnqn  Poqo
Y L 5 P 2
(a) 7 (b) 4  Poqn
(c) (d) None of these
(c) 3 (d) 9  Pnqn

410 BUSINESS MATHEMATICS, LOGICAL REASONING & STATISTICS (Paper 3) [CA Foundation]
21. Marshall-edge worth Index formula after 26.
interchange of p and q is expressed in terms (a) Base year quantities are taken as weights
of: in Laspeyre’s price Index number.
(b) F isher ’s ideal index is equal to the

 qn P0 + Pn  
 Pn q0 + qn  Arithmetic mean of Laspeyre’s and
(a) (b)  qP P + P

 q0 P0 + Pn  0 0 n
  Paasche’s index numbers.
(c) Laspeyre’s index number formula does
not satisfy time reversal test.

 P0 q0 + qn  (d) None of these.
(c) (d) None of these

 Pn P0 + Pn  27.
(a) Current year quantities are taken as
22. If  Pn qn = 249,  Po q o = 150, Paasche’s Indexx weights in Paasche’s price index number.
Number = 150 and Drobiseh and Bowely’s (b) Edge worth Marshall’s index number
Index number = 145, then the Fisher’s Ideal formula satisfies Time, Reversal Test.
Index Number is
(c) The Arithmetic mean of Laspeyre’s and
(a) 75 (b) 60 Paasche’s index numbers is called
(c) 145.97 (d) None of these. Bowely’s index numbers.
23. Consumer Price index number for the year (d) None of these.
1957 was 313 with 1940 as the base year 96 the 28.
Average Monthly wages in 1957 of the workers
into factory be ` 160/- their real wages is (a) Current year prices are taken as weights
in Paasche’s quantity index number.
(a) ` 48.40 (b) ` 51.12
(b) Fisher’s Ideal Index formula satisfies
(c) ` 40.30 (d) None of these. factor Reversal Test.
24. If  Po q o = 3500,  Pn q o = 3850, then the (c) The sum of the quantities of the base
Cost of living Index (C.L.I.) for 1950 w.r. to base period and current period is taken as
1960 is weights in Laspeyre’s index number.
(a) 110 (b) 90 (d) None of these.
(c) 100 (d) None of these. 29.
25. From the following table by the method of (a) Simple Aggregative and simple
relatives using Arithmetic mean the price Geometric mean of price relatives
Index number is formula satisfy circular Test.
Commodity Wheat Milk Fish Sugar (b) Base year prices are taken as weights in
Laspeyre’s quantity index numbers.
Base Price 5 8 25 6
(c) Fisher’s Ideal Index formula obeys time
Current Price 7 10 32 12
reversal and factor reversal tests.
(a) 140.35 (b) 148.95
(d) None of these.
(c) 140.75 (d) None of these.
30. In 1980, the net monthly income of the
From the Q.No. 26 to 29 each of the following employee was ‘ 800/- p. m. The consumer price
statements is either True or False with your index number was 160 in 1980. It rises to 200 in
choice of the answer by writing F for False. 1984. If he has to be rightly compensated.
The additional D. A. to be paid to the employee
is

INDEX NUMBER AND TIME SERIES 411


(a) ` 175/- (b) ` 185/- (a) 117.3 (b) 115.43
(c) ` 200/- (d) ` 125. (c) 118.35 (d) 116.48
31. The simple Aggregative formula and weighted 37. Which statement is False?
aggregative formula satisfy is
(a) The choice of suitable base period is at
(a) Factor Reversal Test best a temporary solution.
(b) Circular Test (b) The index number is a special type of
(c) Unit Test average.
(d) None of these. (c) Those is no such thing as unweighted
32. “Fisher’s Ideal Index is the only formula which index numbers.
satisfies” (d) Theoretically, geometric mean is the best
(a) Time Reversal Test average in the construction of index
(b) Circular Test numbers but in practice, mostly the
arithmetic mean is used.
(c) Factor Reversal Test
(d) a & c. 38. Factor Reversal Test is expressed in terms of
33. “Neither Laspeyre’s formula nor Paasche’s  P1Q1
formula obeys” : (a)  P0Q 0
(a) Time Reversal and factor Reversal Tests
of index numbers.
 P1Q1  P1Q1
(b) Unit Test and circular Tests of index 
number. (b)  P0 Q 0  P0Q 1
(c) Time Reversal and Unit Test of index
number.  P1Q1
(d) None of these. (c)  P0Q 1
34. Bowley’s index number is 150. Fisher’s index
number is 149.95. Paasche’s index number is
 P1Q 0  P1Q1
(a) 158 (b) 154 (d) 
 P0Q 0  P0Q 1
(c) 148 (d) 156
35. With the base year 1960 the C. L. I. in 1972 stood 39. Circular Test is satisfied by
at 250. x was getting a monthly Salary of ` 500 (a) Laspeyre’s Index number.
in 1960 and ` 750 in 1972. In 1972 to maintain
his standard of living in 1960 x has to receive (b) Paasche’s Index number
as extra allowances of (c) The simple geometric mean of price
(a) ` 600/- (b) ` 500/- relatives and the weighted aggregative
with fixed weights.
(c) ` 300/- (d) none of these.
(d) None of these.
36. From the following data base year :-
Commodity Base Year Current Year 40. From the following data for the 5 groups
combined
Price Quantity Price Quantity
Group Weight Index
A 4 3 6 2
Number
B 5 4 6 4
C 7 2 9 2
Food 35 425
D 2 3 1 5 Cloth 15 235
Fisher’s Ideal Index is Power & Fuel 20 215

412 BUSINESS MATHEMATICS, LOGICAL REASONING & STATISTICS (Paper 3) [CA Foundation]
Rent & Rates 8 115 45. The price level of a country in a certain year
has increased 25% over the base period. The
Miscellaneous 22 150
index number is
The general Index number is
(a) 25 (b) 125
(a) 270 (b) 269.2
(c) 225 (d) 2500
(c) 268.5 (d) 272.5
46. The index number of prices at a place in 1998
41. From the following data with 1966 as base year is 355 with 1991 as base. This means
Commodity Quantity Units Values (`) (a) There has been on the average a 255%
A 100 500 increase in prices.
B 80 320 (b) There has been on the average a 355%
increase in price.
C 60 150
(c) There has been on the average a 250%
D 30 360 increase in price.
The price per unit of commodity A in 1966 is (d) None of these.
(a) `5 (b) `6 47. If the price of all commodities in a place have
(c) `4 (d) ` 12 increased 125 times in comparison to the base
42. The index number in whole sale prices is 152 period prices, then the index number of prices
for August 1999 compared to August 1998. for the place is now

During the year there is net increase in prices (a) 100 (b) 125
of whole sale commodities to the extent of (c) 225 (d) None
(a) 45% (b) 35% 48. The wholesale price index number or
(c) 52% (d) 48% agricultural commodities in a given region at a
given date is 280. The percentage increase in
43. The value Index is expressed in terms of prices of agricultural commodities over the
base year is :
 P1Q1
(a)  100 (a) 380 (b) 280
 P0Q 0
(c) 180 (d) 80
49. If now the prices of all the commodities in a
 P1Q1
place have been decreased by 85% over the
(b)  P0Q 0 base period prices, then the index number of
prices for the place is now (index number of
 P0Q 0 prices of base period = 100)
(c) × 100 (a) 100 (b) 135
 P1Q1
(c) 65 (d) None of these.
 P0Q1   P1Q1 50. From the data given belowCommodity
(d) Price Relative Weight
 P0Q 0  P1Q 0
A 125 5
44. Purchasing Power of Money is B 67 2
(a) Reciprocal of price index number.
C 250 3
(b) Equal to price index number. Then the suitable index number is
(c) Unequal to price index number. (a) 150.9 (b) 155.8
(d) None of these. (c) 145.8 (d) None of these.

INDEX NUMBER AND TIME SERIES 413


51. Bowley’s Index number is expressed in the 55. For constructing consumer price Index is used:
form of : (a) Marshall Edge worth Method.
Laspeyre's index + Paasche's index (b) Paasche’s Method.
(a) (c) Dorbish and Bowley’s Method.
2
(d) Laspeyre’s Method.
Laspeyre's index × Paasche's index
(b) 56. The cost of living Index (C.L.I.) is always :
2 (a) Weighted index
Laspeyre's index - Paasche's index (b) Price Index.
(c) (c) Quantity Index.
2
(d) None of these (d) None of these.
52. From the following data 57. The Time Reversal Test is not satisfied to :
Commodity Base Price Current Pricet (a) Fisher’s ideal Index.
(b) Marshall Edge worth Method.
Rice 35 42
(c) Laspeyre’s and Paasche Method.
Wheat 30 35
(d) None of these.
Pulse 40 38
58. Given below are the data on prices of some
Fish 107 120 consumer goods and the weights attached to
The simple Aggregative Index is the various items Compute price index number
(a) 115.8 (b) 110.8 for the year 1985 (Base 1984 = 100)
(c) 112.5 (d) 113.4 Items Unit 1984 1985 Weight
53. With regard to Laspeyre’s and Paasche’s price Wheat Kg. 0.50 0.75 2
index numbers, it is maintained that “If the Milk Litre 0.60 0.75 5
prices of all the goods change in the same ratio,
the two indices will be equal for them the Egg Dozen 2.00 2.40 4
weighting system is irrelevant; or if the Sugar Kg. 1.80 2.10 8
quantities of all the goods change in the same Shoes Pair 8.00 10.00 1
ratio, they will be equal, for them the two
Then weighted average of price Relative
weighting systems are the same relatively”.
Index is :
Then the above statements satisfy. (a) 125.43 (b) 123.3
(a) Laspeyre’s Price index ? Paasche’s Price (c) 124.53 (d) 124.52
Index. 59. The Factor Reversal Test is as represented
(b) Laspeyre’s Price Index = Paasche’s Price symbolically is :
Index.
 P1Q1
(c) Laspeyre’s Price Index may be equal (a) p01 × Q 01 =
Paasche’s Price Index.  P0Q 0
(d) None of these. (b) I01 ×I10
54. The quantity Index number using Fisher’s
formula satisfies :  P0Q 0
(a) Unit Test (c)  P1Q1
(b) Factor Reversal Test.
(c) Circular Test.  P1Q1  P0Q 0
(d) ×
(d) Time Reversal Test.  P0Q 0  Q10P0

414 BUSINESS MATHEMATICS, LOGICAL REASONING & STATISTICS (Paper 3) [CA Foundation]
60. If the 1970 index with base 1965 is 200 and 64. Time Reversal Test is represented symbolically
1965 index with base 1960 is 150, the index by :
1970 on base 1960 will be :
(a) P01 × p10 (b) P01 × p10 = 1
(a) 700 (b) 300
(c) 500 (d) 600 (c) P01 × p10  1 (d) None of these
61. Circular Test is not met by :
65. In 1996 the average price of a commodity was
(a) The simple Geometric mean of price 20% more than in 1995 but 20% less than in
relatives. 1994; and more over it was 50% more than in
(b) The weighted aggregative with fixed 1997 to price relatives using 1995 as base (1995
weights. price relative 100) Reduce the data is :
(c) Laspeyre’s or Paasche’s or the fisher’s (a) 150, 100, 120, 80 for (1994–97)
Ideal index. (b) 135, 100, 125, 87 for (1994–97)
(d) None of these. (c) 140, 100, 120, 80 for (1994–97)
62. From the following data (d) None of these
Commodity Base Year Current Year 66. From the following data
Price Quantity Price Quantity Commodity Base Year Current Year
A 4 3 6 2 1922 1934
B 5 4 6 4 Price (`) Price
C 7 2 9 2 A 6 10
D 2 3 1 5 B 2 2
Then the value ratio is: C 4 6
D 11 12
59 49 E 8 12
(a) (b)
52 47 The price index number for the year 1934 is :
(a) 140 (b) 145
41 47
(c) (d) (c) 147 (d) None of these.
53 53 67. From the following data
63. The value index is equal to : Commodity Base Year Current Year
(a) The total sum of the values of a given 1964 1968
year multiplied by the sum of the values Rice 36 54
of the base year.
Pulse 30 50
(b) The total sum of the values of a given
year Divided by the sum of the values of
Fish 130 155
the base year. Potato 40 35
(c) The total sum of the values of a given Oil 110 110
year plus by the sum of the values of the The index number by unweighted methods :
base year. (a) 116.8 (b) 117.25
(d) None of these. (c) 115.35 (d) 119.37

INDEX NUMBER AND TIME SERIES 415


68. The Bowley’s Price index number is 72. From the following data
represented in terms of : Group A B C D E F
(a) A.M. of Laspeyre’s and Paasche’s Price Group Index 120 132 98 115 108 95
index number.
Weight 6 3 4 2 1 4
(b) G.M. of Laspeyre’s and Paasche’s Price
index number. The general Index I is given by :
(c) A.M. of Laspeyre’s and Walsh’s price index (a) 11.3 (b) 113.45
number. (c) 117.25 (d) 114.75
(d) None of these.
73. The price of a commodity increases from ` 5
69. Fisher’s price index number equal is : per unit in 1990 to ` 7.50 per unit in 1995 and
(a) G.M. of Kelly’s price index number and the quantity consumed decreases from 120
Paasche’s price index number. units in 1990 to 90 units in 1995. The price and
(b) G.M. of Laspeyre’s and Paasche’s Price quantity in 1995 are 150% and 75% respectively
index number. of the corresponding price and quantity in
(c) G.M. of Bowley’s price index number and 1990. Therefore, the product of the price ratio
Paasche’s price index number. and quantity ratio is :
(d) None of these. (a) 1.8 (b) 1.125
70. The price index number using simple G.M. of (c) 1.75 (d) None
the n relatives is given by : 74. Test whether the index number due to Walsh
give by :
1 P
(a) log Ion =2 -  log n
n Po  p1 Q 0Q1
I= × 100 Satisfies is :-
1 P  P0 Q 0Q1
(b) logIon =2 +  log n
n Po
(a) Time reversal Test.
1 P (b) Factor reversal Test.
(c) log Ion =  log n (c) Circular Test.
n Po
(d) None of these.
(d) None of these
75. From the following data
71. The price of a number of commodities are
given below in the current year 1975 and base Group Weight Index Number
year 1970. Base :
Commodities A B C D E F 1952–53 = 100
Base Price 45 60 20 50 85 120 Food 50 241
Current Price 55 70 30 75 90 130 Clothing 2 21
For 1975 with base 1970 by the Method of price Fuel and Light 3 204
relatives using Geometrical mean, the price Rent 16 256
index is :
Miscellaneous 29 179
(a) 125.3 (b) 124.3
The Cost of living index numbers is :
(c) 128.8 (d) None of these.
(a) 224.5 (b) 223.91
(c) 225.32 (d) None of these.

416 BUSINESS MATHEMATICS, LOGICAL REASONING & STATISTICS (Paper 3) [CA Foundation]
76. Consumer price index number goes up from 81. From the following data
110 to 200 and the Salary of a worker is also Commodity Base Year Current Year
raised from ` 325 to ` 500. Therefore, in real
terms, to maintain his previous standard of Price Quantity Price Quantity
living he should get an additional amount of : A 7 17 13 25
(a) ` 85 (b) ` 90.91 B 6 23 7 25
(c) ` 98.25 (d) None of these. C 11 14 13 15
77. The prices of a commodity in the year 1975 D 4 10 8 8
and 1980 were 25 and 30 respectively taking The Marshall Edge Worth Index number is :
1980 as base year the price relative is :
(a) 148.25 (b) 144.19
(a) 109.78 (b) 110.25
(c) 147.25 (d) None of these.
(c) 113.25 (d) None of these.
82. The circular Test is an extension of
78. The average price of certain commodities in
(a) The time reversal Test.
1980 was ` 60 and the average price of the same
commodities in 1982 was ` 120. Therefore, the (b) The factor reversal Test.
increase in 1982 on the basis of 1980 was 100%. (c) The unit Test.
80. The decrease in 1980 with 1982 as base is:
(d) None of these.
using 1982, comment on the above statement
is : 83. Circular test, an index constructed for the year
‘x’ on the base year ‘y’ and for the year ‘y’ on
(a) The price in 1980 decreases by 60% using
the base year ‘z’ should yield the same result
1982 as base.
as an index constructed for ‘x’ on base year ‘z’
(b) The price in 1980 decreases by 50% using i.e. I01 × I12 × I20 equal is :
1982 as base.
(a) 3 (b) 2
(c) The price in 1980 decreases by 90% using
(c) 1 (d) None of these.
1982 as base.
84. In 1976 the average price of a commodity was
(d) None of these.
20% more than that in 1975 but 20% less than
79. Cost of Living Index (C.L.I.) numbers are also that in 1974 and more over it was 50% more
used to find real wages by the process of than that in 1977. The price relatives using 1975
(a) Deflating of Index number. as base year (1975 price relative = 100) then
(b) Splicing of Index number. the reduce date is :
(c) Base shifting. (a) 8,.75 (b) 150,80
(d) None of these. (c) 75,125 (d) None of these.
80. From the following data 85. T ime Reversal Test is represented by
symbolically is :
Commodities A B C D
(a) P01 x Q01 = 1
1992 Base Price 3 5 4 1
(b) I01 x I10 = 1
Quantity 18 6 20 14
(b) I01 x I12 x I23 x …. I(n–1)n x In0 = 1
1993 Price 4 5 6 3
(d) None of these.
Current Quantity 15 9 6 15
86. The prices of a commodity in the years 1975
Year
and 1980 were 25 and 30 respectively, taking
The Passche price Index number is : 1975 as base year the price relative is :
(a) 146.41 (b) 148.25 (a) 120 (b) 135
(c) 144.25 (d) None (c) 122 (d) None of these.

INDEX NUMBER AND TIME SERIES 417


87. From the following data 98.4, then the purchasing power of money
Year 1992 1993 1995 1996 1997 (Rupees) of 1950 will in 1960 is
(a) ` 1.12 (b) ` 1.25
Link Index 100 103 105 112 108
(c) ` 1.37 (d) None of these.
(Base 1992 = 100) for the years 1993–97. The
construction of chain index is : 93. If
(a) 103, 100.94, 107, 118.72  P0Q 0 =1360,  PnQ 0 =1900,  P0Q n =1344, PnQn =1880
(b) 103, 108.15, 121.3, 130.82 then the Laspeyre’s Index number is
(c) 107, 100.25, 104, 118.72 (a) 0.71 (b) 1.39
(d) None of these. (c) 1.75 (d) None of these.
88. During a certain period the cost of living index 94. The consumer price Index for April 1985 was
number goes up from 110 to 200 and the salary 125. The food price index was 120 and other
of a worker is also raised from ` 330 to ` 500. items index was 135. The percentage of the
The worker does not get really gain. total weight of the index is
Then the real wages decreased by : (a) 66.67 (b) 68.28
(a) ` 45.45 (b) ` 43.25 (c) 90.25 (d) None of these.
(c) ` 100 (d) None 95. The total value of retained imports into India
89. Net monthly salary of an employee was ` 3000 in 1960 was ` 71.5 million per month. The
in 1980. The consumer price index number in corresponding total for 1967 was ` 87.6 million
1985 is 250 with 1980 as base year. If the has to per month. The index of volume of retained
be rightly compensated then, 7th dearness imports in 1967 composed with 1960 (= 100)
allowances to be paid to the employee is : was 62.0. The price index for retained inputs
(a) ` 4.800.00 (b) ` 4,700.00 for 1967 our 1960 as base is

(c) ` 4,500.0 (d) None of these. (a) 198.61 (b) 197.61

90. Net Monthly income of an employee was ` (c) 198.25 (d) None of these.
800 in 1980. The consumer price Index number 96. During the certain period the C.L.I. goes up
was 160 in 1980. It is rises to 200 in 1984. If he from 110 to 200 and the Salary of a worker is
has to be rightly compensated. The additional also raised from 330 to 500, then the real terms
dearness allowance to be paid to the is
employee is : (a) Loss by ` 50 (b) Loss by 75
(a) ` 240 (b) ` 275 (c) Loss by ` 90 (d) None of these.
(c) ` 250 (d) None of these. 97. From the following data
91. When the cost of Tobacco was increased by Commodities Q0 P0 Q1 P1
50%, a certain hardened smoker, who
maintained his formal scale of consumption, A 2 2 6 18
said that the rise had increased his cost of living B 5 5 2 2
by 5%. Before the change in price, the
C 7 7 4 24
percentage of his cost of living was due to
buying Tobacco is Then the fisher’s quantity index number is
(a) 15% (b) 8% q (a) 87.34 (b) 85.24
(c) 10% (d) None of these. (c) 87.25 (d) None of these
92. If the price index for the year, say 1960 be 110.3
and the price index for the year, say 1950 be

418 BUSINESS MATHEMATICS, LOGICAL REASONING & STATISTICS (Paper 3) [CA Foundation]
98. From the following data 104. ________ is particularly suitable for the
Commodities Base year Current year construction of index numbers.
(a) H.M. (b) A.M.
A 25 55
(c) G.M. (d) none
B 30 45 105. Index numbers show _________ changes
Then index numbers from G. M. Method is : rather than absolute amounts of change.
(a) 181.66 (b) 185.25 (a) relative (b) percentage
(c) 181.75 (d) None of these. (c) both (d) none
106. The ________ makes index numbers time-
99. Using the following data
reversible.
Commodity Base Year Current Year (a) A.M. (b) G.M.
Price Quantity Price Quantity (c) H.M. (d) none
X 4 10 6 15 107. Price relative is equal to
Y 6 15 4 20 Price in the given year ×100
(a)
Z 8 5 10 4 Price in the base year
the Paasche’s formula for index is : Price in the year base year × 100
(a) 125.38 (b) 147.25 (b)
Price in the given year
(c) 129.8 (d) None of these. (c) Price in the given year x 100
100. Group index number is represented by (d) Price in the base year x 100
108. Index number is equal to
Price Relative for the year
(a) × 100 (a) sum of price relatives
Price Relative for the previous year (b) average of the price relatives
(c) product of price relative
  Pr ice Relative × w  (d) none
(b)
w 109. The ________ of group indices given the
General Index
  Pr ice Relative × w  (a) H.M. (b) G.M.
(c)
w (c) A.M. (d) none
(d) None of these 110. Circular Test is one of the tests of
(a) index numbers (b) hypothesis
101. A series of numerical figures which show the (c) both (d) none
relative position is called 111. ___________ is an extension of time reversal
test
(a) index number (b) relative number
(a) Factor Reversal test (b) Circular test
(c) absolute number (d) none
(c) both (d) none
102. Index number for the base period is always
112. Weighted G.M. of relative formula satisfy
taken as
________test
(a) 200 (b) 50 (a) Time Reversal Test (b) Circular test
(c) 1 (d) 100 (c) Factor Reversal Test (d) none
103. _________ play a very important part in the 113. Factor Reversal test is satisfied by
construction of index numbers. (a) Fisher’s Ideal Index
(a) weights (b) classes (b) Laspeyres Index
(c) estimations (d) none (c) Paasches Index
(d) none

INDEX NUMBER AND TIME SERIES 419


114. Laspeyre’s formula does not satisfy 123. P01 is the index for time
(a) Factor Reversal Test (a) 1 on 0 (b) 0 on 1
(b) Time Reversal Test (c) 1 on 1 (d) 0 on 0
(c) Circular Test (d) all the above
124. P10 is the index for time
115. A ratio or an average of ratios expressed as a
percentage is called (a) 1 on 0 (b) 0 on 1 (c) 1 on 1 (d) 0 on 0
(a) a relative number 125. When the product of price index and the
(b) an absolute number quantity index is equal to the corresponding
value index then the test that holds is
(c) an index number (d) none
(a) Unit Test
116. The value at the base time period serves as
the standard point of comparison (b) Time Reversal Test
(a) false (b) true (c) Factor Reversal Test (d) none holds
(c) both (d) none 126. The formula should be independent of the
unit in which or for which price and quantities
117. An index time series is a list of _______
are quoted in
numbers for two or more periods of time
(a) Unit Test
(a) index (b) absolute
(b) Time Reversal Test
(c) relative (d) none
(c) Factor Reversal Test (d) none
118. Index numbers are often constructed from the
127. Laspeyre’s method and Paasche’s method do
(a) frequency (b) class
not satisfy
(c) sample (d) none
(a) Unit Test
119. ________ is a point of reference in comparing
(b) Time Reversal Test
various data describing individual behaviour.
(c) Factor Reversal Test (d) b & c
(a) Sample (b) Base period
128. The purpose determines the type of index
(c) Estimation (d) none
number to use
120. The ratio of price of single commodity in a
(a) yes (b) no
given period to its price in the preceding year
(c) may be (d) may not be
price is called the
129. The index number is a special type of average
(a) base period (b) price ratio
(a) false (b) true
(c) relative price (d) none
(c) both (d) none
Sum of all commodity prices in the current year × 100 130. The choice of suitable base period is at best
121.
Sum of all commodity prices in the base year is temporary solution
(a) Relative Price Index (a) true (b) false
(b) Simple Aggregative Price Index (c) both (d) none
(c) both (d) none 131. Fisher’s Ideal Formula for calculating index
122. Chain index is equal to numbers satisfies the _______ tests
(a) Unit Test
link relative of current year × chain index of the current year
(a) (b) Factor Reversal Test
100
(c) both (d) none
link relative of previous year × chain index of the current year
(b) 132. F isher ’s Ideal Formula dose not satisfy
100 _________ test
link relative of current year × chain index of the previous year (a) Unit Test (b) Circular Test
(c)
100 (c) Time Reversal Test (d) none
link relative of previous year × chain index of the previous year
133. _________________ satisfies circular test
(d) (a) G.M. of price relatives or the weighted
100
aggregate with fixed weights

420 BUSINESS MATHEMATICS, LOGICAL REASONING & STATISTICS (Paper 3) [CA Foundation]
(b) A.M. of price relatives or the weighted Original Price × 100
aggregate with fixed weights 141.
Price Index of the year on which it has to be shifted
(c) H.M. of price relatives or the weighted
aggregate with fixed weights (a) True (b) false
(d) none (c) both (d) none
134. Laspeyre’s and Paasche’s method _________ 142. The number of test of Adequacy is
time reversal test (a) 2 (b) 5
(a) satisfy (b) do not satisfy (c) 3 (d) 4
(c) are (d) are not 143. We use price index numbers
135. There is no such thing as unweighted index (a) To measure and compare prices
numbers
(b) to measure prices
(a) false (b) true
(c) to compare prices (d) none
(c) both (d) none
144. Simple aggregate of quantities is a type of
136. Theoretically, G.M. is the best average in the
construction of index numbers but in practice, (a) Quantity control
mostly the A.M. is used (b) Quantity indices
(a) false (b) true (c) both (d) none
(c) both (d) none
137. Laspeyre’s or Paasche’s or the Fisher’s ideal ---0---0---
index do not satisfy
(a) Time Reversal Test(b) Unit Test
(c) Circular Test (d) none
138. ___________ is concerned with the
measurement of price changes over a period
of years, when it is desirable to shift the base
(a) Unit Test (b) Circular Test
(c) Time Reversal Test (d) none
139. The test of shifting the base is called
(a) Unit Test
(b) Time Reversal Test
(c) Circular Test (d) none
140. The formula for conversion to current value
Price Index of the current year
(a) Deflated value =
previous value

Price Index of the current year


(b) Deflated value =
current value

Price Index of the previous year


(c) Deflated value =
previous value

Price Index of the previous year


(d) Deflated value =
previous value

INDEX NUMBER AND TIME SERIES 421


8. Most commonly used index number is
(a) Diffusion index number
HOME WORK-1 (b) price index number
(c) value index number
(d) none
1. Index numbers are also known as
9. The index number of the base year is
(a) economic barometers
(a) 1 (b) 100
(b) signs and guide posts
(c) 0 (d) none
(c) both (a) and (b)
10. A fixed year is taken as the base year in the
(d) neither (a) nor (b) chain base method.
2. Index number is a (a) Yes (b) No
(a) measure of relative change (c) May be (d) none
(b) a special type of an average 11. The cost of living index number for the working
(c) a percentage relative class families can not be applied to the middle
(d) all the above class families.
3. Index numbers are expressed (a) Yes (b) No
(a) in percentages (c) May be (d) none
(b) in ratios 12. The price of a commodity in the current year
(c) in terms of absolute value has increased by 250 per cent compared to that
(d) all the above in the base year. Therefore the index number
is 250.
4. Index numbers help
(a) Yes (b) No
(a) in framing of economic policies
(c) May be (d) none
(b) In assessing the purchasing power of
money. 13. Each commodity is given equal importance in
the construction of index number.
(c) for adjusting national income
(a) Yes (b) No
(d) all the above
(c) May be (d) none
5. Index numbers reveal the state of
14. If the price of the current year is eight times to
(a) inflation
that of the base year, the index number is
(b) deflation
(a) 8 (b) 100
(c) both (a) and (b)
(c) 800 (d) none
(d) neither (a)nor (b)
15. In the ........ method the preceding year is taken
6. The error(s) involved in the construction of as the base year.
index numbers is/are
(a) Fixed base (b) chain base
(a) error of sampling
(c) Fisher’s (d) none
(b) formula error
16. Consumer price index number is constructed
(c) error in collection of data for
(d) all the above (a) a well defined section of people
7. Element of subjectivity is involved in index (b) all people
numbers due to
(c) factory workers only
(a) choice of base year
(d) all the above
(b) selection of weights
(c) choice of commodities
(d) all the above

422 BUSINESS MATHEMATICS, LOGICAL REASONING & STATISTICS (Paper 3) [CA Foundation]
17. Base period for an index number should be 25. Paasche was
(a) a year only (a) an English mathematician
(b) a normal period (b) a French economist
(c) a period at distant past (c) a German statistician
(d) none (d) none
18. The most appropriate average for index 26. Paasche’s index number was given in the year
number is (a) 1871 (b) 1901
(a) arithmetic mean (c) 1874 (d) 1918
27. The geometric mean of Laspeyre’s and
(b) geometric mean
Paasche’s price indices is also known as:
(c) harmonic mean (d) none
(a) Fisher’s price index
19. Most frequently used index number formulae (b) Kelly’s price index
are
(c) Drobish-Bowely price index
(a) weighted formulae (d) Walsh price index
(b) unweighted formulae 28. Geometric cross formula for index number is
(c) fixed weight formulae known as :
(d) none (a) Walsh ideal formula
20. Unweighted price index formula is (b) Fisher’s ideal formula
(a) most frequently used (c) Bowley’s standard formula
(b) seldom used (d) none
(c) the best 29. Fisher named the geometric cross formula as
(d) all an ideal index formula in the year.
(a) 1901 (b) 1910
21. Laspeyre’s index formula uses the weights of
the (c) 1920 (d) 1950
30. Dorbish-Bowley gave the formula for price
(a) base year
index in:
(b) current year
(a) 1910 (b) 1901
(c) average of the weights of a number of
(c) 1801 (d) 1871
years.
31. The Drobish-Bowely price index formula is the:
(d) none of the above. (a) geometric mean of Laspeyre’s and
22. Laspeyre’s index number was given in the year Paasche’s price index formulae.
(a) 1901 (b) 1871 (b) arithmetic mean of Laspeyre’s and
(c) 1874 (d) none Paasche’s price index formulae
23. Laspeyre’s index number is also known as : (c) Weighted mean of Laspeyre’s and
(a) fixed base index Paasche’s price index formulae
(b) given year method index (d) none of the above.
(c) base year method index 32. Cost of living index number from the following
data is
(d) none
Group Index numbers Weight
24. Paasche’s index number formula uses the
A 250 44
weights of the
B 320 20
(a) the base period
C 150 16
(b) the current period
D 170 8
(c) to any arbitrary chosen period
E 280 12
(d) none
(a) 200.2 (b) 245.2
(c) 300.2 (d) none

INDEX NUMBER AND TIME SERIES 423


33. The price of five commodities have increased 39. For the data given below, find x if the ratio
by 100%, 200%, 300%, 400 % and 500% in the between Laspeyre’s (L) and Paasche’s (P) index
current year compared to those in the base numbers is 28 : 27
year and their relative importance are in the Commodities Commodities
proportion 5 : 4 : 3 : 2 : 1. The index number of A B A B
prices is
p0 1 1 p1 2 x
(a) 100 (b) 133.33
q0 10 5 q1 5 2
(c) 333.33 (d) none
(a) 5 (b) 6
34. From the following data, Laspeyre’s index
(c) 4 (d) none
number is
40 The price index as the arithmetic mean of
Commodity Base year Current year
Laspeyre’s and Paasche’s indices was
Price Quantity Price Quantity
expounded by
A 6 50 10 50
B 2 100 2 200 (a) Kelly (b) Irving Fisher
C 4 200 6 100 (c) Drobish and Bowely (d) Walsh
(a) 100 (b) 133.33 41. Marshall and Edgeworth price index number
(c) 333.33 (d) none formula utilizes the weights as
35. From the following data, Fisher ’s index (a) quantities of the base year
number is (b) quantities of the current year.
Commodity Base year Current year (c) combined quantities of base and given
Price Quantity Price Quantity year
A 6 50 10 50 (d) any of the above
B 2 100 2 200 42. The weighted price relatives with selective
C 4 200 6 100 weights can be reduced to
(a) 100 (b) 133.33 (a) Laspeyre’s formula
(c) 333.33 (d) none (b) Paasche’s formula
36. From the following data, Paasche’s index (c) both (a) and (b)
number is
(d) neither (a) nor (b).
Commodity Base year Current year
43. Sampling error in respect of index numbers
Price Quantity Price Quantity
can be reduced by
A 6 50 10 50
(a) taking a random sample of items.
B 2 100 2 200
(b) including large number of items.
C 4 200 6 100
(a) 100 (b) 133.33 (c) both (a) and (b)
(c) 333.33 (d) none (d) neither (a) nor (b)
37. The index number of base year is 100. 44. If the index number is independent of the units
of measurements, then it satisfies
(a) Yes (b) N
(c) May be (d) none (a) times reversal test
(b) factor reversal test
38. In usual notations,
(c) unit test
P q 1 0  500,  P0 q0  425, (d) all the above
45. Factor reversal test was given by
P q 1 1  540,  P0 q1  480
(a) Walsh (b) A.L.Bowely
the Fisher’s index number is
(c) John I. Griffin (d) Irving Fisher
(a) 100 (b) 115
46. Circular test for price indices is satisfied by the
(c) 120 (d) none formula :

424 BUSINESS MATHEMATICS, LOGICAL REASONING & STATISTICS (Paper 3) [CA Foundation]
(a) based on geometric mean of price (c) 50, 52.5, 60, 75, 200, 125, 150, 160, 175
relatives (d) none of them
(b) obtained by Kelly’s fixed weight method. 54. From the following data, the index numbers
(c) both (a) and (b) taking 1995 as the base year are
(d) neither (a) nor (b) Year I. N. I. N.
47. Fisher’s ideal formula does not satisfy: (Base 1990) Base 1995
(a) time reversal test 1990 100
(b) circular test 1991 120
(c) factor reversal test 1992 125
(d) unit test 1993 140
48. Year-to-year indices in the chain-base method 1994 150
are called : 1995 160 100
(a) Chain indices (b) link relatives 1996 110
(c) fixed base indices (d) all the above 1997 115
49. Indices calculated by the chain-base method 1998 120
are almost free from: 1999 128
(a) homogeneity error 2000 130
(b) seasonal variations (a) 62.5, 75, 78.12, 87.5, 93.75, 100, 110, 115,
(c) rigidity of weights 120, 128, 130
(d) all the above (b) 100, 120, 125, 140, 150, 160, 110, 115, 120,
50. Combining of two index number series having 128, 130
different base periods into one series with
(c) 100, 120, 125, 140, 150, 100, 110, 115, 120,
common base period is known as :
128, 130
(a) Splicing (b) base shifting
(d) none of them
(c) both (a) and (b) 55. Fisher’s ideal index number satisfies circular
(d) neither (a) nor (b) test.
51. If the old series is connected with the new (a) Yes (b) No
series of index numbers, it is known as: (c) May be (d) none
(a) base sifting 56. An index is at 100 in 1991. It rises 5% in 1992,
(b) backward splicing falls 6% in 1993, falls 5% in 1994, rises 4% in
(c) forward splicing 1995 and 7% in 1996. The index numbers for all
(d) none these years with 1991 as base are
52. If the new series is connected with the old (a) 100,105,94,105,95,107
series, it is known as : (b) 100,105,94,105,107,95
(a) base shifting (c) 100,105,94,107,95,94
(b) backward splicing (d) 100,105,94,95,104,107
(c) forward splicing 57. The cost of living index number from the
following data :
(d) all
Group Weights Group/Commodity
53. The following are the index number of prices
Commodities W Index Number I
on base 1990.
Food 71 370
Year 1990 1991 1992 1993 1994 1995 1996 1997 1998
Clothing 3 423
I. N. 100 105 120 150 200 250 300 320 350
Fuel etc. 9 469
the index number of prices on base 1994 are
House rent 7 110
(a) 66.67, 70, 72, 76.67, 80, 100, 106.67, 110, Miscellaneous 10 279
113.33 (a) 353.2 (b) 300
(b) 50, 52.5, 60, 75, 100, 125, 150, 160, 175 (c) 100 (d) none

INDEX NUMBER AND TIME SERIES 425


58. Index number is unit free. their respective group weights are 60, 20 and
(a) False (b) True 20, the consumer price index is
(c) may be (d) none (a) 108.33 (b) 97.00
59. If Laspeyre’s price index is 324 and Fisher’s (c) 98.49 (d) none
ideal index is 216, then Paasche’s price index 68. An appropriate method for working out
is consumer price index is
(a) 234 (b) 180 (a) weighted aggregate expenditure
(c) 216 (d) none (b) family budget method method.
60. Deflation of index numbers is meant for (c) Price relative method.
calculating (d) none
(a) real wages 69. For consumer price index, price quotations are
(b) money income index collected from :
(c) real income index (a) retailers
(d) all the above. (b) wholesale dealers
61. If a family spends on food, housing and clothing (c) fair price shops
in the rate of 5:3:2 and experiences the rise in (d) government depots
prices of these heads by 40, 30 and 20 per cent
70. The consumer price index in 1990 increases by
respectively, the family budget will be
80% as compared to the base 1980. A person in
increased by
1980 getting ` 60,000 per annum should now
(a) 33% (b) 40%
get :
(c) 37% (d) none
62. If Laspeyre’s price index is 324 and Paasche’s (a) ` 1,08,000 per annum
price index 144, then Fisher’s ideal index is (b) ` 72,000 per annum
(a) 234 (b) 180 (c) ` 54,000 per annum
(c) 216 (d) none (d) none
63. If the consumer price index for 1994 is 800,
then the purchasing power of a rupee is
ANSWER KEYS
(a) 0.125 paise (b) 12.5 paise 1 c 21 a 41 c 61 a
(c) 8 paise (d) none 2 d 22 b 42 c 62 c
64. The consumer price index numbers for 1981 3 a 23 c 43 c 63 b
and 1982 to the base 1974 are 320 and 400 4 d 24 b 44 c 64 b
respectively. The consumer price index for 5 c 25 c 45 d 65 a
1981 to the base 1982 is 6 d 26 c 46 c 66 c
(a) 125 (b) 80 7 d 27 a 47 b 67 b
(c) 128 (d) none
8 b 28 b 48 b 68 b
65. The index number for 1985 to the base 1980 is
9 b 29 c 49 d 69 a
125 and for 1980 to the base 1985 is 80. The
given indices satisfy 10 b 30 b 50 a 70 a
(a) time reversal test 11 a 31 b 51 c
(b) factor reversal test 12 b 32 b 52 b
(c) circular test 13 c 33 c 53 b
(d) all the above 14 c 34 d 54 a
66. The price relatives for three commodities are 15 b 35 d 55 b
125, 120 and 130 with their respective weights 16 a 36 d 56 d
5, w and 8. If the price index for the set is 125.25, 17 b 37 a 57 a
the value of w is 18 b 38 b 58 b
(a) 6 (b) –7
19 a 39 c 59 d
(c) 7 (d) none
67. If the group indices are 80, 120 and 125 and 20 b 40 c 60 d

426 BUSINESS MATHEMATICS, LOGICAL REASONING & STATISTICS (Paper 3) [CA Foundation]
6. The ideal average particular suitable for the
HOME WORK-2 construction of Index number is
(a) AM (b) GM
(c) HM (d) None
1. Fishers Ideal Formula satisfied 7. Consumer price Index number from a year 2004
to 2010 changed 100 to 200. The salary of an
(1) Unit Test employee has changed from Rs.3,000 to
(2) Circular Test (a) Rs.3,500 (b) Rs.2,500
(3) Factor Reversal Test (c) Rs.6,000 (d) Rs.3,500
(4) Time Reversal Test 8. Which of the following statement is true?
(a) 1 and 2 (b) 3 and 4 (a) Paache's Index Number is based on the
(c) 1 and 3 (d) 1,2 and 3 base year quantity
(b) Fisher's Index Number is the Arithmetic
2. While construction of Index numbers which
Mean of Laspeyre's Index Number and
of the following has to be considered as point
Paache's Index Numbers
of the following has to be considered as point
of reference in company various data (c) Arithmetic Mean is the most appropriate
describing individual behaviour. average for constructing the index
number
(a) Selection of weights
(d) Fisher's Index Number is an Ideal Index
(b) Base Period
Number
(c) Selection of Formulae
9. If Laspeyre's Index Number is 250 and Paache's
(d) Choice of variables Index Number is 160. then F isher's index
3. Fishers’ Ideal Index number is number is:
(a) The median of Laspyre’s and Paasches Index 25
numbers (a) 40,000 (b)
16
(b) The Arithmetic mean of Laspyres and
Paasche’s Index numbers 16
(c) 200 (d)
25
(c) The geometric mean of Laspyres and
Paasche’s Index Numbers
(d) None of these ANSWER KEYS
4. Which of the options does not contain the
proper use of Index numbers 1 (b) 2 (b)
(a) Helpful in policy determination 3 (c) 4 (c)
(b) Useful in Forecasting 5 (b) 6 (b)
(c) Equally useful in all condition for different 7 (c) 8 (d)
purpose
9 (c)
(d) Helpful in comparison
5. ____is an extent of time reversal test
(a) Factor reversal test
(b) Circular Test
(c) Both (d) None of them

INDEX NUMBER AND TIME SERIES 427


UNIT - 2

TIME SERIES

INTRODUCTION
Time series: Statistical data collected at regular interval of time (decade, annual, months, days, hours etc.)
and arranged in a chronological order. E.g. population of a country for last few decades, annual agricultural
production and the hourly temperature measured in a city, etc.
Components of time series:
1. Secular or long term trend
2. Seasonal variations
3. Cyclical variations
4. Irregular or random variations

1) Secular or long term trend: It refers to tendency of time series to move in upward or downward
directions or remain stagnant. It is the effect of long term forces that gradually affect the variable
under consideration. It is the behaviour of the time series over the entire period under consideration.
E.g. increasing trend of population of a country over a period, increase in industrial production due to
modern technology, decrease in death rate due to improvement of medical facilities etc. Most of the
time series relating to Economic Business and commerce show upward trend. We notice downward
trend in death rate, birth rate and share price of a company.
2) Seasonal variations: It refers to periodic and regular variations occurring in the time series. The period
of oscillation of such variations is less than a year. Such variations have similar pattern year after year.
Such variations are seen if the data are collected quarterly, monthly, weekly, daily, or hourly basis.
Variations may be due to various season or weather conditions, conventions, habits, customs or
traditions. E.g. increase in sale of cold drinks in summer, increase in sale during Diwali, decrease in sale
of ice-cream during winter.
3) Cyclical variations: Such variations occur at more or less regular interval of more than a year. They are
less regular as compared to seasonal variations. Business cycles are common examples. They have four
phases consisting of boom, recession, depression and recovery. All these phases together may last for
7 to 9 years may be more or less.
4) Irregular or random variations: It refers to variations due to unforeseen and unpredictable forces. They
do not have definite pattern and cannot be predicted in advance. E.g. variations due to war, strikes,
floods earthquakes etc.

428 BUSINESS MATHEMATICS, LOGICAL REASONING & STATISTICS (Paper 3) [CA Foundation]
Models of time series:

1. Additive model: when four components are believed to be independent we use additive model Yt
= T+C+S+I
Yt = Time series
T= Trend variations
S= Seasonal variation
C= Cyclical variations
I= Irregular variations
2. Multiplicative model: When four components are believed to be interdependent we use
multiplicative model Yt = T x C x S x I This model is generally used in business and economic data.
Measurement of secular trend:
1. Graphical or freehand curve method
2. Method of semi average
3. Method of moving averages
4. Method of least squares
Measurement of Seasonal variations:
1. Simple average method
2. Ratio to trend method
3. Ratio to moving average method
4. Link relative method

INDEX NUMBER AND TIME SERIES 429


4. Calculate the seasonal Indices for each quarter
CLASS WORK from the following percentages of whole sale
prices indices to their moving averages.

1. Calculate fi ve yearly moving averages for the Year Quarter


following data. I II III IV
Year Value (‘000 `) 2003 - - 11.0 11
1991 123 2004 12.5 13.5 15.5 14.5
1992 140 2005 16.8 15.2 13.1 15.3
1993 110 2006 11.2 11.0 12.4 13.2
1994 98 2007 10.5 13.3 - -
1995 104
5. Assuming no trend in the series, Calculate
1996 133 seasonal indices for the following data.
1997 95 Year Quarter
1998 105 I II III IV
1999 150
2004 78 66 84 80
2000 135
2005 76 74 82 78
2. From the following data verify that 5 year
2006 72 68 80 70
weighted moving average with weights 1, 2, 2,
2, 1 respectively is equivalent to the 4 year 2007 74 70 84 74
centred moving average: 2008 76 74 86 82
Year Value (‘000 `) 6. The annual production in commodity is given
1999 5 as follows:
2000 3 Year Production (in tonnes)
2001 7 2000 70
2002 6 2001 80
2003 4 2002 90
2004 8
2003 95
2005 9
2004 102
2006 10
2005 110
2007 8
2006 115
2008 9
(a) Fit a straight line trend by the method of
2009 9
least squares.
3. Calculate the Seasonal Indices by the method
(b) Convert the annual trend equation into
of Link Relatives for the following data.
monthly trend equation.
Quarterly Figures for Five years
7. An orderly set of data arranged in accordance
Quarter 2003 2004 2005 2006 2007 with their time of occurrence is called:
I 45 48 49 52 60 (a) Arithmetic series
II 54 56 63 65 70 (b) Harmonic series
III 72 63 70 75 83 (c) Geometric series
IV 60 56 65 72 86 (d) Time series

430 BUSINESS MATHEMATICS, LOGICAL REASONING & STATISTICS (Paper 3) [CA Foundation]
8. A time series consists of: 16. Wheat crops badly damaged on account of
(a) Short-term variations rains is:
(b) Long-term variations (a) Cyclical movement
(b) Random movement
(c) Irregular variations
(c) Secular trend
(d) All of the above
(d) Seasonal movement
9. The graph of time series is called:
17. The method of moving average is used to fi nd
(a) Histogram (b) Straight line the:
(c) Historigram (d) Ogive (a) Secular trend
10. Secular trend can be measured by: (b) Seasonal variation
(a) Two methods (b) Three methods (c) Cyclical variation
(c) Four methods (d) Five methods (d) Irregular variation
11. The secular trend is measured by the method 19. Most frequency used mathematical model of
of semi-averages when: a time series is:
(a) Time series based on yearly values (a) Additive model
(b) Trend is linear (b) Mixed model
(c) Time series consists of even number of (c) Multiplicative model
values (d) Regression
(d) None of them 20. A time series consists of:
12. Increase in the number of patients in the (a) No mathematical model
hospital due to heat stroke is: (b) One mathematical model
(a) Secular trend (c) Two mathematical models
(b) Irregular variation (d) Three mathematical models
(c) Seasonal variation 21. In semi-averages method, we decide the data
into:
(d) Cyclical variation
(a) Two parts
13. The systematic components of time series
which follow regular pattern of variations are (b) Two equal parts
called: (c) Three parts
(a) Signal (b) Noise (d) Diffi cult to tell
(c) Additive model 22. Moving average method is used for
measurement of trend when:
(d) Multiplicative model
(a) Trend is linear
14. The unsystematic sequence which follows
(b) Trend is non-linear
irregular pattern of variations is called:
(c) Trend is curvi linear
(a) Noise (b) Signal
(d) None of them
(c) Linear (d) Non-linear
23. When the trend is of exponential type, the
15. In time series seasonal variations can occur moving averages are to be computed by using:
within a period of:
(a) Arithmetic mean
(a) Four years (b) Three years
(b) Geometric mean
(c) One year (d) Nine years
(c) Harmonic mean
(d) Weighted mean

INDEX NUMBER AND TIME SERIES 431


24. The long term trend of a time series graph 31. Damages due to fl oods, droughts, strikes fi
appears to be: res and political disturbances are:
(a) Straight-line (b) Upward (a) Trend (b) Seasonal
(c) Downward (c) Cyclical (d) Irregular
(d) Parabolic curve or third degree curve 32. The general pattern of increase or decrease in
25. Indicate which of the following an example of economics or social phenomena is shown by:
seasonal variations is: (a) Seasonal trend
(a) Death rate decreased due to advance in (b) Cyclical trend
science (c) Secular trend
(b) The sale of air condition increases during (d) Irregular trend
summer
33. In moving average method, we cannot fi nd
(c) Recovery in business the trend values of some:
(d) Sudden causes by wars (a) Middle periods
26. The most commonly used mathematical (b) End periods
method for measuring the trend is:
(c) Starting periods
(a) Moving average method
(d) Between extreme periods
(b) Semi average method
34. Moving-averages:
(c) Method of least squares
(a) Give the trend in a straight line
(d) None of them
(b) Measure the seasonal variations
27. A trend is the better fi tted trend for which the
sum of squares of residuals is: (c) Smooth-out the time series

(a) Maximum (b) Minimum (d) None of them

(c) Positive (d) Negative 35. The rise and fall of a time series over periods
longer than one year is called:
28. Decomposition of time series is called:
(a) Secular trend
(a) Historigram
(b) Seasonal variation
(b) Analysis of time series
(c) Cyclical variation
(c) Histogram
(d) Irregular variations
(d) Detrending
36. A time series has:
29. The fi re in a factory is an example of:
(a) Two Components
(a) Secular trend
(b) Three Components
(b) Seasonal movements
(c) Four Components
(c) Cyclical variations
(d) Five Components
(d) Irregular variations
37. The multiplicative time series model is:
30. Increased demand of admission in the subject
of computer in Uttar Pradesh is: (a) Y=T+S+C+I (b) Y = TSCI

(a) Secular trend (c) Y= a + bx (d) y = a + bx + C x 2

(b) Cyclical trend 38. The additive model of Time Series

(c) Seasonal trend (a) Y=T+S+C+I (b) Y = TSCI

(d) Irregular trend (c) Y= a + bx (d) y = a + bx + C x 2

432 BUSINESS MATHEMATICS, LOGICAL REASONING & STATISTICS (Paper 3) [CA Foundation]
39. A pattern that is repeated throughout a time
series and has a recurrence period of at most
one year is called: HOME WORK-1
(a) Cyclical variation
(b) Irregular variation 1. The annual trend equation with 2002 as origin
(c) Seasonal variation is Y = 30 + 10X. If the origin is shifted to 2005
then the equation will be
(d) Long term variation
a) Y = 30 + 13X b) Y = 60 + 13X
40. If an annual time series consisting of even
number of years is coded, then each coded c) Y = 60 + 10X d) Y = 33 + 10X
interval is equal to: 2. If the annual trend equation Y = 36 + 28.8X is
(a) Half year (b) One year converted into monthly trend equation then
the equation will be
(c) Both (a) and (b) (d) Two years
a) Y = 3 + 2.4X b) Y = 3+ 0.2X
41. In semi averages method, if the number of
values is odd then we drop: c) Y = 36 + 0.2X d) Y = 36 + 2.8X
3. If the equation Y = 2400 + 432X with 2002 as
(a) First value (b) Last value
origin , X unit = 1 year and Y = average monthly
(c) Middle value sales is converted into monthly equation then
(d) Middle two values the equation will be
42. The trend values in freehand curve method a) Y = 200 + 3X b) Y = 200+ 36X
are obtained by: c) Y = 2400 + 3X d) Y = 2400 + 36X
(a) Equation of straight line 4. If the annual trend equation Y = 36 + 28.8X is
(b) Graph converted into quarterly trend equation then
the equation will be
(c) Second degree parabola
a) Y = 9 + 1.8X b) Y = 9+ 7.2X
(d) All of the above
c) Y = 36 + 7.2X d) Y = 36 + 1.8X
5. Convert Y = 20 + 0.5X monthly equation into
yearly equation
a) Y = 240 + 72X b) Y = 20+ 72X
c) Y = 240 + 6X d) Y = 20 + 6X
 6. The annual trend equation with 2002 as origin
is Y = 30 + 10X. predict the trend value for 2006
a) 100 b) 70
c) 90 d) 30
7. Year 2008 2009 2010
2011 2012 2013 2014
2015 2016
Values 506 620 1036
673 588 696 1116
738 663
4 yearly moving average for 2011
(a) 705 (b) 720.3
(c) 738.75 (d) 776.4

INDEX NUMBER AND TIME SERIES 433


8. If the total sales forecasted for 2018 is 60,000
and seasonal indices are HOME WORK-2
Quarter 1 2
3 4
1. Using the following table for trend values
S.I. 80 120 taken three year Moving Averages using a, b
150 50 and C are
(a) 22,500 (b) 25000 Year Profit Yearly Moving
(c) 90,000 (d) 30,000 Averages
9. If the sales for 1st quarter is 48,000 then 2002 40 ---
estimate the sale for 2nd quarter 2003 60 a
Quarter 1 2 2004 68 b
3 4 2005 70 c
S.I. 80 120 2006 90 ---
150 50 (a) AP (b) HP
(a) 22,500 (b) 25000 (c) GP
(c) 72000 (d) 30,000 (d) Neither AP or nor HP or GP
10. In a straight line equation Y = a + bX; a is the: 2. The simple average method is used to
calculate:
(a) X-intercept (b) Slope
(a) Trend Variation
(c) Y-intercept (d) None of them
(b) Cyclical Variation
11. In a straight line equation Y = a + bX; b is the:
(c) Seasonal Variation
(a) Y-intercept (b) Slope (d) Irregular Variation
(c) X-intercept (d) Trend
3. If  P0Q0 = 240,  P1Q1 = 480,  P1Q1 = 600

 P0Q1 = 192, then Laspeyre's index number is


ANSWER KEYS :
(a) 250 (b) 300
1 (c) 2 (b) 3 (d) (c) 350 (d) 200
4 (a) 5 (a) 6 (b) 4. The Sale of Cold Drink would go up in summers
7 (c) 8 (a) 9 (c) and go down in the winters is an example of:
10 (c) 11 (b) (a) Trend Variation
(b) Cyclical Variation
(c) Seasonal Variation
---0---0---
(d) Irregular Variation

ANSWER KEYS

1 (a) 2 (c) 3 (a) 4 (c)

G G G

434 BUSINESS MATHEMATICS, LOGICAL REASONING & STATISTICS (Paper 3) [CA Foundation]

You might also like